You are on page 1of 577

6th Edition

(Solved GATE Questions 1991-2020)


B. K. Das

GATE
Architecture/ Planning
(6 ed.)

Review of GATE Questions


© B. K. Das
6th edition, 2020
ISBN: 978-93-5406-410-4
e mail: dear.bijay@gmail.com
facebook: https://www.facebook.com/groups/1143797872301025/
Rs 1,200/- (Rupees One Thousand Two Hundred only)
Available through popular online stores.
Photocopying or unauthorized distribution is prohibited.
Allowed for distribution to StoneHenge GATE Architecture Coaching at New Delhi,
Hyderabad, Nagpur, Kolkata and Lucknow.

Printed at Maa Laxmi Printers, Nayatola, Patna


Published by KUSH BROTHERS Patna

The author expresses his sincere thanks to Students of Arch/Planning institutes


all over India, who helped in compilation of this study material, and pointing out
the error from time to time.
Contents
GLOSSARY
GATE 1991
GATE 1992
GATE 1994
GATE 1995
GATE 1996
GATE 1997
GATE 1998
GATE 1999
GATE 2000
GATE 2001
GATE 2002
GATE 2003
GATE 2004
GATE 2005
GATE 2006
GATE 2009
GATE 2007
GATE 2008
GATE 2010
GATE 2011
GATE 2012
GATE 2013
GATE 2014
GATE 2015
GATE 2016
GATE 2017
GATE 2018
GATE 2019
GATE 2020
GLOSSARY
Books and Authors

No Author Book
1. Adolf Loos Ornamentation and Crime
2. Alvin Toffler Future Shock
3. Allan Dobby Conservation and Planning
4. Alvar Alto An Experimental Town
5. Alvar Alto The humanizing of Architecture
Amos
6. Rapoport Human Aspects of Urban Form
Amos
7. Rapoport House form and Culture
Andrea
8. Palladio Four Books of Architecture
9. Ayan Rand Fountainhead
Leonardo da
10. Vinci Codex Atlanticus
11. Brian Hackett Planting Design
12. Brian Hackett Landscape planning
13. Brian Hackett Man Society and Environment
14. Bruno Zevi Towards an Organic Architecture
Buckminster
15. Fuller The Dimaxian World
16. Burgees The City
17. C P Kukreja Tropical Architecture
Cecil Maurice
18. Bowra Golden Ages of Great Cities
Charles
19. Abraham The future of Housing (GATE 1991)
Charles
20. Correa The New Landscape
21. Charles Correa Urbanisation in the third world
Language of Post- Modern Architecture
22. Charles Jenks (GATE 1991)
Christopher
23. Alexander Pattern Language
Christopher
24. Alexander Synthesis of Form
Christopher
25. Alexander Oregon Experiment
Christopher
26. Alexander Community and Privacy
Christopher The city as a mechanism for sustaining
27. Alexander Human contact
Christopher
28. Alexander Pattern of streets
Christopher
29. Tunnard Gardens in Modern Landscape
No Author Book
Clarence
30. Stein Towards new Towns in America
31. Devid Lewis Urban Structure
32. Doxiadis Ekistics
33. Doxiadis Dynapolis
Urban Renewal and The Future of
34. Doxiadis Urban Cities
35. E. Saarinen City- its growth its decay its future
Ebenezer Tomorrow a Peaceful Path to Social
36. Howard Reform
Edmund N.
37. Bacon Design of Cities
38. F. A Gutkind Our World from Air
39. F A Gutkind Revolution in Environment
40. F L Wright Writings and Buildings
41. F L Wright Disappearing City
42. F L Wright The living city
43. Faber Birren New Horizon in Color
Francis D. K.
44. Ching Architecture: Form, Space & Order
Francis D. K.
45. Ching Architectural Graphics
Francis D. K.
46. Ching A Visual Dictionary of Architecture
Fredric
47. Gibberd Town Design
48. Felix Candela Structural digressions on style
49. Felix Candela Simple concrete shell structure
50. Felix Candela New ways to span space
51. Gallion Fisher Urban Pattern
52. G A Atkinson Introduction to tropical Building design
53. Geoffrey Scott The Architecture of Humanism
George F
54. Chadwick The park in the town
Gorden
55. Cullen Townscape
56. Garret Eckbo Landscape living
57. Garret Eckbo Urban Landscape design
58. Guy Gruer Your City Tomorrow
Harold Van
59. Dorsen Industrial Design
H V
60. Lanchaster The art of town planning
61. Hasan Fathy Mud Architecture
Hubbard and Introduction to study of Landscape
62. Kimball design
63. Ian Macharg Design with Nature
64. Jean Gottman Megalopolis
65. John Marshall Mohenjo-Daro and Indus civilization
No Author Book
66. John O Simon Earthscape
The Seven Lamps of Architecture
67. John Ruskin (GATE 1991)
Kenneth Meaning of 20th century the great
68. Boulding transition
Environment Technology in
69. Kenzeyz Sharp Architecture
70. Kenzeyz Sharp Acoustics in buildings
71. Kevin Lynch Image of City
72. Kevin Lynch Site Planning
73. Kevin Lynch City Form
74. Kevin Lynch What time is this place
Principles and Practice of Town and
75. Lewis Keeble Country Planning
L
76. Hilbersheimer The New city
77. Le Corbusier The Radiant City
78. Le Corbusier The Modular
79. Le Corbusier Concerning Town Planning
80. Le Corbusier The City of Tomorrow and its Planning
81. Le Corbusier New World of Space
82. Le Corbusier Towards New Architecture
Lewis
83. Mumford The city in History
Lewis
84. Mumford Culture of cities
85. Lewis Mumford The Brown Decades
86. Lewis Mumford The city development
87. Lewis Mumford Condition of man
88. Lewis Mumford Techniques and civilization
Lewis Mumford The city in History, its origin, its
89. transformation and its prospects
90. Lewis Mumford Pattern of streets
Lewis
91. Mumford The story of Utopians
92. Lloyd Rodwin Future of Metropolis
93. Mahony - Nagy Vision in Motion
94. Mahony - Nagy Matrix of man
Aesthetics of Proportion in Nature and
95. Matila Ghyka in the Arts
96. Met Scott Cities are for people
Margaret An Introduction to Town Planning
97. Roberts Technique
Nicholas
98. Negnoponte The Architecture Machine
Nicholas
99. Pevsner The Indian Metropolis
Norberg Meaning in Western Architecture
100. Schulz (GATE 1991)
No Author Book
Norman
101. Evenson A history of Building type
Oscar
102. Newman Defensible space (GATE 1991)
103. Paul Heyer Architects on Architecture
Arcology- the city in the image of
104. Paolo Soleri man
Patrick
105. Abercrombie Town and Country Planning
106. Patric Geddes Cities in Evolution
Paul D. The Architecture of Towns and
107. Spreiregen Cities
108. Pawley Architecture versus housing
Peter Geoffrey
109. Hall Cities of Tomorrow
110. Peter Wolf The future of the city
R
Buckminister
111. Fuller The Dimaxion World
112. R Scott Design Fundamentals
Richard N
113. Wagner Environment and Man
Raymond
114. Unwin Nothing gained by overcrowding
Reyner Theory and Design in the first machine
115. Banham age
Richard
116. Neutra Survival through Design
117. Robert Ardray The territorial Imperative
Robert Gillam
118. Scott Design Fundamentals
Robert Moore
119. Fisher Twenty Years of Public Housing
Robert
120. Sommer Personal space
Robert Complexity and Contradiction in
121. Venturi Architecture
Robert
122. Venturi Learning from Las Vegas
123. Robin Boyd Victorian Modern
124. Robin Boyd New Dimensions in Japan Architecture
Rose
125. Roseman The ideal city
126. S Geidon Space, time and Architecture
127. S Geidon Works of Joseph Paxton
128. Stuart Chapin Urban Landuse Planning
129. Sylvia Crowe The Landscape of Roads
130. Sylvia Crowe Garden Design
131. Talbot Hamlin Architecture through the Ages
No Author Book
Thomas
132. Adams Outline of Town and Country Planning
Thomas D
133. Church Gardens are for people
Thomas D
134. Church Your Private World
135. Tunnard The city of man
136. Victor Gruen The Emerging Urban Pattern
137. Victor Olgay Design with Climate
138. Victor Papanoc Design for real world
139. W D Teagni Design this Day
140. W Kennet Preservation
W Kohler/ Kurt
141. Koffle Gestalt Psychology
142. Wilfred own The accessible city
Yoshinobu
143. Ashihara Exterior Design in Architecture
1. Astropolis – star-scaled city/industry area; complex space station.
2. Aerotropolis- settlement around a major airport
3. Acropolis- Citadel located close to Athens, Greece and has ruins of Parthenon, stoa.
4. Artisanopolis- Settlement over sea.
5. Cosmopolis – a large urban centre with a population of many different cultural
backgrounds
6. Decapolis – a group of ten cities
7. Dodecapolis – a group of twelve cities.
8. Dynapolis- Islamabad, new capital of Pakistan, was designed by Coastantinos.
Doxiadis (1913-75), a famous Greek planner, in 1960 on the basis of his theory of
dynapolis.
9. Ecumenopolis – a city that covers an entire planet, usually seen in science
fiction.(50,000 million Population as estimated by Doxiadis)
10. Eopolis - the period of village (GATE 1997 )
11. Megalopois- built by merging several cities and their suburbs (Coined by Patric
Geddes)
12. Metropolis- city with population of 40 lacs and above in India
13. Necropolis- city of dead (graveyard) (GATE 1999)
14. Pentapolis – a group of five cities
15. Technopolis- a city with high-tech industry. (GATE 2004)

City Forms
1. Star Shaped City- Moscow, Washington D.C.
2. Linear City- Mumbai, Leningrad, Kolkata, Stalingrad
3. Ring form- Delhi, San Francisco
4. Sheet form- Tokyo
5. Poly Centred net- Detroit
6. Finger Plan- Copenhagen

Diagrams Charts and Nomograms


Hazen William Nomogram- physical property of pipe and pressure drop.
Psychrometric chart- RH using WBT and DBT
Ringlemann chart- smoke level
Sun Path diagram- Altitude and Azimuth of Sun
Waldram diagram- sky factor
Mahoney table- guide to climate appropriate design.

Ancient India (measurement)

Angul: width of one finger.


Pada: 8 angula
Hasta: 24 angula
Danda: 4 hasta
Cubit: distance from elbow to tip of middle finger. (equivalent to 45.72 cm)
Dhanurmusti: 26 angulas make one dhanurmusti (to measure buildings)
Dhanurgraha: 27 angulas make one dhanurgraha (to measure village and town)

Temple styles and their geographical extents (overlaps existed)


1. Nagara style: Mount Kailash to Vindhayas
2. Visera style: Vindhayas to River Krishna (hybrid of Nagara and Dravidian)
3. Dravidian style: River Krishna to Kanya Kumari.
Famous Sayings
1. Form follows Function- Louis Sullivan
2. Form Follows Sun- Raj Rewal
3. Less is more- Ludwig Meis van der Rohe
4. God is in Details- Ludwig Mies van der Rohe
5. Less is bore- Robert Venturi
6. Small is beautiful- E. F. Schumacher
7. Ornamentation and crime- Adolf Loos
AR Architecture and Planning
SYLLABUS
Section 1: Architecture and
Design
Visual composition in 2D and 3D; Principles of Art and Architecture; Organization
of space; Architectural Graphics; Computer Graphics– concepts of CAD, BIM, 3D
modeling and Architectural rendition; Programming languages and automation.
Anthropometrics; Planning and design considerations for different building types;
Site planning; Circulation- horizontal and vertical; Barrier free design; Space
Standards; Building Codes; National Building Code.
Elements, construction, architectural styles and examples of different periods of
Indian and Western History of Architecture; Oriental, Vernacular and Traditional
architecture; Architectural developments since Industrial Revolution; Influence of
modern art on architecture; Art nouveau, Eclecticism, International styles, Post
Modernism, Deconstruction in architecture; Recent trends in Contemporary
Architecture; Works of renowned national and international architects.
Section 2: Building Materials, Construction and Management
Behavioral characteristics and applications of different building materials viz. mud,
timber, bamboo, brick, concrete, steel, glass, FRP, AAC, different polymers,
composites.
Building construction techniques, methods and details; Building systems and
prefabrication of building elements; Principles of Modular Coordination;
Estimation, specification, valuation, professional practice; Construction planning
and equipments; Project management techniques e.g. PERT, CPM etc.
Section 3: Building and Structures
Principles of strength of materials; Design of structural elements in wood, steel
and RCC; Elastic and Limit State design; Structural systems in RCC and Steel;
Form and Structure; Principles of Pre-stressing; High Rise and Long Span
structures, gravity and lateral load resisting systems; Principles and design of
disaster resistant structures.
Section 4: Environmental Planning and Design
Ecosystem- natural and man-made ecosystem; Ecological principles; Concepts of
Environmental Impact Analysis; Environmental considerations in planning and
design; Thermal comfort, ventilation and air movement; Principles of lighting and
illumination; Climate responsive design; Solar architecture; Principles of
architectural acoustics; Green Building- Concepts and Rating; ECBC; Building
Performance Simulation and Evaluation; Environmental pollution- types, causes,
controls and abatement strategies.
Section 5: Urban Design
Concepts and theories of urban design; Public Perception; Townscape; Public
Realm; Urban design interventions for sustainable development and
transportation; Historical and modern examples of urban design; Public spaces,
character, spatial qualities and Sense of Place; Elements of urban built
environment – urban form, spaces, structure, pattern, fabric, texture, grain etc;
Principles, tools and techniques of urban design; Urban renewal and
conservation; Site planning; Landscape design; Development controls – FAR,
densities and building byelaws.

Section 6: Urban Planning and Housing


Planning process; Types of plans - Master Plan, City Development Plan, Structure
Plan, Zonal Plan, Action Area Plan, Town Planning Scheme, Regional Plan;
Salient concepts, theories and principles of urban planning; Sustainable urban
development; Emerging concepts of cities - Eco-City, Smart City, Transit Oriented
Development (TOD), SEZ, SRZ etc.
Housing; Concepts, principles and examples of neighbourhood; Housing
typologies; Slums; Affordable Housing; Housing for special areas and needs;
Residential densities; Standards for housing and community facilities; National
Housing Policies, Programs and Schemes.
Section 7: Planning Techniques and Management
Tools and techniques of Surveys – Physical, Topographical, Landuse and Socio-
economic Surveys; Methods of non-spatial and spatial data analysis; Graphic
presentation of spatial data; Application of G.I.S and Remote Sensing techniques
in urban and regional planning; Decision support system and Land Information
System.
Urban Economics; Law of demand and supply of land and its use in planning;
Social, Economical and environmental cost benefit analysis; Techniques of
financial appraisal; Management of Infrastructure Projects; Development
guidelines such as URDPFI; Planning Legislation and implementation – Land
Acquisition Act, PPP etc.; Local self-governance.
Section 8: Services, Infrastructure and Transportation
Building Services: Water supply; Sewerage and drainage systems; Sanitary
fittings and fixtures; Plumbing systems; Principles of internal and external
drainage system; Principles of electrification of buildings; Intelligent Buildings;
Elevators and Escalators - standards and uses; Air-Conditioning systems;
Firefighting Systems; Building Safety and Security systems.
Urban Infrastructure – Transportation, Water Supply, Sewerage, Drainage, Solid
Waste Management, Electricity and Communications.
Process and Principles of Transportation Planning and Traffic Engineering; Road
capacity; Traffic survey methods; Traffic flow characteristics; Traffic analyses and
design considerations; Travel demand forecasting; Land-use – transportation -
urban form inter-relationships; Design of roads, intersections, grade separators
and parking areas; Hierarchy of roads and level of service; Traffic and transport
management and control in urban areas,; Mass transportation planning; Para-
transits and other modes of transportation, Pedestrian and slow moving traffic
planning; Intelligent Transportation Systems.
Principles of water supply and sanitation systems; water treatment; Water supply
and distribution system; Water harvesting systems; Principles, Planning and
Design of storm water drainage system; Sewage disposal methods; Methods of
solid waste management - collection, transportation and disposal; Recycling and
Reuse of solid waste; Power Supply and Communication Systems, network,
design and guidelines.
GATE 1991
Q.1
(i) Ratio ‘Golden Mean’ is:
(A) 1: 2.216 (B) 1: 1.618 (C) 1: 1.50 (D) 1: 1.44

Answer: (B) 1: 1.618

=
Let the ratio be x: 1 then putting a=x and b=1

= by cross multiplication we get = x+1 (this equation is quadratic and we


solve for x)

-x-1= 0 we use the formula where a= 1, b= -1 and c= -1

Ignoring negative value we get x= 1.618


Fibonocci series : 0, 1, 1, 2, 3, 5, 8, 13, 21, 34, 55……….
Division of ‘n’th term with (n-1)th term in Fibonnoci series = golden ratio;

for ‘n’ a higher value we see that =


1.617

(ii)‘Mihrab’ is found
(A) on the west wall of a mosque
(B) inside wall of a masoleum
(C) in the stepped well of Gujarat
(D) on the crown of minaret
Answer: (A) on the west wall of a mosque
Mihrab is a semicircular niche in the wall of a mosque that indicates the qibla; that
is, the direction of the Kaaba in Mecca and hence the direction that Muslims
should face when praying. The wall in which a mihrab appears is thus the "qibla
wall".
In India it is found in west wall, as Mecca/Medina is in west of India.

) Which one falls under Indo-Sarasanic architecture?


(A) Qutab Minar (B) Taj Mahal (C) Sher Shah’s tomb (D) Bahai Temple
Answer: No appropriate answer
Indo-Saracenic building in India, incorporate elements and motifs of Hindu and
Islamic Architecure. It was introduced by British Architect and used in Public buildings
of all sorts such as railway stations, banks and insurance buildings, educational
institutions, clubs and museums .
Chepauk Palace in Chennai designed by Paul Benfield is said to be the first
example of Indo- Saracenic Architecture.
The Central Railway Station, Law courts, Victoria Public Hall, Museum and
University Senate House in Chennai
Muir college at Allahabad
Napier Museum at Thiruvananthapuram
Post Office, Prince of Wales Museum, University Hall and Library, Gateway of
India in Mumbai
M.S. University, Lakshmi Vilas Palace at Baroda,
Palaces at Mysore and Bangalore.
New Delhi Secretariat, North and South Block, Rashtrapati Bhawan, New
Delhi.

) Which of the following is not a function of a good mulch:


(A) Aid in water retention (B) Prevent soil temperature fluctuations
(C) Encourage weed growth (D) Improve landscape appearance
Answer: (C) Encourage weed growth
A mulch is a layer of material applied to the surface of soil. Reasons for applying
mulch include
conservation of soil moisture,
improving fertility and health of the soil,

reducing weed growth


enhancing the visual appeal of the area.

(v)Byzantine architecture is famous for:


(A) Stone carving (B) Pointed arches
(C) Fluted columns (D) New type dome construction
Answer: (D) New type dome construction
Dome have openings at springing level.
Hagia Sophia (Istanbul, Turkey) is the most important Byzantine Architecture.

(vi)‘Gopuram’ refers to
(A) Temple (B) Gateway (C) Village (D) Brick dome
Answer: (B) Gateway
A Gopuram is a monumental tower, at the entrance of any temple, especially in
Southern India.
Tallest Gopuram in India- Sri Ranganathaswamy Temple (Srirangam,
Tiruchirappalli) –UNESCO World Heritage site. Srirangam is a river island town
on Kaveri river.
Srirangam- town planning Sarvatobhadra

(vii) Most efficient arch in transferring load:


(A) Semi circular (B) Flat

(C) Pointed (D) Catenary


Answer: (D) Catenary
In geometry, a catenary is the curve that an idealized hanging chain or cable
assumes under its own weight when supported only at its ends. The catenary curve
has a U-like shape, superficially similar in appearance to a parabolic arch, but it is not
a parabola.

(viii) Dimension of Corbusier’s modular human scale are in:


(A) Arithmetic progression
(B) Geometric progression
(C) Unrelated natural order
(D) None of the above
Answer: (C) Unrelated natural order
Le Corbusier described it as a "range of harmonious measurements to suit the
human scale, universally applicable to architecture and to mechanical things".
(ix) Find odd one out of the following:
(A) T-trap (B) Q- trap
(C) S- trap (D) P- trap
Answer: (A) T-trap

(x)pH value of treated domestic water supply should be between :


(A) 4.0 – 5.5 (B) 5.5 – 6.5
(C) 6.5 – 8.0 (D) 8.0 – 9.5
Answer: (C) 6.5 – 8.0

xi) PERT is
(A) Node oriented(B) Cost oriented (C) Activity oriented (D) Event oriented
Answer: (D) Event oriented
CPM uses activity oriented while PERT uses event oriented network.

xii)Gypsum is added in cement manufacture for the property of:


(A) Increasing the binding capcity (B) Retarding the setting time
(C) Reducing heat of hydration (D) Increasing strength
Answer: (B) Retarding the setting time
Superplasticizers- Increase flowability of concrete, reduce water cement ratio.

Functions of admixtures to modify fresh concrete


properties:
To increase workability without increasing water content or to decrease the water
content at the same workability.
To retard or accelerate both initial and final setting times.
To reduce or prevent settlement.
To create slight expansion in concrete and mortar.
To modify the rate or capacity for bleeding or both.
To reduce segregation of concrete, mortars and grouts.
To improve penetration and or pumpability of concrete, mortars and grouts.
To reduce rate of slump loss.
Functions of admixtures to modify hardened concrete properties:
To retard or reduce heat generation during early hardening.
To accelerate the rate of strength development.

No Type of Desired effect Material


admixture
1 Accelerator Accelerate setting Calcium chloride,
and early-strength Triethanolamine, sodium
development thiocyanate, calcium formate,
calcium nitrite, calcium
nitrate
2 Corrosion Reduce steel Calcium nitrite, sodium
inhibitors corrosion activity in nitrite, sodium benzoate,
a chloride-laden certain
environment phosphates or fluosilicates,
fluoaluminates, ester amines
3 Damp- Retard moisture Soaps of calcium or
proofing penetration into dry ammonium stearate or oleate
admixtures concrete Butyl stearate
Petroleum products
4 Retarders Slow the setting Gypsum
process of cement
(xiii)Critical path in CPM connects the initial and end events and:
(A) consists of events all having zero or minimum slack times (B) consists of events
all having maximum slack times
(C) consists of events without dummy events (D) none of the above
Answer: (A) consists of events all having zero or minimum slack times
(with input from Anindita Dey, SPA Bhopal,2013-15)

(xiv) Maximum distance for discerning facial expression in a theatre:


(A) 12.5 m (B) 18.0 m (C) 22.5 m (D) 30.0 m
Answer: (A) 12.5 m
Solution provided by Surabhi Jethani, B. Arch (MITS Gwalior)
Maximum distance for seeing people= 4000 ft (1200 m)
Maximum distance for discerning action= 450 ft (137 m)
Maximum distance for recognizing a face= 80 ft (24 m)
Maximum distance for discerning facial expression= 40 ft (12m )
Range of conversational distance= 10 ft (3 m)
(source: The Architecture of Town and Cities, Paul D. Spreiregen)
(xv) Outer layer of timber log is
(A) Heart wood (B) Strong wood
(C) Sap wood (D) Knot wood
Answer: (C) Sap wood

(xvi) Sun temple of Konark predominantly built with


(A) Basalt stone (B) Soap stone
(C) Sand stone (D) Granite stone
Answer: (C) Sand stone
Sun Temple Konark-Chlorite was used in door frame, Laterite was used in
foundation, staircase and centre of the platform and Khondalite stone (Red
sandstone) was used elsewhere.
Khajuraho Temples- Sandstone
Brihadeshwara, Tanjavur- Granite
Mughal mostly utilized sandstone, with the exception of marble in Taj Mahal.

(xvii) Average power output (in microwatt) of human speech in ordinary conversation is:
(A) 2.5 (B) 4.8 (C) 6.2 (D) 10.0
Answer: (D) 10.0
Ordinary conversation is at 65 dB at a distance of 0.5 m.

Finding out the intensity of sound wave at a


distance of 0.5 m

dB= 10 log10(

65= 10 log10( solving this we get


I= 10 watt/m
-5.5 2

Power at the source = Intensity × 4π(d)2


= 10-5.5 ×4×3.14×(0.5)2
= 3.14 × Watt
(Now the ans. is in microwatt, so the power should come in
Here we do a bit of mathematics !
we multiply and divide 3.14 by

= ×(
= 3.14 × ×( watt
= 3.14 × ×( watt
= 3.14 × 3.16 × ( watt
= 9.92 × watt
Intensity of Various Sounds
Source of the Sound Level (dB) Intensity (Watt/m²)
sound
Jet Plane at 30 m 140 100
Treshold of pain 120 1
Loud rock concert 120 1
Siren at 30 m 100

Busy street traffic 80

Noisy restaurent 70

Talk, at 50 cm 65

Quiet Radio 40

Whisper 30

Rustle of leaves 10

Threshold of 0
hearing 1×

(xviii) Principal determinants of a residential neighbourhood size is based on:


(A) Landuse composition
(B) Availability of vacant land
(C) Education facility
(D) Residential density
Answer: (C) Education facility
School within a radius of half km.The concept of the neighborhood unit, was
propogated by Clarence Perry, is an early diagrammatic planning model for
residential development in metropolitan areas (1929).
(xix) Minimum height of habitable room as prescribed in NBC :
(A) 1.85 m (B) 2.75 m(C) 3.0 m (D) 3.2 m
Answer: (B) 2.75 m (as per earlier code of NBC 1983), Now it is 2.6 m (NBC 2016)
Modifications as per NBC 2016
C-3.3.1 Habitable Room
Every dwelling unit to be provided should have at least two habitable rooms. Even if one
room house is provided initially it should be capable of adding a new second room in
future. However, in case single room tenements are required to be provided where future
additions are not possible, the carpet area of multipurpose single room should be at
least 12.5 m². Such one room dwelling units with 12.5 m² carpet area of habitable space is
permitted only in case of on-site rehabilitation of slum dwellers. In a house of two rooms,
first room shall not be less than 9.0 m² with minimum width of 2.5 m and second room
shall not be less than 6.5 m² with a minimum width of 2.1 m provided the total area of both
the rooms is not less than 15.5 m². In incremental housing the bigger room shall always
be the first room.

C-3.5 Minimum Height


The minimum height of rooms/spaces shall be as follows:
a) Habitable room : 2.6 m
b) Kitchen : 2.6 m
c) Bath/water-closet : 2.1 m
d) Corridor : 2.1 m
source: NBC (2016) –Vol 1 . page 138

(xx) Desirable housing layout of buildings from acoustic point of view is :


(A) Courtyard type (B) Stilted multistoried flats
(C) Open type single or semidetached houses (D) none of the above.
Answer: (C) Open type single or semidetached houses

Q.2 Match the following:


Q. 2 (i)
(a)The city in history (A) Patrick Geddes (g)
(b) Life and death of great American cities (B) Norberg Schulz (j)
(c)The Modular (C) John Ruskin (e)
(d) The Future of Housing (D) Charles Abraham (d)
(e)Seven Lamps of Architecture (E) Oscar Newman (h)
(f) Language of Post –Modern Architecture (F) Lewis Mumford (a)
(g) Cities in Evolution (G) Jean Jacob (b)
(h) Defensible Space (H) Le Corbusier (c)
(i) The New Landscape (I) Charles Jenka (f)
(j) Meaning in Western Architecture (J) Charles Correa (i)
(ii)(a) Crystal Palace (A) Le Corbusier (c)
(b) Johnson Wax factory (B) Joseph Paxton (a)
(c)Shoden Villa (C) B. V. Doshi (i)
(d) German Pavillion, Barcelona (D) Mies Van der Rohe (d)
(e)Pompidou Centre Paris (E) Frank Lloyd Wright (b)
(f) TWA Terminal, Kennedy Airport (F) Charles Correa (g)
(g) Kanchanjunga Apartment, Bombay (G) Eero Saarinen (f)
(h) National Institute of Bank Management, Pune (H) A.P. Kanvinde (h)
(i) I. I. M. Bangalore (I) Sarat Das (j)
(j) Indira Gandhi Sports Complex, Indraprastha (J) R. Rogers and Renzo Piano (e)
iii)(a) Grid Iron Pattern (A) Kautilya (e)
(b) Vastu Shastra (B) Hippodamus (a)
(c)Satellite Town (C) E. Howard (d)
(d) Garden City (D) Raymond Unwin (c)
(e)Arthasastra (E) Mansara (b)

(iv)(a) Chandigarh (A) C.A. Doxiadis (d)


(b) New Delhi (B) Otto Koenigsberger (e)
(c)Gandhi Nagar (C) Mewada (c)
(d) Islamabad (D) Le Corbusier (a)
(e)Bhubaneswar (E) Edwin Lutyen (b)

(v)
(a)Munsell Atlas (A) Gothic (e)
(b) Low air speed (B) Landscape (c)
(c)Kinaesthetia (C) Colour (a)
(d) Jantar Mantar (D) Raja Jai Singh (d)
(e)Flying Buttress (E) Kata Thermometer (b)
(f) Hypostyle Hall (F) Egyptian (f)
(g) Humidity (G) Heliometer (h)
(h) Solar radiation (H) Microbar (i)
(i) Sound Pressure (I) Hygrograph (g)
j) Patina (J) Corrosion (j)

Q.3 Indicate the following statement as TRUE or FALSE.


(i) Over compaction of concrete is bad as it causes disintegration. TRUE
(ii) Zero hardness water is unsuitable for distribution because it is likely to be
corrosive. TRUE
(iii) “Mulguf” is a ventilating device used first by the Romans. FALSE
(iv) Aerobic bacteria require oxygen for their existence and they thrive in the presence
of light.TRUE
(v) Laurie Baker is famous for slum upgradation scheme in India. FALSE

Q.4 Fill in the blanks:


(a) The purpose of vibration is to expel voids and air bubbles in the concrete mass
entrapped during mixing.
(b) In a computer system printer is an external device.
(c) Floppy diskette is an external storage device.
(d) In a post-tensioned beams, the tendons are not initially embedded to the
concrete.
(e) Amoeba is an organism associated with the biological treatment of waste water.
(f) Sun light facilitates stack effect by warming the air and causing gentle convection
current.
(g) Bhilai is an example of Steel Town.
(h) New Delhi is an example of Capital Town.
(i) Notre-Dame is an example of Gothic architecture.
(j) Ratio of built up area to plot area is defined as FAR

Q.5 What do the following abbreviations stand for:


(a) INTACH : Indian National Trust for Art and Cultural Heritage
(b) N.B.O: National Building Organization
(c) E.W.S: Economically Weaker Section
(d) T.C.P.O: Town and Country Planning Organization
(e) FORTRAN: Formula Translation

Q.6 Distinguish between:

(i) Urban and Rural


(ii) Renewal and Redevelopment
(iii) Intimate scale and Monumental scale
(iv) Forum and Agora
(v) Loop street and cul-de-sac
(vi) P.C.C and R.C.C
(vii) Micro climate air and Macro climate air
(viii) Harmony and contrast in colour
(ix) Walk up and high rise apartments
(x) In-situ and Pre-cast concrete.

Q.7 Answer the following briefly:


(i) Indicate the factor that you would consider regarding the terrain for suitability of a
site.
(ii) Write the names according to hierarchy of streets and roads in an urban area.
Neighbouhood Road→ Collector/Distributor Road→ Sub Arterial Road→ Arterial
Road
(iii) Highlight the major considerations for the preparation of land sub-division plan in
an urban area.

(iv) Discuss the meaning and concept of a development plan.


Development plan prepared within the framework of the approved perspective
plan is a medium-term (5 years) comprehensive plan of spatio-economic
development of the urban centre.These plans could be
traffic and transportation plan,
tourism development plan,
environmental conservation plan,
heritage conservation plan,
mining sites reclamation plan,
coastal area development plan,
highway corridor development and such others.
(v) Outline the principle aspects for consideration in the development of settlement
planning thoughts.

(vi) What is meant by Day Light Factor? Name


the components required for estimation of Daylight Factor.
DF component = SC + ERC + IRC
SC – Sky Component
ERC – Externally Reflected Component
IRC – Internally Reflected Component

(vii) Briefly describe the importance of chlorination in water treatment.


Water chlorination is the process of adding chlorine or chlorine compounds such
as sodium hypochlorite to water. ... In particular, chlorination is used to prevent
the spread of waterborne diseases such as cholera, dysentery, and typhoid.
(viii) Why Roman dome looks hemispherical inside but saucer shaped from outside?
(ix) Outline the role of public participation in urban planning.
(x) State special architectural features associated with the following buildings which
made them different from one another:
(a) Jama Masjid, Gulbarga- No minaret, One main dome.
(b) Jama Masjid, Mandu- no minaret (three domes)

(c) Jama Masjid, Ahmedabad- originally


two minaret in the front. (fifteen domes)

Q.8 Explain the following with sketches:


(i) Ekistics grid

(ii) Schematic diagram of four different urban forms.

(iii) Difference between symmetric balance and asymmetric balance in visual

composition.

(iv) A square shape resting on its side looks vertically elongated or horizontally
elongated.
(v) Given one dimension ‘a’, draw a set of dimensions in Golden Mean Rectangle.

Q.9 (a) An urban area with a population of 2,15,000 is having housing stock of 39,000 and
average household size of 5.0. The city is expected
to have a population of 2,70,000 by 2001 with an average family size of 4.5.
Estimate the housing demand of the city by 2001 assuming there will
be depletion of existing housing stock by 3,500 during the
period.
Soln:
Given Population = 2,15,000
Housing stock = 39,000
Avg. Household size= 5.0

From above data, DU needed is = = 43,000


So there is housing backlog of 43,000- 39,000= 4000 DU’s
Year 2001
Population = 2,70,000
House hold size = 4.5

DU needed = = = 60,000 DU

Available DU in 2001 = 39,000 – 3500 = 35,500 (3500 units depleted due to obsolence)
Housing Demand in (2001) 60,000 – 35,500 = 24,500 DU

(b) Define the term “Affordable cost” in housing.


Definition: Affordable housing refers to housing units that are affordable by that
section of society whose income is
below the median household income.

(c) Distinguish between ‘home’ and ‘housing’.


A home has a more psychological impact, compared to physical. A place of
residence or refuge. It can be a house, trailer, motor home, apartment or any
place that a person is comfortable in residing. A person’s most personal
belongings are kept in a home and it is where a person feels safe and accepted.
Every house is not a home.
House is defined as a building or structure, occupied for habitation by humans.
The house can be a stand-alone structure or community with single floor or
multiple floors.

Q.10 (a) The residential landuse of an urban area accounts for 50 % of the developed land of
the city. The vacant undeveloped land is about 30 %
of the total urban area, which amounts to 2,400 hectare of land. Estimate the
quantum of land put to residential uses and also the overall density of the
urban area if the population is of 2,00,000 size.
Soln:
Residential Area = 50 %
Vacant (undeveloped) = 30 % = 2400 ha
Let total urban area of land be ‘A’ hectare so that Residentail area = 0.5A and vacant
undeveloped land = 0.3A
Now 0.3A= 2,400 hectare

Therefore A = = 8000 hectare (Total urban area)


Residential Use = 50 % of 8000 hectare= 4000 hectare

Gross Density = = = 25 person/ hectare

Net Density = = = 50 person/ hectare


(b) Illustrate with sketches the ‘Radburn principle’ of housing layout.
When Clarence Stein was commissioned in 1929 to design a Masterplan for the
Radburn estate in New Jersey he set out to build a ‘garden city for the motor
age’.
The housing layout used at Radburn was the first to create a pedestrian
circulation system that allowed people to walk to the local centre, park and the
school without the need to cross a road.

It does this by the simple expedient of super blocks 300m by 600m with a series
of cul-de-sacs pointing into the centre of each block.

These cul-de-sacs provide car access to the front of each home while a separate
pedestrian network links to the back gardens via which residents can walk
through a central area of open space to local facilities.

Q.11 (a) Draw the bending moment and shear force diagram for the following:

(b)Sketch the bending moment and shear force diagram (values not required) :

Q.12 Explain the following planting techniques:

(a) Grafting-
Graftage is a horticultural technique whereby tissues of
plants are joined so as to continue their growth together. The upper part of the
combined plant is called the scion while the lower part is called the rootstock

(b) Layering- The development of roots on a stem while the stem is still attached to the
parent plant is called layering.

Cutting- A piece of the stem or root of the source plant


is placed in a suitable medium such as moist soil. If the conditions are suitable, the plant
piece will begin to grow as a new plant independent of the parent, a process known as
striking. A stem cutting produces new roots, and a root cutting produces new stems.

(c) Transplantation- moving a plant from one location to another.

Q.13 Draw the CPM network diagram with the activities as shown below:
Sl.No Activity Preceding activity

1 A -
2 B A
3 C A
4 D C
5 E B
6 F E
7 G D

Q.14 Find the errors in the following FORTRAN program:


READ (5)A, B, C
20FORMAT (3F10.4)
D= (A+B) * C/5.0
PRINT *A,B,C
END
Q.15 Illustrate with sketches the optical correction in Architecture developed by the Greek.
GATE 1992

Q.1
(i) The delay in transfer of thermal energy from outside to inside is called
(A) Thermal conductivity (B) Insulation
(C) Radiation (D) Thermal lag.
Answer :( D) Thermal lag

(ii)Flying buttresses were used in


(A) Egyptian Architecture (B) Greek Architecture
(C) Gothic Architecture (D) None of the above
Answer: (C) Gothic Architecture

(iii) PERT Analysis is based on:


(A) Optimistic time (B) Pessimistic time
(C) Most likely time (D) All the above.
Answer: (D) All the above.

(iv) ‘Ashihara’ refers to


(A) Prayer hall
(B) Tower above main chamber of God
(C) Crown of Minaret
(D) Hall of offerings.
Answer: (B) Tower above main chamber of God (also
known as Vimana)
Wrong Question. It is ‘Shikhara’.(seems it was typing error in original GATE Question)
(This Question was corrected by Ipsita Acharya, B. Arch. CET, Bhubaneswar, M Arch SPAD)

(v) Guggenheim museum is in:


(A) New York (B) Copenhagen
(C) London (D) Philadelphia
Answer: (A) New York
Today Guggenheim museum is in New York, Venice, Bilbao and Abu Dhabi

vi) According to the National Building Code of India the minimum clear height prescribed for
mezzanine floor is :
(A) 2.2 metres (B) 2.5 metres (C) 2.8 metres (D) 3.0 metres
Answer: (B) 2.5 metres

(vii) Mottle is a defect which causes the stone to have


(A) Minute cracks containing calcium (B) Cavities filled up with sand
(C) Spotted appearance due to chalky substance (D) Cavities filled up with sand

Answer: (C) Spotted appearance due to chalky


substance

(viii) The study of spatial factors in face to face interpretation is known as


(A) Schemata (B) Personal space
(C) Proxemics (D)Territoriality
Answer: (C) Proxemics
Propogated by Edward T Hall (Part of non verbal communication)
Intimate distance for embracing, touching or whispering
Close phase – less than 6 inches (15 cm)
Far phase – 6 to 18 inches (15 to 46 cm)
Personal distance for interactions among good friends or family members
Close phase – 1.5 to 2.5 feet (46 to 76 cm)
Far phase – 2.5 to 4 feet (76 to 122 cm)
Social distance for interactions among acquaintances
Close phase – 4 to 7 feet (1.2 to 2.1 m)
Far phase – 7 to 12 feet (2.1 to 3.7 m)
Public Distance used for public speaking
Close phase – 12 to 25 feet (3.7 to 7.6 m)
Far phase – 25 feet (7.6 m) or more.
Territoriality is a term associated with nonverbal communication that refers to how
people use space (territory) to communicate ownership or occupancy of areas
and possessions.

(ix) Wedge-shaped bricks or stone blocks forming an arch are called:


(A) Voussoirs (B)Cornices (C) Springers (D)Abuttments.
Answer: (A) Voussoirs

(x) The following compound of cement contributes most to the strength of concrete:
(A) Calcium Aluminate (B) Dicalcium Silicate (C) Tricalcium Silicate (D)Gypsum
Answer: (B) Dicalcium Silicate
Tricalcium silicate gives initial strength and hardness to concrete.It is
responsible for heat of hydration.
Dicalcium Silicate gives strength after one week and contributes to maximum
strength.
Gypsum retards the setting time of concrete.

(xi) The roof type suitable for minimum wind resistance is


(A) Hemispherical (B) Flat (C) Conical with top opening (D)None of the above
Answer: (B) Flat

(xii) Optical corrections were employed predominantly in:


(A) Gothic architecture (B) Greek architecture (C) Indian
architecture (D) Islamic architecture
Answer: (B) Greek architecture

(xiii) Maximum length of cul-de-sac type of street is:


(A) 450 m (B) 220 m (C) 139 m (D) 80 m
Answer: (B) 220 m

(xiv) Find the odd one out of the following:


(A) Extended Aeration process (B) Activated sludge process
(C) Oxidation pond (D) Oxidation ditch
Answer: No appropriate answer

(xv) Dilwara temples are famous for:


(A) New method of dome construction (B) Intricate carving
(C) Wooden construction (D) Clear-storey windows
Answer: (B) Intricate carving

(xvi) The minimum area of a combined bath and w.c. as specified in the National building
Code of India is
(A) 2.5 sq.metres (B) 2.8 sq. metres (C) 3.0 sq. metres (D) 3.2 sq. metres
Answer:(B) 2.8 sq. metres (as per latest revision of NBC)
(xvii) Cohort is
(A) grafting technique (B) Groups of people aggregated by one or more
characteristics
(C) A type of shading devices (D) Places with similar soil condition
Answer: (B) Groups of people aggregated by one or more characteristics

(xviii) Reverberation inside a theatre depends on:


(A) Structural strength of the roof (B) Shape of the roof
(C) Volume of the hall (D) Heat generated in the hall
Answer: (C) Volume of the hall

(xix) In a room in hot dry climate the minimum aggregate area of openings excluding doors
will be
(A) One-fifth of the floor area (B) One-sixth of the floor area
(C) One-eighth of the floor area (D) One-tenth of the floor area
Answer: (D) One-tenth of the floor area
(soln provided by Chitra Mishra, Sunderdeep College of Architecture, Ghaziabad)
Ratio of opening of window
Hot and Dry= 1/10
Warm Humid= 1/8
Temperate and Composite= 1/8
Cold= 1/12

xx)According to the National Commission of urbanisation Report, the type of urban


development suitable for Indian condition is
(A) Low-rise low density development (B) Low-rise high density development
(C) High-rise low density development (D) High-rise high density development
Answer: (B) Low-rise high density development

Q.2 Match the following:


(i) (a) Theory and Design in the first Machine age (A) Ammos Rapoport (g)
(b)The Oregon experiment (B) Geoffrey Scott (e)
(c)The Dimaxion world (C) Richard Neutra (d)
(d) Survival through Design (D) C. P. Kukreja (i)
(e) The Architecture of Humanism (E) Nikolaus Pevsner (j)
(f) Tomorrow: A peaceful path to social reform (F) Norman Evenson (h)
(g) House- form and culture (G) Christopher Alexandre and others (b)
(h) The Indian Metropolis (H) Reiner Banham (a)
(i) Tropical Architecture (I) R. Buckminister Fuller (c)
(j) A History of Building Type (J) Ebenezer Howard (f)

(ii) (a) Lake shore Drive apartments (A) Frank Lloyd Wright (j)
(b) Sydney Opera (B) Uttam C. Jain (g)
(c)Centre of Development Studies, Trivandrum(C) Michael Graves (d)
(d) Portland Building (D) Charles Correa (h)
(e)U.S. Embassy Building, New Delhi (E) Joseph Paxton (i)
(f) Asiad Village Complex (F) Jorn Utzon (b)
(g) Jodhpur University (G) Meis van der Rohe (a)
(h) Hotel Cidad- de- Goa (H) Raj Rewal (f)
(i) Crystal Palace (I) Edward Durrel Stone (e)
(j) Falling Water (J) Laurie Baker (c)
(iii)(a) Dom-ino (A) Patrick Geddes (d)
(b) The Neighbourhood unit (B) Kevin Lynch (e)
(c)Cite Industrielle (C) Le Corbusier (a)
(d) Connurbation (D) Clarence A. Perry (b)
(e)Imageability (E) Tony Garnier (c)
(iv)(a) Letchworth (A) Hippodamus (e)
(b) Vidyadhar nagar (B) H. P Berlage (c)
(c)Amsterdam South (C) Raymond Unwin (a)
(d) Brasilia (D) B.V. Doshi (b)
(e)Miletus (E) Lucio Costa (d)

(v)(a) C.I.E. diagram (A) Ventillation (i)


(b) Globe thermometer (B) Climate Analysis (h)
(c)Natural Bed (C) Concrete strength (e)
(d) Stalactite Brackets (D) Sand stone (g)
(e)Water Cement Ratio (E) Meeting point (j)
(f) Comfort Scale (F) Qutub Minar (d)
(g) Alabaster (G) Colour (a)
(h) Mahoney Tables (H) Effective Temperaature (f)
(i) Stack Effects (I) Mean Radiant temperature (b)
j) Node (J) Marble (c)

Q.3Indicate the following statements as TRUE or FALSE.


(i) External shading devices are the most effective means of blocking out solar heat
transmission. TRUE
(ii) The Zoning ordinance provide standards for plot sizes and arrangement, utilities and
street improvements. FALSE
(iii) Floor Area Ratio is plot area divided by total covered area of all floors. FALSE
(iv) Luminous efficacy of fluorescent lamp is greater than that of incandescent lamp. TRUE
(v) A plywood board is weaker than a piece of unlaminated timber of the same size and
thickness. FALSE

Q.4 Fill in the blanks.


(a) The standard illumination level in general office is 100 lux.
(b) In land use planning land allocation between uses remains major criteria in the
decision making process.
(c) Lines of intersection of Roman cross vaults are called groin
(d) The pruning of plants into unusual shapes is known as topiary
(e) A mixture of all colour pigments produces black
(f) Cul –de sac type of street system is primarily meant to eliminate traffic in a
residential area.
(g) A ‘bit’ can take two values.
(h) Residential density indicates number of dwellings in relation to the area of land.
(i) Haldia is an example of port Town.
(j) Unit of measurement of plastering is square metre.

Q.5 What do the following abbreviations stand for?


(a)H.D.F.C. : Housing Development Financial Corporation
(b) C.I.A.M. : Congrès International d'Architecture Moderne
(c)M.I.G. : Middle Income Group
(d) N.B.C.C : National Building Construction Company
(e)C.P.U.: Central Processing Unit

Q.6 Distinguish between:

(i) Dry Rot and wet rot.


(ii) Arterial street and Collector street
(iii) Trabeated and Arcuated construction
(iv) Receding and Advancing colours
(v) Exotic plants and native plants
(vi) One-way slab and two-way slab
(vii) Absolute Humidity and Relative Humidity
(viii) Scale and Proportion
(ix) Raft foundation and pile foundation
(x) PERT and CPM.

Q.7 Answer the following briefly :


(i) The percentage articulation in an auditorium depends upon four factors. What are
these?
(ii) What do you understand by determinate structure? Give an example.
(iii) Name three most important criteria that guide the designing of an art gallery.
(iv) What are the factors affecting comfort conditions.
(v) Explain efflorescence on plastered brick wall.
(vi) Enumerate three advantages of linear type of city plan.
(vii) What is accent or focal colour?
(viii) What are basic functional zones of urban settlement.
(ix) What is meant by ‘U’ value of a construction.
(x) Name important components of a typical Hindu temple.

Q.8 Explain the following with sketches:


(i) Typical cross-section of a major street of aprox. 40 metres right of way, showing
essential features.
(ii) Block board and composite board.
(iii) How would you make a square shape resting on its side look vertically elongated.
(iv) The special features of Taj Mahal dome.
(v) Tongue and groove joint and Dove-tail joint in wood work.
Q.9 Draw sketch plan of Chandigarh city and explain Le Corbusier’s basic concepts of
town planning.
Q.10 Following information about income pattern is available from household survey of a
community.

Category Monthly Number of


family families
income in
rupees
1 Below 500 45
2 501-1000 62
3 1001-1500 213
4 1501-2000 171
5 2001-3000 76
6 above 3000 33

(i) Find out percentage of families earning a monthly income above Rs 1,500.
(ii) Draw a pi-diagram showing percentage distribution of various income groups.

Q.11 Draw the bending moment and shear force diagram for the following :
Q.12 List six factors that are considered while selecting a tree for a landscape.
Q.13 Draw the CPM network and determine the critical path from the following data:
Sl. No Activity Duration Preceding
(days) activity
1 A 4 -
2 B 10 -
3 C 6 -
4 D 6 A
5 E 8 B
6 F 3 C
7 G 7 D
8 H 2 E

Q.14 Find the errors in the following FORTRAN program:


DIMENSION I (10)
REAL M
DO 100 M = 1,10
READ (*, 200) │ (M)
100 CONTINUE
200 FORMAT (F 10.2)
STOP
END

Q.15 Draw a sketch of a typical Greek Temple façade and name the important elements.
GATE 1994
SECTION A
Q.1
(i) The visual principle of optical correction was invented and used in
(A) Byzantine Architecture (B) Islamic Architecture (C) Greco Roman
Architecture (D)Greek Architecture

Answer: (D) Greek Architecture

(ii)Interface of two ecological zones is called


(A) Ecosystem (B)Ecotone (C) Profile (D)Promontories
Answer: (B) Ecotone
(iii) Greater London Plan, 1941, was prepared by
(A) Christopher Wren (B) Ebenezer Howard
(C)Prince Charles (D)Patrick Abercrombie
Answer: (D) Patrick Abercrombie

(iv) Density control of residential area is expressed in terms of


(A) Dwelling units per floor (B) Dwelling units per plot (C) Age-sex ratio (D)
Floor area ratio
Answer: (D) Floor area ratio

(v) Write (17, *) A is a write statement with


(A) Free format (B) Open format (C) F 17.0 format (D)A 17 format
Answer:

(vi) Grey value of colour refers to


(A) Wave length (B) Reflectivity (C)Lightness (D)None of the above
Answer: (D) None of the above

(vii) Coarse textured plants are characterised by


(A) Dense small leaves with many branches and full growth habits
(B) Large leaves with massive branches and loose growth habits
(C) Sharp edged long leaves with thin branches and restricted growth habits
(D) Uncontrolled growth with sparsely developed leaves and trunk
Answer: (C) Sharp edged long leaves with thin branches and restricted growth habits

(viii) In a two dimensional composition spatial depth can be created by the principle of
(A) Balance (B) Rhythm (C) Perspective (D) Harmony
Answer: (C) Perspective

(ix) Settlement with ‘mile high’ structure was conceived by


(A) Le Corbusier (B) Antonio Gaudi (C) Frank Lloyd Wright (D) Kevin Lynch
Answer: (C) Frank Lloyd Wright

(x) The permissible height of a building on a plot is determined by


(A) Density of the area and floor space index (B) Uncovered rear space and ground
coverage
(C) Abutting road width and floor area ratio (D) None of the above
Answer: (C) Abutting road width and floor area ratio

(xi) AUTOEXE . BAT is a


(A) Data file(B) ASCII text file (C) Autocad drawing file (D)Write protected file
Answer: (B) ASCII text file

(xii) Photo chemical reaction in the atmosphere begins with


(A) the act of absorption of radiation (B) the formation of smog
(C) the depletion of ozone layer (D) None of the above
Answer: No appropriate answer

(xiii) Trickling filter is used for

(A) Treatment of drinking water (B) Treatment of waste


water
(C) Oxidation of water (D) Air conditioning plant
Answer: (B) Treatment of waste water

(xiv) Complementary colour of violet is


(A) Red (B) Yellow (C) Black (D) Green
Answer: (B) Yellow

(xv) The Radburn pattern of neighbourhood layout was conceived by


(A) Doxiadis (B) Clarence Stein (C) Clarence Perry (D)Soriya Y. Mata
Answer: (B) Clarence Stein

(xvi) UCS in Autocad


(A) Can not be altered (B) Can only be rotated (C) Can only be
translated (D) Can be rotated and translated
Answer: (D) Can be rotated and translated

(xvii) The principle of ‘Conservative Surgery’ was suggested by


(A) Oscar Neimeyer (B) Patrick Geddes (C) Charles Abraham (D)Lewis Mumford
Answer: (B)Patrick Geddes

viii) Plants which can block wind and view throughout the year when branching very near the
ground are known as
(A)Decidous(B) Evergreen Conifers (C) Broad leaf evergreen (D) Mixed evergreen
deciduous
Answer: (C)Broad leaf evergreen

(xix) The Tien An Mein Square in Peking is an example of


(A)Ornamental park (B) Shopping Centre (C) Large Plaza (D) Exhibition ground
Answer: (B) Large Plaza

(xx) The National Commission on Urbanisation was chaired by


(A) Rajiv Gandhi (B) Charles Correa (C) B. V. Doshi (D) A. P. Kanvinde
Answer: (B) Charles Correa

Q.2 What do the following abbreviation stand for


(a) BTU: British Thermal Unit
(b) HVAC: Heating Ventillation and Air Conditioning
(c) SFS: Self Financing Scheme
(d) COPP:
(e) ASCORAL: Assemblee de COnstructeurs pour une Renovation Architecturale
(f) NCR: National Capital Region
(g) RAM: Random Access Memory
(h) BPS : Business Process Services
(i) BOPE :
(j) NGO: Non Government Organization

Q.3 Match the following


(i) (a) Mud Pushta (A) DOS (f)
(b) Ballast (B) Relationship (h)
(c)Fineness (C) Colour Separator (j)
(d) Wblock (D) Water proofing (a)
(e) Copy (E) Order (g)
(f) Print (F) Accentuation (i)
(g) Proportion (G) Fluorescent (b)
(h) Schemata (H) Aggregate (c
(i) Vista (I) Autocad (d)
(j) Newton (J) BASIC (e)
(ii) (a)Laurie Baker (A) Site Planning (d)
(b) Christopher Alexander (B) World cities (e)
(c)Eugen P. Odum (C) New Bombay (j)
(d) Kevin Lynch (D) Sector Theory (h)
(e)Peter Hall (E) Small is Beautiful (g)
(f) Lewis Mumford (F) Arcology (i)
(g) E.S. Schumacher (G) Architect of the poor (a)
(h) Homer Hoytt (H) A city is not a Tree (b)
(i) Soleri Paolo (I) Fundamentals of Ecology (c)
(j) Charles Correa (J) The city in History (f)

Q.4 Fill in the blanks:


(i) Physiological comfort is a major ………………….issue in planting design.
(ii) ‘Dwelling’ 67 Montreal’ is an example of Housing complex
(iii) The first garden city Letchworth is 35 km away from London.
(iv) OPEN (15, file…….’C 10 OUT’)
(v) Two parallel lines in perspective meet at vanishing point.
(vi) Chlorine is added to water for removal of pathogens.
(vii) Exhaust Fan are provided to remove foul air inside the toilet.
(viii) Asymmetrical space can be made to feel more dynamic when pure element is placed
symmetrical
in the space.
(ix) Brasilia is a variant of ……………………………. urban form.
(x) A complete periodic enumeration of total population in a given country made at a
specific point of
time is known as Census

Q.5 Indicate the following as TRUE or FALSE


(i) Unity in composition can also be increased by proximity. TRUE
(ii) Perception of size of an object is independent of its relation to human size.
FALSE
(iii) The Milletus demonstrates Hippodomian plan. TRUE
(iv) Agora was locating the Royal Palace. FALSE
(v) Harappa is situated by the side of river Sind. FALSE
(vi) The Central city with satellite towns was originated by Raymond Unwin.TRUE
(vii) The Aranya Housing Project was designed by B.V. Doshi. TRUE
(viii) The intensity of commercial development is expressed in terms of F.A.R. FALSE
(ix) Sodium vapour lamps are suitable for industrial lighting. FALSE
(x) In an auditorium if the near ground temperature is higher than the upper level the
sound from source will deflect upwards. TRUE
(xi) FORTRAN is a machine language. FALSE
(xii) Hogarth’s Line of Beauty represents variety in unity. TRUE
(xiii) DOS is an operating system software. TRUE
(xiv) Landscape is to be seen as a physical entity only and not to be translated into
economic unit.FALSE
(xv) Human scale refers to space with a maximum horizontal distance of 72 feet.
TRUE
.6 Write a computer program for drawing a specific line and a specific circle on the terminal
screen in BASIC.
.7 In a square frame of 6 cm x 6 cm make balanced composition using two straight lines in
each square so that in first a point at the centre is emphasised, in second a point near
left upper corner is emphasised and in the third a point outside the point is
emphasised.
.8 Discuss with sketch the principles of a residence sector plan of a new town.
.9 Explain briefly the relation of climatic elements to comfort.
.10 Plant form is a combination of overall plant shape and habit of growth. Each shape has
its own unique characteristics and design potentials- Sketch the different types of plant
forms stating their design potentials.

ECTION B (PART-I)
. 11 Critically examine the architectural works developed by Laurie Baker and its influence in
modern architecture in India.
.12 Explain with sketches the working of Aqua Privy.
.13 Design a cantilevered R.C. roof slab to carry a live load of 1.5 KN/m². The overhang of
the slab is 1.2 m. Use M 15 grade concrete Fe 415 grade steel. (Given R= 0.658
N/mm²; j= 0.9; Area of 8 TOR bars = 0.5 cm²).
.14 The average completion time of the following construction activities are given below. As
a construction manager for which activities you would like to maintain very strict
schedule? Draw the network to justify your decision.

Activity Average
completion time
(in weeks)
1-2 11
1-3 14
2-4 6
2-5 16
3-4 7
4-5 3
Q.15 Mention briefly the salient architectural concepts and features of Khajuraho group of
temples.
Q.16 Find out the quantity of each types of materials required for a 12 cm thick R.C.C. (1:2:4)
roof slab 3.5m x 5.0 m with 0.8% steel (wt of steel = 7850kg/m³; 1 cum 1: 2: 4) concrete
requires 6.2 bag of cement, 434 lit sand and 868 lit of stone aggregate).
Q.17 Define Glare and indicate the causes of glare with it remedy.
Q.18 Determine the eccentric load per metre which a 250mm thick work section can carry.
The eccentricity of
load is 3cm from the centre line of the wall. The allowable compressive stress for brick
work (1:4 mortar) is 2000KN/m2; slenderness ratio for the wall is 10 for which the
allowable stress is to be reduced by 40%.
Q.19 Sketch two alternative courses of brick layers showing ‘English Brick bond’ for a
Tee junction of a half brick wall with a one brick thick wall.
Q.20 Outline briefly the architectural features developed during Islamic period in India.
Q.21 ‘Hall of Nation’constructed at the Permanent Exhibition complex (Pragati Maidan), New
Delhi as the forum for the demonstration of India’s industrial and technological potential-
critically evaluate the structure from the point of contemporary Indian architectural
development.
Q.22 Out of several water softening processes for hard water which process produces zero
hardness. Explain the reasons.
Q.23 State the advantages of pre stressed concrete.
Q.24 Sketch the details of ribbed slabs with precast panels and discuss the method of
construction of such a slab.
Q.25 Explain with sketches the method of forming pendentive for dome construction over a
square plan. Give example of a famous structure where it was adopted.
Q.26 Elaborate the considerations you will make in designing a toilet for male students hostel.
Sketch one such toilet having 2 W.C.’s, 3 baths, 4 urinals, 3 W.H. Basins and 1 Janitor’s
closet.
Q.27 Distinguish with help of sketches the construction of dome and supporting system for St.
Paul’s Cathedral, London and St.Peter Rome.
Q.28 Outline with sketches the principles in simplest form the different stages involved in
summer and winter air conditioning.
Q.29 Compare with sketches the proportion of Greek and Roman Doric order and label the
parts.
Q.30 Sketch the longitudinal section through a water closet with S-trap sowing fittings, fixtures
and connections to soil pipe leading to septic tank.

ECTION B (PART-I)
.11 Stating the concept of Primate City, outline how the primacy index could be computed.
.12 The distance measured between two identifiable location on a topographic map (scale 1:
50000) is 52 mm. The distance between the same two locations in an aerial
photograph is 260 mm. The aerial photograph just covers a settlement within its 200
mm x 200 mm size. 45 % of the area of the photograph is under residential use. Find
(a) the scale of the aerial photograph
(b) the total area of the settlement
(c) the area under residential use
.13 Outline the considerations for design and layout of street lighting system in an urban
area.
.14 Write ten major functions of urban government as per the Twelfth Schedule of the
Constitution (Seventy Fourth Amendment Act 1992).
.15 Calculate the time mean speed and space mean speed from Vehicle Spot
the following observed data of spot speeds of the No speed
vehicles. 1 10 m/sec
Soln: 2 15 m/sec
3 12 m/sec
Time mean speed =
4 11 m/sec
= 5 18 m/sec
= 13.2 m/s
Space mean speed = =

= = = 13.15 m/s
.16 Elaborate on the concept of “Spill- over Effect.”
.17 In a housing area of 18.7 Hectares, 1000 residential plots are to be provided. It is
contemplated that 50 % of the plots will be A-type. 30 % B-type and 20 % C-type. The
B-type plots will be 20 % larger in size than A- type plots and C type plots will be 30 %
larger than A-type plots. 15 % of the residential area will be under roads and 25 % area
will be used for open space and community facilities. Find the maximum area for each
type of plots.
Soln:
Total no of Plots = 1000 Let the area of Type ‘A’ be ‘a’ units, then type ‘B’ = ‘1.2a’ and C=
‘1.3a’
Type ‘A’= 500 (a)
Type ‘B’= 300 (1.2a)
Type ‘C’= 200 (1.3a)
Out of 18.7 ha, 15% is under road and 25% under open space/community facilities, Rest
60% is Residential

Residentail area = × 18.7= 11.22 ha


Equating 500a + 360 a + 260a = 11.22 × 10,000 sqm
1120a= 112200 sqm
a = 100 sqm
Type ‘A’= 100 sqm
Type ‘B’= 120 sqm
Type ‘C’= 130 sqm

.18 Find out the quantity of run-off for 70 cm rainfall for an area of 100 Hectares out of which
55 % is residential area, 20 % is under roads and rest of the area is under open space
with 30 % paved area. The co-efficient of run off for residential area is 0.6, for roads
0.9, for open space is 0.5 and for paved area is 0.7.
Answer: 4,55,000 cubic m
Total area= 100 ha
Residential (55 %) = 55 ha
Road (20%) = 20 ha
Open space= 25 ha out of which 30% is paved

Paved area × 25= 7.5 ha (Paved)


Unpaved = 17.5 ha
Total Runoff= (area in ‘m’ × rainfall in ‘m’ × ruoff coefficient)
From Residential = 5,50,000 × 0.7 × 0.6 = 2,31,000 cum
From Road = 2,00,000 × 0.7 × 0.9 = 1,26,000 cum
From paved area = 75,000 × 0.7 × 0.7 = 36,750 cum
From unpaved area = 1,75,000 × 0.7 × 0.5 = 61,250 cum
Total Runoff= 4,55,000 cubic m
.19 Sketch a typical diversion curve based on travel time ratio and discuss the use of such a
curve for trip assignment.
.20 Suggest the area of urban functions where ‘Users charges’ could be used most
effectively.
Q.21 Outline the Public Interest as Determinants of urban landuse.
.22 What data and information you will collect if you are required to decide the price per unit
floor area of a proposed shopping complex in an existing residential area.
Q.23 Explain with sketches the different systems of laying water distribution lines for a city.
.24 Sketch with dimension the cross section of urban arterial road with 4- lanes, a median of
1.5 m width and footbath on either side. The pedestrian volume during morning and
evening peak hours is 13500 pedestrians/hr in the major direction of flow. The design
pedestrian flow is 75 pedestrian per minute per metre.
Q.25 Indicate the administrative measures to be adopted under the Land Acquisition Act,
1894. (Steps only)
Q.26 Suggests the indicator of a settlement pattern Study for a Region.
Q.27 Explain with examples how demographic data on age sex structure of population can be
used for estimation of primary school students, secondary school students and labour
force.
Q.28 Mention the information to be collected for the design of sewerage system for an urban
area
Q.29 Discuss the advantages of one way street system in an urban area.
Q.30 Highlight on ‘Plot Reconstitution Scheme’ as a successful legal tool.
GATE 1995
Q.1
(i) Rhythmic pattern is created by
(A) Repeating a set of elements in a space at regular intervals
(B) Repeating a single element in linear fashion only
(C) Symmetrically arranging elements along an axis
(D) None of the above.
Answer: (A) Repeating a set of elements in a space at regular intervals
Repetition refers to one object or shape repeated;
Pattern is a combination of elements or shapes repeated in a recurring and regular
arrangement;
Rhythm--is a combination of elements repeated, but with variations.

(ii) Of the following type of landscape , which one produces heat- island
(A) Islands (B) Coastal areas (C) Urban areas (D) Mountain ranges.
Answer: (C) Urban areas
(iii) Combination of Red and Green in a colour scheme is
(A) Simultaneous contrast (B) Complementary harmony
(C) Split complimentary (D) Analogous
Answer: (B) Complementary harmony
Composite is opposite colour in colour wheel.

(iv) The maximum distance for recognizing faces is


(A) 15 meters (B) 40 meters (C) 25 meters (D) 50 meters
Answer: (C) 25 meters

(v) The optimum reverberation time for public lecture halls should not exceed
(A) 1.0 sec (B) 1.5 sec (C) 2.0 sec (D) 2.5 sec
Answer: (A) 1.0 sec
Around 2 seconds is desirable for a medium-sized, general purpose
auditorium that is to be used for both speech and music. A classroom should
be much shorter, less than a second. And a recording studio should minimize
reverberation time in most cases for clarity of recording.
The maximum level of background noise allowed in the classroom is 35
decibels (dBA).
Maximum acceptable noise level as per CPCB norms in India.
Day time Night time
Industrial area 75 70
Commercial area 65 55
Residential area 55 45
Silence Zone 50 40
Silence zone is referred as areas upto 100 meters around such premises as
hospitals,educational institutions and courts.

(vi) Higher the elevation of a town above the sea level, larger the dose human population is
exposed to
(A) Nuclear radiation (B) Cosmic rays (C) Electromagnetic waves (D) All the above.
Answer: (B) Cosmic rays

(vii) A rectangular area specified in world co-ordinates is called


(A) Window (B) Door (C) Ventilator (D) Mirror
Answer: (A) Window

(viii) The term Conurbation was initiated by


(A) Lewis Mumford (B) Stuart Chapin
(C) Patric Geddes (D) Charles Correa
Answer: (C) Patric Geddes
an extended urban area, typically consisting of several towns merging with the
suburbs of a central city.

(ix) Permissible building coverage is expressed in terms of


(A) People per hectare (B) Dwelling units per hectare
(C) Ground coverage (D) None of the above.
Answer: (C) Ground coverage
Residential Density= Dwelling units per hectare

(x) F.A.R. of a an area varies with respect to


(A) Size of the plot (B) Abutting road width and use (C) Location (D) Height of the
building
Answer: No appropriate Answer.
(A), (B), (C) all are correct

(xi) WBLOCK command in autocad creates


(A) Wooden block (B) Drawing file (C) ASCII file (D) Data file
Answer: (B) Drawing file
(xii) House near Luxor, Egypt was designed by-
(A) Robert Venturi (B) Hassan Fathy (C) I.M. Pie (D) Lucio Costa
Answer: (B) Hassan Fathy, an Egyptian architect.

(xiii) Self-help housing concept was propagated by


(A) Charles Abraham (B) UNCHS (C) Laurie Baker (D) Burger R. Antipode
Answer: (A) Charles Abraham
"Man's Struggle for Shelter in an Urbanizing World" by Charles Abram's.

(xiv) Broad acre city was proposed by


(A) Ebenezar Howard (B) Raymond Unwin (C) Frank Lloyd Wright (D) C. A.
Doxiadis
Answer: (C) Frank Lloyd Wright

(xv) Desirable spacing of trees along a highway is


(A) 3m to 7 m (B) 9m to 15m (C) 20m to 25m (D) 26m to 30m
Answer: (B) 9m to 15m
Across the highway it should be planted 10-12 m away from the centre line of
extreme traffic lane.

Q.2 What do the following abbreviations stand for


(a) U C S: Universal Coordiate System
(b) U N C H S: United Nations centre for Human Settlement
(c) G L: Ground Level
(d) L O T : Line of travel
(e) B O D: Biological Oxygen Demand

Q.3 Match the following


(i) (a) String (A) Estuaries (c)
(b) Pyrocell (B) Hygrometer (e)
(c)Mangroves (C) Scanner (d)
(d) Raster (D) Character sequence (a)
(e)Humidity (E) Fire resistance (b)

(ii)(a)Colour scale (A) Renaissance (i)


(b) Plan Libre (B) Greek Period (h)
(c)Dyer D.J. (C) Town Design (f)
(d) Calcutta (D) Concentric zone theory (g)
(e)Doxiadis (E) Linear City (d)
(f) Fredrick Gibbard (F) People and housing in third World (c)
(g) E. W. Burgess (G) Ekistics (e)
(h) Miletus (H) Future of Metropolis (j)
(i) Karlsruhu (I) Le Corbusier (b)
(j) Lloyd Rodwin (J) Munsell (a)

Q.4 Fill in the blanks:


(i) Renaissance period in India synchronises with Mughal Rule.
(ii) A public record survey or map showing details of value, extent and ownership of land
is known as Cadastral Map
(iii) Laurie baker is the receipent of Habitat Award 1987 and Padamshree in 1990.
(iv) Aerobic bacteria require oxygen for their existence.
(v) Mansara is the elaborate treatis on building and town planning in ancient India.
(vi) Incineration involves the burning of refuse in the incinerator.
(vii) Glare increases the apparent size of the object
(viii) Visual survey involves examination of the form, appearance and texture of built form.
(ix) A straight line movement of an object from one position to another on the display
screen is known as cursor of the object.
(x) A form of pruning in which plants are severely sheared into unnatural shapes such as
animals or chess pieces is called topiary

Q.5 Indicate the following as TRUE or FALSE:


(i) The saturation scale of any colour can be achieved by mixing white or black to that
colour. FALSE
(ii) If the pollutants are dispersed in a large volume of air the resultant pollution increases.
FALSE
(iii) Transmission loss of sound varies directly with the frequency of sound. TRUE
(iv) The minimum height permitted for habitable room is 2.1 m. FALSE
(v) Timgad is an example of Roman Town. TRUE
(vi) “The Future of Housing” was written by Charles Abraham. TRUE
(vii) Arthashastra was written during Vedic Period.FALSE
(viii) Shrubs cannot reduce the velocity of wind. FALSE
(ix) END CIRCLE, FOR is a valid FORTRAN statement
(x) The collective term for the foliage of a tree is canopy. FALSE

Q.6 Distinguish between:


(i) Freehold and Leasehold property
(ii) Betterment Levy and Development Levy
(iii) Core House and Shell House
(iv) Dandaka and Padmaka
(v) Year’s purchase in perpetuity and Sinking fund
(vi) Value and Hue of colour
(vii) Full enclosure and Loss of enclosure
(viii) Mouse and Digitizer
(ix) Coagulant and Flocculent
(x) Herbaceous and Herbicide

SUB SECTION A2
Q.7 Write a program in FORTRAN or BASIC for computing the volume of a sphere for
variable radius.
Q.8 In a square frame of 10 cm x 10 cm make a composition of figure and ground
(positive and negative) indicating expansion of figure on ground and ground on
figure.
Q.9 Explain “Venturi Effect” of air flow inside a building.
Q.10 Proposed land use distribution for 10 hectare housing complex is 65 %, 10%,
12.5 %, for residential, public-semi public, open space and roads respectively.
Costs of land acquisition, site development and construction are Rs 1200/ m²,
Rs 100/ m² and Rs 4000/ m² respectively. Permissible FAR being 1.5, find out
the cost of a dwelling unit of 85 m².
Q.11 Illustrate with sketches highlighting the major features of any planned Indian city.

SECTION B (PART I)
Q.12 Discuss briefly the important contribution of reinforced cement concrete to
modern Architecture.
Q.13 Greek Architecture is sometimes called “A Carpentry in Marble” – Justify.
Q.14 Describe the fire proofing treatment of steel columns in public buildings.
Q.15 Determine the allowable tensile force P that may be applied across two plates
connected to one another by fillet weld joints as shown in the Fig- B-I
15.Allowable working stress of weld is 75 MPa. Consider only shearing stress in
the weld. The load is applied midway between the two welds.

Q.16 The optimistic (t0) , most likely (tm) and permissible (tp ) times of activities on the
critical path of a PERT network are given below. Calculate the mean and
standard deviation of the critical path duration.
Activities on to tm tp
the
Critical path
A 5 10 15
B 8 16 24

Q.17 Explain the important Architectural Design features of any ONE of the following:
(i) Belgian Embassy
(ii)Bahai Temple
(iii)CMC building- Bombay
Q.18 Explain with illustration the construction of dome over a square plan as
developed during Indo-Islamic phase.
Q.19 Explain sound-focii and Dead Spot in an auditorium.
Q.20 Sketch a wall footing for a 250 mm wall of a two-storied residential building on a
ground having a safe bearing capacity of 10 tons/m² at a depth of 1 m below the
surface. The load from the wall at the ground level is 7 tons/m length of the wall.
Q.21 Sketch the connection of the house water supply pipe with the municipal water
main showing the position of Ferrule, water meter, goose neck and stop cock.
Q.22 Outline briefly the Architectural features of Gandhi Smarak Sangrahalay,
Sabarmati, and Ahmadabad.
Q.23 State the factors considered by the Buddhists while selecting the monastic sites.
Q.24 State the advantages of Fibre Reinforced Plastic (FRP) as structural materials
and mention some areas of their use in building.
Q.25 Draw elevation and details of an Aluminium window 2 m long and 1.5m high with
four vertical divisions out of which the middle two should consist of side-hung
openable shutter of 1.0 m height.
Q.26 Sketch a semi circular brick arch over an opening of 1.5 m and label all its
components.
Q.27 Design a dining hall with sitting capacity of 30 for a guest house showing the
furniture layout, relationship to the kitchen and washing area for the diners.
Q.28 Explain with sketches the characteristic features of a Mughal Garden.
Q.29 Discuss in brief the process of preparation, properties and the uses of plywood.
Q.30 Design illumination for a 6m x 4m computer lab so as to achieve good working
environment. The ceiling height is 3 m and the false ceiling is at 2.4 m. Draw an
inverted ceiling plan and a section of the room.
Q.31 Sketch two consecutive layers of right- angle corner of a 25 cm thick brick wall in
English bond.

SECTION B (PART II)


Q.12 Indicate the functions of a structure Plan.
Q.13 The passenger demand during the two hours morning peak period along a bus
route is 4000 passengers. The round trip time along the route is 50 minutes and
the average vehicle occupancy is 75 passengers. Calculate the hourly flow of
buses and the number of buses required to provide this flow assuming 5 % of
the buses will be under service and repair.
Q.14 Explain how the population projection can be estimated for any settlement by
Employment method.
Q.15 Explain in brief the composting method of refuse disposal.
Q.16 Prepare a flow chart for residential site planning by adopting modern
management technique.
Q.17 Briefly describe the indicators of Physical Planning.
Q.18 Sketch the common types of on- street parking. Discuss their advantages and
disadvantages.
Q.19 Describe the survey to be conducted for developing a shopping centre of an
urban area.
Q.20 A city with a population of 1,00,000 discharges sewage of 120 lpcd in a stream
having a flow of 1 cu.m/sec. The BOD of sewage is 220 mg/lit. The BOD
content in the upstream before the outfall is 0.8 mg/lit. Calculate the BOD
content of the stream in mg/lit just down stream of outfall.
Q.21 State the objectives and ceiling limits under Urban Land (Ceiling and Regulation)
Act, 1976.
Q.22 Outline the process involved in Multi- Level planning.
Q.23 Determine the angle at which a pavement should be banked in order to avoid
outward sliding of vehicles along a horizontal circular curve of radius R = 300 m.
The maximum allowable speed limit along the curve is 100 km/hour. The
coefficient of side friction is 0.2.
Q.24 Explain the stratified sampling and the cluster sampling.
Q.25 Explain how Mass Curve can be used to determine the capacity of a balancing
water tank.
Q.26 Indicate the Obligatory Functions of an Urban Local Body.
Q.27 Highlight the concept of systems Approach in planning and comment on its merits
and demerits.
Q.28 Discuss the form of the Gravity Model commonly used for estimating trip
distributing in transportation planning. Why is it necessary to calibrate such a
model?
Q.29 Aerial photograph on scale 1:25000 was taken with an aerial camera lens of 15
cm focal length, Calculate the flying height above mean ground level of 1200 m.
Find the flying height of the aircraft above mean sea level.
Q.30 Explain break point chlorination.
Q.31 Comment on the PPBS system and its applicability in Indian context.
GATE 1996
Q.1.1‘New Town Intown’ concept involves
(A) Increasing the population density in the city centre
(B) Provision of large scale recreational facilities in city core
(C) Encouraging strip commercial within the city core
(D) Large scale rehabilitation, modernization and redevelopment of the city core.
Answer: (D) Large scale rehabilitation, modernization and redevelopment of the city core.

Q.1.2An ‘Aquiclude’ is
(A) Confined bed of impervious material between acquifers
(B) Perched acquifer
(C) Artesian acquifer
(D) Large water body underground
Answer: (A) Confined bed of impervious material between acquifers
An aquitard may store water and may transmit appreciable water to and from
adjacent aquifers.
A completely impermeable aquitard is called an aquiclude or aquifuge.

Q.1.3If ‘a’ is the optimistic time, ‘b’ is the pessimistic time and ‘m’ is the most likely time of
an activity, the expected time of the activity is

(A) (B) (C) (D)


Answer: (B)

Q.1.4The most commonly used base for timber painting is


(A) Red lead (B) White lead (C) Titanium white (D) Zinc oxide
Answer: (B) White lead

Q.1.5Absorption of sound in porus material occurs mainly due to


(A) Creep loss (B) Vacuum in holes (C)Frictional loss (D)None of the above
Answer: (C) Frictional loss
When sound from a loudspeaker collides with the walls of a room part of the sound's
energy is reflected and part is absorbed into the walls. As the waves travel through the
wall they deform the material thereof (just like they deformed the air before). This
deformation has mechanical losses which convert part of the sound energy into heat
through acoustic attenuation, mostly due to the wall's viscosity. The same attenuating
mechanics apply for the air and any other medium through which sound travels.
Width of formation for various classes of roads
Q.1.6In sewers, velocity of flow should not
be
(A) More than self cleansing velocity Road Roadway width in m
(B) Less than velocity of water at
classification Plain and Mountainous and
flushing
(C) Less than dry water flow velocity rolling terrain steep terrain
(D) Less than self cleansing velocity NH/SH 12 6.25-8.8
Answer: (D) Less than self cleansing MDR 9 4.75
velocity
ODR 7.5-9.0 4.75
Q.1.7As per Indian Road Congress the VR 7.5 4.0
‘Width of formation of highway in plain
land’ is
(A) 6.0 m (B) 12.0 m (C) 18.0 m (D) 21.0 m
Answer: (B) 12.0 m
Width of formation or roadway width is the sum of the widths of pavements or
carriage way including separators and shoulders. This does not include the extra
land in formation/cutting.
The width of road or width of carriage way for single lane road is 3.75, for
intermediate lane road 5.5 m, two lanes without raised kerb is 7 m, two lane with
raised kerb is 7.5 m and for multi-lane road width per lane is 3.5 m.

Q.1.8 In a lecture auditorium the seating pattern from the speaker should fall within
maximum angle of
(A)70º (B)90º (C)120º (D)140º
Answer: (D) 140º
Q.1.9When two pigments of contrasting value are mixed, the most striking change observed
is in
(A) Hue Dimension (B) Occult rhythm (C)Occult Balance (D)None of the above
Answer:

Q.1.10 In meandering river the location of intake point for city water supply should be
(A) On concave bank (B) On convex bank
(C) Down stream point of waste water disposal (D) Middle of the river
Answer: (A) On concave bank
Location of intake.
As far as possible the Intake should be near the treatment plant.
The intake must be located in the purer zone of the source.
The Intake must never be located at the downstream or in the vicinity of a point
of disposal of waste water.
The intake should never be located near navigation channels
The intake site should be such that it will permit grater withdrawals of water, if
required in the future.
The intake must be- located at a place where it can draw water even during
driest periods
The intake site should be easily accessible even during floods and should not
get flooded.
In meandering rivers, the intake should not be located on curves as much as
possible. Even if they have to be located on curves It should be located on the
concave banks.

Q.1.11 Number of common clay bricks required to make one cubic meter of brick masonry
is
(A) 350 (B) 420 (C) 500 (D) 550
Answer: (C) 500

No of bricks = = 500 bricks.


For India, a brick of standard size 190 mm x 90 mm x 90 mm is recommended by the BIS.
With mortar thickness, the size of such a brick becomes 200 mm x 100 mm x 100 mm
and it is known as the nominal size of the modular brick. Thus the nominal size of brick
includes the mortar thickness.

Q.1.12 Sinking fund refers to


(A) Reserve fund (B)Fund loss due to damage (C)Bad debts (D)Fund for
underground construction
Answer: (A) Reserve fund
Sinking fund is an account that is used to deposit and save money to replace an
asset in the future. Some amount of money is earmarked annually so that a fund is
available to reerect the building after the life of building is over.
Bad debt- a loan that cannot be recovered by the bank or any lending agency.

Q.1.13 BASIC is a
(A) Compiler (B) Hardware item (C) Interpreter (D) Plotter type
Answer: (A) Compiler

Q.1.14 Acidity of soil is indicated by


(A) BOD (B) pH (C) GL (D) CLO
Answer: (B) pH

Q.1.15 The unit measurement for space in urban society is


(A) Family (B) Group of family (C) Community (D) Individual
Answer: (A) Family

Q.1.16 DIM in AutoCAD refers to


(A) Dimension of array (B)Dimension of function(C)Brightness of visual
screen (D)None of the above
Answer: (B) Dimension of function

Q.1.17 Sense of movement in a visual composition can be achieved by


(A) Disturbing the visual balance (B) Placing elements in diagonal arrangement
(C) Placing elements symmetrically (D) Overlapping elements
Answer: (B) Placing elements in diagonal arrangement

Movement- How the artist leads the viewer’s eye


around the page

Q.1.18 Minimum strength of cement mortar used in load bearing brick masonry is
(A) 50 N/cm² (B) 60 N/cm² (C) 80 N/cm² (D) 100 N/cm²
Answer: (D) 100 N/cm²
Min Compressive Strength (IS Code)
S. Item Min
No Compressive
Strength
(N/mm²)
1 Mortar for brick 1 N/mm²
Masonry
2 Brick 3.5 N/mm²
3 Plain Cement 15 N/mm²
Concrete (PCC)
4 RCC 20 N/mm²
Mortar (cement : sand ratio must not be less than 1:3 for any type of work)
1:3 – for repair work
1:4 – for external plaster / ceiling plaster
1:5 – for brick work/ internal plaster
1:6 – for plaster work when fine sand is available

Q.1.19 If ‘P’ denotes the total population, the age dependency ratio is expressed as

(A) (B) (C) (D)

Answer: (D)
Age dependency ratio (% of working-age population) in India was last measured at
52.45 in 2015, according to the World Bank. Age dependency ratio is the
ratio of dependents--people younger than 15 or older than 64--to the working-age
population--those ages 15-64

Q.1.20 Concept of self supporting ‘Industrial Town’ was proposed by


(A) Lewis Mumford (B) Henry Wright (C) Robert Owen (D) Ebenezer Howard
Answer: (C) Robert Owen

Q.1.21 Of the following types of forests, which one is least likely to be destroyed by fire
(A) Deciduous forest (B) Coniferous forest (C) Rain forest (D) Broad leaf evergreen
forest.
Answer: (C) Rain forest
Deciduous forest- loose the leaves seasonally
Coniferous forest- vegetation composed primarily of cone-bearing needle-leaf
Tropical rain forests grow around the equator in South America, Africa, and
Southeast Asia. They have the highest species diversity per area in the world,
containing millions of different species. The temperature is stable year-round, around
27°C. Most tropical forests receive at least 200 cm of rain in a year.
Evergreen forest A forest in which there is no complete, seasonal loss of leaves (i.e.
trees shed old leaves and produce new ones partially, and sometimes throughout the
year, rather than during particular periods)

Q.1.22 Workability of concrete mix with low water-cement ratio is determined by

(A) Slump test (B) Tensile strength test


(C) Flexural strength test (D) Compaction factor test
Answer: (A) Slump test
Slump test- Workability
Tensile Strength test- for steel
Flexural Strength- adjoining figure is an example of Flexural test.
Q.1.23 In hot dry climate the form and planning of settlement should be
(A) High rise high density (B) Low rise high density
(C) Low rise low density (D) High rise low density.
Answer: (C) Low rise low density
Characteristic of Urban Form as suggested by National Commission on Urbanization.(set
up in 1986)- this commission was headed by Charles Correa.
Urban Form
Low rise, High density development should be the predominant built form in
urban India.
Municipal regulations regarding minimum plot sizes, buildable plot area, etc should be
amended, building envelopes designed and building codes modified so that the
desired built form is achieved.
Controlled streetscapes should be achieved through mandatory building lines and
developing appropriate building envelops.
Public squares, parks, promenades and other nodal points or urban centers should be
rehabilitated by restricting vehicle entry and ensuring controlled development. Civic
land marks should be treated at urban events which are identity to a city
neighborhood and enhance civic pride.

Land allocation must be consistently


monitored and readjusted to ensure equitable city growth.

Q.1.24 The extent of wind shadow on the leeward side is more dependent on
(A) Height of the building (B) Depth of building
(C)Width of building (D) All of the above
Answer: (D) All of the above

Q.1.25 ‘Letchworth’ was designed based on the concept of


(A) New town (B) Garden City (C) Factory town (D) Linear city
Answer: (B) Garden City
New Town: The new town movement refers to towns that were built after World War II
(1939-45) and that have been purposefully planned.

Q.1.26 ‘Jharokha’ is an architectural element used in building as


(A) Gateway (B) Balcony (C) Column decoration (D)Ceiling decoration
Answer: (B) Balcony

Q.1.27 The concept ‘Architecture as an expression of inner structure’ is attributed to


(A) Alvar Alto (B) Mies van der Rohe(C) Walter Gropius (D) Le Corbusier
Answer: (B) Mies van der Rohe

Q.1.28 Psychrometric scale deals with


(A) Subjective weighting (B) Frequency of sound (C) Thermal comfort (D) Colour
intensity
Answer: (C) Thermal comfort

Q.1.29 The concept of ‘La Ciudad Linear’ is associated with


(A) Le Corbusier (B) Tony Garnier (C)S. Y. Mata (D)Robert Owen
Answer: (C) S. Y. Mata

Q.1.30 The term ‘Megalopolis’ was coined by


(A) Lewis Mumford (B) Patrick Geddes (C) Norman Cousins (D)Jean Gottman
Answer: (B) Patrick Geddes
A megalopolis is a very large city, or an urban area that consists of several towns and
cities.
Megalopolis= Urban Agglomeration= Conurbation

Q. 2 Match the following


Q. 2.1 (a) Exterior design in Architecture (A) Margeret Robers (i)
(b) Urban Landuse Planning (B) Brian Hackett (j)
(c)Image of a city (C) Gorden Cullen (g)
(d) Architecture through the ages (D) Stuart Chapin(b)
(e)Space Time and Architecture (E) Yoshinobu Ashihara (a)
(f) Earthscape (F) Paul D Spreiregen (h)
(g) Townscape (G) S. Giedion (e)
(h) The Architecture of Towns and Cities (H) Talbot Hamlin (d)
(i) An introduction to Town Planning Techniques (I) Kevin Lynch (c)
(j) Planting Design (J) John O Simon (f)

Q.2.2(a) Cavity Wall (A) Population (g)


(b) Plumb (B) B. M. Fuller (f)
(c)Goose Neck (C) Sabine (i)
(d) Chaitya (D) Arch (h)
(e)Anemometer (E) Rural Community (j)
(f) Deodesic Dome (F) Prayer Hall (d)
(g) Histogram (G) Fire resistance (a)
(h) Voussoir (H) Wind Force (e)
(i) Reverberation (I) Masonry (b)
(j) Eoplois (J) Water supply (c)

Q.3 Select out of the given list of architects, the ones associated with the design of each of
the following ten buildings:
(a) Victoria Terminus, Bombay (A) A.P Kanvinde
(b) India International Centre, Delhi (B) Pierre Jenneret (f)
(c)Centre for Development Studies, (C) Aditya Prakash
Thiruvananthpuram
(d) Kashmir Conference Complex, Srinagar (D) Hasmukh Patel
(e)I.I.M. Bangalore (E) Madhav Nayak
(f) Gandhibhavan, Chandigarh (F) Le Corbusier
(g) Raj Bhavan, Calcutta (G) B. V. Doshi (e)
(h) Hall of Nations, Pragati Maidan (H) Laurie Baker (c)
(i) Tamilnadu Legislative Complex, Madras (I) F.W Stephens (a)
(j) Sriram Centre, New Delhi (J) U C Jain
(K) Habib Rehman
(L) J. A. Stein (b) (d)
(M) Raj Rewal (h)
(N) Ram Sharma
(O) Shiv Nath Prasad (j)
(P) D. A. Nair
(Q) Kuldip Singh
(R) Charles Correa
(S) Leo Pereira
(T) Charles Wyatt (g)

Q.4 What do the following abbreviations stand for?


(a) NBO: National building Organisation
(b) IDSMT: Integrated Development of Small and Medium Towns
(c) PCU: Passenger Car Unit
(d) BPE : ? Before Present Era (Question might be BPL- Below Poverty Line)
(e) HDFC: Housing Development Financial Corporation

Q.5 Distinguish between


i. Landmark and Node ii Cul-de-sac and Loop street
iii Land use and Land cover iv Loudness of sound and Intensity of sound
v Grey value contrast and Colour contrast

SUB SECTION A2
Q.6 Write a simple interactive programme in BASIC or FORTRAN to compute the
volume of a regular cone.
Q.7 State, to the point, the environmental attributes of trees in urban planning context.
Q.8 Mention basic pattern of city forms and illustrate them with diagrammatic sketches.
Q.9 State the effects of pollution on microclimate in an industrialized urban area.
.10 Compute the capacity and work out the dimension of a septic tank for a small colony of
500 persons with average daily sewage flow of 75 litres per head. Detention period is
36 hours. Cleaning interval is six months. Assume rate of deposited sludge as 25 litres
per capita per year.
SECTION B (PART –I)
Q.11 What are the influencing factors for the development of regional style in Indo- Islamic
architecture?
Q.12 Discuss, to the point, the contributions made by Le Corbusier in Modern
Architecture.
Q.13 Explain and illustrate with sketches the principal differences between Greek Agora
and Roman Forum in terms of
planning and architectural features.
Q.14 ‘The sense of urban spatial scale is based on enclosure’. What are the different
degrees of enclosure
and how do they help in perception of urban spaces and forms?
Q.15 In five square frame of equal size make compositions using lines and square shapes
depicting (i) rhythm (ii) pattern
(iii) bi-axial symmetry (iv) spatial depth and distance and (v) rotation.
.16 Compute the thermal transmittance (U) value for a 230 mm brick wall with 12.5 mm
thick cement plaster on both sides (values of thermal conductivity for

brick wall and cement plaster are 69 and 80 respectively in .


Thermal conductance of outside and inside walls is 0.05 and 0.16
respectively.
.17 Draw two consecutive layers of (i) octagonal brick pier and (ii) rat-trap bond at right
angled corner brick wall.
Q.18 Draw and label an isometric single line diagram of a plumbing system for a first floor
toilet (shown below)

.19 A newly built property fetches an annual rent of Rs 18000. As per agreement tenant is
liable to pay out-goings equivalent to 18 % of the annual rent. Calculate the present
value of the property. (Assume rate of interest as 8.5 %)
Q.20 Highlight briefly the features of a typical Japanese garden.
.21 a) What are the controls essential for comfort air conditioning in India? Mention the
range of each control element.
b) Why is dehumidification essential in summer air conditioning?
Q.22 Why is artificial lighting preferred to natural lighting in a museum or art gallery?
.23 Mention the common acoustical defects encountered in the design of a big auditorium.
Mention their causes.
Q.24 Determine which of the following (Fig. B-I-24) are determinate or indeterminate
structures.

.25 A reinforced concrete slab (having balanced section) has an overall depth of 100 mm.
The effective cover is 20 mm. If the stress in concrete and steel are not to exceed
5N/mm² and 140 N/mm², find the safe uniformly distributed load which can be placed
on the slab. The slab is supported on beam at 3.0 m c/c on both sides. The maximum

bending moment for a meter strip of slab may be taken as . Take the moment of
resisance of the balanced section as equal to 0.85 bd² N mm.
Q.26 The time estimates of various activities of a project are indicted in the following Table.
Determine the standard deviation of the critical path.
Activity Optimistic Most Pessimistic
time likely time
(Weeks) time (Weeks)
(Weeks)
1-2 8 12 22
1-3 6 12 18
2-4 1 4 7
3-4 5 9.5 11
2-5 9 15 21
4-5 3 4 5
Q.27 What are hue, value and intensity in a colour scale? Indicate their position in a three
dimensional diagram.
Q.28 Mention the factors that made the architecture and planning of Fatehpur-Sikri unique.
Q.29 Mention the means of controlling external noise which should be considered while
designing a building.
Q.30 State the function of DXFOUT, DXBIN and XREF commands in AutoCAD.

SECTION B (PART –I)

Q.11 A main sewer is to be designed to receive a flow from 2 sq km area of a community,


where the population density is 250
persons/hectare. The average sewage flow is 120 litres per capita per day.
What will be the design flow of the main sewer? Assume peak factor as 3.
Q.12 Sketch a label a common Rotary Intersection. State the advantages and
disadvantages of Rotary Intersections.
Q.13 Explain the concept of zero –base budgeting in urban development.
Q.14 Outline the basic elements which constitute the mental image of a city.
Q.15 What is meant by Land Pooling Technique in urban Land Management. Discuss the
operational steps for
preparing a Land Pooling scheme.
Q.16Discuss the role of co-operative housing in mitigating the housing shortage in India.
Mention the merit of
Co-operative housing.
Q.17 Mention the factors that govern the determination of most appropriate housing density
for a particular area. Enumerate each of them
briefly.
Q.18 Discuss the applicability of Scalogram Analysis in spatial development planning.
Q.19 Distinguish between ‘Multi-objective’ and ‘Multi-criteria’ model for decision and evaluation
of planning projects. Mention at least three techniques for each category.
.20 (a) Explain the concept of relief displacement on aerial photography.
(b) A flat area is photographed in a scale of 1: 10,000 with a camera of 15 cm focal
length.
The bottom of a chimney stack is found to lie at a distance of 12.01 cm from the
principle point of the photograph and the top at a distance of 12.22 cm. Find the
height of the chimney stack.
Q.21 Explain the significance of ‘Z’ score in the stastical analysis and mention its properties.
.22 State briefly the various Site Analysis Techniques for appropriate decision making in
urban landscape projects.
.23 Highlight the principles of ‘Ekistics’ and illustrate graphically.
.24 Mention the importance of Environmental Impact Assessment in handling large projects.
State various methods for identifying environmental effects and impacts.
.25 State the objectives of subdividing a planning area into traffic zones.
.26 Mention the different shapes suggested by Mansara for town and villages in ancient
India. Sketch atleast four such shapes.
.27 State the objectives of Sites and Services Scheme. Mention its advantages and
limitations in Indian Context.
.28 What is meant by ‘Decibel Zoning’? Mention principal types of urban noises and the
planning measures to control them.
.29 (a) Differentiate between ‘Mohallas’ of traditional cities and ‘Neighbourhood units’ in
modern cities.
(b) Explain the differences between ‘housing need’ and ‘housing demand’.
Q.30 Illustrate with sketches the important planning features for any ONE of the following
cities:
1) Brasilia 2) Islamabad
GATE 1997
Q.1.1The colonial word ‘Bungalow’ is originated from
(A) Ahmadabad (B) Goa (C)Bengal huts (D)Rajasthan desert cottage
Answer: (C) Bengal huts

Q.1.2The ‘Piazza del Campo’ is an example of


(A) Medieval period (B) Roman period (C) Greek period (D) Renaissance
Answer: (A) Medieval period
In the history of Europe, the Middle Ages or Medieval Period lasted from the 5th to the
15th century. It began with the fall of the Western Roman Empire and merged into
the Renaissance and the Age of Discovery. Piazza del Campo’ is a World Heritage
site and is in city of Siena, Italy. Sienna is known as Town of Human Scale and
Piazza del Campo is designed on Human Scale.

Q.1.3The unit of measurement, cusecs or cu.ft/sec is equivalent to


(A) 15.48 gal/min (B) 0.353 cu.m/sec (C)101.9 cu.m./hr. (D)None of the above
Answer: (D) None of the above
1 cusec = 28.317 litre/sec= 7.48 gallons/sec

Q.1.4The fully urbanized country is


(A) Taiwan (B) Vatican (C) Thailand (D) Finland
Answer: (B) Vatican

Q.1.5The Programming Language particularly suitable for commercial application is


(A) BASIC (B) FORTRAN (C) COBOL(D) ALGOL
Answer: (C) COBOL

Q.1.6Control of phenols with heavy metal contamination is possible through


(A) Ion – exchange treatment (B) Stabilization pond with water hyacinth
(C) Trickling filtration (D) Activated sludge process
Answer: (B) Stabilization pond with water hyacinth

Q.1.7‘Years Purchase’ in perpetuity can be computed by (where ‘n’ is number of years and ‘i’
is the rate of compound interest)

(A) (B) (C) (D)

Answer: (C)

Q.1.8The ‘Campidoglio’ was planned by


(A) Michelangelo (B) George Washington (C) Carlo Fontana (D)Paleadio
Answer: (A) Michelangelo

Q.1.9The Landscape planner for the Versailles was


(A) Richelieu (B) Andre Lenotre (C) Vauban (D) Sir Christopher Wren
Answer: (B) Andre Lenotre

Q.1.10 Two major components of gas generated through anaerobic digestion of sewage
sludge are
(A) CO2 and H2 (B) CO and CH4 (C) CH4 and CO2 (D) H2 and CO
Answer: (C) CH4 and CO2
Methane (CH4) generated is 70 % while Carbon di oxide (CO2) is 30 %. This mixture is
known as Bio gas.
Q.1.11In a simply supported beam, the maximum bending moment occurs at
(A) The point where shear force is zero (B) The point where shear force is
maximum
(C) Mid span point (D) None of the above
Answer: (A) The point where shear force is zero

Q.1.12The proportion and sizes of Greek Architecture – its scale were based on
(A) A module (B) Various parts of a building
(C) Optical corrections (D) Human measurements
Answer: (B) Various parts of a building

Q.1.13Use of Aluminium handles is prohibited in steel door or window frames due to


(A) Risk of electrolytic corrosion (B) Difference in tensile strength
(C) Difference in Modulus of Elasticity (D) Difference in ductility
Answer: (A) Risk of electrolytic corrosion

Q.1.14Walls and roofs should be massive and heavy in


(A)Cold Climate (B)Hot-dry climate (C) Warm-humid climate (D) Composite climate
Answer: (B) Hot-dry climate

Q.1.15Barium plaster is employed as a final coat


(A) For the walls of X-ray room (B) For acoustical purpose
(C) For superior types of wall finishes (D) For extra- hard finish
Answer: (A) For the walls of X-ray room

Q.1.16Eopolis is
(A) A city (B)A city is cosmos (C)A village (D )The universal city
Answer: (C) A village

Q.1.17The Aranya Project in Indore is planned by


(A) U. C. Jain (B) B. V. Doshi (C) C. P. Kukreja (D) Amos Rappoport
Answer: (B) B. V. Doshi

Q.1.18U.S. Embassy in New Delhi was designed by


(A)Laurie Baker (B)Raj Rewal (C)Richard Neutra (D)Edward D. Stone
Answer: (D) Edward D. Stone

Q.1.19Largest urban area by population


(A)Peking (B)Mexico (C)Brasilia (D)Calcutta
Answer: (B) Mexico

Q.1.20Float valves are used in distribution reservoir or other tanks


(A) To maintain constant water level (B) To relieve excessive pressure
(C) To drain out sediments collected (D) To permit the air to enter
Answer: (A) To maintain constant water level

Q.1.21Fire tower is
(A) An arrangement of pipes (B) A tower open to sky (C) A spiral chute (D) An
enclosed staircase
Answer: (D) An enclosed staircase

Q.1.22Individual parking space for car is


(A)3 m x 6 m (B)3.75 m x 7.5 m (C)2.5 m x 5 m (D)3 m x 5 m
Answer: (A) 3 m x 6 m
For parking space of a car, ‘Space requirement’ are as follows
- 3m x 6m in case of individual parking
- 2.75 m x 5 m in case of common parking
For scooter it is 1.25 sqm and for cycle 1.0 sq.m.
(Source: NBC 2016)
Q.1.23The maximum acceptable noise level inside library is
(A) 25 to 30 dB (B) 40 to 45 dB (C) 50 to 55 dB (D) 60 to 65 dB
Answer: (A) 25 to 30 dB

Q.1.24‘Perspective View’ concept was developed in Europe during


(A) Roman Period (B) Romanesque (C)Renaissance (D)Rococo
Answer: (C) Renaissance

Q.1.25The term ‘City of Light’ is associated with


(A)Paris (B)Vienna (C)Karlsruhu (D)Canberra
Answer: (A) Paris

Q.1.26Interstitial condensation refers to


(A)Surface Condensation (B) Dew point temperature
(C) Dry Bulb temperature (D) Relative humidity
Answer: (B) Dew point temperature
Interstitial condensation is a form of structural damping that occurs when
warm, moist air penetrates inside a wall, roof or floor structure, reaches the
dew point and condenses into liquid water.

Q.1.27A series of nearly equal-size cities in close proximity is known as


(A) Satellite (B) Radio centric (C) Constellation (D) Star
Answer: (C) Constellation

Q.1.28Architecture was defined as ‘Utilitas Fermitas Venustas’ by


(A) Michelangelo (B) Vitruvius (C)Leonardo da Vinci (D)Geoponti
Answer: (B) Vitruvius

Q.1.29Gray areas of the city are


(A) Service quarters of the city (B)Glamorous areas (C)Slums (D) Blighted areas
Answer: (C) Slums

.1.30When the contractor is asked to execute the various items of works specified in the
schedule at the tendered rates, it is called
(A) Lump sum contract (B) Cost plus a fixed percentage contract
(C) Unit –price contract (D) Labour contract
Answer: (C) Unit –price contract
(Corrected by Sumeet Singh Malik, Jai Bhagwan Institute)

Q. 2 Match the following


Q. 2.1 (a) Malabar Cement Township, Kerala (A) Raj Rewal (e)
(b) Indian Institute of Forest Management, Bhopal(B) Ralph Lerner (f)
(c) Sainik Guest House, Bhopal (C) Robert Millart (j)
(d) Yale University Art and Architecture building (D) Charles Correa (a)
(e) Parliament Library Extension (E) Le Corbusier (h)
(f) Indira Gandhi National Centre of Art (F) Anant d Raje (b)
(g) Bhubaneswar (G) Sen Kapadia (c)
(h) La Villa Radieuse (H) Otto Koenigsberger (g)
(i) Art Nouveau (I) Paul Rudolph (d)
(j) Mushroom Construction (J) Victor Horta (i)

Q.2.2 (a) Golden Ages of the Great Cities (A) Victor Gruen
(b) The Landscape of Roads (B) Fisher Robert M.
(c) Design Fundamentals (C) Edmund N Bacon
(d) The Emerging Urban Pattern (D) Crowe Sylvia
(e) Design of Cities (E) Bowra, Sir Maurice (a)
(f) Tomorrow : a peaceful path to social reform (F) C.A. Doxiadis
(g) New Horizon in Colour (G) Ebenezer Howard (f)
(h) Twenty Years of Public Housing (H) Robert Gillaim Scott
(i) Mastering Turbo C (I) Faber Birren
(j) Urban Renewal and the Future of the Urban Cities (J) Stan Kelly- Bootle

Q.2.3 (a) Calistemon lanceletus (A) Heliodome (e)


(b) Isovist (B) Computing (d)
(c) Float (C) Landscape (b)
(d) Iteration (D) Activity (c)
(e) Sunpath (E) Bottle Brush (a)
A single isovist is the volume of space visible from a given point in space, together
with a specification of the location of that point. Isovists are naturally three-
dimensional, but they may also be studied in two dimensions: either in horizontal
section ("plan") or in other vertical sections through the three-dimensional Isovist.
Q.3.0 Distinguish between:
i. Turbidity and Colour in drinking water
ii Cactii and Succulent
iii Algae and Algicide
iv d-Base and LOTUS
v Urban Space and Open Space
vi Evaluation and Evolution
vii Sheet Glass and Float Glass
viii Satellite town and Dormitory town
ix Alluvial soil and Black cotton soil
x Physical distance and Perceptual distance

SUBSECTION A2
Q.4 Mention briefly the issues to be considered while planning a terrace garden.
Q.5 Explain with sketches a town developed during the Greek period highlighting the major
features.
Q.6 Illustrate with sketches distance relationship for seeing facade details of a building.
Mention the angles.
Q.7 A housing project is initiated to build 30,000 dwelling units of different categories (20 %
H.I.G. of 80 sq.m., 30 % M.I.G. of 60 sq.m. and 50 % E.W.S. for 25 sq.m.) Assuming the
cost of land, site development, construction cost and overhead cost to be Rs. 1000 per
sq.m., Rs 500 per sq.m., Rs 3000 per sq.m. and Rs 500 per sq.m. respectively
for the upper income group. The total cost of the self financing project is estimated as
Rs 62,250 lacs.
Work out the various cost components for EWS housing.
Q.8 Write an interactive programme in FORTRAN or BASIC for computation of V, where

SECTION B
PART I
.9 Explain important architectural contributions made by Mies van der Rohe in his domestic
building.
Q.10 Explain with sketches construction of gothic wall and vaults.
.11 Illustrate with sketches the difference between random rubble and ashlars works in
stone masonry.
.12 Draw conceptual diagram and label the different components of typical fire fighting
system in an apartment building above 24 m.
.13 An investor has capital of Rs 15 Lakhs from which he expects the return of 14.5
percent. He intends to purchase a small workshop from which the net annual income is
expected to become Rs 5 lakhs. Calculate the maximum price which the investor can
invest for the workshop if money can be borrowed in mortgage at 16% for 8 years.
.14 Draw a sun path diagram labeling all the elements in it for tropic. What information does
a sun path diagram convey to architects?
Q.15 Explain salient architectural features as evident in Quit Miner.
Q.16 Distinguish between ‘setting of cement’ and hardening of cement?
Q.17 What is meant by ‘occult balance’? illustrate it through a sketch within a frame of
10cmX10cm.
Q.18 Highlight the principle of Jain temple planning.
Q.19 Draw the Bending Moment and Shear Force diagrams for the following:

.20 Critically compare between two well known network analysis techniques ‘PERT’ and
‘CPM’ with respect to architectural projects.
.21 Design and illustrate a general classroom for 40 students in hot-humid climate, and
mention the design criteria adopted.
.22 Outline the usefulness of Building Bye-laws.
.23 Highlight, with suitable examples, the characteristics features of Spanish garden.
.24 Mention the factors to be considered for thermal insulation of buildings.
.25 A studio has dimension 10 m. x 8 m. x 5 mother ceiling of studio is provided with
acoustical tiles having absorption coefficient = 0.40. Curtains in heavy folds (absorption
coefficient = 0.50) are provided on one short wall. The absorption power of other
surfaces of the studio may be taken as 8 sq. m. sabins. What will be extra absorption
units required to make reverberation time t= 0.75 sec.?
.26 What is colour harmony? How it differs from Colour Contrast? Give some examples.
.27 Explain the function of ‘BLOCK’ and ‘WBLOCK’ commands in Autocad highlighting their
attributes.
.28 Draw the sketches of a ‘Balanced Cantilever’ and a ‘Propped Cantilever’ highlighting
their functions.
SECTION B
PART II

Q.9 Outline the cause of soil degradation as major concern on terrestrial environment.
.10 Comment on the existing mechanism of devolution of urban development fund between
State and Local Governments.
.11 Stating the scope of ‘MILE STONE TECHNIQUE’, prepare a mile stone chart schedule
for residential site planning project.
.12 Briefly discuss the system adopted for urban solid waste disposal.
.13 Discuss the elements of PERT to establish its logical network in scheduling projects.
.14 Evaluate the Fagence techniques of Peoples’ participation in Planning.
.15 Explain through sketch a typical plan and section of Pour-Flush Water seal Latrine.
.16 Briefly outline the contents of a Development Plan.
.17 (a) Illustrate to the point, the significance of Landform in Landscape.
(b) Mention the factors which govern the decision related to Landform.
.18 There are four routes from a person’s house to his work place. there are two parking lots
near his office building. His office building has 4 entrances, three elevators to reach his
office floor, and a corridor from each to his office door.
(a) How many ways can the person reach his office door?
(b) While starting from home he realizes that route 1 and 3 are no entry, parking lot 1 is
closed for the day and elevator no. 2 is on repair. What is
probability of selecting route no. 4, parking at lot no. 2 use the cast
entrance and use elevator 1?
.19 Distinguish between the ‘Spacing’ and ‘Headway’ in traffic stream and mention the way
they can be measured.
.20 Discuss the Professional Code of Conduct of a Planner to his fellow Professional.
.21 State the major data inputs from Satellite Remote Sensing System for Urban Planning.
.22 Draw a schematic layout of a ‘Trumpet Interchange’ showing direction of flow of traffic.
Mention at what situation it is more appropriate.
.23 Highlight the importance of G.I.S. (Geographic Information System) in the planning
decision making.
.24 Analyze ‘St. Mark’s Square, Venice’ from the urban design point of view, highlighting the
merits and achievements.
.25 The distance between two points in a map on 1: 100,000 is 2 cm. Distance between the
same two points in an aerial photograph is 10 cm. The camera of the aerial photograph
was flown with a focal length of 6 inches. Find out the scale of the photograph and
calculate the flying height.
.26 Highlight the Demographic characteristics for delineation of a Region.
.27 Illustrate with sketches the major features of Mohenjidaro Plan.
.28 A property has been sold by the Housing Board on a conditional sale. The Board is to
receive Rs 24,000 at the end of every year for 10 years, and further, the Board is to
receive an amount of Rs. 200,000 at the end of 10 years. A period of 4 years has
already lapsed. Estimate the current value of the property (Interest rate for Years
Purchase is 8 %)
GATE 1998
Q.1.1‘Pompidou Centre’ in Paris is an example of
(A) Symbolic analogy (B) Mechanical analogy (C) Bio-morphic analogy (D)Mathematical
analogy
Answer: (B) Mechanical analogy

Q.1.2Cement (in cu.m) required for preparing 10.0 cum. of cement concrete in the
proportion of 1: 2: 4 is
(A) 0.80 (B) 1.00 (C) 1.20 (D) 1.40
Answer: (D) 1.40

Q.1.3The Pritzker Architecture Prize, 1996 has been awarded to


(A) Charles Correa (B) Jose Rafael Moneo(C) Robert Venturi (D) Ricardo Legorretta
Answer: (B) Jose Rafael Moneo

Q.1.4Maximum tensile stress is possible in


(A) Wood (B) Stabilised Mud block (C) Stone Block (D)Steel
Answer: (D) Steel

Q.1.5‘Kailash Temple’ of Ellora is an example of rock-cut architecture of


(A) Brahmanical style (B) Dravidian Style (C) Pallava Style (D) Mamalla Style
Answer: (B) Dravidian Style
Architectural styles, Dravidian architecture (Rastrakuta style)

Q.1.6‘Zeolite’ is
(A) Hydrated alumino-silicate (B) Sodium carbonate
(C) Hydrated Calcium hydroxide (D) Calcium bi-carbonate
Answer: (A) Hydrated alumino-silicate Diameter Gradients Discharge
mm m3/min
Q.1.7The gradient of a horizontal branch in building 100 1 in 57 0·18
drainage system should not be steeper than 150 1 in 0·42
(A) 1 in 60 (B) 1 in 30 (C) 1 in 20 (D) 1 in 10 100
Answer: (A) 1 in 60 200 1 in 0·73
Normally, the sewer shall be designed for 145
discharging three times the dry-weather flow flowing
230 1 in 0·93
half-full with a minimum self-cleansing velocity of
175
0·75 m/s. The approximate gradients which give this
velocity for the sizes of pipes likely to be used in 250 1 in 1·10
building drainage and the corresponding discharges 195
when flowing half-full are as follows: 300 1 in 1·70
250
Q.1.8‘Timgad’ is an example of
(A) Greek town (B) Roman Town (C) Sumerian Town (D) Egyptian Town
Answer: (B) Roman Town

Q.1.9‘Architect’s Credo’ is
(A) Liberty, Equality and Fraternity (B) Firm, Commodity and Delight
(C) I came, I built, I overchanged (D) Man, Hot dogs and Apple pie
Answer: (C) I came, I built, I overchanged

Q.1.10 ‘Dimmer’ is used for


(A) Energy saving (B) Shortening bulb life
(C) Flexible lighting (D) Change of emphasis of lights
Answer: (C) Flexible lighting

Q.1.11 The phenomenon of degrading water quality of natural water bodies in


decomposition is called
(A) Purification (B) Eutrophication (C) Fermentation (D) Hydration
Answer: no appropriate answer

Q.1.12 In computer program a valid real constant is


(A) 1 (B) - ½ (C) 1.5 (D) 58, 634.2
Answer:

Q.1.13 The ‘Water Temple, Awaji Island’ is designed by


(A) Fumihiko Maki (B) Kenzo Tange (C) Arata Isozaki (D) Tadeo Ando
Answer: (D) Tadeo Ando

Q.1.14 An ‘activity’ in Project Management means


(A) start or completion of task in time and resource
(B) total project time for completion of work
(C) least total time required to complete the work
(D) actual performance of task consumes time and resources
Answer: (A) start or completion of task in time and resource
An activity has precise starting and ending dates, incorporates a set of tasks to be
completed, consumes resources, and results in work products.

Q.1.15 The pH scale runs from 0-14, the nearest value of drinking water is
(A) 0 (B) 4.5 (C) 7.0 (D) 12.6
Answer: (C) 7.0

Q.1.16 The emission of auto exhaust consists of


(A) CO (B) SO2 (C) NOx (D) All the three
Answer: (D) All the three

Q.1.17 According to architecture of Mansara a building is called female when its plan is
(A) Octagonal (B) Rectangle (C) Circular (D) Square
Answer: (B) Rectangle
Square plan is for male and rectangle plan is for female. Male God can be
placed in Rectangle but female God not in square plan.

Q.1.18 The unit of measurement for intensity of sound level is


(A) Bel (B) Decibel (C) Hertz (D) Sones
Answer: (B) Decibel

Q.1.19 To prevent excessive contrast between wall and window which creates glare, the ratio
of wall to window area should not generally exceed
(A) 1 : 1 (B) 2 : 1 (C) 3 : 1 (D) 4 : 1
Answer: (D) 4 : 1

Q.1.20 Permissible stress in bending compression (N/mm²) for M20 grade concrete is
(A) 3.0 (B) 5.0 (C) 7.0 (D) 8.5
Answer: (C) 7.0

Q.1.21 Non-degradable pollutant is


(A) Sewage (B) Algae (C) Detergents(D) DDT
Answer: (D) DDT
Q.1.22 Maximum air flow at body level in a room can be achieved through
(A) High inlet and high outlet (B) High inlet and low outlet
(C) Low inlet and high outlet (D) Low inlet and low outlet
Answer: (C) Low inlet and high outlet

Q.1.23 The bread basket region refers to


(A) Tropical desert (B) Coniferous forest (C)Temperate grassland (D)Tropical grassland
Answer: (C) Temperate grassland
Prairies in Temperate grassland is bread basket region of USA.

Q.1.24 ‘Tap root’ concept was initiated by


(A) Le Corbusier (B) F L Wright (C) Tony Garnier (D) Robert Millart
Answer: (B) F L Wright
Tap root foundation and mushroom column was propogated by F. L. Wright.

Q.1.25 ‘Extreme Simplicity’ was propagated by


(A) J.J.P.Oud (B) C.A. Doxiadis (C) Mies Van der Rohe (D) Walter Gropius
Answer: (C) Mies Van der Rohe
‘Less is more’- Mies Van der Rohe

Q.1.26 Botanical name for ‘Gulmohar tree’ is


(A) Hamelia Patens(B) Cordia Sabestina (C) Delonix Regia (D)Gmelina Phillippensis
Answer: (C) Delonix Regia

Q.1.27 Type of village should be built on a bank of river or a sea as per architecture of
Mansara is
(A) Nandyabarta (B) Karmukha(C) Swastika (D) Prastara
Answer: (B) Karmukha

Q.1.28 CPCB is an organization which deals with


(A) Pest Control (B) Poverty Control(C) Population control(D) Pollution Control
Answer: (D) Pollution Control

Q.1.29 Heliodome is used to measure (GATE 2016)


(A) Atmospheric pressure (B) Sound level (C) Sun Path (D) Intensity of light
Answer: (C) Sun Path

Q.1.30 Multi layer dome was found during


(A) Greek period(B) Renaissance period (C) Indus valley period (D) Egyptian Civilization
Answer: (B) Renaissance period

Q. 2.0 Match the following


Q. 2.1 (a) Principles and practices of Town and Country Planning (A) A. Toffler (e)
(b) Site Planning (B) Andrea Palladio (d)
(c)The City in History (C) Charles Correa (h)
(d)Four books of Architecture (D) C.P. Kukreja (j)
(e)Future Shock (E) Lewis Keeble (a)
(f) Complexity and contradiction in Architecture (F) F. L. Wright (i)
(g) Human Aspects of Urban Form (G) Kevin Lynch (b)
(h)The New Landscape (H) Amos Rappoport (g)
(i) Writings and Buildings (I) Robert Venturi (f)
(j) Tropical Architecture (J) Lewis Mumford (c)
Q.2.2
(a) Ringlemann chart (A) Suspended Particles (c)
(b) Waldram Diagram (B) Hardware (e)
(c) Electrostatic Precipitator (C) Smoke Density (a)
(d) Stereoscope (D) Sky Factor (b)
(e) Micro Processor (E) Aerial photo (d)

Q.2.3
(a) Linear City (A) P.L. Nervi
(b) Dynapolis (B) Anant Raje
(c)National Institute of Immunology, Delhi (C) D. A. Nair
(d) Exhibition Hall (D) Turin (a)
(e)Loyola Chapel, Trivandrum (E) Le Corbusier
(f) Kinemax, France (F) Oscar Periera
(g) Kalakshetra Theatre, Chennai (G) Raj Rewal (c)
(h) Metabolism (H) I. M. Pei (j)
(i) New Parliament Complex, Sri Lanka (I) Shivnath Prasad
(j) National Gallery of Arts, Washington D.C.(J) Lawrie Baker (e)
(K) Denis Laming (f)
(L) Gunar Mydral
(M)Geoffrey Bawa (i)
(N) Charles Correa
(O) Kenzo Tange (h)
(P) C. A. Doxiadis (b)
Q.3.0Distinguish between:
(i) Transplantation and Transpiration
(ii) Hardening and Tempering of steel
(iii) curtain wall and cavity wall
(iv) Gully and Gutter
(v) Heartwood and Laminated wood
(vi) Food Producers and Food Consumers
(vii)Repitition and Rhythm
(viii) Easement and Prescriptive Rights
(ix) Earthernware and Stoneware
(x) S-trap and P-trap

SUB SECTION A-2


Q.4 Outline the procedure to convert a part of an Auto CAD drawing into a separate
drawing File.
Q.5 Describe the salient features of English style Garden.
Q.6 Outline the mechanism to scale down a large public square flanked by buildings all
round.
Q.7 Outline with example the evolution of settlement planning thoughts during Egyptian
period.
.8 A developer is planning to develop 50 hectare of land for residential complex. 65 % of
the total land will be available for residential plots. The land acquisition cost is
Rs 60.00 per sqm. Mentioning all the outgoings and assuming appropriate cost for
them with 20 % profit for the developer, workout the land cost component for the
project per unit area of built space if FAR is 1.5.

SECTION- B

Q.9 Explain with sketches the evolution of Gothic vaulting during Roman and Romanesque
period.
Q.10 Explain the term ‘Vernacular Architecture’. Mention the contribution of Lawrie Baker in
the development of vernacular
architecture.
Q.11 Mention the advantages of Poly-carbonate sheets or fiber glass reinforced plastic.
Q.12 Draw the sketches (atleast five) of the different ways to cover long span uninterrupted
space.
Q.13 Critically analyze the architectural features of Taj Mahal building complex at Agra.
Q.14 State the special considerations for the fire fighting measures for an auditorium.
Q.15 Bring out the advantages of PVC pipes over conventional C.I. pipes in building sewer
system.
Q.16 Enumerate the design considerations to ensure ventilation in engineering industrial
structures.
Q.17 Discuss with illustrations the application of ‘Organic Architecture’ by F. L. Wright.
Q.18 Show the exploded details of
a) Dove-tailed joint in wood
b) Double Tenon and Mortise joint in wood.
Q.19 Outline the utility of mixing sand in mortar.
Q.20 Explain the visual principles of landscape design with building mass or block.
Q.21 Draw the essential features of St. Peter Plaza, Rome, explaining the design feature.
Q.22 The rateable value of a building is Rs 20,000/- p.a.when interest on capital is 18% and
on sinking fund is 6 %. The owner of the building gets an offer from a bank for a net rent
of Rs 25,000/- p.a. for 21 years lease period, provided he modifies the internal layout at
a cost of Rs. 30,000/-. As a valuer what would be your advice to the owner
regarding the bank offer.
Q.23 Determine the size and the end span reinforcement (due to moment) for a beam to
support a live load of 12 KN/m on a single span of 8 m, using M15 concrete grade σst =
230 N/mm² (R= 0.658 N/mm² (R= 0.658 N/mm² and j= 0.9)
Q.24 Highlight the critical design features of Asiad Village, New Delhi.
Q.25 Compare between Early start schedule and Late start schedule in Project
Management.
Q.26 Mention the architectural characters developed in the construction of Khajuraho group
temples which distinguish them from any other temple design and style.
Q.27 Starting the concept of ‘Taxonomy of Space’, explain atleast four methods of creating
spaces within a large enveloping space.
Q.28 Suggest the elements in normal public buildings to be stressed upon for the use of
physically handicapped persons.
SECTION -B

Q.9 Enumerate the decision factors for outdoor recreation.


Q.10 Mention five Government sponsored Slum Upgradation Schemes in urban areas.
Q.11 Draw a sketch of Diamond Interchange of a Freeway with a Highway and label all the
parts.
Q.12 Suggest the components of Financial Appraisal of a Project.
Q.13 Evaluating the ‘Master Plan Approach’ suggest the future of Spatial Plan.
Q.14 Highlight the principles with sketches “Hoyt’s Sector Theory.”
.15 The weights of 40 students in a college are recorded to the nearest kilogram. Construct
a histogram and frequency polygon for the weight distribution at 5 kg class
interval.

51 52 54 56 57 58 58 59
59 60 61 62 62 63 63 64
64 64 65 65 66 66 67 67
68 68 69 69 70 71 72 72
73 74 76 77 78 79 81 84

Q.16 Mention the different ways of decreasing the Traffic Noise level from a Highway.
Q.17 Identify various methods for conducting Environmental Impact Assessment.
Q.18 Outline the details required to be incorporated in the preparation of project estimate for
water supply scheme of a town.
Q.19 Suggest an action program for top soil conservation or protection.
Q.20 Explain the principles for serial vision and truncation in Urban Design.
Q.21 State the importance and purpose of channelization of modern highways.
Q.22 Outlining the ‘Vastupurush Mandala’ discuss its applicability in modern urban structure.
Q.23 Comment on the performances of Urban Land Ceiling and regulation Act, and suggest
requisite reforms.
Q.24 State the ‘Elements of Public Interest’ in formulating Development control.
Q.25 Suggest the characteristics of “Growth Centre” in regional planning.
.26 The total population of an area for 1981 and 1991 was given as 30 lakhs and 35 lakhs
respectively. Extrapolate the expected population for 2001 based on two different
methods.
a) Arithmetic mean
b) Geometric mean
Q.27 Explain the merits and demerits of a large dam in regional planning.
.28 An aircraft flying at an altitude of 5000 m above mean sea level takes aerial photographs
of a terrain having an average elevation of 1000 m above mean sea level.
a) Find the scale of photograph if focal length of camera is 20 cm.
b) Find the area covered in ground by each photo format of 23 cm x 23 cm.
GATE 1999
Q.1.1Peristyle in architecture means
(A) a row of free sanding columns surrounding an area
(B) perimeter wall of an enclosed shrine
(C) perishable materials in buildings
(D) the triangular part above the entablature in the classic order
Answer: (A) a row of free sanding columns surrounding an area
Hypostyle: roof supported by columns only. Eg. Hypostyle hall of Karnak,
Temple: Egypt.
Pediment: the triangular part above the entablature in the classic order

Q.1.2Washington DC. is an example of


(A) linear urban form (B) star shaped urban form
(C) poly centred net urban form (D) the sheet urban form
Answer: (B) star shaped urban form

Q.1.3SON lamps operate on the principle of discharge in


(A) Sodium vapour (B) Krypton vapour
(C) Mercury vapour (D) Zinc and Cadmium vapour
Answer: (A) Sodium vapour

Q.1.4A method of control survey, in which a network of triangles is used, is


(A) Triangulation (B) Three-point resection (C) Trilateration (D) None of these
Answer: (A) Triangulation

Q.1.5Hyperbolic paraboloid can be generated by


(A) a curve moving over two straight lines at obtuse angles.
(B) a straight line moving over a curve at acute angle.
(C) a curve moving over two other parallel curves
(D) a straight line moving over two other straight lines at an angle to one another.
Answer: (D) a straight line moving over two other straight lines at an angle to one another.

Q.1.6‘Savannas’ are
(A) grasslands with drought-resistant trees (B) parts of arctic region with moving
glaciers
(C) estuaries, where delta is formed (D) parts of the desert with perennial water pockets.
Answer: (A) grasslands with drought-resistant trees

Q.1.7Variability of project duration in PERT analysis is measured in terms of


(A) pessimistic time difference (B) optimistic time difference
(C) time difference of activities (D) square of standard deviation of activity duration
Answer: (D) square of standard deviation of activity duration

Q.1.8The term Necropolis refers to


(A) small size metropolis (B) the new metropolis(C) dead city (D) the city in
space
Answer: (C) dead city

Q.1.9Phenomenon of contorted growth of trees due to unequal irradiation of light on two


sides is known as
(A) Photosynthesis (B) Phototropism (C)Photoperiodism (D)Photorespiration
Answer: (C) Photoperiodism
Photosynthesis: food preparation in the presence of sunlight
Phototropism: movement of plants towards sunlight
Photorespiration: It occurs in a hot dry day when stomata of plants close to
prevent excessive loss of water thereby increasing CO2 concentration.

.1.10 The total quantity of runoff of an area of 1.8 hectares in a lateritic region (runoff
coefficient = 0.5 and rainfall = 10 mm/hr) is
(A) 55 m³/hr (B) 108 m³/hr (C) 90 m³/hr (D) 180 m³/hr
Answer: (C) 90 m³/hr
Run off= CIA where C= Runoff Coefficient
I= Intensity of rainfall (here annual rainfall is given)
A= Area

Q.1.11 The average Lux required on a pavement, having width 4 m, is 8. The mounting height
of the lamp (lumen 2000) is 4 m. The spacing of the lamps (for coefficient of the
utilization is 0.5 and maintenance factor is 0.8) is
(A) 25 m (B) 75 m (C) 10 m (D)15 m
Answer: (A) 25 m
Let a stretch of 100 m of road be considered. Then total area to be illuminated is 100
× 4= 400 m²
Totals lumens required to get Lux of 8 = 400× 8= 3200 lumens
Given lumen by each lamp = 2000 lumens
Actual lumens reaching ground=
2000 × maint. Factor × utilization coeff.
= 2000× 0.8× 0.5= 800 lumens

Total no of lamps required for 100 m sretch = = 4 lamps

Spacing of lamp= = 25 m

.1.12 In BOT based project, the most important evaluation criteria is


(A) financial internal rate of return (B) internal rate of return (C) benefit- cost
ratio (D) present value
Answer:

Q.1.13 Addition of Decibel levels 92 dBA and 88 dBA amounts to


(A) 97 dBA(B) 150 dBA (C) 180 dBA (D) 93 dBA
Answer: (D) 93 dBA

Q.1.14 The velocity head of water supply line is measured in terms of


(A) m/sec (B) m/sec² (C) m (D) m²/sec
Answer: (C) m
Velocity head is due to the bulk motion of a fluid (kinetic energy).
Elevation head is due to the fluid's weight, the gravitational force acting on a
column of fluid.
Pressure head is due to the static pressure, the internal molecular motion of a
fluid that exerts a force on its container.
Resistance head (or friction head or Head Loss) is due to the frictional forces
acting against a fluid's motion by the container.
Note: All heads are expressed in ‘m’ or ft.

Q.1.15 ‘After-image’ in visual perception is


(A) same image same colour (B) same image complementary colour
(C) inverted image complementary colour (D) mirror image same colour.
Answer: (B) same image complementary colour

Sub Section A2
Q.2 Match the following from 2.1 to 2.10
(2.1) Distomat (A) Odour
(2.2) Trowel (B) Truss (2.5)
(2.3) Lamelle (C) Plaster (2.2)
(2.4) Aziotoc (D) Mud architecture (2.6)
(2.5) Purlin (E) Survey (2.1)
(2.6) Adobe (F) Order (2.10)
(2.7) Jalousie (G) Luminaire
(2.8) Latourette (H) Plumbing
(2.9) Column (I) Window(2.7)
(2.10) Doric (J) Temple
(K) Le Corbusier(2.8)
(L) Buckling (2.9)
(M) Computer
(N) Air conditioning

Match the following from 2.11 to 2.20


(2.11) Express Tower, Bombay (A) Paolo Soleri (2.20)
(2.12) Semi Conductor Complex, Chandigarh (B) Charles Correa (2.18)
(2.13) IIT Campus, Kanpur (C) Edward D Stone (2.19)
(2.14) Aranya Housing project, Indore (D) Jorn Utzon (2.17)
(2.15) Amsterdam South Plan (E) John Nash (2.16)
(2.16) Park Crescent, London (F) B.V. Doshi (2.14)
(2.17) Sydney Opera House (G) H. P. Berlage (2.15)
(2.18) Cidade de Goa (H) A. P. Kanvinde (2.13)
(2.19) U S Embassy, New Delhi (I) Joseph Allen Stein (2.11)
(2.20) Arco Santi, USA (J) Romi Khosla and associates (2.12)
(K) Anant Raje
(L) Richard Neutra

Match the following from AR 2.21 to AR 2.25


(2.21) Defensible Space (A) Euges P Odum (2.23)
(2.22) Th Economics of Urban Areas (B) Robert Wenkam
(2.23) Fundamentals of Ecology (C) Arthus B Gallion (2.25)
(2.24) Design for the Real World (D) Edgar R Neff
(2.25) Urban Pattern (E) Brian Goodall (2.22)
(F) Oscar Newmann (2.21)
(G) Victor Papanek (2.24)
(H) Richard Meyer

SubSection A3
Q.3.0Distinguish between :
Q.3.1Viewshed and Watershed.
Q.3.2Dormer Window and Bay Window.
Q.3.3Economic life and Physical life of building
Q.3.4Aqueduct and Acquifer
Q.3.5Ventillation and Air- conditioning.
Q.3.6Gradient and Camber
Q.3.7Revolving Fund and Sinking Fund
Q.3.8Raster images and Vector Images
Q.3.9Value and Intensity of colour
Q.3.10 Arbitrator and Umpire.
Section B (Part I)
Q.4 Draw the following Brick paving patterns.
(A) Running Bond (bricks laid on edge)
(B) Herringbone (bricks laid flat)
(C) Basket Weave (bricks laid on edge)
(D) Stacked Bond (bricks laid on edge)
(E) Basket weave variation (bricks laid flat)
Q.5 What is a Psychrometric Chart? Draw a typical Psychrometric Chart with appropriate
labeling and explain its explanation.
Q.6 Discuss the salient features of French style garden through illustration.
Q.7 Illustrate with sketches the function of the following AutoCAD commands:
(A) REVSURF
(B) RULESURF
(C) EDGESURF
(D) TABSURF
(E) THICKNESS
Q.8 Draw a schematic plan and a section of an Open Air Theatre designed for good
acoustics, showing all the design elements used for achieving the
objective.
Q.9 A loan of Rs 8,00,000 Rs been granted by a financial institution to an individual for the
construction of his house. The loan has to be repaid by way of annuity at the rate of
13.5 % interest per annum in 15 equal installments from the year in which the loan is
taken by the house owner. Calculate the yearly installment for repaying the
loan.
Q.10 Indicate five major advantages for which you will recommend steel structure for a multi
storied building.
Q.11 Sketch the section of an overhead water reservoir, showing the float valve, overflow pipe,
drain outlet, supply inlet and outlet and other valves.
Q.12 What is Mastic Asphalt? Where is it used in built environment?
Q.13 An activity in a CPM network has duration of 4 days. The free float for the activity is 10
days and the total float is also 10 days. Find the maximum delay that can be allowed for
the activity from the occurrence of the preceding event.
Q.14 With the help of the diagrammatic plan and as idea view, show the various important
elements and space components of a typical Orissan temple.
Q.15 Explain the phenomenon ‘Stack effect’ in a building. Write the equation for estimating
stack effect.
Q.16 With the help of three sketches explain the variation of the horizontal thrust at the
springing point of an arch with respect to its rise for a fixed span and load.
Q.17 What is contrast in visual design ? Explain the various applications of contrast in
architecture.
Q.18 Explain the importance of Variance, Easement, and Deed restrictions in architectural
preservation.
Q.19 What is the utility of central court in a housing cluster under hot-humid climate? Explain
with sketches.
Q.20 Indicate the various possibilities of use of the following industrial and agricultural
wastes in buildings.
(A) Blast furnace slag
(B) Waste glass
(C) Slate and late rite stone waste
(D) Coconut waste and husk
(E) Rice Husk
Q.21 Compare between ‘Chaityas’ and ‘Viharas’ in rock cut architecture, in terms of their
layouts, elements and facade treatment.
Q.22 What were the major driving forces in development of modern architecture in post
industrial revolution?
Q.23 Explain through plan and section the illumination scheme of the design studio in an
architect’s office having six drawing desk units and three computer work
stations.

Section B (Part II)


Q.4 Draw a population pyramid for an urban area with significant in- migration of working
population.
Q.5 Indicate five major factors which affect the capacity of a road in urban areas.
Q.6 Work out the net and gross population densities for a neighbourhood, given the
following
a) ground coverage = 30 %
b) F.A.R. = 1.5
c) plinth area per residential block= 372 sq.m.
d) number of dwelling units per floor in each block= 2
e) average family size = 4.5
f) area under residential plots= 62 %
g) area under access roads = 2 %
h) area under others including major roads = 36%

Q.7 Indicate with sketches, the different types of water supply distribution network in an
urban area.
Q.8 Briefly enumerate the advantages as well as the application of Remote Sensing in
planning.
Q.9 In a 200 m x 150 m corner plot with vehicular traffic on two abutting roads, prepare a
schematic housing layout plan with six four –storied blocks of 48 HIG units , with the
objective of minimum pedestrian –vehicular conflict and maximum defensible spaces.
Q.10 In the light of 74th Amendment of the Indian Constitution, discuss the new role of the
State Town and Country Planning Departments.
Q.11 Write, step by step, the AutoCAD commands to perform the following operations in
mapping.
(a)to draw a site plan of irregular configuration, closed traversed dimensions of edges
and angles between the pairs of edges.
(b)to estimate the area and perimeter of the site
(c)to create hatches for the landmass and water bodies.
Q.12 What are the major considerations for designing an Oxidation pond for a small town?
Q.13 What is Net Present Value? Which are the major parameters that decide the Net Present
Value of a development project?
Q.14 Mention the significant factors for designing a neighbouhood level park.
Q.15 Explain the major design features of ‘Campidoglio’ in Rome.
Q.16 A primary road within a city has to bend along a horizontal curve having a radius of 150
m. what should be the design speed of the road at that section if the maximum super
elevation of 0.07 is not to be exceeded and the safe limit of transverse coefficient of
friction is 0.15.
Q.17 Explain the salient features of Kenzo Tange’s plan for New Tokyo.
Q.18 In urban and rural systems highlight the social conditions that influence the social class
structure.
Q.19 What are the primary considerations for planning of settlements in a desert region?
Q.20 What are the factors to be considered in landscape planning for cyclone- prone coastal
zones?
Q.21 Mention the points to be considered while designing a Signage system for the India Gate
precinct in New Delhi.
Q.22 The spot elevation of four points A, B, C and D, in an area are 900 m, 1650 m, 1200 m,
and 1800 m respectively. Determine the maximum scale and the average scale of the
survey photo, when the flying height is 4500 m. The focal length of the camera is 140
mm.
Q.23 Indicate five major factors that you would consider for the assessing the house
demand in a town.
GATE 2000
Q.1.1J. N. U., New Delhi campus has been designed by
(A) A. P. Kanvinde (B) Louis I. Kahn (C) C. P. Kukreja (D) J. A. Stein
Answer: (C) C. P. Kukreja

Q.1.2Occupancy rate refers to


(A) number of buildings per unit area. (B) number of persons per habitable rooms.
(C) number of habitable rooms per acre. (D) number of persons working in an office.
Answer: (B) number of persons per habitable rooms.

Q.1.3The new Guggenheim Museum in Bilbao, Spain was designed by


(A) Frank Gehry (B) F. L Wright (C) Roger Anger (D) orman Foster
Answer: (A) Frank Gehry

Q.1.4Vidhyadhar nagar was planned by


(A) H. K. Mewada (B) Vidhyadhar Bhattacharya (C) B. V. Doshi (D) Charles Correa
Answer: (C) B. V. Doshi

Q.1.5For Indian Metropolitan cities the quantity of solid waste accumulation per head per
day is approximately
(A) 0.5 Kg (B) 1.5 Kg (C) 5 Kg (D) 3.5 Kg
Answer: (A) 0.5 Kg

Q.1.6The maximum gradient of a ramp leading to a car park is


(A) 1 in 5 (B) 1 in 10 (C) 1 in 15 (D) 1 in 20
Answer: (A) 1 in 5

Q.1.7‘Less is Bore’ is propagated by


(A) Eero Saarinen (B) Philip Johnson (C) Robert Venturi (D) Joseph Paxton
Answer: (C) Robert Venturi

Q.1.8Turbidity of water is due to


(A) Algae (B) Fungi (C) Organic salt (D)Suspended matters.
Answer: (D) Suspended matters.

Q.1.9The only architect-president of a nation was


(A) Richard Nixon (B F. Marcos (C) Thomas Jefferson (D)L. B. Johnson
Answer: (C) Thomas Jefferson

Q.1.10 The Pruitt housing project in St. Louis failed because of


(A) natural calamity (B) structural failure (C) un- affordability (D)functional inaccessibility
Answer: (D) functional inaccessibility

Q.1.11 To ensure comfort condition inside the room, the temperature and relative humidity
values should preferably be
(A) 20º C and 65 % respectively. (B) 15º C and 45 % respectively.
(C) 25º C and 50 % respectively. (D) 30º C and 65 % respectively.
Answer: (C) 25º C and 50 % respectively.

Q.1.12 Intensity of colour refers to


(A) brightness (B) darkness (C) pigment density (D) quantity
Answer: (A) brightness
Value of colour refers to: Lightness or darkness.

Q.1.13 As per the National Building Code the minimum area of a habitable room is
(A) 8.5 sqm. (B) 9.5 sqm (C) 10.5 sqm (D) 11.5 sqm
Answer: (B) 9.5 sqm

Q.1.14 The most commonly used disinfectant for purification of municipal water is
(A) boric powder (B) alum (C) bleaching powder (D) camphor
Answer: (C) bleaching powder

Q.1.15 In completion of a project, Critical path is one which requires


(A) maximum time (B) minimum time (C) optimum time (D) critical time
Answer: (A) maximum time
Input from Chetana, B. Arch. Sunderdeep College of Architecture, Ghaziabad. RMIT, Australia.

Q.1.16 With every doubling of distance from source the noise level will reduce by
(A) 6 dbA (B) 8 dbA (C) 10 dbA(D) 12 dbA
Answer: (A) 6 dbA

Q.1.17 Indicator of poverty line denotes


(A) Persons having no shelter (B) Level of family income
(C) Per capita calorie consumption of food (D) none of the above.
Answer: (C) Per capita calorie consumption of food

Q.1.18 The situation which provides the most intimate scale to an observer is, while walking
along
(A) 9 m wide road in front of one storey buildings.
(B) 12 m wide road infront of two storey buildings
(C) 15 m wide road infront of three storey buildings
(D) 21 m wide road infront of four storey buildings
Answer: (D) 21 m wide road infront of four storey buildings

Q.1.19 The concept of Greek town planning emphasized on


(A) large size (B) more open space (C) human scale (D) compact development
Answer: (B) more open space

Q.1.20 ‘Habit’ of plants refers to


(A) growth rate (B) branching pattern (C) foliage (D) life span
Answer: No appropriate answer
The habit is characteristic appearance of a plant, stem, leaves or other organ
including size, shape, colour, pattern of growth.

Sub Section A2

Q.2 Match the following from 2.1 to 2.10


(2.1) Symbiosis (A) Planting
(2.2) St. Mark Square (B) Rome
(2.3) Autolisp (C) Ecology (2.1)
(2.4) Symbolic Garden (D) Egypt (2.6)
(2.5) Ferrule (E) Wash Basin (2.8)
(2.6) Layering (F) Water Main (2.5)
(2.7) Ziggurat (G) Sensor (2.10)
(2.8) Bib- cock (H) Venice (2.2)
(2.9) Kalasha (I) Habitat
(2.10) Scan line (J) Computer (2.3)
(K) Japan (2.4)
(L) Mesopotamia (2.7)
(M) Lingaraj Temple (2.9)

Match the following from 2.11 to 2.20


(2.11) Geodesic Dome (A) F. L. Wright (2.17)
(2.12) Conservative Surgery (B) Moshe Safdie (2.15)
(2.13) Chek Lap Kok Airport, Hongkong (C) Cesar Pelli (2.19)
(2.14) Calcutta (D) Buckminster Fuller (2.11)
(2.15) Khalsa Heritage Memorial Complex (E) M. N Joglekar
Anandpur Saheb (F) K. T. Ravindran
(2.16) Slum Networking, Indore (G) Patrick Geddes (2.12)
(2.17) Vertical city (H) Norman Foster (2.13)
(2.18) Jawahar Kala Kendra (I) Le Corbusier
(2.19) Petronas Tower, Kualampur (J) Job Charnock (2.14)
(2.20) Gandhinagar, Gujarat (K) H.K.Mewada (2.20)
(L) B.V. Doshi
(M) Himanshu H. Parikh (2.16)
(N) Charles Correa (2.18)
(O) Ebenezar Howard

Match the following from AR 2.21 to AR 2.25


(2.21) Design with Nature (A) Andrew Thomas
(2.22) Small is Beautiful (B) John O Simonds
(2.23) The World Cities (C) Ian McHarg (2.21)
(2.24) Culture of Cities (D) Lewis Mumford (2.24)
(2.25) The Fountainhead (E) Henry F Arnold
(F) Clarence Stein
(G) Peter Hall (2.23)
(H) Ayn Rand (2.25)
(I) E. F. Schumacher (2.22)
Sub Section A3

Q.3Distinguish between.
3.1 Value and Cost
3.2 Urbanism and Urbanisation
3.3 Absolute Humidity and Relative Humidity
3.4 Site Plan and Key Plan
3.5 Fine Sand and Coarse Sand
3.6 Forum and Agora
3.7 Harmony and Contrast
3.8 Artesian Well and Infilteration well
3.9 Restoration and Reconstruction
3.10 Conforming Use and Compatible Use
3.11 Plot Coverage Control and F.A.R. Control
3.12 Aerobic and Anaerobic
3.13 Cadastral Map and Topographic Map
3.14 Percentage Rate Contract and Lump Sum Contract
3.15 Hard Wood and Soft Wood

SECTION B
Part I

Q.4 Describe with sketches various elements of Bisnupur Temple Architecture in West
Bengal.
Q.5 Explain with illustration, the Baroque City Planning concept of Monarchy and
Monumentlism.
Q.6 Comment on the concept of ‘Smart Building’ in contemporary architectural
development.
Q.7 Highlight with example the utility of ‘Floorscaping in a public plaza.
Q.8 Draw a steel truss roof showing north- light and label the components.
Q.9 Outline the cause of failure due to combining old and new materials for building
preservation.
Q.10 What is the significance of water cement ratio in building construction? Indicate the
critical observations required in this context.
Q.11 Illustrate any one of the best example of residential sector planning in post independent
new towns of India.
Q.12 Explain briefly the function of the following commands in AutoCAD.
a) FILTER
b) 3DFACE
c) PEDIT
d) SETVAR
e) LTSCALE
Q.13 Highlight briefly, a) Planning criteria and b) Environmental requirements for Indoor
plantations.
Q.14 State the parameters governing the lightening design of a room.
Q.15 Highlight the design philosophy through project examples of architect Norman Foster
and his contributuion to contemporary
architecture.
Q.16 List all the information required in a Notice Inviting Tender.
Q.17 Sketch various techniques of water proofing on R.C.C. roofs.
Q.18 Explain the concept of ‘After Image’ as a visual phenomenon.
Q.19 Highlight the structural significance of Flying Buttress with reference to historic
architectural evolution.
Q.20 Indicate the criteria for selection of an Elevator System in a building.
Q.21 Draw a neat sketch of a domestic kitchen, 3.0 m x 2.4 m showing arrangement of
different functional areas as well as the necessary
services layout.
Q.22 Discuss the significance of air changes in a given room and mention the factors
governing the air change requirements.
Q.23 List the various forms of plants used generally in planting design.

SECTION B
PART II

Q.4 Explain with the help of sketches the Hippodamus planning concept.
Q.5 Mention the key elements used for interpretation of Aerial Photographs.
Q.6 Outline the concept of ‘Vermiculture’ in Solid Waste Management.
Q.7 What do you understand by ‘Sight Distance’ in a traffic flow?
Write the expression for ‘Braking Distance’ of a vehicle on a roadway.
Q.8 Explain the concepts of ‘Eminent Domain’ and ‘Police Power’ in relation to town
planning.
Q.9 An urban area is expected to accommodate during the next decade an additional
population of 65,000 to the existing population of 2,25,000. Estimate the existing
housing need and also the need at the end of the next decade based on the following
information.
Present household size - 4.75 persons
Future household size - 4.50 persons
Housing stock to be replaced
At the end of the decade - 4,500 units
Q.10 A residential neighbourhood with a population of 15,000 has to be serviced by a water
supply pipe 700 m in length. Assuming average rate of supply of water at 175 litres per
capita per day, maximum permissible velocity of flow of 1.5m/sec and head loss across
the pipe length not to exceed 8 m, design the diameter of the pipe using Hazen’s-
Willaim’s Nomogram for C.I. pipes.
Q.11 Explain the ‘Backwash Effect’ in development process.
Q.12 State the administrative procedure under the Land Acquisition Act, 1894 and the
recent amendments to expedite the process of land acquisition.
Q.13 Briefly outline the main recommendations of the National Housing Policy.
Q.14 Explain the hierarchy of recreational open spaces in Urban areas in terms of physical
size and facility standards.

Q.15 What do you understand by the terms Parking


accumulation, parking Index and Parking Turnover?
Q.16 Enumerate the factors which effect runoff from a catchment basin.
Q.17 Describe the salient features of the 73rd Constitutional Amendment Act of India,
and its role in empowering grass- roots level organizations.
Q.18Explain with examples the difference between formal and functional regions.
Q.19 Distinguish between ‘Street Furniture’ and ‘Street Hardware’, giving examples.
Q.20 Establish the relationship between ‘Human Vision’, ‘Distance’ and ‘Scale’ in urban
design.
Q.21 Explain the relevance of the term Amortization to urban finance.
Q.22 Highlight the major functions of water bodies in urban areas.
Q.23 What is the significance of ATTRIBUTES entity in AutoCAD. Mention the steps of
various commands associated with ATTRIBUTES entity.
GATE 2001
Q.1.1According to Aristotle, the ideal range of population for a city or polis is
(A) 2000 – 30,000 (B) 1,000 – 10,000 (C) 10,000 – 20,000 (D) 10,000 – 1,00,000
Answer: (C) 10,000 – 20,000
Aristotle’s ideal state:
Area: 60 km²
Population : 500-1000 household
Concept City
Concept Propagated Population Area
by
1 Plato’s city Plato 5040 -
2 Garden City Ebenezer 32,000 6000 acre
Howard
3 Broadacre City F L Wright - 1-5 acre per
household
4 Radiant City Le - -
Corbusier
5 Arcosanti Paolo Soleri 5000 25 acre
6 Neighbourhood Clarence 5000-6000 160 acre
Perry

Q.1.2‘Crash Time’ of a project indicates the


(A) optimum time in which a project is completed (B) shortest time in which a project
can be completed
(C) maximum delay that the project can undergo (D) point of time at which the
project will fail
Answer: (B) shortest time in which a project can be completed

Q.1.3‘Architrave’ is the
(A) slab of stone on top of classical order (B) lowest part of entablature
(C) wall supporting the weight of an arch or vault (D) topmost part of a classical Greek
column
Answer: (B) lowest part of entablature

Q.1.4The first garden city, Letchworth was designed by


(A) Ebenezar Howard (B) Antonio Sant Elia (C) Raymond Unwin (D) Clarence Perry
Answer: (C) Raymond Unwin
Garden City movement was propagated by Ebenezar Howard.
Clarence perry is known for Neighbourhood Development.

Q.1.5The minimum water supply requirement per head per day for residential purpose
including drainage and sanitation is
(A) 70 litre (B)135 litre (C)180 litre (D)210 litre
Answer: (B) 135 litre

Q.1.6The term ‘Conservative Surgery’ was coined by


(A) Patrick Geddes (B) Le Corbusier (C) Edwin Lutyens (D) Frank Lloyd
Wright
Answer: (A) Patrick Geddes

Q.1.7Level of Services (LOS) for a signalized intersection is defined in terms of


(A) traffic flow characteristics (B) mix of modes (C) delay (D) profile of
intersection
Answer: (A) traffic flow characteristics

Q.1.8The designer of the garden of Versailles, France was


(A) Andre LeNotre (B) Ebenezer Howard (C) Andrea Palladio(D) Patrick Geddes
Answer: (A) Andre LeNotre

Q.1.9The noise level for lecture rooms should be kept at


(A) 30 dB (B) 40 dB (C)50 dB (D)60 dB
Answer: (A) 30 dB

Q.1.10 The architectural movement, which was also popularly referred to as ‘Jazz Moderna’
is
(A) Arts and Crafts movement (B) Art Nouveau movement
(C) Art Deco movement (D) Ecole de Beaux Arts movement
Answer: (B) Art Nouveau movement
Q.1.11 Temples of many Sikharas are
(A) Indo- Aryan temples (B) Dravidian temples (C) Besara temples (D) Khajuraho
temples
Answer: (D) Khajuraho temples
Vesara is one of a number of terms for a distinct stylistic tradition of Indian
Hindu temple architecture primarily used in the Deccan and Central India,
between the Vindhyas and the river Krishna.

Q.1.12 Hardness of water is measured in parts per million by weight in terms of


(A) Calcium carbonate (B)Ferrous oxide (C) Carbon dioxide (D)Magnesium sulphate
Answer: (A) Calcium carbonate

Q.1.13 The concept of ‘Ekistics’ was propagated by


(A) Amos Rapoport (B) Constantine Doxiadis (C) Buckminster Fuller (D) Adam
Hardy
Answer: (B) Constantine Doxiadis

Q.1.14 Water seal in water closet is used to


(A) prevent foul gases from entering the house through external sewer pipe
(B) prevent water from leaking through the pipe
(C) keep the water closet moist throughout
(D) None of the above
Answer: (A) prevent foul gases from entering the house through external sewer pipe

Q.1.15 Which of the following is generally the largest internal source of revenue for Indian
municipalities?
(A) Property Tax (B) Stamp duty on property transfer (C) Conservancy Tarrif (D)
Water Tariff
Answer: (A) Property Tax
Governance Highest Revenue
Centre Union Excise Duty
State Stamp duty on
property transfer
Local Property Tax

Q.1.16 For proper ventilation, the ratio of opening area in a room to the total floor area of
the room, should be
(A) 1 : 6 (B) 1 : 10 (C) 1 : 12 (D) 1 : 20
Answer: Marks to all (It depends upon climate of the region)
As per NBC 2016
No Climate Opening area:
floor area
1 Warm and 1: 6
Humid
2 Temperate 1:8
3 Hot and Dry 1:10
4 Cold 1:12
Q.1.17 Resemblance of Graeco-Roman basilica is evident in a Buddhist
(A) Temple (B) Vihara (C) Chaitya (D) Stupa
Answer: (C) Chaitya

.1.18 Among the following architect- planner of foreign origin, the only one who is NOT
associated with an Indian city is
(A) Otto Koenigsberger (B)Frei Otto (C)Le Corbusier (D)Edwin Lutyens
Answer: (B) Frei Otto

Q.1.19 The most appropriate plan type for comprehensive physical planning of a
metropolitan region is
(A) District Plan (B) Action Area Plan (C)Town Planning Scheme (D) Structure Plan
Answer: (D) Structure Plan

Town Planning Schemes : In India, local


governments have increasingly relied on town planning schemes (TPS) to
influence urban growth and to finance affordable housing and basic infrastructure. TPS is a
hybrid land readjustment system that requires owners of agricultural land on the
urban fringe to transfer up to 40 percent of their land to the government for
redevelopment. In return, they receive cash compensation for the land taken and retain the
remaining 60 percent of their land, which is reconstituted as urban plots with public
infrastructure.The landowners can either build new homes on these serviced plots or sell the
plots to developers.The government builds roads and other public facilities on a portion of
the land received from the landowners and reserves a portion to sell at auction to cover the
costs of infrastructure development.

.1.20 In network analysis, the amount of time by which the start of an activity may be delayed
without hampering the start of a succeeding activity, is called
(A)Total Float (B)Free Float (C)Time Lag (D)Start Lag
Answer: (B) Free Float

Q.1.21 The most common method of protecting or preserving timber is


(A)Smoke drying (B) Salt Seasoning (C)Dry Seasoning (D) Creosoting
Answer: (C) Dry Seasoning

Q.1.22 Codex Atlanticus, a book propagating a new concept in urban planning, was
authored by
(A)Leonardo da Vinci (B)Biaggio Rossetti(C)Michelangelo (D)Leon B. Alberti
Answer: (A)Leonardo da Vinci
Q.1.23 The book ‘Design with Nature’ was written by
(A) Lawrence Halprin (B) Frederick Law Olmsted (C)Ian Mcharg (D) Andre Le Notre
Answer: (C) Ian Mcharg

Q.1.24 The extreme limit to which a body can be


repeatedly strained without fracture or permanent change of shape, is known as
(A) Compressibility (B) Resiliency
(C) Density (D) Elasticity
Answer: (D) Elasticity
Compressibility is a measure of the relative volume change of a fluid or solid as
a response to a pressure.
Resilency: the power or ability to return to the original form, position, etc., after
being bent, compressed, or stretched.
Density is mass per unit volume.
Elasticity: elasticity is the tendency of solid materials to return to their original
shape after being deformed.

Q.1.25 The locus of a fixed point outside a circle rotating on a fixed straight line is called
(A) Super Trochoid (B) Hypocycloid (C) Epicycloid (D) Involute
Answer: (A) Super Trochoid
Trochoid is the curve traced out by a point fixed to a circle (where the point may be on,
inside, or outside the circle) as it rolls along a straight line.
Term Position of Rolls over
Point
Trochoid on, inside, or Straight line
outside the
circle
Supertrochoi Outside the Straight line
d circle
Cycloid On the circle Straight line
Inferior Inside the circle Straight line
trochoid
Hypocycloid Within small Within
circle Large circle
Epicycloid Edge of small Outside
circle large circle
The cycloidal arch was used by architect Louis Kahn in his design for the Kimbell Art
Museum in Fort Worth, Texas. It was also used in the design of the
Hopkins Center in Hanover, New Hampshire, USA (Architect Wallace Harrison)
Involute: In mathematics, an involute is a particular type
of curve that is dependent on another shape or curve. An involute of a curve is the locus of a
point on a piece of taut string as the string is either unwrapped from or wrapped around the
curve.

Questions 2.1 – 2.25 carries two marks each


Q.2.1Which of the following is the closest approximation to a land of area measuring 1.5
acres?
(A)500 sq m. (B) 3035 sq m. (C) 6060 sq m. (D)6070 sq m.
Answer: (D) 6070 sq m.

.2.2 Two critical activities A and B need 5 and 8 days respectively, to complete. Another non-
critical activity C needs 5 days to complete with a free float of 2 days. If C is completed
in 4 days now, how much compression in project time is achieved?
(A) 0 days (B)1 day (C)2 days (D)3 days
Answer: (A) 0 days
Compression of project imply completing project before normal completion time. Since the poject completion time
is dependent on Critical path, activities along critical path has to be crashed to complete in shorter time
duration.Any non critical activity crash will not have any effect on Project completion time.
(explanation provided by Amit Das, Manager (Commercial) Air India)

Q.2.3In a seminar room of area 200 sq.m, 4 m height and total absorbing power of 120 m²
sabines, what is the reverberation time?
(A) 0.24 secs. (B)1.06 secs (C)1.52 secs (D)4.16 secs
Answer: (B) 1.06 secs
Soln: A standard reverberation time has been defined as the time for the sound to die
away to a level 60 decibels below its original level. Formula for calculation of
reverberation time.

Where RT60 = Reverberation time in sec


V = Volume in m³
Se = Total Absorption
Here Volume = 200 × 4= 800 m³and Se = 120 m² sabines

RT 60=

Q.2.4A town has a basic employment of 25,000 workers. If the basic: non basic ratio is 1:2.5
and the workers dependency ratio is 4:1, what is the population size of the town?
(A) 2,50,000 (B) 4,37,500 (C) 3,50,000 (D) 3,12,500
Answer: (B) 4,37,500
Basic employment= 25,000 ( basic activities produce export based goods/services)
Non Basic employment= 25,000 ×2.5= 62,500 9Non- basic activity produce goods/service
for domestic consumption)
Total Employment = basic employment + non basic employment
= 25,000 + 62,500= 87,500
Age dependency ratio is the ratio of dependents people younger than 15, or
older than 64 to the working-age population of those ages between 15-64.
Here, for every one working population there are 4 non working population.
So, non working population =
Total Population is 87,500 + 350000 = 4,37,500

Q.2.5For large and closed buildings, the fire hydrants should be located at distance of
(A) 90- 120 metres (B) 50 - 80 metres (C) 120- 150 metres (D) 150- 300 metres
Answer: (A) 90- 120 metres

Q.2.6 What is the rate of ventilation due to wind action if the free area of the window is 1
sq.m., and the wind speed is 1 m/hr. Assume the wind to be perpendicular to the
window.
(A) 1.0 cu.m/hour (B) 0.6 cu.m/hour (C) 0.3 cu.m/hour (D)0.1 cu.m/hour
Answer: (B) 0.6 cu.m/hour
Soln: for the wind which is perpendicular to window plane effective ventillation is 60 %
and for diagonal winds it is 30-35%.

Here Rate of ventillation = 1 ×1 × = 0.6 cu.m/hour

.2.7 The diagram below shows the relative distribution of different types of housing within a
total residential area of 150 hectares. If the net density of the plotted housing area is
350 ppha, how many people will be accommodated there?

(A) 20,000 - 25,000 (B) 25,000 - 30,000 (C) 30,000 - 35,000 (D) 35,000 -
40,000
Answer: no appropriate answer
(pie diagram angle not given, so technically an error)

Q.2.8As per 1991 Census, the urban component of India’s total population was between
(A) 10% and 20% (B) 20% and 30% (C) 30% and 40% (D) 40% and 50%
Answer: (B) 20% and 30%
2011 Census= 31.6 % urbanized.

Q.2.9Zinc coating is given over the steel reinforcement to


(A) increase tensile strength (B) reduce bending capacity (C) reduce
corrosion (D) increase bond strength
Answer: (C) reduce corrosion
Q.2.10 Which of the following sequence of names constitute the botanical name for
identification and use of landscape plants?
(A) species and order (B) order and genera (C) family and
species (D) genera and species
Answer: (D) genera and species

Q.2.11 For a four-way road intersection the following alternative traffic management schemes
are proposed.
i)signalized intersection;
ii) manually controlled intersection;
iii) rotary intersection
Considering the above alternatives, which of the following statement is INCORRECT ?
(A) All are equally space consuming
(B) ‘i’ is more power consuming than ‘ii’ and ‘iii’
(C) ‘ii’ is more power consuming than ‘i’ and ‘iii’
(D) ‘iii’ is less power and manpower consuming than ‘i’ and ‘ii ’
Answer: (A) All are equally space consuming

Q.2.12 A residential plot of 20 metre frontage and 25 metre depth is governed by the
development regulations of maximum F.A.R. of 200 and maximum plot coverage of 50
%. Upto what maximum height can the plot be built?
(A) 2 floors (B) 3 floors(C) 4 floors (D) 10 floors
Answer: (C) 4 floors

Q.2.13 Minimum visibility distance at a major road intersection, for a design speed of 80
kmph is
(A) 200 metres (B) 180 metres (C) 80 metres (D) 100 metres
Answer: (B) 180 metres
Minimum visibility Distance along
major roads at priority Intersections
Design speed of Minimum visibility
major road (kmph) distance along
major roads
(metres)
100 220
80 180
65 145
50 110
source: IRC-66-1975
Q.2.14 For accumulation of Re. 1/- for n years at a given rate of compound interest ‘i’, the
annual sinking fund is equal to
(A) 1 / i (B) (1 + i)n
(C) {(1 + i)n -1} / i (D) i / {(1 + i)n -1}
Answer: (D) i / {(1 + i) -1}
n

Calculation of Sinking fund

R=
R : periodic payment.
A : required to accumulate a sum of ‘A’ Rs
n: number of periods.
i: interest rate.

Q.2.15 Which of the following parametric conditions will provide the most suitable land for
intensive development of settlement?
(A) slope ≈ 4 %; soil ≈ silty loam aggregate; depth of water table ≈ 6 metres; vegetation ≈
moderate
(B) slope ≈ 20 %; soil ≈ aggregate sand; depth of water table ≈ 30 metres; vegetation ≈
barren
(C) slope ≈ 2 %; soil ≈ clay; depth of water table ≈ 1 metre; vegetation ≈ moderate
(D) slope ≈ 10 %; soil ≈ sandy loam; depth of water table ≈ 15 metres; vegetation ≈ dense
Answer: (A) slope ≈ 4 %; soil ≈ silty loam aggregate; depth of water table ≈ 6 metres;
vegetation ≈ moderate

Q.2.16 A rectangular room (internal dimension 5 m x 3 m) is made of 250 mm walls.


Calculate the volume of concrete needed for 25 mm Damp Proof Course.
(A) 0.425 cu.m. (B) 4.25 cu.m. (C) 0.4 cu.m. (D) 4.0 cu.m.
Answer: No appropriate answer
Soln: Using Centre line method
Length of centre line= 2 × (5.25 + 3.25) = 17 m
Surface area= 17 × 0.25= 4.25 m2
Volume of DPC = 4.25 × 0.025 = 0.106 cu.m (for wall)
Volume of DPC for room = 5 × 3× 0.025= 0.375
Total DPC = 0.106 + 0.375 = 0.481 cu. m

Q.2.17 With the following given data the velocity of undisturbed water flow in a rectangular
open drain channel is equal to
(Given data: coefficient of roughness = 0.11; liquid width = 1 metre; Depth of liquid = 60 cm; and slope = 1 in 200)
(A) 43 metres per sec (B)38 metres per sec (C) 31 metres per sec (D) 27 metres per
sec
Answer: No appropriate answer in options.
Coefficient of roughness (N)= 0.11, Liquid depth = 0.6 m, Liquid width = 1.0 m
Slope (S) = 1 in 200 = 0.005

Manning formula V= × ×

Where R= (R is known as hydraulic radius)

= = = 0.27

V= × ×
= 9.09 × 0.42 × 0.07 = 0.267 m/s
(soln. provided by Jamshed Baig, MURP, NIT Patna)

Q.2.18 The term ‘gentrification’ refers to


(A) Migration of fresh population into a crowded urban area.
(B) Settling of population in a new urban area.
(C) Settling of military forces in an urban area.
(D) Evacuation of population from a rural area.
Answer: (A) Migration of fresh population into a crowded urban area.
The term “gentrification” is coined by ‘Ruth Glass’ a British sociologist. The buying
and renovating of houses and stores in deteriorated urban neighborhoods by
wealthier individuals, which in effect increases property values and displaces low-
income families and small businesses.

Q.2.19 A tree of 13 metres height is required to shade the entire southern wall of a building of
4 metres height. At a solar altitude of 45º, what should be the maximum distance of a
tree from the building wall?
(A) 4 metres (B) 9 metres (C)13 metres (D)17 metres
Answer: (B) 9 metres

Q.2.20 The Dampers placed in the air conditioning duct are provided to control the
(A) Velocity and volume of air (B) Exhaust air and velocity (C) Foul air and exhaust
air (D) Volume of air and foul air
Answer: (A) Velocity and volume of air

Q.2.21 A site map drawn to scale 1: 10,000 shows six contour lines at 5 metres contour
interval. The highest contour elevation is 250 metres. The average distance between
the highest and the lowest contour lines on the map is 2.1 cms. What is the average
slope between the highest and the lowest contour elevations?
(A) 1 in 25 (B) 1 in 21 (C) 1 in 30 (D) 1 in 7
Answer: No appropriate answer
Highest contour = 250 and lowest contour = 225 so elevational difference= 25 m
2.1 cm on map = 2.1 × 10,000 = 21,000 cm = 210 m on actual terrain
So average slope = 25 : 210 = 1: 8.4

Q.2.22 One cm square area on a map represents 9 hectares. What is the scale of the map?
(A) 1 : 300 (B)1 cm = 30 metres (C) 1 : 30000 (D) None of the above
Answer: (C) 1 : 30000
1 cm × 1 cm ≈ 9 hectares = 90,000 m² = 300 m × 300 m.

1 cm ≈ 300 m = 30,000 cm

Q.2.23 If the original cost of the building is Rs 1,00,000/-, scrap value is Rs 1,000/- and the life
of the property is 99 years, The Annual Depreciation calculated by Straight line Method
is
(A)100 (B)1000 (C)90 (D)999
Answer: (B) 1000

Q.2.24 Loam category in soil texture is represented by


(A) sand : 20; silt: 50; clay : 30
(B) sand : 30; silt: 40; clay : 30
(C) sand : 60; silt: 20; clay : 20
(D) sand : 40; silt: 40; clay : 20
Answer: (D) Loam= sand : 40; silt: 40; clay : 20

Q.2.25 Which of the following operations should be the first one for setting up plane table
survey?
(A) Orientation (B) Levelling (C) Centering (D) Resection
Answer: (B) Levelling

SECTION B
PART I

Q.3 How was Roman Architecture influenced by the availability of natural building materials
during classical period?
Q.4 Through a schematic section, explain the function of ‘wind scoop’ in hot-dry climate.
Q.5 Name and sketch the five major elements governing ‘imageability’ of a place propagated
by Kevin Lynch.
Q.6 Enumerate the function of each of the following commands in Auto CAD
i) BLOCK
ii) DDEDIT
iii)DIMSCALE
iv)OFFSET
v) UCS
Q.7 Highlight the significance of ‘Building Automation Systems’ and identify its application
area in a building.
Q.8 Name and draw atleast fivetypes of foundation used for heavy R.C.C. structures.
Q.9 Discuss the concept and features of ‘Post Occupancy Evaluation’ in architecture.
Q.10 ‘Green trees in front of a building do not create fatigue to our eyes’- Why?
Q.11 How are the recesses on external surfaces of Hindu temple functional?
Q.12 Explain the influence of wooden construction style in Buddhist Architecture.
Q.13 Draw a layout of a toilet of size 1.8 metres x 3.0 metres, showing wasbasin with counter,
bathtub, European W. C., water heater for hot water supply and all other fixtures.
Draw the isometric view of all the pipelines (water supply, sanitary and drainage)
showing the positions of fixtures, valves, etc. (Drawing need not be scale).
Q.14 Mention the salient features of the architectural work of Alvar Alto. Give any one example
of his famous works.
Q.15 Explain the factors to be considered in the prevention of heat gain of the structure during
summer months.
Q.16 Explain the differences between ‘Sound Focci’ and ‘Dead Spots’ in an auditorium.
Q.17 Illustrate with sketches any five different defects of timber.
Q.18 How does purity of air play a significant role in the comfort of people affected by
ventilation system?
Q.19 What are the reasons for efflorescence on porous building materials?
Q.20 Explain with illustration, the philosophy behind the design of Jawahar Kala Kendra,
Jaipur.
Q.21 Outline the salient features of the historical landscape gardening style of Japan.
Q.22 With an illustrative example, highlight the characteristics, utilities and effectiveness of
Ferro- cement concrete in building construction.

PART II
Q.3 Outline the salient features of the ‘Central Place Theory’ of Christaller. Use a diagram
to explain your answer.
Q.4 Briefly state the Environmental Impacts of highways.
Q.5 What was the major planning issue that the plan of Radburn helped to resolve? Show
through neat schematic diagram how the Radburn plan achieved this objective.
Q.6 State the common methods of domestic refuse disposal.
Q.7 In project management, differentiate between PERT and CPM. How the expected time of
an activity is computed in PERT?
Q.8 Distinguish between ‘linear’ and ‘radio-centric’ pattern of physical form of cities in terms
of
(i) Transportation (ii)Utility Networks
Illustrate your answer with schematic diagrams.
Q.9 Highlight briefly, the features and application of Geographical Information Systems
(GIS) in decision making in planning.
Q.10 (a)Differentiate between ‘gross’ and ‘net’ residential densities for a planned sector.
(b) For a sector of 2 km x 1 km size and gross density of 350 ppha, what will be the net
density if non residential land- use constitutes 30 % of the land area?
Q.11 Outline the differences between hot- humid and hot- dry climate, in terms of appropriate
building and urban forms. Illustrate their differences in mass-void relationship of urban
forms.
Q.12 Describe briefly the ‘Activated Sludge Process’, highlighting its operations.
Q.13 What is difference between ‘direct subsidization’ and ‘cross subsidization’ in the
provision of urban land, housing or
infrastructure in India? Which among these is relatively advantageous and why?
Q.14 (a)Distinguish between ‘Time Mean Speed’ and ‘Space Mean Speed’.
(b) Estimate the average travel speed of an urban road based on the following
information. (Length of the segment of
the urban road = 3 km)
Vehicle No Travel time (min.)
1 4.5
2 3.45
3 3.0
4 6.0
Q.15 Discuss the essence of ‘Concordance’ and ‘Discordance’ analysis technique, through a
suitable example.
Q.16 Name the specific laws governing each of the following activities.
(i) Permission of building construction in a city.
(ii) Notification of master plan area of a city.
(iii) Permissible use of a plot of land in a city
(iv) Impact assessment of a highway development.
(v) Eviction of a tenant by a landlord.
Q.17 The 1991 Housing Census for a town of 2.5 lakhs with average household size of 5
persons shows a total number of 40,000 dwelling units. If the annual exponential
population growth rate is 2.0 %, what should be the average annual rate of dwelling unit
supply to meet housing shortage in 2001? Assume a constant average household size
and annual obsolesce rate of 5%.
Q.18 In SWOT Analysis, what do the individual letters stand for? Give one example for each
of these elements in the context of a cities/ place or an organization.
Q.19 Outline the various urban planning and design strategies towards conservation of ground
water resources.
Q.20 Explain the characteristics of ‘maritime desert climate’. Name a place located in this
climate.
Q.21 What are the basic components of an ideal remote sensing system?
.22 Name the Amendment Act and the Schedule of the Constitution of India that provides for
the responsibilities and Functions of urban local bodies. Outline the steps necessary to
empower the local bodies to carry out these functions.
GATE 2002
. 1.1 The colour code for the storage of objects with radiation hazards according to the
Occupational Safety Hazards Act (OSHA) is
(A) red with black (B) purple with yellow(C) orange with blue (D) green with white

Answer: (B) purple with yellow


OSHA-Specified Barricade Tape Colors

1. Red / white for Fire Prevention and Protection Equipment

2. Black / white for Housekeeping and Aisle Marking

3. Magenta / yellow for Radiation Hazards

4. Green / white for Safety and First Aid

5. Blue / white for Defective Machinery

6. Orange / white for Traffic and Caution Warning


7. Black / yellow for Physical
Hazards

Helmet Colour code for construction site.


White: Engineers, Managers, Supervisors, and foreman
Blue: Electrician, Carpenter, Tech Operators
Red: Fire fighters
Green: Safety officer
Grey: Site visitors
Yellow: Labourers, Earth movers
Brown: Welders, and works with high heat application

Q.1.2 Terra- cotta is a kind of earthern work processed by


(A) drying in the sun
(B) burning at high temperature
(C) mixing with lime
(D) compacting under pressure
Answer: (B) burning at high temperature
adobe: sun dried bricks
cob: contains proportioned amounts of soil, clay, water, manure, and straw and
left to dry naturally.

Q. 1.3 Sand blasting is a technique used for cleaning the surface of


(A) stone work (B) concrete work (C) wood work (D) earth work
Answer: (A) stone work

Q. 1.4 Series of column in a straight row arrangement is termed as


(A) Orthostyle (B) Peristyle
(C) Tracery (D) Colonia
Answer: (A) Orthostyle (adjacent figure is orthostyle)
Distyle- A Distyle is a small temple-like structure with only two columns in front.
Distyle in
Antis- Distyle in
antis denotes a
temple with the
side walls
extending to the
front of the
porch

Prostyle- Prostyle is
an architectural term
defining a row of
columns in front of a
building.

Amphiprostyle- Column in both front and back


Peristyle: Peri means “around” and style means “column”, so a peristyle is a
place with columns all the way around it like in a courtyard.
eg- Durg Temple at Aihole has peristyle column all around
- Parthenon temple has peristyle column all around

hypostyle: roof supported only on column.


eg- Temple of Luxor in Egypt.

tracery is the stonework elements that support the glass in a Gothic window.

Q. 1.5 When the picture plane moves towards the observer, the size of the picture in
perspective
(A) does not change (B) increases (C) decreases (D) gets elongated
Answer: (C) decreases

Q. 1.6 The term ‘Vedika’ in Buddhist architecture is used for


(A) railing around stupa (B) crown umbrella of stupa
(C) decorative entrance of stupa (D) niche in the wall of stupa
Answer: (A) railing around stupa
Hermika: railing on top of stupa
Chhatri: crown umbrella of stupa
Anda: hemispherical mass of stupa
Toran: decorative entrance of stupa

Entrance (Gateways) nomenclature:


Japanese: torii
Chinese: paifang
Korean: hongsalmun
Egyptian: pylons
Stupas: torana
South Indian temple: gopuram
Great Southern Gate of Todai-ji located in the city of Nara, Japan : Nandaimon
(UPSC 2016)

Q. 1.7 Capital value of a property is equal to


(A) net rent value per year multiplied by sinking fund (B) annual rent minus
outgoings
(C) net annual rent multiplied by years purchase (D)gross annual rent plus
overhead costs
Answer: (C) net annual rent multiplied by years purchase

Q. 1.8 ‘Demac gauge’ is a device for


(A) measuring mechanical strain in structures (B) measuring the dampness of inner
walls
(C) checking the thermal conductivity of walls (D) gauging air temperature
Answer: (A) measuring mechanical strain in structures

Q. 1.9 Funicular polygons are


(A) Stress diagrams (B)Force diagrams (C) Regular shapes (D)Irregular shapes
Answer: (B) Force diagrams

Q.1.10 The duct size for an air conditioning system is dependent on amount of air flow and
its
(A) velocity (B) temperature (C)relative humidity (D) latent heat
Answer: (A) velocity

Q.1.11 The botanical name of ‘Neem’ tree, used for roadside landscaping is
(A) Lagerstroemia speciosa (B) Ficus benghalensis (C) Kleinhama hospital
(D)Azadirachata Indica
Answer: (D) Azadirachata Indica

Q.1.12 Symbolic representation of water by sand is frequently found in


(A) Mughal Garden (B) English Garden (C) Japanese Garden (D)Moorish Garden
Answer: (C) Japanese Garden

Q.1.13 Kenzo Tange developed the concept of


(A) Deconstructivism (B) Arcology (C) Metabolism (D) Anamorphosis
Answer: (C) Metabolism

Q.1.14 The maximum slope in a for wheel chair movement is


(A) 1: 8 (B) 1: 12 (C) 1: 20 (D) 1: 25
Answer: (B) 1: 12
The CPWD manual recommends a slope of 1:12 for wheelchairs.

Q.1.15 Preserving timber by Charring is carried out for


(A) Wooden piles (B) Wooden partitions(C) Wooden doors (D) Wooden truss
Answer: (A) Wooden piles
Charring: This process is useful for such timbers that are partly placed in the
ground, such as posts or timber used in the foundation of buildings, as
the carbonic coat or charcoal covering which incrusts it by this
process, preserves for many years what otherwise from continual moisture
would sooner decay.

Q.1.16 In construction work, Hoes are used for


(A) transporting materials (B) excavating trenches (C) mixing concrete (D)
compacting sand
Answer: No appropriate answer
Both B and C is correct.

Q.1.17 ‘Swastika’ form of settlement layout in ancient Indian town planning is basically
(A) a grid iron pattern(B) a radial pattern (C) a ring radial pattern (D) an informal
pattern
Answer: (A) a grid iron pattern

Q.1.18 Pompidou Centre in Paris , designed by Renzo Piano, illustrates the concept of
(A) Biology analogy (B) Mechanical analogy (C) Romantic analogy (D) Pattern
Language analogy
Answer: (B) Mechanical analogy
Richard Rogers & Renzo Piano: Designed Pompidou Centre in Paris which is a
high tech. architecture building.

Q.1.19 The Kandariya Mahadeo Temple in Khajuraho is an example of


(A) Rock Cut architecture(B) Nagara architecture (C) Dravida Architecture (D) Indo
Saracenic architecture
Answer: (B) Nagara architecture
Rock cut Architecture: Ajanta, Ellora, Elephanta caves
Three prominent styles of Temples and geographical boundaries.
Nagara style: Mount Kailash to Vindhayas
Visera style: Vindhayas to River Krishna
Dravidian style: River Krishna to Kanya Kumari.
Indo Saracenic Architecutre: Influence of Indian style on Persian and Collonial
style.
Q.1.20 Decision on phasing is required in the design of
(A) channelized intersection (B) priority intersection (C) grade separated
intersection (D) signalized intersection
Answer: (D) signalized intersection

Q.1.21 For street lighting maximum lumen per watt is obtained from
(A) sodium vapour lamps (B) mercury vapour lamps (C) incandescent
lamps (D) fluorescent lamps
Answer: (A) sodium vapour lamps

Q.1.22 ‘Logit Model’ for discrete choice analysis has been developed by
(A) A.O. Hirshman (B) D. McFadden (C) P. Nijkamp (D) T. Scitovsky
Answer: (B) D McFadden

Q.1.23 ET index can be obtained from nomograms showing


(A) DBT, WBT and air velocity (B) DBT, AH and RH (C) AH, RH and air velocity
(D) WBT and air velocity
Answer: (A) DBT, WBT and air velocity

Q.1.24 SKPOLY command in AutoCad helps in


(A) sticking together two polylines (B) drawing street kerbs with polyline
(C) creating polylines while sketching (D) creating shortest path along polylines
Answer:

Q.1.25 Class II towns as defined in the Indian Census relate to a minimum population size
of
(A) 2,00,000 (B) 1,50,000 (C) 1,00,000 (D) 50,000
Answer: (D) 50,000

Question 2.1 -2.25 carries two marks each.


Q.2.1A town having a population of 88,236 (in the year 1971) has shown an arithmetic
increase in its population. The growth rate was 12 % per decade. The population in
2001 was
(A) 97,000 (B) 1,03,007 (C) 1,20,000 (D) 2,59,000
Answer: (C) 1,20,000

Decadal growth rate = 88,236 × = 10,588. Since growth is arithmetic increase, in


three decade total growth will be 10,588 × 3= 31,764
Total Population = 88,236 + 31,764 = 1,20,000

Q.2.2The series 1/1, 1/3, 1/5, 1/7, 1/9 ……… is known as


(A) geometric series (B) harmonic series (C) Fibonacci series (D)golden mean
series
Answer: (B) harmonic series

Q.2.3For a Split Complementary colour scheme, the primary red colour should be
associated with
(A) blue violet – red violet (B) red orange –yellow orange
(C) blue green – yellow green (D) red violet – blue green
Answer: (C) blue green – yellow green

Q.2.4The fixed end moment of a cantilever beam with a u.d.l. of 20 kN/m and span of 3 m is
(A) 60 kN m (B) 90 kN m (C) 120 kN m (D) 150 kN m
Answer: (B) 90 kN m

Q.2.5The buildable area available on a plot of one hectare with a FAR of 2.25 is
(A) 15500 sqm (B)20500 sqm (C)22500 sqm (D)25500 sqm
Answer: (C) 22500 sqm

Q.2.6For an open drain if the ratio of the area to the wetted perimeter is tripled, then the
velocity of water in the drain increases by
(A) two times (B) three times (C)four times (D)five times
Answer: (A) two times

According to Manning’s formula V= (R )2/3(S) ½


Where V= velocity in m/s
N= co-efficient of Resistance
R= hydraulic radius=
S= slope

In the above Question=


Which gives ans= 3 2/3 = 2

Q.2.7In transportation planning Link- Node diagrams are required for


(A) estimation of zonal trips (B) determination of trip characteristics
(C) trip assignment along roads (D) determination of mode characteristics
Answer: (C) trip assignment along roads
In the case of transportation networks, the nodes may represent points (or locations)
from which traffic is produced or to which traffic is attracted. The nodes also represent
intersections. The links represents roads or movements.
In the conventional transportation forecasting model, following four steps are
involved.
a. trip generation- predicting the total number of trips generated and attracted to
each zone of the study area. (Regression model)
b. trip distribution- number of trips that occur between each origin zone and each
destination zone. (gravity model)
c. mode choice- what mode of transport will be used. (multinomial logit model)
d. Route assignment- selection of route between origin and destination.
(minimum tree search)

Q.2.8Water while falling down from a horizontal surface with an edge lip creates
(A) glassy sheet flow (B )vertical angular rhythm
(C) agitated frothy surface in multiple layers (D) none of the above
Answer: (A) glassy sheet flow

Q.2.9The R. L. at the basement floor of a building and the R. L. at the road surface are
93.00 and 94.85 respectively. If the plinth height from the road is 600
mm then the depth of the basement is
(A) 1.25 (B) 1.85 (C) 2.35 (D) 2.45
Answer: (B) 1.85
Depth will be 1.85 and height of basement will be 1.85 m + 0.60 m= 2.45 m.

Q.2.10 Along a critical path crashing is carried out for the activity that has
(A) minimum cost-time slope (B) maximum cost- time slope (C) minimum cost
slope (D) maximum time slope
Answer: (C) minimum cost slope

Q.2.11 Archimedean solids have


(A) regular faces (B) right angles (C) irregular faces (D) none of the above
Answer: (A) regular faces
Archemedian Solid: Soild with two or more regular polygonal face with identical
vertex.There are 13 Archemedian solids.
Platonic solids: Solids with regular polygonal face with identical vetex. There
are five platonic solids.

Q.2.12 If the scale of the map is 1: 30,000, then 1 sq cm area of the map would represent
(A) 9.00 Hectare (B)15.00 Hectare (C)17.30 Hectare (D)30.00 Hectare
Answer: (A) 9.00 Hectare

.2.13 The oxygen demand in the initial stage of the biological decomposition of sewage is due
to the presence of
(A) nitrogenous matter (B) phosphate matter (C) carbonaceous matter (D) calcerous
matter
Aswer: (C) carbonaceous matter

Q.2.14 Number of modular bricks required for 10 cum brickwork is


(A) 4500 (B) 4750 (C) 5000 (D) 5225
Answer: (C) 5000

Q.2.15 GPS instrument is used for determining


(A) Temperature of a place (B) Lat- Long of a place (C) Rainfall of a
place (D) Wind velocity of a place
Answer: (B) Lat- Long of a place
GPS gives lat-long, time and speed if in motion, along with direction of motion and
altitude.

Q.2.16 An ideal example of a tensile roofing


system is found in
(A) Sydney Opera House by J Utzon
(B) Olympic stadium in Tokyo by Kenzo Tange
(C) TWA airport terminal by Eero Saarinen
(D)Bahai Temple in New Delhi by Fairburz Sahba
Answer: (B) Olympic stadium in Tokyo by Kenzo Tange

Q.2.17 In measuring traffic noise, the noise level L90 represents


(A) background noise level (B) lowest noise level (C)average noise
level (D) peak noise level
Answer: (A) background noise level
L90 is the value that the sound level was above 90 % of the time.

Q.2.18 The process of acquiring excess land for selling at a higher price after the
completion of the project is known as
(A) Encroachment (B) Enforcement (C) Betterment (D) Recoupment
Answer: (C) Betterment

Q.2.19 Cadastral map shows


(A) cropping pattern (B) land subdivisions (C) land ownership (D) depth of water
bodies
Answer: (C) land ownership

Q.2.20 Participation ratio is obtained from


(A) work force and population in work age group (B) work force and total population
(C) male work force and female work force (D) work force and in-migration
Answer: (A) work force and population in work age group
Q.2.21 To perceive the backdrop of a building more than its facade detail and the total
facade simultaneously, the height distance relationship should be
atleast
(A) 1:1 (B) 1:2 (C) 1:4 (D) 1:6
Answer: (D) 1:6
1:1 - Tends to see detail more than façade
1:2 – Object in relation with surrounding
1:3 – All together with details
1:4 – object as an edge.
1:6 - backdrop of a building more than its facade detail

Q.2.22 Marble stone used in buildings, is a type of


(A) Metamorphic rock(B) Calcerous rock (C) Igneous rock (D) Sedimentary rock
Answer: (A) Metamorphic rock
Igneous rock- Granite
Sedimentary rock- sand stone, limestone.
Metamorphic rock- gneiss, slate, marble, schist, quartzite.

Q.2.23 IRDP in India is associated with


(A) regional development (B) rural development(C) riverfront development (D) road
development
Answer: (B) rural development
Integrated Rural Development Program.

Q.2.24 Incentive Zoning is related to


(A) provision of extra advantages to developers (B) provision of extra advantage to
common people
(C) provision of extra advantage to traffic (D) provision of extra advantage to
handicapped citizens
Answer: Both A and B correct.

Q.2.25 When the arrival rate of bank customers is lower than the service rate at the
counters
(A) queues ae quite often formed (B) queues are never formed
(C) queues of infinite length are formed (D) queues of particular length are always
formed
Answer: (B) queues are never formed

Section B (75 marks)


Attempt EITHER PART- I OR PART- II
Q.3 Indicate the design principles of interior landscaping for an air conditioned exhibition
pavilion.
Q.4 In architectural design what is represented by ‘Blue Series’ and ‘Red Series’? Explain
the underlying concepts and their applications.
Q.5 Enlist the advantages of adopting membrane structural system in architecture
Q.6 Design and draw the application of ‘Occult Balance’ and ‘Multidirectional Symmetry’ in
window grills of size 1200 mm x 1200mm.
Q.7 Mention five important factors in designing a reading enclosure for a college library.
Q.8 Explain the ‘Unit and Mullion curtain walling system for an office exterior with an
exploded view highlighting all individual
components.
Q.9 Explain the basic difference between ‘Minimalism’ and ‘Deconstruction’ in modern
architectural movements.
Q.10 What should be the considerations for externally illuminating the famous shore temple
at Mahaballipuram?
Q.11 Explain the principles of ‘Noticeable Absence’ and Distant Netting in the urban built
form design.
Q.12 Indicte the characteristics of a ‘Turn-key Job’ in building construction
Q.13 A three-hinged parabolic arch hinged at the crown and springing points, has a horizontal
span of 16 m and a central rise of 8 m. A concentrated load of 100 kN is applied on the
arch at a distance of 4 m from one of the springing point. Find out the reactions at the
springing points.
Q.14 Indicate AutoCad commands for drawing a regular tetrahedron illustrating all the steps.
Q.15 Sketch a three dimensional view of a shower cubicle showing the tray, water supply
connection, waste water outlet and other accessories.
Q.16 Explain the advantages of an ‘Open Office System’ as compared to traditional ‘Box
Office System’.
Q.17 How does the concept of “Territoriality” ensure natural survillence of an entrance lobby
in a residential apartment building?
Q.18 Explain with annotated sketches, similarities and difference between the spatial
organizations of a typical mosque and a typical church.
Q.19 Mention the environmental utility of preserving water bodies in densely built urban
areas.
Q. 20 An academic campus of 80 Hectares has 30 % area under student’s enclave, 25 %
area under staff housing and 15 % area under major roads (of which 50 %
is within academic complex). There are 5000 students and 1800 faculty and
staff members having average family size of 4.45 living in the campus.
Find out the gross residential population density of the campus.
Q.21 Draw the cross section of a passenger lift well for an eight storied building, mentioning
all the important components and
critical dimensions.
Q.22 Enlist special planning and design considerations for a housing complex in an
earthquake prone area.
PART II
Q.3 What should be the major considerations for landscaping a steep terrain along a
highway?
Q.4 A city had grown geometrically at a rate of 7 % per annumfrom 1991 to 2001. In the year
2001, the city had a population of 701276. The net migration rate for the city during the
above perioed had been 10 per thousand population. What was the net migration to the
city during 1991 to 2001.
Q.5 Mention the speed calming technique to be adopted for a busy street passing through a
residential neighbourhood.
Q.6 Mention the factors, which weaken the sense of enclosure for an open space
surrounded by buildings.
Q.7 A project network has two activities, A & B on the critical path. The pessimistic time
estimate for A & B is 25 weeks and 29 weeks respectively. The optimistic time estimates
are 10 weeks and 12 weeks for A & B respectively. Determine the standard deviation for
the critical path.
Q.8 Indicte the factors that govern the utility of para- transits in urban areas.
Q.9 What are the parameters to be considered for determinimg the sample size for a
household survey in an urban area?
Q.10 Why green spaces should be given more importance in the building of towns in hot
climates?
Q.11 State the major advantages of vertical aerial photographs over a map for its application
in urban planning.
Q.12 Where check valves are used? Sketch a check valve labeling its parts.
Q.13 What is ‘Human Development Index’? What are the advantages of using this index?
Q.14 State the important stages in land acquisition procedure while acquiring land for public
purpose.
Q.15 Explain through illustration the oxygen sag curve for determining the DO level of a
stream?
Q.16 What are various forms of non-property taxes that can be imposed by a municipality for
mobilization of development funds?
Q.17 What is ‘Incremental Housing’? Why it is one of the preferred options in providing
housing for economically weaker section of people?
Q.18 Indicate the factors to be considered for describing the economic profile of a region.
Q.19 Mention various criteria for enlisting heritage structure and precincts.
Q.20 Enlist the precautionary measures for disposal of hazardous wastes produced in urban
areas.
Q.21 Which factors are to be considered for ‘ Life Cycle Cost Analysis’ while evaluationg
alternative materials in any urban construction project?
Q.22 Mention the limitations of GIS as a planning tool. Which are the other support systems
that are to interlinked with GIS to make it an effective planning tool?
GATE 2003
Q. 1 The Sanskrit school of Auroville is a work of
(A) modular precast concrete blocks (B) ferro cement and terra forming
(C) advanced steel trusses and space frames (D) adobe and mud plastering
Answer: (A) modular precast concrete blocks

Q. 2 The Baroque city of European Renaissance and the city Beautiful Movements though
separated by a period of 300 years have one of the following things in common
(A) palaces and sculptures (B) colonnades and forum
(C) grand open spaces and waterways (D) radial streets and concentric landscaping
Answer: (D) radial streets and concentric landscaping

Q. 3 The survey of India toposheet stating 1:50,000 refer to


(A) graphic scale (B) spatial scale (C) time scale (D) numeric scale
Answer: (A) graphic scale

Q. 4 Improvement of physical condition of existing population in inner city areas is called


(A) incumbent upgradation (B) gentrification (C) slum
upgradation (D) homesteading
Answer: (A) incumbent upgradation

Q. 5 The landscape designer of Tara Group of Housing, New Delhi was


(A) Charles Correa (B) Jasbir Sawhney (C) Ravindra Bhan (D) Raj Rewal
Answer: (C) Ravindra Bhan

Q. 6 Mline command in AutoCAD 2000


(A) create multiple parallel lines (B) draw two parallel lines (C) draws
polyline (D) moves a line
Answer: (A) create multiple parallel lines

Q.7 In Greek structures, distyle, prostyle, and amphiprostyle are processes of


(A) symmetry (B) repitition (C) hierarchy (D) clustered organization
Answer: (C) hierarchy

Q. 8 The proportion of Parthenon corresponds to


(A) blue series and the spiral (B) geometric series and Archimedian solids
(C) rules of Protogoras and Euclidian anthropometrics (D) entasis and the golden
mean
Answer: (D) entasis and golden mean

Q. 9 Resistance of water to the passage of light through it is the measure of


(A) colour (B) turbidity (C) hardness (D)portability
Answer: (B) turbidity

Q. 10The term Directrix and Eccentricity are mainly connected with


(A) anti classic shells (B) trabeated systems
(C) gothic arches (D) pneumatic or tensigrity structures
Answer: (A) anti classic shells

Q. 11Identify the variety of plastic used to prepare ‘Perspex’


(A) PVC (B) nylon (C)acrylic (D)polysterene
Answer: (C) acrylic
Q. 12 The number of vehicles moving in a specified direction on a given lane and passing a
given point during a specified unit of time is called
(A) traffic density (B) traffic volume (C) traffic capacity (D) basic capacity
Answer: (B) traffic volume

Q. 13The tool to recover the cost of providing new services and infrastructure is known as
(A) property tax (B) land tax (C) development charge (D) professional tax
Answer: (C) development charge
Q. 14 In an urban electricity network, load factor is defined as
(A) average demand divided by maximum load (B) average demand divided by
minimum load
(C) maximium demand divided by average demand (D) minimum demand divided by
average demand.
Answer: (A) average demand divided by maximum load
The value of load factor is less than one.

Q. 15The literal meaning of Feng- shui is


(A) house and its houseland (B) gender sensitiveness (C) efficiency and design (D)
wind and water
Answer: (D) wind and water

Q. 16Dhanurmusti and Dhanurgraha are basic modular units used in ancient India. They
respectively represented the twin system of
(A) environment and ecology (B) architecture and townplanning
(C) house and temples (D) microcosm and macrocosm
Answer: (B) architecture and townplanning

Q. 17With respect to the contents of a base map, identify the odd one out
(A) physical features (B) topological features
(C) economic features (D) planning and administrative boundaries
Answer: (C) economic features

Q. 18A Psychrometric chart concerns the behavior of


(A) mixture of air and water vapour (B) air and ambient temperature
(C) operative and outdoor temperature (D) people in their communities
Answer: (A) mixture of air and water vapour

Q. 19The following names are associated with the Bahaus movement


(A) Louis Sullivan, Paulo Soleri (B) Paul Klee, Marcel Breuer
(C) Richard Neutra, Paul Rudolph (D) Jane Drew, Ralph Rapson
Answer: (B) Paul Klee, Marcel Breuer

Q. 20 The sampling technique used to “select from every sub- group of population in atleast in
at least one stage of the procedure” is known as
(A) systematic sampling (B) cluster sampling
(C) stratified sampling (D) multi- stage- random sampling
Answer: (C) stratified sampling
Random sampling : Suppose there are N=850 students in a school from which a
sample of n=10 students is to be taken. The students are numbered from 1 to 850.
Since our data runs into three digits we use random numbers that contain three
digits. All numbers exceeding 850 are ignored because they do not correspond to
any serial number in the data. In case the same number occurs again, the repetition
is skipped.
Systematic sampling: In this method first we have to number the data items from 1
to N. Suppose the sample size be n, then we have to calculate the sampling interval
by dividing N by n. And generate a number between 1 and N/n and select that data
item to be in the sample. Other items in the sample are obtained by adding the
sampling interval N/n successively to the random number.
Cluster sampling is a sampling technique where the entire population is divided into
groups, or clusters, and a random sample of these clusters are selected.
Stratified Sampling : When sub-populations vary considerably, it is advantageous to
sample each subpopulation (stratum) independently. Stratification is the process of
grouping members of the population into relatively homogeneous subgroups before
sampling.
Multistage sampling is a complex form of cluster sampling. Cluster sampling is a
type of sampling which involves dividing the population into groups (or clusters).
Then, one or more clusters are chosen at random and everyone within the chosen
cluster is sampled.

Q.21 Which one of the following sequences is in the correct ascending order
(A) lithosphere, barysphere, mesosphere, stratosphere (B) hydrosphere, troposphere,
ionosphere, exosphere
(C) barysphere, lithosphere, ionosphere, atmosphere (D) ionosphere, exosphere,
troposphere, barysphere
Answer: (B) hydrosphere, troposphere, ionosphere, exosphere

Q. 22In a land use map, identify the colour code used for the manufacturing industry
(A) yellow (B) blue (C) purple (D)black
Answer: (C) purple

Q. 23The technique that can alter the scale of the building is


(A) facade articulation (B) visual analysis (C) photogrammetry (D)space programming
Answer: (A) facade articulation

Q. 24Buddhist and Greek elements met each other to form


(A) Pandya style in South India (B) Gandhara style in Bactria
(C) Assyrian phase in Persia (D) Catacombs in Judea
Answer: (B) Gandhara style in Bactria

Q. 25 In AutoCAD 2000, which command will allow you to select two point pairs, move, rotate
and scale the selected objects in 2D or 3D to adjust with other objects.
(A) Rotate (B) Qselect (C) Align (D) Skpoly
Answer:

Q. 26Ideal width to depth ratio of an air conditioning duct is


(A) 1: 6 (B) 3: 1 (C) 1: 3 (D) 6: 1
Answer: (B) 3: 1
Aspect ratio of AC duct should be within range of 4:1. Square is best but it will take more
depth.

Q. 27Increase in water cement ratio in concrete


(A) decreases strength (B) increases strength(C) increases durability (D) decreases
workability
Answer: (A) decreases strength

Q. 28The concept of obtaining maximum utility per unit length of road is called
(A) capacity building (B) facilities information system (C) saturation
system (D) trip distribution
Answer: (C) saturation system
Q. 29The hanging gardens of Babylon were built in
(A) the flood planes of Tigris (B) the banks of river Euphrates
(C) the slopes of the Median hills (D) the coastlines of the Persian Gulf
Answer: (B) the banks of river Euphrates

Q. 30Devolution of fund refers to


(A) income from property tax
(B) specific project grant
(C) mechanism of sharing revenue by the highr level government to the lower level
government.
(D) income from revenue resource
Answer:(C) mechanism of sharing revenue by the highr level government to the lower
level government.
How do States get revenue from the Center
a. As states share of taxes from the Gross Tax Revenue. (This is extra-budgetary)
b. As Centrally Sponsored Schemes from the Scheme Expenditure. (Based on
Budget Allocations).
c. As Transfer to States from the Transfers, Expenditure, and Other Expenses.
d. Other grants or loans. (Based on Budget Allocations).

Q. 31Components of Lowry model may be identified as


P transportation modes
Q basic employment
R residential population
S public utilities
T service employment
(A) Q, R,T (B) Q, R, S (C) P, R,T (D) P, Q, R, T
Answer: (D) P,Q, R, T
The Lowry model takes three major components of a city, namely, population,
Service employment/basic employment and the means of transport between
them, and describes the interaction between them and these interactions determine
urban change by allocating population (Residential Land Use) and employment
(Service and Commercial Land Use)) across the various zones of the city.
LandUse Models
Lowry Model: Considered to be the first transportation / land use model (1964),
it links two spatial interaction components. The first calculates spatial
interactions between basic employment activities and zones of residence,
while the second calculates spatial interactions between service
employment activities and zones of residence.
ITLUP: The Integrated Transportation and Land Use Package is composed of
a residential allocation model, an employment allocation model, and a travel
demand model.
MEPLAN: This model is a derivative of the Lowry model, since it is based on
the economic base theory. It considers the two components of the
transportation / land use system as markets, one market for land use and one
market for transportation.
Cellular automata: A new range of models where space is represented as a
grid (raster) with a set rule enforced to govern the state of a cell depending on
the configuration of its adjacent cells.

Q. 32Assume that a camera with a 152 mm focal length lens was flown over a flat terrain
located at 1200 m above mean sea level. If the scale of the aerial photography is 1:
50,000 calculate the flying height above the terrain
(A) 6400 m (B)7600 m (C)8800 m (D)4166 m
Answer: (B) 7600 m
Soln:

Q. 33A town of 2,25,000 population in 1981 has exhibited decadal growth rates of 25%, and
30% during 1991 and 2001 respectively. Estimate the population in 2011 having a 40 %
decadeal growth rate
(A) 3,50,000 (B) 6,14,250 (C) 5, 11,875 (D) 4,50,000
Answer: (C) 5, 11,875
Population in 1981= 2,25,000
In 1991 (25% growth) = 2,25,000 ×1.25= 2,81,250
In 2001 (30% growth) = 2,81,250 ×1.30 = 3,65,625
In 2011 (40% growth) = 3,65,625× 1.40 = 5,11,875

Q. 34Match the following


Group I Group 2
(Category of plants) (Region)
P. hydrophytes 1. Semi- arid
Q. xerophytes 2. Live on woody vegetation
R. epiphytes 3. In or near water
S. halophytes 4. Polar region
5. salt marshes
6. saline water
(A) (B) (C) (D)
P- 3 P- 1 P- 5 P- 3
Q-4 Q- 6 Q- 1 Q- 4
R- 5 R- 4 R- 3 R- 2
S- 6 S- 3 S- 2 S- 6
Answer: D

Q. 35Hippodamus and Vitruvius pioneered in important areas of built environment. They are
famous for
(A) the planning of Priene and ‘De Architecture’
(B) the planning of Memphis and ‘De Mensura’
(C) the planning of Constantinople and ‘Treatise of Vesta’
(D) the planning of Alexandria and ‘The ionic order’
Answer: (A) the planning of Priene and ‘De Architecture’
Hippodamus (498- 408 BC) is known as father of Urban planning and
designed Piraeus, Olynthus, Priene, and Miletus.
De Mensura was written by Deculi for astronomical calculation.
Constantinople was planned by Anthemius, a mathematician.
Alexandria was planned by Dinocrates.

Q. 36A group housing plot of 400 sqm, abutting a 20 m road is permitted to have 35 % ground
coverage and maximum FAR of 1.25. Assuming 70 sqm of super built up area, calculate
the maximum number of units that can be accommodated
(A) 5 (B) 7 (C) 9 (D)12
Answer: (B) 7
FAR = 1.25 (Maximum built up area = 400

70 sqm for each unit, so no of units = = 7.1 (say 7 units)

Q. 37Match the following


Group 1 Group 2
P. UCSICON 1. Creates a surface that is not filled in
Q. UCSFOLLOW 2. Creates a surface that is filled in
R. TILEMODE 3.Controls the visibility and placement of UCS icon
S. 3DFACE 4. When set to zero make paper space active
5. Sets the orientation of UCS in 3-D space
6.Displays new UCS in plan view whenever the UCS is changed
7. Allows multiple copies across the screen.
(A) (B) (C) (D)
P- 5 P- 6 P- 3 P- 3
Q- 6 Q- 5 Q- 6 Q- 6
R- 4 R- 7 R- 4 R- 7
S- 2 S- 2 S- 1 S- 2
Answer:

Q. 38As speed of the vehicle increases, the driver experiences one or more of the following
P planes perpendicular to road become more prominent
Q distance of focusing point decreases
R peripheral vision increases
S foreground details begin to fade
T importance of the roadside treatment decreases
(A) P, Q, S, T (B) S, T (C) P, Q, R, S, T (D) P, S, T
Answer: (A) P, Q, S, T

Q. 39The ‘caryatid’ mode of columnar construction is a composition of


(A) linear columns and beam forms of a three dimensional framework in Japanese
palaces
(B) sculptured female figure as columnar supports for entablature in Greek temples
(C) vertical linear elements defining volume of transparent spaces in ancient Turkish
domes
(D) basic relationships between structural elements supporting roof and the roofing
materials in Gothic
churches.
Answer: (B) sculptured female figure as columnar supports for entablature in Greek
temples

Q. 40The contemporary style that turns down nature and becomes dependent on huge energy
supply is referred as
(A) Aerodynamic and Ergonomic technology – City gate, Dusseldorf
(B) Local climatological conditions- New Caledonia Cultural Centre, UK
(C) Glass Box and automated systems- CityCorp Centre, New York City
(D) Solar Active controls- Sophia University (Tokyo)
Answer: (C) Glass Box and automated systems- CityCorp Centre, New York City

Q. 41A 25- storied office building is served by a group of 4 lifts of 25 capacity each. Calculate
the probable number of stops
(A) 14 (B) 10 (C) 25 (D) 11
Answer:
The number of stops that the elevator makes in one round trip is called the probable
number of stops, S.
If there was only one lift then the formula to determine the number of probable stops is

S= N [ 1- ( )P]
Where S= number of probable stops
N= No of floors served
P= No of passengers

Putting the values we get S= 25 [ 1- ( )25]


S= 25(1- 0.3603) which comes out to be 15.99 (say 16)
Note: This Question is based on Probabilty and is beyond the scope of B. Arch/B. Plan students.
Q. 42Find the value of slenderness ratio of a wall of thickness 0.19 meter with the following
considerations
(i) Effective height = 2.74 m
(ii) Effective length = 4.32 m
(iii) Stiffness co-efficient =1
(iv) Effective thickness = Actual thickness
(A) 22.7 (B) 10.8 (C) 14.4 (D) 18.6
Answer: (C) 14.4

Slenderness ratio = = = 14.4


Note:
Maximum permissible Slenderness ratio of masonry wall is 20.
In case of column the unsupported length between end restraints shall not exceed 60
times the least lateral dimension of the column.
source: IS code
[Ref: Q-69 (2007), Q-42(2003), Q18 (1994)]

Q. 43Match the following


Group I (Vehicle type) Group 2(PCU)
P. car 1. 1.5
Q. 2- wheeler automobiles 2. 1.0
R. bus 3. 4.0
S. agricultural tractor trailor 4. 0.5
5. 2.2
6. 0.2
(A) (B) (C) (D)
P- 1 P- 2 P- 5 P- 2
Q-4 Q- 6 Q- 2 Q- 4
R- 5 R- 3 R- 4 R- 5
S- 6 S- 5 S- 6 S- 3
Answer: (D)
See new URDPFI Guidelines (GATE 2016, Q- 40) . PCU and ECS (equivalent car space
is different)

Q. 44Choose the correct sequence of waste management


P treatment
Q disposal
R source reduction
S recycling
(A) P, R, Q, S (B) Q, S, R, P (C) S, P, R, Q (D) R, S, P, Q
Answer: (D) R, S, P, Q

Q. 45Choose the correct sequence of the evolution of U. K. planning laws


(A) Uthwatt Commission, Scott Commission, PAG Report, Barlow Commission
(B) Barlow Commission, Uthwatt Commission, Scott Commission, PAG Report
(C) Barlow Commission, PAG Report, Scott Commission, Uthwatt Commission
(D) PAG Report, Scott Commission, Uthwatt Commission, Barlow Commission
Answer: (C) Barlow Commission, PAG Report, Scott Commission, Uthwatt Commission
The Barlow Commission (1940) into the distribution of industrial population
PAG Report
The Scott Committee into rural land use (1941)
The Uthwatt Committee into compensation and betterment (1942)
The Reith Report into New Towns (1947)

Q. 46Participatory plan development having scope like detailed planning, coordinated detailed
design, legal services, coordinating tender process, coordinating construction,
mobilization of funds, mortgage brokerage, operation and maintenance, sales and
marketing, is known as
(A) build operate and transfer (B) development initiatives
(C) leasing contract (D) direct management services contract
Answer: (B) development initiatives

Q. 47The period between 1870 and 1940 was the era of metropolitan development in the
USA. Identify the factor that was not responsible for metropolitan development
(A) The great migration from Europe
(B) Technological advances in highway and high –rise engineering
(C) The comparative advantages of World Wars and Marshall Plan
(D) The process of Urban decentralization.
Answer: (D) The process of Urban decentralization
The Marshall Plan (officially the European Recovery Program, ERP) was an
American initiative passed in 1948 to aid Western Europe, in which the United
States gave over $12 billion (nearly $100 billion in 2018 US dollars) in economic
assistance to help rebuild Western European economies after the end of World
War II.

Q. 48The Bariloche model defines the key development factor as the Basic Needs to ensure
growth. Choose the correct combination that applies to it
(A) (B) (C) (D)
death rate birth rate food birth rate
literacy rate calorie intake shelter school enrolment
employment rate school enrolment education health care
income rate urbanization urbanization clothing
Answer:

Q. 49Identify the criteria that are best suited for building design in hot and dry climate
P high building density
Q plantation parallel to air movement
R small openings
S thick walls
T large openings
U plantation perpendicular to air movement
(A) P, Q, R, S (B) Q, S, T (C) P, R, S, U (D) P, T, U
Answer: (C) P, R, S, U

Q. 50Which of the following are reasons to use mulch on the topsoil of newly planted shrubs
?
P aid in water retention
Q prevent soil temperature fluctuation
R discourage weed growth
S improve landscape appearance
T protect from insects
(A) P, Q (B) P, R (C) P, R, T (D) P, Q, R, S
Answer: (D) P, Q, R, S

Q. 51Match the following


Group I Group II
P digital governance 1. visual simulation of environment
Q genetic algorithm 2. image enhancement, manipulation, management
R virtual reality 3. e- governance
S digital image processing 4. access to information on internet
5 evolutionary visual simulation
6 image interpretation
(A) (B) (C) (D)
P- 4 P- 3 P- 4 P- 3
Q-5 Q-5 Q-5 Q- 2
R-2 R-1 R-1 R- 5
S- 6 S-2 S- 2 S- 1
Answer: B

Q.52 Calculate the number of light fixtures required in an office room of 8m x 7m, requiring
an illumination level of 400 lux on the work plane. Each light
fixture has a rated output of 7350 lumens. Assume a utilization factor of
0.5 and a maintenance factor of 0.8
(A) 8 (B) 7 (C) 10 (D) 12
Answer: (A) 8

Solution : Total No of Lumainaire= = = 7.6 (say 8)

Q. 53The concept of “pattern language” deals with one or more of the following
P description of a problem that occurs in our environment and the core of solution to hat
problem
Q communication and public participation using sign language
R recognition of styles and patterns of various languages
S devising patterns to create a building or a town
(A) P, Q, S (B) Q (C) Q, R (D) P, S
Answer: (D) P, S
Pattern Language was written by Cristopher Alenxander.

. 54 In Indian system of Anthropometrics, two known and one unknown data are given, i.e., 8
angulas = 1 pada, 3 pada = 1 hasta, 1 hasta and 1 danda = x padas. Given
this, find out the perimeter of a square site in angulas and dandas if its sides
are measuring 2 hastas each
(A) 164 angulas and 4 dandas (B) 192 angulas and 2 dandas
(C) 126 angulas and 3 dandas (D) 108 angulas and 6 dandas
Answer: Insufficient data (No appropriate answer)
From the given data:
1 danda = ? angula (Actually 1 danda = 4 hasta = 96 angula)
1 hasta= 24 angula
1 pada = 8 angula
1 angula= 1 angula

Q. 55The contemporary architectural theory which undermines conventional notions of


harmony, unity, and stability, breaks continuity and disturbs relationship between interior
and exterior, is known as
(A) brutalism (B) iconography
(C) transformation (D) deconstruction
Answer: (D) deconstruction

Q. 56Identify the attributes that best apply to Ring type distribution of electric supply.
P only one cable is laid from substation into each building
Q only one fused switch is required in distribution board in each building to isolate
the system
R current flows in both directions from intake room.
S repairs to any point disrupts supply to other buildings
T provides better and balanced supply than radial system
U repairing any cable fault at any point does not lead to loss of supply to any
building
(A) R, S, T, U (B) P, Q, S, T (C) P, R, T, U (D) Q, S, T, U
Answer:

Q. 57Match the defects in painting with their description


Group I (Description) Group II(Defects)
P Dislocation or loosening of some portion of painted surface 1. Flashing
Q Formation of bubbles under the film of paint 2. Fading
R Formation of glossy patch on the painted surface 3. Blistering
S Gradual loss of colour of paint due to effect of sunlight 4. Flaking
on pigments 5. Grinning
6. Crawling
(A) (B) (C) (D)
P- 4 P-6 P- 4 P- 6
Q-3 Q-4 Q-3 Q- 3
R-2 R-2 R-1 R- 4
S- 1 S-1 S- 2 S- 5
Answer: (C)

Q. 58Which is the correct match for the elements (Group1) with behavior (Group 2) ?
Group I Group II
P Beam/ Slab 1. Flexure
Q Arch 2. Compression
R Cable 3. Meriodional Stress
S Shell 4. Tension
5. Indeterminancy
(A) P -1, Q- 2, R- 3, S- 4 (B) P -1, Q- 2, R- 4, S- 3 (C) P -5, Q-1, R- 3, S-
4 (D) P -2, Q- 3, R- 4, S-5
Answer: (B) P -1, Q- 2, R- 4, S- 3

Q. 59What is the theoretical capacity of a traffic lane with one-way traffic flow at a speed of
60 kmph? Assume that the average cente to centre spacing of
vehicles at this speed is 16.68 m
(A) 1000 (B) 3597 (C) 500 (D) 4000
Answer: (B) 3597

Soln: = 3597

Q. 60Choose the attributes that apply to social infrastructures


(A) schools, parks, club, water supply (B) parks, library, club, water supply
(C) hospitals, sanitation, water supply, electric supply (D) schools, hospitals, parks,
library, club
Answer: (D) schools, hospitals, parks, library, club

Q. 61Choose the right kind of statutory provision to enforce integrated urban conservation
(A) Conservation of Ancient Monuments and Sites Act (B) Urban Arts Commission Act
(C) Conservation of Energy Act (D) Conservation of Natural Resources Act
Answer: (A) Conservation of Ancient Monuments and Sites Act

. 62 A community comprising 120 hectares is having 60% of the land put to residential plots,
and a population of 30,600. Calculate the net residential density
(A) 375 pph (B) 400 pph (C) 425 pph (D) 500 pph
Answer: (C) 425 pph

Area under residence = 120 =72 hectare Net Residential density= =


425 pph

. 63 Poor academic performancesof school children were attributed to missing breakfast and
subsequent malnutrition. The investigating survey schedule questioned every student,
how many days the students have missed the breakfast? Their response may be
referred as
(A) nominal scale (B) ordinal scale (C) longitudinal scale (D) interval scale
Answer: (D) interval scale
For qualitative data
i. Nominal: scales are exclusive, no order or overlap eg. What is your
religion: Hindu/ Muslim/ Sikh/……..
ii. Ordinal: data can be rank ordered and can overlap, eg. Mild,
moderate, severe
iii. Dichotomous: nominal data fall into only two categories. (Yes/No)
For quantitative data
i. Interval: Numeric interval, eg: height group
ii. Ratio: Interval data with necessary zero

. 64 A site map shows five contour lines. A line drawn across the lowest and the highest
contour measures 125 m and has a slope of 8 %. Determine the contour interval
(A) 5 m (B) 2.5 m (C) 10 m (D) 2 m
Answer: (B) 2.5 m

Soln: = solving for h = 10m

Contour interval = = 2.5 m (for five contour lines there are four contour intervals)

Q. 65Identify the components of Geographical Information System


P computer system and software
Q spatial data
R uniform energy source
S a super sensor
T real time data handling system
U data management and analytical tools
(A) P, Q, R, S, T, U (B) P, Q, S, U (C) P, Q, U(D) P, S, T, U
Answer: (A) P, Q, R, S, T, U

. 66 In energy technology, efficiency is equal to energy output by energy input. If the


efficiency level of a particular fossil fuel is 90 % and the mechanical efficiency of the
vehicle is 25 % then how much is the total efficiency of the vehicle?
(A) 62.5% (B) 22.5 % (C) 33.33% (D)82.5%
Answer: (B) 22.5 %
Efficiency is multiplied so × = × 100 = 22.5 %

Q. 67Identify the missing prime factor for design


investigation based on ergonomics of the figure
(A) shape (B) functions (C) size (D)aesthetics
Answer: (B) functions

Q. 68The Bahai temple of New Delhi has a lotus plan. The lotus has x number of petals, y
number of equal angles and z type of architectural tradition. Which of the following is
correct?
Type x y z
(A) 12 30 Egyptian Mastaba
degrees
(B) 9 40 Persian Sepharial
degrees
(C) 10 36 Santa Sophia,
degrees Turkey
(D) 8 45 Ideal city of Vineezo
degrees Seamozzi

Answer: (B)

Q. 69Michael Wilford, Peter Salter and Richard Rogers are associated with
(A) Potsdamer Plaza (Berlin), Kyoto Centre (Japan), Music School and Theatre
Academy (Stuttgart)
(B) Marlenbone Gate (UK), islamic Arts Centre (London), Stone Henge Visitors Club (UK)
(C) Reich Stag (Berlin), Camden Arts Center (France), Tete Gallery (Cornwall)
(D) Fountain Abbey Visitors Centre (Yorkshire), Flat Conversion (London), More Cambe
(UK)
Answer:

. 70 Match the fittings shown in Group 1 with the sketches shown in Group 2

Group 1
P Single socketed pipe
Q Double equal junction
R Pipe- shoe
S Reducing piece

(A) (B) (C) (D)


P- 5 P-4 P-8 P- 3
Q-4 Q-2 Q-6 Q- 1
R-1 R-7 R-5 R- 2
S-7 S-5 S-1 S- 7
Answer: (A)

Q. 71Which is the correct match for items given in Group 1 with items given in Group 2?
Group 1 Group 2
P Plastering 1 Volume
Q Earthwork 2 PERT
R Reinforcement 3 Schedule of bending
S Management 4 Area
5 Temperature
(A) P -4, Q-1, R- 3, S- 2 (B) P -2, Q-1, R-3, S- 4 (C) P -3, Q- 5, R-2, S- 4 (D) P
-2, Q- 3, R- 4, S- 5

Answer: (A) P -4, Q-1, R- 3, S- 2

. 72 Match the attributes with the corresponding shapes of central island in a rotary shown in
the sketches below
P reduction of vehicular speed on entering rotary and enable speeding of going
out vehicle
Q enables excessive speeding of vehicles
R where two equally important roads crosses
S accomodates four or more intersecting roads and allow traffic flows
along direction of elongation
(A) P – 4, Q-1, R- 3, S- 2 (B) P – 2, Q-1, R- 3, S- 4 (C) P – 3, Q- 1, R- 2, S-
4 (D) P – 2, Q- 3, R- 4, S- 1
Answer: No appropriate answer

Q. 73 Identify the attributes that apply to sanitary landfill


P waste is deposited in 0.90 – 4.50 m thick layers
Q low lying marshy waste land are transformed into useful areas
R attract flies, insects and rodents
S causes health and pollution hazards
T chances of fire hazards in wet weather
U skilled personnel are required
V semi- skilled personnel are required
(A) R, S, T, U (B) P, Q, T, V (C) P, Q, S, T (D) Q, R, S, T
Answer: No appropriate answer

Q. 74Match the urban forms in Group I with their corresponding urban areas in Group 2.
Group I Group II
(Description) (Cities)
P Sheet 1. Delhi
Q Star 2. Tokyo
R Ring 3. Copenhagen
S Linear 4. Ohio
5. Bombay
6. Chandigarh
(A) (B) (C) (D)
P- 3 P- 2 P- 1 P- 4
Q- 2 Q- 3 Q- 2 Q- 1
R- 4 R- 1 R- 3 R- 2
S- 6 S- 5 S- 6 S- 3
Answer: B

. 75 Four sequences of development are given below. Choose the one that is appropriate for
slums
(A) land → services → house → people
(B) land → services → people → house
(C) people → land → house → services
(D) land → people → house → services
Answer: (A) land → services → house → people

Q. 76The neoclassical school and the modern ecological school of environment studies differ
on
(A) their historical evaluation of environmental resources
(B) their relative emphasis on cost- benefit test vis- a-vis resource development
(C) their variable interpretation of local and global ecosystems.
(D) their variable interpretation of Malthusian model of sustainability explaining
environmental disaster
Answer: (A)

Q. 77The term Imperial Garden, Viewing hill, Middle Sea, South Sea, Gate of Heavenly
Peace, Land and God of Grain are associated with
(A) Mongolian Garden – Hukado Garden, Ulan Bator
(B) Japanese Garden – le Garden at Kobe
(C) Chinese Garden – Forbidden city at Peking
(D) Persian Garden – Palace Court at Persopolis
Answer: (C) Chinese Garden – Forbidden city at Peking

Q.78 Find the thermal transmittance (U) of a 50 cm thick wall with 5 cm cavity, 1.8 cm thick
plaster internally and having a normal exposure
Given
Thermal resistivity of brickwork = 0.96 kcal per m² h ºC
Thermal resistivity of plaster = 2.02 kcal per m² h ºC
Thermal resistivity of 5 cm cavity = 0.205 kcal per m² h ºC
Internal Surface resistance = 0.1440 kcal per m² h ºC
External Surface resistance = 0.0620 kcal per m² h ºC
(A) 1.672 kcal per m² h ºC (B) 1.078 kcal per m² h ºC (C) 0.935 kcal per m² h ºC (D)
2.015 kcal per m² h ºC
Answer: Unit of thermal resistivity is confusing. Again in options the unit of U- value is
same as thermal resistivity

. 79 Domes of Islamic period are found in three distinct phases. Given below are four phases
and their respective examples. Identify the incorrect one
Phases Type
(A) Imperial Lodhi Tomb Dome,
style Delhi
(B) Provincial Bijapur Dome
style
(C) Early style Samarkand Dome
(D) Mughal Sultan Ghori Dome
style

Answer: (D)
Samarkand Dome is in Uzbekistan.

Q. 80Identify the building that has open passageways, inner outer courts, influences from
Lodhi gardens, use of local stones for horizontal elements and precast concrete slabs
for vertical elements, in –situ concrete frames and burnt clay
(A) French Embassy staff quarters in New Delhi
(B) India International Center in New Delhi
(C) Akbar Hotel in New Delhi
(D) YMCA Guests House Complex and staff quarters in New Delhi
Answers: (B) India International Center in New Delhi

Q. 81Find the total volume of liquid in a septic tank with the following criteria.
i. Numbers of users =20
ii Cleaning intervals =365 days
iii The capacity required for sludge digestion = 0.033 m³ per capita at 25 ºC
iv Volume of digested sludge = 0.0002 m³ per capita per day
(A) 4080 litres (B) 2580 litres (C) 5250 litres (D) 3289 litres
Answers: Marks awarded to all.
Amount of sludge deposited 0.0002 × 20× 365 = 1.46 m3
Capacity required for sludge digestion = 0.033 × 20= 0.66 m3 (at 25 ºC)
Total volume of liquid = 1.46 + 0.66= 2.12 m3 which equals 2120 litres.

Q. 82Safe axil load for a short R.C. square column having cross section as 300 mm x 300 mm
and 4 longitudinal bars of diameter 20 mm (using M 20 concrete and Fe 415 steel) is
near to
(A) 100 kN (B) 10 kN (C) 1000 kN (D) 10,000 kN
Answers: (C) 1000 kN
According to IS Code 456 in limit state
load carrying capacity of short column is Age Strength
P=0.4F(c) x A(c) + 0.67F(y) x A(s) percent
P=Load Carrying capacity of column 1 day 16%
A(c)=Area Of Concrete 3 days 40%
A(s)=Area Of Steel
7 days 65%
F(c)=Characteristic strength of Concrete
F(y)=Characteristic Strength of Steel 14 days 90%
28 days 99%
Here area of one steel bar = 2
= 3.14 x 102 = 314 mm2
Area of four steel bar : 314 x 4= 1256 mm2
Area of concrete = 300 x 300 – 1256 = 88744
P= (0.4 x 20 x 88744) + (0 .67 x 415 x1256)
= 709952 + 349230
= 1059182 N/mm2
= 1059 kN/mm2

The strength of concrete increases with age. Table shows the strength of concrete at
different ages in comparison with the strength at 28 days after casting.
Grade of Minimum Specified characteristic
Concrete compressive compressive strength
strength (N/mm2) at 28 days
N/mm at 7 days
2
M15 10 15
M20 13.5 20
M25 17 25
M30 20 30
M35 23.5 35
M40 27 40
M45 30 45

. 83 Given below is a table of five types of roads and their characteristics. Which of the
following types represent the collector and the urban arterial street?
Type Width Recommended
(meters) speed
(km/hr)
P 10-20 30
Q 20-30 50
R 30-40 60
S 50-60 80
T 80-100 80-100

(A) R, S (B) Q, S (C) Q, R (D) S, T


Answers: (A) R, S

Q. 84Match the hierarchy of Chandigarh road pattern


Group 1 Group 2
(Hierarchy of road) (Connection)
P V1 1 sector connector
Q V3 2 cross country
R V4 3 to building
S V5 4 sector divider
5 pedestrin connection

(A) (B) (C) (D)


P- 2 P- 1 P-3 P- 4
Q- 4 Q- 2 Q-4 Q- 1
R- 1 R- 5 R- 5 R- 3
S- 3 S- 3 S-2 S- 5
Answer: (A)

Q. 85Public interest determinants of urban development may be stated to comprise of


(A) health, infrastructure, entertainment, and safety
(B) economy, amenity, convenience, and entertainment
(C) health and safety, convenience, economy and amenity
(D) entertainment, economy, health and infastructure
Answer: (A) health, infrastructure, entertainment, and safety
Q. 86Which housing layout variation does not follow a strict geometrical (orthogonal)
pattern?
(A) Prastara variation (B) Finger variation
(C) Cluster variation (D) Finger and cluster variation
Answer: (D) Finger and cluster variation
Q. 87 Choose the appropriate concept propagated in the urban landuse theory based on
economic determinants.
(A) concentric zone theory
(B) sector theory
(C) axial development theory
(D) multiple nuclei theory
Answer: (D) multiple nuclei theory

Q. 88Identify the characteristics of a “district” as an element of imageability


P has two-dimensional extent
Q is channel of movement
R used as lateral references, boundaries, or breaks
S observer enters “inside of”
T presence of a common theme
(A) P, S, T (B) P, R, S (C) Q, R, S (D) P, R, S, T
Answer: (D) P, R, S, T

. 89 A settlement pattern has a 4 x 4 matrix with a cross like principle axis of main roads and
four smaller cross like networks of arterial roads, opening upto 4 gateways cardinally.
The pattern is called
(A) Dandaka (B) Nandavartya (C) Chaturmukha (D) Sarvatobhadra
Answer: (C) Chaturmukha

Q. 90 If the expected duration of a project is 30 months along with a standard deviationof 6


months, what is the probability that the project will be completed in 26 months?
(A) 0.100 (B) 0.500 (C) 0.253 (D) 0.340
Z Table
Z Probability of meeting due Z Probability of meeting due date

1.0 0.841
0.8 0.788
0.6 0.726
0.4 0.655
0.2 0.579
0.0 0.500
-0.2 0.421
-0.4 0.345
-0.6 0.274
-0.8 0.212
-1.0 0.519

Answer: (C) 0.253

Soln: Z value calculation= = = - 0.66


From table probability will lie between 0.274 and 0.212.
GATE 2004
Q.1 The designer of Horizon City in Texas is
(A) Lucio Costa (B) Le Corbusier (C) Frank Lloyd Wright (D) Soria Mata
Answer: (A) Lucio Costa
Lucio Costa and Oscar Niemeyer- City planner of Brasilia, capital of Brazil.
(Brasilia designed in 1960 is UNESCO world heritage city in virtue of its
modernist architecture).Ahmadabd is India’s first Heritage City designated by
UNESCO.
F L Wright- Broadacre city
Soria Y Mata- (Madrid) In 1882, the Spanish planner Arturo Soria y Mata
envisioned a 30-mile-long city built along a Madrid tramline.
Le Corbusier- Chandigarh

. 2 A dark brown organic compound found in the soil due to partial or complete
decomposition of vegetable matter is known as
(A) Loam (B) Texture (C) Humus (D) Mulch
Answer: (C) Humus
Loam- soil with 40% sand. 40% silt and 20 % clay.
Humus- partial or complete decomposition of vegetable or animal matter.
Mulch- a layer of material applied to the surface of an area of soil.

Q.3 The format for transferring the AutoCAD drawing file to an ASCII file is known as
(A) DXB (B) DXF (C) JPEG (D) DGN
Answer: (B) DXF
Drawing Interchange Format (DXF) files enable the interchange of drawings between
AutoCAD and other programs. The term “ASCII file” refers to a “text” file that is
readable by the naked eye (it only contains the letters a-z, numbers, carriage returns,
and punctuation marks). Conversely, a binary file is not readable by the
naked eye (it contains the ASCII characters in addition to binary codes).

Q.4 Technopolis means


(A) Ten million size city (B) Dynapolis
(C) Settlement designed by high technology (D) Urban conurbation
Answer: (C) Settlement designed by high technology
Megacity- Cities with over ten million (one crore) population.
City with largest population- Tokyo (34.3 million)
Dynapolis- Doxiadis parabolic shaped scheme applied to Islamabad.
Urban conurbation- The term conurbation was coined by Patrick Geddes in his
book ‘Cities in Evolution’ which means a polycentric urban agglomeration
(many urban areas, through population growth have merged as one
continuous area known as Urban conurbation). Agglomeration is extended city
with suburbs.
NCR (National Capital Region) comprises Delhi, and 22 districts of UP,
Haryana and Rajasthan.

Q.5 Earthwork for the mounds in the landform work in a landscape is estimated by
(A) Average- end- area method (B) Borrow pit method
(C) Dead-man- method (D) Prismoidal method
Answer: (B) Borrow pit method
Average end area calculation is used to calculate volume between two cross
sections i.e., two cross sectional areas are averaged and multiplied by the length
(distance) between two cross sections to get the volume.
Volume = L x (A1 + A2) cubic meter where L is distance between two cross section
and A1 and A2, area of cross sections.
Borrow pit method: The area is divided into grid and average height of each grid is
multiplied with the area of grid to get the volume.
Prismoidal Method: Prismoidal formula is accurate in finding the volume of prisms,
pyramids, wedges, and prismoids having irregular end sections. The estimation of
earthwork gives nearly an accurate volume.

Volume= [Sum of areas of end two sections+4(sum of the even sections)+2(sum


of the remaining sections)]

Q.6 The word Bionomics refers to


(A) management of land (B) management of project (C) management of fund (D)
management of life
Answer: (D) management of life
In ecology, bionomics is the comprehensive study of an organism and its relation to
its environment.

Q.7 For proper ventilation, the ratio of opening area to the total floor area of the room
should be
(A) 1 : 6 (B) 1 :10 (C) 1: 12 (D) 1:20
Answer: (A) 1 : 6
Standards of ventilation
Cubic space- 3000 cubic feet per person per hour. (range is 300- 3000)
Air change- 2 to 3 times per hour in living room and 4 to 6 times per hour in
work rooms and assemblies.
(Note: For volume of room calculation height above 10-12 ft are not taken into
account)

Q.8 Emerald necklace is well known as


(A) Ornamented garden in Italy (B) Large public park in New York
(C) Parkway in Boston (D) Street lighting in Marine Drive in Mumbai
Answer: (C) Parkway in Boston
The Emerald Necklace consists of a 1,100-acre chain of parks linked by
parkways and waterways in Boston and Brookline, Massachusetts. It gets its
name from the way the planned chain appears to hang from the "neck" of the
Boston peninsula.
Central Park (1857) is a public park at the center of Manhattan in New York
City. The park was designed by Frederick Law Olmsted.
Marine Drive is a 3.5-kilometre-long boulevard in South Mumbai in the city of
Mumbai. It is a 'C'-shaped six-lane concrete road along the coast, which is a
natural bay. Marine Drive is also known as the Queen's Necklace because, if
viewed at night from an elevated point anywhere along the drive, the street
lights resemble a string of pearls in a necklace.
(with input from Vipin V Mukkawar, Mumbai)

Q.9 Indicate the minimum horizontal and vertical distance for any building from a 33kV
electric line
(A) 3.7m (Vertical) & 2.0m (Horizontal) (B) 1.5m (Vertical) & 4.6m (Horizontal)
(C) 2.5m (Vertical) & 1.2m (Horizontal) (D) 4.5m (Vertical) & 6.9m
(Horizontal)
Answer: (A) 3.7m (Vertical) & 2.0m (Horizontal)
Voltage-Categories:
(a) Low Voltage upto 250 V
(b) Medium Voltage 250 V to 650 V
(c) High Voltage 650 V to 33 KV
(d) Extra High Voltage above 33 KV

Q.10 Effective temperature in buildings integrates the effect of


(A) temperature, humidity and air movement
(B) radiation, temperature, humidity and air movement
(C) temperature, air movement, conduction, vegetation
(D) temperature, humidity, density, vegetation
Answer: (A) temperature, humidity and air movement
Corrected Effective Temperature is - radiation, temperature, humidity and air
movement
Refer Page 48-49 (Manual of Tropical Housing and Building by Koenigsberger)
(input by Chitra Mshra, B. Arch, Sundardeep College of Architecture, Ghaziabad.)

Q.11 The ‘Five points for a New Architecture’ was defined by


(A) Louis Sullivan (B) H H Richardson (C) Gustav Eiffel (D) Le Corbusier
Answer: (D) Le Corbusier
Pilotis, Free Façade, Open Plan, Ribbon Window, Roof Garden

Q.12 Gate Valves are used in water supply lines to


(A) isolate parts of the network (B) drain out excess water
(C) release air trapped in pipe lines (D) limit the flow of water
Answer: (A) isolate parts of the network
Gate valves are primarily used to permit or prevent the flow of liquids, but typical gate valves
shouldn’t be used for regulating flow, unless they are specifically designed for that purpose.
Drain out excess water- Scour valve
Release air trapped in pipe line- Relief valve
Limit the flow of water- Globe valve

Q.13 Earnest money is required to be deposited by a tenderer


(A) before acquiring a tender document (B) before the bid is open
(C) after the contract is awarded (D) before getting the payment against the last bill
Answer: (B) before the bid is open
An earnest money is a specific amount of security deposit made in some major transactions
such as real estate dealings or required by some official Procurement processes to
demonstrate that the applicant is serious and willing to demonstrate an earnest of good faith
about wanting to complete the transaction.

Q. 14 Designer of Roofless Church is


(A) Kenzo Tange (B) Philp Johnson
(C) Bruce Goff (D) Ero Saarinen
Answer: (B) Philp Johnson (Pritzker Prize- 1979)
adjacent figure is Roof less church, Indiana

Q. 15 Homelessness refers to
(A) number of people per 1000 urban area population sleeping outside dwelling units
(B) total number of peple living on footpath
(C) percentage of people living on unauthorized land
(D) share of urban population living in slums
Answer: (A) number of people per 1000 urban area population sleeping outside dwelling
units

Q. 16 Dummy activities are activities having


(A) zero time
(B) time difference between additional event and preceding event
(C) time difference between additional event and succeeding event
(D) earliest event time
Answer: (A) zero time

Dummy activity is an imaginary activity. It does not


exist in the Project activities. It is used in the network diagram to
show dependency relationship or connectivity between two or more activities. It is
represented by a dotted arrow.

Q.17 The roof above the millennium dome in England (2000 AD) is made of
(A) P. V. C (B) Ferro- cement (C) Teflon (D) Titanium
Answer: (C) Teflon

Architect: Richard Rogers. The canopy is made of


PTFE-coated glass fibre fabric, a durable and weather-resistant plastic, and is 52
m high in the middle – one metre for each week of the year.
Polytetrafluoroethylene (PTFE) is a synthetic
fluoropolymer of tetrafluoroethylene that has numerous
applications. The best known brand name of PTFE is Teflon by DuPont
Co.

Q. 18 Grease trap is provided in house drain connected to


(A) kitchen (B) bathroom (C) water closet (D) urinal
Answer: (A) kitchen

Q. 19 Hydra house is an example of


(A) space architecture (B) modern skyscraper
(C) mobile architecture (D) desert architecture

Answer: (C) mobile architecture

Jennifer Siegal (Los Angeles based ) and her


‘Office of Mobile Architecture’ proposed the “Hydra House.” In her brief, the project is
described as “a mass-customized mobile modular structure that is responsive to
environmental issues of global warming and water desalination and recycling.”
‘space structure’ refers to a structural system that involves three dimensions. This is
in contrast with a ‘plane structure’, such as a plane truss, that involves no more than two
dimensions.
supernal structures: structures for outer space.
Japanese architect Arata Isozaki teamed up with
Indian-born Anish Kapoor to design the orb-like structure, which is on the inflatable
Leviathan sculptures. It can be inflated and carried to other location.

Q. 20Housing in Compliance refers to


(A) housing affordability (B) Mortgage rate
(C) Zoning regulation (D) Authorised Housing

Answer: (D) Authorised Housing

Q. 21The Secession Building in Vienna was designed by


(A) August Perret (B) Carlo Lodilo
(C) Joseph Maria Olbrich (D) Josef Hoffman
Answer: (C) Joseph Maria Olbrich

Q. 22The curved arris formed by the intersection of vaults is


(A) Groin (B) Glyph (C) Impost (D) Frieze
Answer: (A) Groin

Q. 23For a truss under vertical loading the top chord members of the truss are subjected
to
(A) torsion (B) axial tension
(C) tension and compression (D) axial compression
Answer: (C) tension and compression

Q. 24The Ian Potter Centre in Australia designed by


(A) Vibas Gore (B) Group Seven Architecture

(C) Lab Architecture Studio (D) B. V. Doshi


Answer: (B) Group Seven Architecture

Q. 25Solid masonry which resists the lateral pressure of an arch, is called


(A) Abutment (B) Alae(C) Abacus(D) Aisle
Answer: (A) Abutment

Q.26 Which plan form is most suitable for a building located in an earthquake zone:
(A) Rectangular (B) U- Shape (C) L- Shape (D Square
Answer: (D) Square

Q.27 If intensity of incident light increases on a totally black object


(A) The object appears blacker (B) The object produces a shining effect
(C) The form is flattened (D) The object appears whitish
Answer:

Q. 28Square plan of Garbh Griha in Hindu temple is very much linked with
(A) Climate (B) Orientation (C) Corbelling (D) Façade treatment
Answer: (D) Façade treatment

Q. 29An external enclosure for any space is most pleasing when its
walls are
(A) 1/ 2 to 1/3 rd as high as the width of space
(B) Height is greater than the width
(C) Height is four times the width
(D) Height is equal to the width

Answer: (A) 1/ 2 to 1/3 rd as high as the width of space


Q.30 When the picture plane, located between the object and the observer, is moved
towards the observer the size of the perspective view
(A) increases in size (B) remains same (C) is distorted (D) decreases in size
Answer: (D) decreases in size

Q. 31 In the treatise of Vastushastra the cardinal boundaries are known as


(A) (B) (C) (D)
Eastern Boundary Aditya Varuna Aditya Chandra
Western Boundary Yama Aditya Varuna Aditya
Southern Boundary Chandra Yama Yama Varuna
Northern Boundary Varuna Chandra Chandra Yama

Answer: (C)
North

North- (Kuber)- Ruled by lord of wealth (Finance)


South- (Yama)- Ruled by lord of death – Yama (Damaging)
East- (Indra)- Ruled by the solar deity- Aditya (Seeing the world)
West- (Varuna)- Ruled by lord of water (Physical)
Northeast (Ishan) – Ruled by Shiva
Southeast- (Agni)- Ruled by the fire deity – Agni (Energy Generating)
Northwest- (Vayu)- ruled by the god of winds (Advertisement)
Southwest- (Pitru/Nairutya) Niruthi- Ruled by ancestors (History)
Center- (Brahma)- Ruled by the creator of the universe (Desire)

In accordance with the position occupied by the gods in the mandala, guidelines are
given for zoning of site and distribution of rooms in a building. Some of these are:

North – treasury
Northeast – prayer room
East – bathroom
Southeast – kitchen
South – bedroom
Southwest – armoury
West – dining room
Northwest – cowshed.

Q.32 The purpose of the Building code is to protect the interests of the citizen. Choose the
right combination from the following factors.
P General Welfare
Q Aesthetic Quality
R Protection of Health
S Safety
T Convenience
U Affordability
(A) P, R, S, U (B) P, R, S, T (C) P, S, T, U (D) Q, R, S, T
Answer: (B) P, R, S, T

Q. 33 The items associated with the garden styles listed in Group- I are shown under
Group II
Group I Group II
P. French Baroque 1. William Kent
Q. Moghul 2. Temple Garden
R. English Romantic 3. Varsailles
S. Japanese 4. Charbag
Select the appropriate combination from the following options
(A) P-1, Q-2, R-3, S-4 (B) P-3, Q-4, R-1, S-2
(C) P-3, Q-2, R-4, S-1 (D) P-4, Q-2, R-3, S-1
Answer: (B) P-3, Q-4, R-1, S-2

. 34 Choose the right combination of information, which should accompany an application for
Building Permission from any urban local government.
P Site Plan
Q Building Plan
R Key Plan
S Ownership Title
T Power of Attorney
U Services Plan
V Quantity Estimate
(A) P – Q – R- S – U (B) R – Q – T – V (C) S – T- U – V (D) Q – R – S – T
Answer: (A) P – Q – R- S – U

Q. 35Which of the following are the phenomena exhibited by the sound waves
P Reflection
Q Refraction
R Interference
S Polarization
T Diffraction
Indicate the correct combination
(A) P, Q, R (B) Q, R, S, T (C) P, R, S, T (D) P, Q, R, T
Answer: (D) P, Q, R, T

. 36 The net rent received from a property is Rs. 1,00,000/- per annum. The outgoings is Rs
20,000/- per annum. It is expected that the rent will be received for a long period.
Estimate the rental value of the property for 10% rate of interest.
(A) Rs 9,00,000/- (B) Rs 10,00,000/- (C) Rs 10,20,000/- (D) Rs 11,00,000/-
Answer: (B) Rs 10,00,000/-

Rental value = Net Annual Rent ×

= 1,00,000 ×
= 10,00,000 Rs
Net Rent: Amount received by land lord from tenant after deduction of maintenance,
taxes, insurance. If gross rent was given instead of Net Rent, then 20,000/- was to be
deducted to get Net Rent.
(corrected by Kavinaz Kaur, B. Arch, GNDU Amritsar)

Q. 37Reservoir capacity can be determined from the graph drawn between


(A) Cumulative runoff and time (B) population and time
(C) runoff and time (D) water demand and time
Answer: (A) Cumulative runoff and time

Q. 38All the meanings of the following architectural terms are not correct. Select the correct
answer
Term Meaning
P PLAZZA - an open public square
Q COLUMN - a horizontal support
R STUCCO - art of high relief decoration by moulding plaster
S PORTICO - entry porch or vestibule supported by columns
(A) P, Q (B) P, S (C) Q, R (D) R, S
Answer: (B) P, S
Stucco- plaster used for coating and making decorative casts.

Q. 39Number of modular bricks required for Ten cubic metre brickwork in a house is
(A) 4500 (B) 4750 (C) 5000 (D) 5225
Answer: (C) 5000
Size of modular bricks= 20 cm × 10 cm × 10 cm

No of bricks = = 5000

Q. 40 A residential area of 80 hectares has the following residential plot subdivision. Each plot
has one dwelling unit and the average household size is 5 persons. The rest area is
devoted to roads, schools, parks and shops.
Plot Size Numbers
500 sq m 100
300 sq m 500
200 sq m 1000
What are the gross and net density respectively of the area in person per hectare (ppha) ?
(A) 100 ppha and 200 ppha (B) 500 ppha and 300 ppha
(C) 200 ppha and 100 ppha (D) 200 ppha and 300 ppha
Answer: (A) 100 ppha and 200 ppha
Total no of Dwelling units = 100 + 500 + 1000= 1600 Dwelling units.
Total Population = 1600 ×5= 8000
Total Residential Area= (500× 100) + (300 ×500) + (200 ×1000) = 4,00,000 sqm= 40
ha

Gross Density = = = 100 ppha

Net Density = = = 200 ppha

. 41 A site map shows six contour lines at 5 metre contour interval. The mid point distances
between the top and the bottom most contour lines is 600 metres. The site measures
100 metre x 100 metre square. What is the slope between the above two midpoints?
(A) 1 in 60 (B) 1 in 30 (C) 20% (D) 5%
Answer: None of these.(Question not clear)

. 42 A small commercial plot measuring 20 metre x 30 metre located in a community centre


is subjected to maximum ground coverage of 60 % and FAR 300. With fullest land
utilization and uniform floor areas how many floors can be built on the plot?
(A) Six (B) Three (C) Five (D) Ten
Answer: (C) Five
Area of plot = 20×30= 600 m²

Ground coverage= 60 %=
FAR= 300 so total built up area= 600 ×3= 1800 m²

No of floors=

. 43 Which of the following percentage distribution of different land use is the most
representative one for a medium size urban area.
[Note : R- Residential, C- Commercial, P- Public and Semi Public, T- Transportaion
and Communication, I- Industrial] ?
(A) R : 45%, C : 4%, P : 12%, T : 14%, I : 8%
(B) R : 30%, C : 4%, P : 35%, T : 14%, I : 10%
(C) R : 30%, C : 20%, P : 12%, T : 14%, I : 10%

(D) R : 45%, C : 4%, P : 12%, T : 14%,


I : 20%
Answer: (A)

Q. 44The most appropriate hierarchical sequence of plans is


(A) Regional Plan →Master Plan →Action Plan → Perspective Plan
(B) Regional Plan →Perspective Plan →Master Plan →Action Plan
(C) Perspective Plan→Regional Plan →Master Plan→Action Plan
(D)Master Plan →Perspective Plan → Action Plan →Regional Plan
Answer: (C) Perspective Plan → Regional Plan → Master Plan → Action Plan
Q.45 A country has the largest city size of 5 million. What will be the population size of its
fourth largest city according to the rank size rule.
(A) 1 million (B) 1 lac (C) 1.25 Lacs (D) 1.25 million
Answer: (D) 1.25 million
According to rank size rule a rank 2 city would have half the population of a
country’s largest city, a rank 3 city would have one-third the population of a
country’s largest city, a rank 4 city would have one-fourth the population of the
largest city.
Zipf’s law: the most frequent word will occur approximately twice as often as the
second most frequent word, three times as often as the third most frequent word,
etc.
For example, in the Brown Corpus of American English text, the word “the” is the
most frequently occurring word, and by itself accounts for nearly 7% of all word
occurrences. True to Zipf’s Law, the second-place word “of” accounts for slightly
over 3.5% of words, followed by “and”.
Pareto’s distribution: ( 80- 20 rule) Pareto originally used this distribution to
describe the allocation of wealth among individuals. A larger portion of the wealth
of any society is owned by a smaller percentage of the people in that society. He
also used it to describe distribution of income. This idea is sometimes expressed
more simply as the Pareto principle or the “80-20 rule” which says that 20% of the
population controls 80% of the wealth.

Q. 46 The following
figures represent four different study areas

Which of the following combination of the study area and number of nursery school
required is the most appropriate as per the planning norms.
(A) P-1, Q-2, R-2, S-2 (B) P-1, Q-2, R-1, S-1
(C) P-1, Q-2, R-2, S-1 (D) P-1, Q-1, R-2, S-2
Answer: (C)
(with input from Shweta Bharti, Ganga Institute of Arch. and Town Planning, MDU,
Rohtak)

Q. 47The following figures P, Q, R represent three areas of different population sizes. The
Small circles within represent distribution of two consecutive hierarchical levels of
commercial centres in each study area. Typologies of these commercial centres are
given below.
Typologies
1. Community Centre
2. Convenient Shop
3. District Centre
4. Sector market

Which of the following combination represents the most appropriate typology of


commercial centres in these areas.
(A) P-2, 1 Q-2, 3 R-3, 1 (B) P-2, 1 Q-1, 4 R-1, 3
(C) P-2, 4 Q-4, 1 R-1, 3 (D) P-2, 4 Q-4, 3 R-3, 1
Answer: (A)

Q. 48On street parking along a road kerb has provision for 45 degree angular parking with car
spaces of 5.4 m by 2.5 m. How many cars can be parked in 400 m stretch along the
kerb ?
(A) 100 cars (B) 110 cars (C) 120 cars (D) 130 cars
Answer: (B) 110 cars

Q. 49 Moment at the fixed end ‘A’ of the beam indicated below is


(A) (-) wl² / 12 (B) (-) wl / 16
(C) (-) wl² / 8 (D) (-) wl / 8
Answer: (C) (-) wl² / 8

Cantilever deflection due to UDL δ1 =

Cantilever deflection due to point load RB δ2 =

Equating both δ1 = δ2 =

= wl and = wl ( since + = wl)

Now moment at A…… -

=
Q. 50Consider the following qualities of building materials:
P Fire resistance
Q Moisture resistance
R Strength
S Thickness
T Stability
Which of the following combination of qualities is most appropriate for thermal insulation?
(A) P, Q, R (B) P, Q, S (C) Q, R, S (D) R, S, T
Answer: No appropriate answer.

Q. 51A 300 metre long and 6 m wide pathway is to be illuminated with 4000 lumen lamps
having maintenance factor of 0.8 and coefficient of utilization 0.45. The desired average
lux on the pathway is 6. What should be the spacing between the lamps?
(A) 20 m (B) 30 m (C) 40 m (D) 50 m
Answer: (B) 30 m
Desired Lux= 6 lumens/m²
Total Area to be illuminated= 300 m²
Total Lumens required= 1800 (effective)
Let the no of lamps be ‘y’

Then y = 10800
y= 7.5 lamps (say 8 lamps)

So spacing will be = 37.5 m ( so, we will go by 30 m)

Q. 52Sight lines are manipulated in the design of a complex by


P Slight shifts in ground level
Q Path directions
R Position of opaque barriers
S Some focal objects
T Rising levels of middle ground
Which of the following combinations provide the best manipulation?
(A) P, Q, R (B) Q, R, S (C) R, S, T (D) P, R, S
Answer: (C) R, S, T

Q. 53 Following factors are considered for architectural design


P Scale of different spaces Q location of elements
R Topographical conditions S Structural system
Which of the following combination of factors is most suitable for proximity diagramming?
(A) R and S (B) Q and R (C) P and Q (D) P and S
Answer: (C) P and Q

. 54 Which of the following combination correctly represent the labels indicated in the
diagram given below?
(A) (B) (C) (D)
RIDGE Q R Q P
VALLEY R Q R Q
HIP P P S R
GABLE END S S T T
EAVES PROJECTION T T P S
Answer: B

.55 Indicte the correct combination of dimensions against the labels shown below in the
diagram of the WC?

(A) (B) (C) (D)


P 70 cm 30 cm 55 cm 75 cm
Q 50 cm 24 cm 34 cm 55 cm
R 130 cm90 cm 100 cm 130 cm
S 15 cm 15 cm 24 cm 30 cm
T 90 cm 65 cm 78 cm 95 cm
U 60 cm 28 cm 39 cm 30 cm
Answer: B

Q. 56 Indicate the correct combination of duals of five platonic solids given below
TETRAHEDRON
CUBE
OCTAHEDRON
DODECAHEDRON
(A) (B) (C) (D)
T–T T–O T–D T–I
C–O C–D C–C D–D
I–D I–I I–O O–C
Answer: (A)
The top row shows the original Platonic solids. The middle row shows the vertex
figures of the original solid as lines superposed on the tangential polygons forming
the corresponding duals. Finally, the polyhedron compounds consisting of a
polyhedron and its dual are illustrated in the bottom row.
Archemedian Solids: Solids with two or more regular polygon with identical vertex.
There are 13 archemidian solids.

Q. 57 Indicte the correct combination of names of the following bolts

(A) (B) (C) (D)


P L – Bolt Wood Bolt Rectangular Bolt Crank Bolt
Q Coach screw Wood screw Needle screw Coach screw
R Purlin bolt J- Bolt Hook Bolt Hook Bolt
S Left crank bolt Left Hand Bolt Skewed Bolt Cranked Hook bolt
Answer: B

Q. 58 Match the following landscape architects listed in Group A with their works listed in
Group B
Group A Group B
P Andre Le Notre 1 Versailles, France
Q Valladier 2 The Piazza del Popolo, Rome
R John Wood 3 The Regents Park
S John Nash 4 Palace de la Concorde
5 Terrace Trivoli Garden
6 The Royal Crescent & Circus at Bath
(A) (B) (C) (D)
P3 P1 P3 P3
Q4 Q2 Q2 Q2
R6 R6 R4 R6
S5 S3 S6 S5
Answer: (B)

Q. 59Identify two attributes which best describe to the works of Michael Graves
P Anthropomorphic Q Aesthetic irony
R Traditional Symmetry S Symbolic
(A) P, S (B) P, Q (C) Q, R (D) R, S
Answer: (D) R, S
Anthropomorphic: resembling or made to resemble a human form.
Aesthetic irony:
Traditional symmetry: extrinsic geometric properties of the shape.
Symbolic architecture: Term coined by Charles Jencks in the 1980s to
describe architecture with cultural ideas, historical references, and other pre-
Modernist themes.
Portland Building at Oregon, USA, Resort World Sentosa, Singapore was designed
by Michael Graves,

. 60 Select three important factors from the following, which were considered by Louis I.
Kahn in his works
P Materials and the ways they can be joined together
Q Heights
R Sensitivity to light
S Grand Spaces
T Glass and steel
(A) P, Q, R (B) P, R, S (C) R, S, T (D) P, S, T
Answer: (B) P, R, S
Important Buildings by Louis I. Kahn
1959 – Salk Institute for Biological Studies, 10 North Torrey Pines Road, La
Jolla, California
1962 – Indian Institute of Management, Ahmedabad, India
1962– Jatiyo Sangshad Bhaban, the National Assembly Building of
Bangladesh, Dhaka, Bangladesh
1963 – President’s Estate, Islamabad, Pakistan (unbuilt)
1974 – Franklin D. Roosevelt Four Freedoms Park, Roosevelt Island, New
York City, New York (Completed 2012)

Q. 61The abstract form, shown below of an ancient Indian Settlement represents

(A) Kurmaka
(B) Nandyavarta
(C) Padmaka
(D) Swastika
Answer: (B) Nandyavarta
Q. 62Match the following commands in the AUTOCAD.
P REVSURF 1 – Reverse the last command
Q SURFTAB 2 – Makes a view portable to other software applications
R VIWPORT 3- revolve an entity
S SHX 4- opens another window
5- surfing the table generated
6- assigning the mesh density
7- related to shape file
(A) P-1, Q-5, R-2, S-6 (B) P-2, Q-6, R-4, S-5
(C) P-5, Q-3, R-7, S-4 (D) P-3, Q-6, R-4, S-7
Answer: (D) P-3, Q-6, R-4, S-7

. 63 Identify the appropriate combination of the following items vis-à-vis their description,
which may be either True or False.
P SPLINE generates curve on the polyline
Q LTYPEGEN generates continuity of linetype in a polyline
R WCS Window compression by scaling
S XREF references X coordinate of the element
(A) P-F, Q-T, R-T, S-T
(B) P-T, Q-T, R-F, S-F
(C) P-F, Q-F, R-F, S-T
(D) P-T, Q-F, R-T, S-F
Answer:

Q. 64Identify the appropriate match between Group A and Group B from the choices given
below.
Group A Group B (Computer Programming Language)
P Main ( ) { ……..} 1 Pascal
Q Begin { …..} End 2 Lisp
R (Defun ( ) …….) 3 Basic
S Format (* ,*) ….. 4 Fortran
T REM…………… 5 C
(A) P-5, Q-3, R-1, S-2, T-4 (B) P-5, Q-4, R-2, S-3, T-1
(C) P-3, Q-1, R-4, S-2, T-5 (D) P-5, Q-1, R-2, S-4, T-3
Answer: (D) P-5, Q-1, R-2, S-4, T-3

. 65 Match the following building materials in Group 1 with their predominant use in places as
mentioned in Group 2.
Group 1 Group 2
P Glazed Bricks 1 Mohenzodaro
Q Tufa 2 Mesopotemia
R Fine grained marble 3 Greece
S Sun dried bricks 4 Rome
5 Egypt
(A) P-1, Q-3, R-2, S-5 (B) P-2, Q-4, R-3, S-1
(C) P-2, Q-5, R-3, S-4 (D) P-1, Q-3, R-4, S-5

Answer: (B) P-2, Q-4, R-3, S-1


Tufa is a variety of lime stone.

Q. 66Match the name of books in Group 1 with the authors listed in Group 2.
Group 1 Group 2
P Design of cities 1 Fredric Gibbard
Q Town design 2 Kevin Lynch
R Site Planning 3 Edward N Bacon
S Comments on Neighbourhood concept 4 Ebenezer Howard
5 Hans Bluemenfeld
6 Charles Abrams
(A) P-3, Q-2, R-6, S-4, (B) P-3, Q-1, R-2, S-5,
(C) P-6, Q-5, R-3, S-1, (D) P-5, Q-6, R-3, S-2,
Answer: (B) P-3, Q-1, R-2, S-5,

Q. 67For a stadium select most suitable sequence of design considerations.


P. Circulation Requirement Q. Easy access of seating
R. Image of technology S. Visual Communication
(A) Q, S, P, R (B) P, Q, R, S (C) S, R, P, Q (D) Q, P, R, S
Answer: (D) Q, P, R, S

. 68 A good thermal insulating materials should possess the several qualities. Select the
combination of best three factors.
P. Fire-resistance
Q. Moisture resistance
R. Strength
S. Stability
T Soundproofing
(A) P, Q, S (B) Q, R, S (C) P, Q, R (D) R, S, T
Answer: (B) Q, R, S

Q. 69Match the towns in Group 1 with their locations in Group 2.


Group 1 Group 2
P Miletus 1 Roman
Q Timgad 2 Greek
R Monpagier 3 French
S Place Daupkine 4 English
5 German
6 Russian
(A) P-4, Q-1, R-4, S-6, (B) P-1, Q-2, R-5, S-3,
(C) P-5, Q-6, R-3, S-4, (D) P-2, Q-1, R-4, S-3,

Answer: (D) P-2, Q-1, R-4, S-3,


Miletus: Ancient Greek Town. Miletus is the birthplace of the Hagia Sophia’s
architect (and inventor of the flying buttress) Isidore of Miletus and Thales. It
is also the birth place of Hippodamus.
Hippodamus (498 BC – 408 BC): According to Aristotle (in Politics),
Hippodamus was a pioneer of urban planning and he devised an ideal city to
be inhabited by 10,000 men (free male citizens), while the overall population
including the correspondent women, children and slaves would reach 50,000
people. He designed the city of Miletus and Rhodes. He is known as Father of
City Planning. (Hippocrates is known as Father of Modern Medicine)
Timgad: Ancient Roman town
Monpagier: French town
Place Daupkine: French Square

. 70 Match the following paints in Group 1 against their most important quality as mentioned
in Group 2.
Group 1 Group 2
P Cement Paint 1 Water proofing
Q Plastic Emulsion Paint 2 Glossy appearance
R Oilbound distemper 3 Decorative appearance
S Synthetic Enamel paint 4 Easily washable
5 Fire- resistant
6 Acid resistant
(A)P-1, Q-3, R-4, S-5, (B) P-2, Q-3, R-5, S-3,
(C) P-1, Q-6, R-3, S-4, (D) P-2, Q-1, R-4, S-6,
Answer: (C) P-1, Q-6, R-3, S-4,
Red Lead- base paint for iron material.
White Lead (Lead Carbonate) – base paint for wood.

Q. 71Strips of two different metals firmly joined together as shown in the figure.

Select the changed profile of the joined strips while heated from the figure shown below.
(A) P (B) Q (C) R (D) S
Answer: (B) Q

Q. 72Match the propagators with the following concepts.


Propagators Concepts
P Total Architecture 1. Patrick Gedes
Q Ekistics 2 Jean Gottman
R Megalopolis 3 Doxiadis
S Radburn 4 Walter Gropius
T Conservative Surgery 5 Clearence Stein
(A) P-4, Q-3, R-2, S-5, T-1 (B) P-3, Q-4, R-2, S-5, T-1
(C) P-5, Q-2, R-4, S-1, T-3 (D) P-4, Q-5, R-3, S-2, T-1

Answer: (A) P-4, Q-3, R-2, S-5, T-1


Ekistics- Doxiadis
Megalopolis –It is defined as a chain of roughly adjacent metropolitan areas.
The term was used by Patrick Geddes in his (1915) book “Cities in
Evolution”, by Oswald Spengler in his (1918) book, The Decline of the
West, and Lewis Mumford in his (1938) book, The Culture of Cities, which
described it as the first stage in urban over development and social decline.
Later, it was used by Jean Gottmann in 1954, to describe the chain of
metropolitan areas along the northeastern seaboard of the U.S. extending
from Boston, Massachusetts through New York City, Philadelphia, Baltimore,
and ending in Washington, D.C. and Northern Virginia.
Radburn: Founded in 1929. Town for the motor age. Planned by Clarence
Stein and Henry Wright. Famous for Pedestrian path system that does not
cross any major roads at grade. Cul de sac was introduced for the first time in
USA.

Q. 73In Gandhi Smarak Sangrahalaya architect Charles Correa used different types of
finishes. Select the combination of finishes used in the building.
P Stone Cladding Q Exposed brick
R Plastering and painting S Marble chip finish
T Exposed concrete U Plain cement concrete finish
(A) R, S, T (B) Q, T, U (C) P, Q, R D) Q, R, S
Answer: (B) Q, T, U
Wooden doors, stone floors, ceramic tile roofs, and brick columns are the features of the
building. Load bearing brick columns support concrete
channels, which support the wooden roof and direct the rainwater
Boards are nailed underneath the joists and tiles are placed atop the joints. The
foundation is concrete and is raised about a foot from the ground

Q. 74Match the correct names of the architectural styles and periods.

1 2 3 4
Moorish Style Romanesque Style Gothic Style Renaissance Style
15th Century 11th- 12th Century 13th- 14th Century 15th- 16th Century
(A) P-1, Q-2, R-3, S-4, (B) P-2, Q-4, R-4, S-1,
(C) P-1, Q-3, R-4, S-1, (D) P-2, Q-1, R-2, S-2,
Answer: (C) P-1, Q-3,R-4, S-1

Q. 75 A 4 cm x 4 cm area on a map represented a land area of 16 hectares of ground. If this


map is transformed to a scale of 1: 5000 the same ground area will be represented by
(A) 80 square (B) 32 square (C) 64 square(D) 40 square
Answer: (C) 64 square

. 76 The figures 1 and 2 represent two different case of building forms, building layouts and
the site features on sites located respectively in hot- humid and hot- dry regions.
Considering the factors listed below which of the following options compared best the
relative presence and absence of these factors in the two cases.
Factors
V Cross Ventillation
P Privacy in outdoor space
E Exposure to outside view

(A) Figure 1 has more V, more P and better E than figure 2


(B) Figure 2 has less V, more P and better E than figure 1
(C) Figure 2 has more V, more P and better E than figure 1
(D) Figure 1 has more V, less P and better E than figure 2
Answer: (B) Figure 2 has less V, more P and better E than figure 1

Q. 77 The graph below shows urban population growths of four countries (P, Q, R and S)
with time.
Which of the following statements is INCORRECT ?
(A) Urbanisation started much later in ‘P’ than in ‘S’ and it has achieved lower growth
rate.
(B) Growth rate is higher in ‘Q’ than in ‘R’.
(C) Growth rate is higher in ‘P’ than in ‘Q’, ‘R’, ‘S’ after 1900.
(D) Urban population is higher in ‘R’ than in ‘Q’.
Answer: (A) Urbanisation started much later in ‘P’ than in ‘S’ and it has achieved
lower growth rate.

Q. 78 The following figures show four different unmanaged traffic intersections in a city.

Which of the following statements shows the


correct numbers of potential traffic conflict points (excluding merging points) at the
intersections ?
(A) P- 8, Q-3, R-3, S-2
(B) P-16, Q-3, R-3, S-0
(C) P- 8, Q-3, R-3, S-0
(D) P-16, Q-3, R-3, S-2
Answer: (B) P-16, Q-3, R-3, S-0
In cross roads there are 16 traffic conflicts points.
- 4 Through traffic
- 4 Right turn
- 8 Right turn- through

Q. 79 Arrange the sequence of events in a Residential site development.


P. Levelling and land filling
Q. Tress and plantation
R. Electrification
S. Allotment
T Provision of roads
U Provision of water supply, sewerage and drainage.
(A) R, S, T, U, P, Q (B) P, R, S,U, T, Q (C) Q, P, S, R, T, U (D) P, T, Q, U, R, S
Answer: (D) P, T, Q, U, R, S
Q. 80Indicate the correct match for items given in Group 1 with items given in Group 2.
Group 1 Group 2
P Stone 1. Pitting
Q Timber 2 Mottle
R Brick 3 Chalking
S Lime 4 Star shake
5 Unburnt

6 Cracking

(A) P-2, Q-4, R-5, S-1 (B) P-2, Q-3, R-1, S-6
(C) P-3, Q-1, R-5, S-2 (D) P-1, Q-3, R-4, S-2
Answer: (A) P-2, Q-4, R-5, S-1
Star shake in timber.
Mottle- it is spot of different shades of colour as in granite.

. 81 A stack of sand measures 2.5 metre x 3.5 metre at the top and 3.5 metre x 4.5 metre at
the base. The average height of the stack is 1 metre. What is the volume of sand in the
stack ?
(A) 10 cum (B) 12 cum (C) 15 cum (D) 18 cum
Answer: (B) 12 cum

Using Average area method to find the volume = 1 x = 12.25


cum.

. 82 In a residential community of 10000 population 20% are Higher Secondary school going
children. The expected enrolment is 80 % and per capita gross floor space required is
3.0 sq m. The ground coverage permissibility is 40 %. Indicate the land area required
for the Higher Secondary school building.
(A) 0.85 hectare (B) 6.0 hectare (C) 1.2 hectare (D) 7.5 hectare

Answer: (C) 1.2 hectare

No of students = 10,000 = 1600 students


Floor space required @ 3.0 sq m / capita= 1600 3 = 4800 sq m.
Considering the school building only one storey total built up area is 4800 sq m which
is 40 % of ground area.
Therefore total land area = 12000 sq m= 1.2 ha

Q.83 Which is the correct match for items given in Group 1 with items given in Group 2.
Group 1 Group 2
P Thermal Inversion 1. Water repellent
Q Efflorescence 2 Stable airflow
R Hydrophobic surface 3 Lecture Hall
S Psychedelic effect 4 Crystallization
5 Globulation
(A) P-2, Q-4, R-1, S-3 (B) P-2, Q-3, R-4, S-1
(C) P-1, Q-4, R-3, S-1 (D) P-5, Q-3, R-1, S-2
Answer: (A) P-2, Q-4, R-1, S-3
In meteorology, an inversion is a deviation from the normal change of an atmospheric
property with altitude. “temperature inversion”, i.e., an increase in temperature with
height. Thermal inversion is an atmospheric condition that occurs when a layer of
warm air settles over a layer of cool air that is near ground level.
Efflorescence (chemistry) – loss of water of crystallization
Efflorescence (botany) - gradual process of unfolding of flower
Efflorescence (civil engg.) – It is a deposit, usually white in color that
occasionally develops on the surface of concrete, often just after a structure is
completed. Water in moist, hardened concrete dissolves soluble salts. This
salt-water solution migrates to the surface by vapor transmission or hydraulic
pressure where the water evaporates, leaving the salt deposit at the surface.
Hydrophobia- fear of water.
Psychedelic effect-a stage of seeing colourful, twisting, spinning, winding, swirling
images and euphoric feeling depending upon drug and the dose.
(Relationship with Lecture Hall confusing)

Q.84 A general hospital has the following functional areas


P Entry Q Emergency
R Out Patient Department S In Patient Department
T Diagnostic Therapeutic Unit U Incinerator
V Mortuary W Kitchen and Laundry

Select from the following group of pairs the one you consider as most appropriate
while designing the hospital.
(A) PR, QW, RS (B) PR, ST, RT (C) RT, PU, RV (D) RV, UW,
QR
Answer: (B) PR, ST, RT

. 85 In high- rise building the walls are built thinner as they get higher. Select the most
important pair of factors.
P. Thinner wall has less weight to bear at the top
Q. Thinner wall are suitable for earthquake
R. Thinner wall look beautiful
S. Thinner wall can take electrical conduit pipe easily
T Thinner wall have moment of inertia
(A) P, R (B) P, Q (C) R, S (D) Q, T
Answer: (B) P, Q

. 86 In the landscape drains correlate the drain types with their positive or negative
attributes, and select the appropriate combinations.
P Open drain 1. Economic
Q Swale 2 easy collection
R French Drain 3 susceptible to clogging
S Pipe drain 4 high rate of flow
5 easy maintenance
6 visual scar
(A) P - 1, 2, 4, 6 (B) Q - 2, 3, 6, 4
(C) R – 1, 2, 5, 6 (D) S – 1, 4, 5, 6

Answer: (A) P - 1, 2, 4, 6
Open drain is open nala.
Swale is a low tract of land, especially one that is moist or marshy. The term
can refer to a natural landscape feature or a human-created one. (GATE 2016)
French drains are primarily used to prevent ground and surface water from
penetrating or damaging building foundations. Alternatively, French drains may
be used to distribute water, such as a septic drain field at the outlet of a typical
septic tank sewage treatment system. French drains are also used behind
retaining walls to relieve ground water pressure. Is a trench filled with gravel or
rock or containing a perforated pipe that redirects surface water and
groundwater away from an area.

Q. 87 Match the tree forms with their corresponding functions as listed below.

1 Visual screening
2 Shading of functions around
3 Backdrop
4 Water side edge planting
5 Boundary edging of a functional area
6 Scale induction of the adjoining object

(A) P-3, Q-1, R-5, S-2


(B) P-1, Q-3, R-4, S-6
(C) P-1, Q-2, R-5, S-4
(D) P-3, Q-6, R-1, S-4

Answer: (D) P-3, Q-6, R-1, S-4


Crown: The crown of a tree consists of the mass of foliage and branches
growing outward from the trunk of the tree.
Canopy : The uppermost layer in a forest, formed by the crowns of the trees.
Also called crown canopy.

. 88 The Age- Sex pyramid of an urban area shows a bulge in the age group of 15 to 25.
Select the likely correct group of inference.
P Birth rate has fallen
Q In- migration is high
R Out- migration is high
S Death rate has fallen
T Males are more than females
(A) P – Q (B) P – R (C) R – T (D) S – T
Answer: (A) P – Q
Demographic cycle of population. (India is in stage III)
S. Stage Birth/death Category
No
1 Stage I High birth/ high death High
stationary
2 Stage II High birth/ death decline Early
expansion
3 Stage III Birth marginal decline/ Late
death decline expansion
4 Stage IV Low birth/ death decline Low
stationary
5 Stage V Low birth/ high death High decline

Q. 89 Identify the elements marked in the


following drawing and match them with the items in the list provided below.

1 Rafter 2 Strut
3 King Post 4 Diagonal tie
5 Flat tie 6 Support
7 Main tie
(A) P – 1, Q- 2, R-3, S- 4, T- 5, U- 6 (B) P – 3, Q- 1, R-3, S- 6, T- 7, U-
4
(C) P – 3, Q- 4, R-5, S- 1, T- 2, U- 7 (D) P – 2, Q- 4, R-5, S- 7, T- 6, U-
3

Answer: (C) P – 3, Q- 4, R-5, S- 1, T- 2, U- 7

Q. 90Select the items in the Group 1, which matches most appropriately with items in Group
2.
Group 1 Group 2
P Air delivery System 1. Adsorption
Q Air distribution system 2 Dampers

R Dehumidifiction 3 Spray washers


4 Diffusers
5 Dry filter
(A) P-1, Q-3, R-5,
(B) P-2, Q-4, R-1,
(C) P-2, Q-4, R-3,
(D) P-1, Q-2, R-3,

Answer: (B) P-2, Q-4, R-1


Cooling coils acts as dehumidifier.
A damper is a plate that helps regulate and control the amount of air that flows
through air conditioning system.
Spray washer cleans the outside air by spraying water over it and adding
moisture to it.
Diffuser is the mechanical device that is designed to control the
characteristics of a fluid at the exit.
Dry filters remove dust particle from the circulated air.

ASHRAE (American Society of Heating, Refrigerating and Air Conditioning


Engineers) recommends that, there be enough ventilation to exchange the air inside
house, once every four hours.
GATE 2005
Q.1 Early Aryan civilization is characterized by
(A) Introduction of arches and domes (B) Wooden construction
(C) Rock cut architecture (D) Flat brick masonry
Answer: (B) Wooden construction

Q.2 Indus valley development is specially known for


(A) Vaulted roofing in masonry (B) Underground drainage system
(C) Great Palaces (D) Stone carving of idols
Answer: (B) Underground drainage system

Q.3 Housing backlog means


(A) Dwelling units back to back
(B) Dwelling units built up to last year
(C) Dwelling units arranged with their backs towards a large courtyard
(D) Accumulated substandard dwelling units to be replaced immediately
Answer: (D) Accumulated substandard dwelling units to be replaced immediately

Q.4 The prefabricated housing project ‘Habitat’ was designed by


(A) Le Corbusier (B) Kenzo Tange (C) VB Doshi (D) Moshe Safdie
Answer: (D) Moshe Safdie
Moshe Safdie: Famous for Habitat 67. A prefabricated concrete forms, residential
complex in Montreal, Canada. Khalsa Heritage
complex, Punjab is his project in India. In Bangladesh, he worked on Asian
University for Women, Chittagong.

Habitat, Montreal
Canada

Moshe Safdie: His important works


1967 Habitat 67 at Expo 67 World’s Fair, Montreal, Quebec, Canada
1989 City plan for Modi’in, Israel
2004 Airside building of Terminal 3, Ben Gurion International Airport, Israel
2005 Yad Vashem Holocaust History Museum, Jerusalem, Israel
2009 Asian University for Women, Chittagong, Bangladesh
2009 Mamilla Mall, Jerusalem, Israel
2011 Marina Bay Sands, Singapore’s second integrated resort and casino
2011 United States Institute of Peace in Washington, D.C., USA
2011 Kauffman Center for the Performing Arts, Kansas City, Missouri, USA
2011 Crystal Bridges Museum of American Art, Bentonville, Arkansas, USA
2011 Khalsa Heritage Memorial Complex, Anandpur Sahib, Punjab, India

Q.5 Buckling of steel column is associated with the stress due to


(A) Shear force and Axial force (B) Axial force and bending moment
(C) Shear force and Torsion (D) Torsion only
Answer: (B) Axial force and bending moment
(Answer by Surbhi Jethani, B Arch, MITS, Gwalior)
Q.6 Belt truss is used in High-Rise buildings with a view to provide
(A) Aesthetic appearance (B) Rigid connection between the columns
(C) Escalators (D) Support to large span floors
Answer: (B) Rigid connection between the columns

Q.7 The sampling rate for household survey of a town with 2 lakhs population is
(A) 1 in 10 (B) 1 in 5 (C) 1 in 8 (D) 1 in 15
Answer: (D) 1 in 15

For 2, 00,000 population (if avg. family size is 5.5) no of household = = 36363
household. 1 in 15 will be 2424 household, which is still high.
Most statisticians agree that the minimum sample size to get any kind of
meaningful result is 100. If your population is less than 100 then you really
need to survey all of them.
A good maximum sample size is usually around 10% of the population, as long
as this does not exceed 1000. For example, in a population of 5000, 10%
would be 500. In a population of 200,000, 10% would be 20,000. This exceeds
1000, so in this case the maximum would be 1000.
Even in a population of 200,000, sampling 1000 people will normally give a
fairly accurate result. Sampling more than 1000 people won’t add much to the
accuracy given the extra time and money it would cost.

Q.8 Identify the most appropriate set that relates to Physical Infrastructure
(A) Water supply, solid waste (B) Housing, education, health Management,
electricity
(C) Petrol pumps, milk booths, LPG (D) Communication, postal services godownsfire
protection services
Answer: (A) Water supply, solid waste Management, electricity

Q.9 Urban design theory that deals with analysis of relationship between building mass and
open space is
(A) Open space theory(B) Figure ground theory(C) Linkage theory (D) Place theory
Answer: (B) Figure ground theory
There are three approaches to urban-design theory which are figure-ground theory;
linkage theory; and place theory.

Figure ground theory : A figure-ground diagram is a


two-dimensional map of an urban space that shows the relationship between built and
unbuilt space.
A figure ground diagram comprises entities called pochés. These are, in simple terms,
groups of structures — or in even simpler terms the black figures on the diagram. A
poché helps to define the voids between the buildings, and to emphasize their
existence as defined objects in their own rights.
Frederick Gibberd was a proponent of the reverse figure-ground diagram, where the
buildings are in white and the spaces black, to focus the perception of the designer
upon the space as an object.
Linkage theory: This theory involves the organization of lines connecting the parts of
the city and indicates flow of movement, axis and building edge. These lines invole
streats, pedestian roads, linear open space.
Place theory: Place theory begins with understanding the cultural and human
characteristics of physical space.

Q.10 Micro- climate refers to


(A) Tropical climate (B) The tropical climate where microorganisms grow fast
(C) Climate created for limited space (D) The climate in and around small islands
Answer: (C) Climate created for limited space

Q. 11Albido refers to
(A) Thermal properties of external surface material (B) Roughness of stone surface
(C) Height of a frame to allow creepers to grow (D) Nosing of steps made in marble
Answer: (A) Thermal properties of external surface material

Q. 12Origin and destination survey helps in identifying


(A) Desire lines of vehicular traffic flow (B) Location of main traffic nodes
(C) Peak capacity of road (D) Peak capacity of road junctions
Answer: (A) Desire lines of vehicular traffic flow.

Q. 13 Water harvesting is
(A) A new method of water purification
(B) Collection of water in paddy fields
(C) Device to allow quick flow of storm water run off into rivers
(D) Device to re direct storm water run off into under ground aquifers
Answer: (D) Device to re direct storm water run off into under ground aquifers
Aquifer: An aquifer is an underground layer of water-bearing permeable rock or
unconsolidated materials (gravel, sand, or silt) from which groundwater can be
extracted.
Aquitard: bed of low permeability along aquifer.
Aquiclude: An impermeable body of rock or stratum of sediment that acts as a
barrier to the flow of groundwater.

Q. 14 Millowners’ Association Building in Ahmedabad was designed by


(A) Charles Correa (B) Bernard Kohn (C) Le Corbusier (D) Louis Kahn
Answer: (C) Le Corbusier
Le Corbusier’s work in India
Town Planning of Chandigarh
High court, Chandigarh
Secretariat, Chandigarh
Governor House, Chandigarh
Govt. College of Art, Chandigarh
Chandigarh College of Architecture, Chandigarh
Museum and gallery of Art, Chandigarh
Shodan House, Ahmedabad (GATE 2016)
Sarabhai House, Ahmedabad
Mill Owners Association Building, Ahmedabad. (also known as ATMA
House)
Sanskar Kendra Museum, Ahmedabad

Louis Kahn (1901- 1974)


Plan of Philadelphia,USA
National Assembly Building, Dhaka, Bangladesh
IIM Ahmedabad, India

Assembly Buildings
Indian Parliament- Leutyens
Pakistan National Assembly- Edward Durel Stone
National Assembly Building, Dhaka, Bangladesh- Louis Kahn
Srilanka Parliament Building- Geoffrey Bawa

Q. 15 Bahai temple in Delhi is a


(A) Tensile structure (B) Shell structure (C) Pneumatic structures (D) Space
structure

Answer: (B) Shell structure


Bahai Temple: Architect F Sahaba. Nine water pools.Shell structure is bulk active
structure.
A tensile structure is a construction of elements carrying only tension and no
compression or bending.
Tensigrity: structural form with both tension and compression elements.
A space frame or space structure is a truss-like, lightweight rigid structure
constructed from interlocking struts in a geometric pattern. Space frames can be
used to span large areas with few interior supports. (adjoining pic)
Pneumatic Membrane structure that is stabilized by the pressure of compressed
air. Air-supported structures are supported by internal air pressure. ... Air-inflated
structures are supported by pressurized air within inflated building elements that are
shaped to carry loads in a traditional manner

Q. 16The leaning tower of Pisa was prevented from further leaning by


(A) Pumping out muddy slurry from under the foundation of the tower that was moving up
(B) Pumping in cement concrete under the foundation of the tower that was moving down
(C) Propping the tower with steel shoring
(D) Relocating the furniture inside the tower
Answer: (B) Pumping in cement concrete under the foundation of the tower that was
moving down

Q. 17 A building is referred to as an ‘Intelligent Building’ primarily due to its capability to


automatically provide
services in terms of
(A) heating, ventilation, air-conditioning and lighting
(B) water supply, laundry, waste water treatment and communication
(C) security, office space facility and office automation
(D) elevator/ escalator operations, smart cards for corridor/ office door operation
Answer: (A) heating, ventilation, air-conditioning and lighting

Q. 18The use of Police Power is most appropriately related to


(A) Fair administration of law
(B) Using powers to pass and enforce laws in public interest
(C) Using powers to pass and enforce laws in private interest
(D) Reduce crime in the neighbourhood
Answer: (A) Fair administration of law
Q. 19 The value of runoff coefficient C in Q= CIA that represents a completely impervious
and wetted surface from which there is a total runoff is
(A) 0 (B) 1 (C) 10 (D) 1 %
Answer: (B) 1
Since the surface is completely impervious all the water will go as runoff.
Q= CIA (also known as Rational Formula)
Where Q = Flowrate
C = Coefficient of Runoff (0 to 1)
I = intensity of the storm (mm/ hr)
A = Area of the Catchment that the rainfall will runoff of.

Q. 20 The role of government agencies in the field of housing started changing since 1970s
in the following way
(A) From provider to facilitator (B) From plot supplier to dwelling unit supplier
(C) From designer to builder (D) From facilitator to provider
Answer: (A) From provider to facilitator

Q. 21A contour line is


(A) An imaginary line connecting points at the same elevation on ground
(B) The line identifying the edge of National Highways
(C) The line identifying the edge of lakes and rivers
(D) An imaginary line connecting hilltops
Answer: (A) An imaginary line connecting points at the same elevation on ground

Q. 22The 74th Constitutional Amendments empowers


(A) The President of India to declare general election without consulting the parliament
(B) The Prime Minister to reschedule Parliament Sessions
(C) Municipal Organizations to undertake planning and development of their respective
towns
(D) Individuals to change political party membership after elections
Answer: (C) Municipal Organizations to undertake planning and development of their
respective towns

Q. 23The best example of synthesis between Indo- Aryans and Dravidian stylistic features in
a temple is
(A) Hoyasaleshwar (B) Madurai (C) Konark (D) Dilwara
Answer: (A) Hoyasaleshwar
Hoyasaleshwar Temple at Halebid is in Hassan District of Karnataka.
Durga Temple at Aihole and other temples of Pattadakal and Badami is in Bagalkot
District of Karnataka.
Both the site of Hassan and Bagalkot represent the synthesis of Nagara and Dravidian
style of temple.

Q. 24Urushringa is the design component used in shikhara of one of the following temple
styles
(A) Bhubaneshwari(B) Dravidian (C) Khajuraho (D) Deccan
Answer: (C) Khajuraho
An urushringa is a subsidiary tower pasted against the main tower (sikhara) in the
Hindu temple architecture. The urushringa is lower and narrower than the sikhara. It
strengthens the feeling of height given by the temple. We find them on the temple of
northern India in the Nagara architecture.

Q. 25 Development that meets the need of the present without compromising the ability of
the future
generations to meet their own need is
(A) Health, education and welfare development (B) Sustainable development
(C) Economic development (D) Infrastructure development
Answer: (B) Sustainable development
This definition first appeared in 1987 from World Commission on Environment and
Development, (the Brundtland Commission) Brundtland was the former Prime
Minister of Norway.
1972 Stockholm Conference on the Human Environment
1980 World Conservation Strategy of the International Union for the Conservation of
Nature (IUCN)
headquartered at Gland, Switzerland.
1992 Earth Summit, Rio de Janeiro (Agenda 21)
2002 Third UN Conference on Environment and Development in Johannesburg,
South Africa in 2002

2003 Kyoto Protocol (Green House Gases)

Q. 26Shell structures derives its strength primarily from its


(A) Shape (B) Wall thickness
(C) Strength of the materials (D) Size
Answer: (A) Shape

Q. 27The primary role of braces in a High- Rise building is to


(A) Resist the gravity load (B) Resist the lateral sway
(C) Improve the elevation (D) Achieve large storey sway
Answer: (B) Resist the lateral sway
Braces resist the buckling of the structure under seismic load.

Q. 28Weep holes refer to


(A) Internal duct in human anatomy connecting eyes and throat
(B) Rain water pipes connecting upper terrace to next lower one
(C) Hole made in a log to remove moisture
(D) Holes in retaining wall to allow water to drain out
Answer: (D) Holes in retaining wall to allow water to drain out

Q. 29A structural device called pendentive is used in


(A) Closing the end of a vault
(B) Decorating the corners of a room
(C) Making the vault on a long passage
(D) Making a dome on a square plan
Answer: (D) Making a dome on a square plan

Q. 30The dome of Gol Gumbaj at Bijapur is supported by


(A) Intersecting walls
(B) Filling inside corners
(C) Intersecting arches
(D) Massive piers
Answer: (C) Intersecting arches
“Eight intersecting arches created by two rotated squares that create interlocking
pendentives” support the dome”.
Q. 31 Match the type of buildings in Group 1 with their type of foundation in Group 2
Group 1 Group 2
P Single storeyed 1 Isolated footing
Q Three storeyed 2 Raft foundation
R 5 to 8 storeyed 3 Pile foundation
S 10 storeyed and above 4 Well foundation
(A) P-1, Q-3, R-4, S-1 (B) P-4, Q-3, R-2, S-1 (C) P-1, Q-1, R-2, S-3 (D) P-2, Q-3, R-
4, S-2
Answer: (C) P-1, Q-1, R-2, S-3

Q. 32Match the figures Group 1 with the concepts in Group 2

P 1 Christaller’s concept

Q 2 Garden City concept

R 3 Multiple nuclei concept

4 Radburn concept

(A) P-1, Q-3, R-4 (B) P-2, Q-1, R-4 (C) P-2, Q-3, R-1 (D) P-1, Q-3, R-2
Answer: (B) P-2, Q-1, R-4

Christaller’s concept: Christaller was a German


geographer. Settlements would tend to form in a triangular/hexagonall lattice,
pattern to serve areas without any overlap.
Garden City Movement: Propagated by Ebenezer Howard. Letchworth, UK was the first
garden city built in 1909.

Multiple nuclei concept: (Chauncy Harris


and Edward Ullman) Even though a city may have begun with a Central business district,
or CBD, other smaller CBDs develop on the outskirts of the city near the more valuable
housing areas to allow shorter commutes from the outskirts of the city. This creates
nodes or nuclei in other parts of the city besides the CBD thus the name multiple nuclei
model.
Radburn Concept: Founded in 1929. Town for the motor age. Planned by
Clarence Stein and Henry Wright. Famous for Pedestrian path system that does not
cross any major roads at grade. Cul de sac was introduced for the first time in USA.

Q. 33Match the appropriate noise level in Group 2 with the building types in Group 1.
Group 1 Group 2
P Radio and TV Studios 1 35- 40 dB
Q Industry 2 40- 45 dB
R Hospitals and auditoria 3 25- 30 dB
S Court rooms & class rooms 4 70- 75 dB
5 50- 55 dB
(A) P-1, Q-2, R-3, S-4 (B) P-2, Q-3, R-4, S-1 (C) P-3, Q-4, R-1, S-2 (D) P-4, Q-5, R-
3, S-2
Answer: (C) P-3, Q-4, R-1, S-2

Q. 34 Match the period/ style in Group 1 with their most significant contribution in the progress
of Architecture in Group 2.
Group 1 Group 2
P Egyptian 1 New bonding material
Q Greek 2 Building skeleton and skin
R Roman 3 Trabeation
S Gothic 4 Optical corrections
(A) P-2, Q-1, R-3, S-4 (B) P-3, Q-4, R-1, S-2 (C) P-1, Q-2, R-3, S-4 (D) P-4, Q-3, R-2,
S-1
Answer: (B) P-3, Q-4, R-1, S-2
New Bonding material (pozzolona: type of cement) was used in Pantheon in
Rome.
Gothic: Flying buttress and rose window with glass.
Trabeation: Having
horizontal beams or lintels rather than arches.
Entasis is the application of a convex curve to a surface for optical correction.
The Classical columns were slightly bulged out a bit above the base.
. 35 A sector has a gross area of 65 hectares and a residential area of 50 hectares. If net
residential density is 325 pph, what is the gross density of the sector?

(A) 275 pph (B) 225 pph (C) 300 pph (D) 250 pph
Answer: (D) 250 pph

= 250 pph

Q. 36 If the height of the building is h and the distance of


viewing is d, then distance relation ratio is given
by h:d. Watch the h:d in Group 1 with their
characteristics in Group 2.
Group 1 Group 2
P 1:1 1 Tend to see object as an edge
Q 1:2 2 Tend to notice details more than facade
R 1:3 3 Tend to see objects as a whole together
with its details
S 1:4 4 Tend to see objects in relation to
surrounding objects
(A) P-2, Q-3, R-4, S-1 (B) P-2, Q-4, R-3, S-1
(C) P-1, Q-3, R-4, S-2 (D) P-4, Q-2, R-3, S-1
Answer: (B) P-2, Q-4, R-3, S-1
source: The Architecture of Town and Cities, Paul D Spreiregen.

Q. 37 Match Auto Cad system variables in Group 1


with their functions in Group 2.
Group 1 Group 2
P Mirrtext 1 Sets drawing units
Q Measurement 2 Mirrors the text object
R Hidetext 3 Sets units for angle
S Aunits 4 Controls how mirror command reflects
text
5 Controls the visibility of text
commands during hide
command
6 Hides the text objects
7 Measures the distance between two points
(A) P-4, Q-1, R-5, S-3 (B) P-2, Q-7, R-6, S-3
(C) P-2, Q-7, R-5, S-1 (D) P-4, Q-7, R-6, S-1
Answer: (D) P-4, Q-7, R-6, S-1
Q. 38The correlation between population size of cities
and area under residential use is depicted by the
curves A, B, C, D. Choose the correct one.
(A) Curve A (B) Curve B (C) Curve C (D) Curve D
Answer: (D) Curve D

Q. 39To describe the traffic characteristics of a city, show how you would identify the
volume of
Group 1 Group 2
P Peak hour traffic 1 By quantifying traffic that stops
Q Through traffic 2 By determining the maximum hourly traffic volume
R Terminating traffic 3 By observing the traffic that does not stop inside the city
(A) P-1, Q-2, R-3 (B) P-2, Q-3, R-1
(C) P-3, Q-1, R-2 (D) P-2, Q-1, R-3
Answer: (B) P-2, Q-3, R-1

Q. 40Match the typology of gardens in Group 1 with their typical features in Group 2.
Group 1 Group 2
P Mughal Garden 1 Moss, manicured trees, rocks, water
Q Italian Garden 2 Cycle tracks, meandering walkways, benches
R Japanese Garden 3 Sculpture of pretty nymphs, angels, fountains
S City Forest 4 Geometric floral patterns, flowing water, stone Shelters.
(A) P-1, Q-4, R-3, S-2 (B) P-3, Q-2, R-4, S-1 (C) P-4, Q-3, R-1, S-2 (D) P-2, Q-1,
R-3, S-4
Answer: (C) P-4, Q-3, R-1, S-2
Mughal Garden:
Shalimar Bag, Lahore (Built by Babur)
Humayu’s Tomb, Delhi.
Shalimar garden in Kashmir (Initiated by Jehangir)
Taj Garden, Agra (Built by Sahjahan)
Mahtab Bagh, Red Fort Agra (Built by Sahjahan)
Khusro Bagh, Allahabad
Nishat Garden, Jammu and Kashmir
Garden of President Estate, New Delhi (Built by Leutyens)
Lal Bagh Fort, Dhaka, Bangladesh
Ismili Centre, Lisbon, Portugal, (Raj Rewal)
Features of Mughal Garden:
Walled, protected from outside world.
Flowing water from four cardinal directions depicting honey, milk, wine
and water.
Gardens in pure geometric pattern.
Backdrop contained fort, museloeum, riverfront or mountain.
pool to reflect the beauties of sky and garden.
trees of various sorts, some to provide shade, and others to produce
fruits and flowers, colorful and sweet-smelling; grass.
Features of Japanese Garden:
Miniature idealized landscape
Japanese gardens always have water, either a pond or stream, or, in the
dry rock garden, represented by white sand. A traditional garden will
usually have an irregular-shaped pond, or, in larger gardens, two or
more ponds connected by a channel or stream, and a cascade, a
miniature version of Japan’s famous mountain waterfalls.
Water body with island.
Use of rock, sand and gravel.
Stepping stones
Features of Chinese Garden:
Chinese gardens are filled with architecture; halls, pavilions, temples,
galleries, bridges, kiosks, and towers, occupying a large part of the
space.
A Chinese garden was not meant to be seen all at once; the plan of a
classical Chinese garden presented the visitor with a series of perfectly
composed and framed glimpses of scenery; a view of a pond, or of a
rock, or a grove of bamboo, a blossoming tree, or a view of a distant
mountain peak or a pagoda.
In addition to these larger halls and pavilions, the garden is filled with
smaller pavilions, (also called ting) which are designed for providing
shelter from the sun or rain, for contemplating a scene, reciting a poem,
taking advantage of a breeze, or simply resting.
The classical garden was surrounded by a wall, usually painted white,
which served as a pure backdrop for the flowers and trees.
A pond of water was usually located in the center. Many structures,
large and small, were arranged around the pond.
The artificial mountain rock garden is an integral element of Chinese
classical gardens.
Flowers and trees, along with water, rocks and architecture, are the
fourth essential element of the Chinese garden.
Features of Italian Garden:
Statues (Sculpture of pretty nymphs, angels)
Fountains
Grottoes (natural or artificial cave)
Water organs
Features of French Garden:
A geometric plan using the most recent discoveries of perspective and optics.
A terrace overlooking the garden, allowing the visitor to see all at once the
entire garden.
All vegetation is constrained and directed, to demonstrate the mastery of man
over nature. Trees are planted in straight lines, and carefully trimmed, and
their tops are trimmed at a set height.
The residence serves as the central point of the garden, and its central
ornament. No trees are planted close to the house; rather, the house is set
apart by low parterres and trimmed bushes.
A central axis, or perspective, perpendicular to the facade of the house, on the
side opposite the front entrance. The axis extends either all the way to the
horizon (Versailles) or to piece of statuary or architecture (Vaux-le-Vicomte).
The axis faces either South (Vaux-le-Vicomte, Meudon) or east-west (Tuileries,
Clagny, Trianon, Sceaux). The principal axis is composed of a lawn, or a basin
of water, bordered by trees. The principal axis is crossed by one or more
perpendicular perspectives and alleys.
The most elaborate parterres, or planting beds, in the shape of squares, ovals,
circles or scrolls, are placed in a regular and geometric order close to the
house, to complement the architecture and to be seen from above from the
reception rooms of the house.
The parterres near the residence are filled with broderies, designs created with
low boxwood to resemble the patterns of a carpet, and given a polychrome
effect by plantings of flowers, or by colored brick, gravel or sand.
Farther from the house, the broderies are replaced with simpler parterres,
filled with grass, and often containing fountains or basins of water. Beyond
these, small carefully created groves of trees, serve as an intermediary
between the formal garden and the masses of trees of the park. “The perfect
place for a stroll, these spaces present alleys, stars, circles, theaters of
greenery, galleries, spaces for balls and for festivities.”
Bodies of water (canals, basins) serve as mirrors, doubling the size of the
house or the trees.
The garden is animated with pieces of sculpture, usually on mythological
themes, which either underline or punctuate the perspectives, and mark the
intersections of the axes, and by moving water in the form of cascades and
fountains.
Features of English Garden: (Kent, William Capability Brown, Humphrey Repton)
The English garden usually included a lake.
Sweeps of gently rolling lawns set against groves of trees.
Ornate carpets of floral designs and walls of hedges, decorated with statues
and fountains.
Recreations of classical temples, Gothic ruins, bridges, and other picturesque
architecture
Grottoes (natural or artificial caves), temples, tea-houses, belvederes
(architectural structure to augment the scenic beauty), pavilions, sham ruins
(false impression of old ruined structure), bridges and statues

Q. 41 Choose TWO abbreviations from the following that are very commonly used in
transportation planning.
P MOU T DJB
Q DPC U PCU
R ROW V RAW
S ITI W EIA
(A) MOU, ROW (B) ITI, DJB (C) ROW, PCU (D) ROW, EIA
Answer: (C) ROW, PCU
MOU: Memorandum of Understanding
DPC: Damp Proofing Course
ROW: Right of Way
ITI: x
DJB: x
PCU: Passenger Car Unit
RAW: x
EIA: Environmental Impact Assessment

Q. 42Find the correct matches in the given list of architectural entities and the architectural
style/ period
Group 1 Group 2
P Temple 1 Mohenjo-Daro
Q Public Bath 2 Greek
R Agora 3 Roman
S Granary 4 Mayan
(A) R-3, R-4 (B) Q-1, Q-3 (C) S-3, S-4 (D) P-2, P-3
Answer: (B) Q-1, Q-3
Thermae in Rome and Great Bath in Mohenjodaro.

Q. 43Stress due to bending moment may be computed through appropriate combination from
the following parameters. Select the appropriate choice out of the ones given below.
M Bending moment I Moment of Inertia
y Distance from neutral axis E Young’s modulus of elasticity
R Radius of curvature Stress due to bending
(A) R= y E (B) M I= E (C) I= M R (D) y= M E
Answer: Marks to all
Correct ans I= M y

Q. 44Match the shape of the age- sex pyramid and their interpretations.
P Q R S

1. City offers high employment opportunity


2. City’s population is aging
3. City has poor employment opportunity
4. City has young population
(A) P-4, Q-1, R-2, S-3 (B) P-1, Q-2, R-3, S-4 (C) P-2, Q-3, R-4, S-1 (D) P-3,
Q-4, R-1, S-2
Answer: (B) P-1, Q-2, R-3, S-4

Q. 45 A plot of land measuring 25 m x 40 m has a Ground+4 storeyed building with uniform


floor areas. If the land has been utilized to its fullest, and FAR is 250, calculate the
permissible ground coverage.
(A) 50% (B) 5% (C) 25% (D) 40%
Answer: (A) 50%
Total Plot area = 25 m x 40 m= 100 sqm.
FAR = 250 means 2.5 (amazing, somewhere we use 2.5 and somewhere 250, no
uniformity!)
Total permissible build up area of all floors = plot area x FAR
= 1000 × 2.5= 2500 sqm
No of floors = G+4= 5 floors

Area of each floor = = 500 sqm

Permissible Ground Coverage= × 100= 50 %

Q. 46Choose the two most appropriate design approaches that signify Laurie Baker’s work.
P Recycling of materials
Q Imaginative use of bricks in construction
R Innovative construction methods to save cost
S New vocabulary in indigenous construction
(A) P, R (B) Q, S (C) R, S (D) P, S
Answer: (B) Q, S

Q. 47A two colour / tone colour scheme is proposed for an office building. Which is the most
appropriate set of combinations between the desired effect and the colour scheme?
Group 1 Group 2
P Contrasting 1 Complementary colours
Q Formal 2 Extreme grey values
R Neutral 3 Light colours
(A) P-2, P-3 (B) Q-1, Q-3 (C) R-1 (D) P-1, P-2
Answer: (A) P-2, P-3

Q. 48The efficiency of a modern housing scheme can be best accessed by


P Efficient planning of a dwelling unit
Q Better carpet area ratio
R Imported finishes and textures
S Good general common facilities
T Good landscaping
(A) R, S, T (B) Q, S, T (C) P, Q, S (D) P, R, T
Answer: (C) P, Q, S

Q. 49Given below are statements related in AutoCad


Group 1 Group 2
P Mspace: switches from model Space to paper space 1 True
Q Undo: restores last object erased 2 False
R Multiple: Repeats the command you enter until you press ESC
S Layerp: restores a deleted layer
(A) P-1, Q-2, R-1, S-1 (B) P-2, Q-2, R-1, S-2 (C) P-2, Q-1, R-2, S-1 (D) P-2, Q-1,
R-2, S-2
Answer:

Q. 50 A tract of land having a concave slope implies that


(A) Contour lines are spaced at decreasing distance in the downhill direction.
(B) Contour lines are equally spaced.
(C) Contour lines are spaced at increasing distance in the downhill direction.
(D) There is no trend in spacing of contour lines.
Answer: (A) Contour lines are spaced at decreasing distance in the downhill direction.

. 51 Given below is a sketch plan of a hilly site and two points to be connected by road.
Select the best road alignment. Contour interval is 2 m.

(A) Alignment A (B) Alignment B (C) Alignment C (D) Alignment D


Answer: (B) Alignment B

Q. 52Select the most appropriate sequence of the major


spaces in a Hindu temple
(A) Antaral → Mandapam → Ardhamandapam → Garbhgruh
(B) Mandapam → Antaral → Ardhamandapam → Garbhgruh
(C) Garbhgruh→ Antaral → Ardhamandapam → Mandapam
(D) Ardhamandapam → Mandapam→ Antaral→ Garbhgruh
Answer: (D) Ardhamandapam → Mandapam→ Antaral→ Garbhgruh

Q. 53 Identify the devices from Group 1 that prevent sliding of truss roof components
in Group 2
Group 1 Group 2
P Screw 1 Battens
Q Bolt 2 Purlins
R Wall plate 3 Rafters
S Cleat 4 Struts
(A) P-3, P-4 (B) S-3 (C) S-2 (D) P-2, R-4
Answer: (C) S-2

Q. 54 Choose the most appropriate set of facilities to be provided at the Sector level for
about 20,000 inhabitants in a town of 3,25,000 inhabitants.
P High School Q College
R Play field S Shopping centre
T Civic Centre
(A) P, Q, S (B) Q, R, T (C) P, R, S (D) Q, S, T
Answer: (C) P, R, S
Q. 55In different climatic zones, different set of devices are used to achieve comfort in a
dwelling. Choose the most appropriate set.
Group 1 Group 2
P Warm and Humid 1 Thick walls
Q Hot and dry 2 Central Courtyard
R Composite 3 Thin walls
4 Small openings
5 Good ventilation
6 Evaporative cooling
(A) Q- 1, Q- 2, Q- 6 (B) P-2, P-3, P-6 (C) R-1, R-4, R-6 (D) P-1, Q- 3, R-2
Answer: (A) Q- 1, Q- 2, Q- 6

Q. 56Which of the following characterize Fatehpur Sikri ?


P Urban design in Mughal era
Q City without roads
R Fine marble carvings
S Synthesis of Hindu and Islamic architectural styles
T Well laid out gardens
(A) R, S, T (B) P, Q, S (C) Q, R, S (D) Q, S, T
Answer: (B) P, Q, S

Q. 57 Match the type of traps in Group 1 with their configuration in Group 2


Group 1 Group 2
P P- trap 1 Outlet is at 60 deg to the inlet
Q Q- trap 2 Outlet is parallel to inlet
R S- trap 3 Outlet is at 45 deg to the inlet
4 Outlet is at 90 deg to the inlet
(A) P-3, Q-1, R-2 (B) P-2, Q-1, R-4 (C) P-1, Q-3, R-4 (D) P-4, Q-3, R-2
Answer: (D) P-4, Q-3, R-2

Q. 58Select the most appropriate combination of elements that can be labeled as “Street
Hardware”.
P Advertisement kiosk T Battens
Q Junction boxes U Letter Box
R Signals V Bus shelter
S Planter beds W Traffic signs
(A) Q, R, T, W (B) P, Q, S, V, W (C) Q, R, S, T, U, W (D) Q, R, S, V, W
Answer: No appropriate answer
Street furniture and street hardware is confusing term.
Street furniture is a collective term for objects and pieces of equipment installed on
streets and roads for various purposes. It includes:

benches
traffic barriers
bollards
post boxes
phone boxes
streetlamps
traffic lights
traffic signs
bus stops
tram stops
taxi stands
public lavatories
fountains
watering troughs
memorials
public sculptures
waste receptacles

Q. 59Match the properties of soil in Group 1 with their characteristics in Group 2


Group 1 Group 2
P Permeability 1 Load which a soil is able to support
Q Shear strength 2 Susceptibility to erosion.
R Bearing capacity 3 The ease with which water can flow through soil .
S Ph level 4 Determines the stability of a soil and its ability to resist failure under
loading.
5 Shows the ability of the soil to support plant grow.
6 Angle at which an unconsolidated soil will naturally slope

(A) P-3, Q-6, R-1, S-2 (B) P-5, Q-1, R-4, S-2
(C) P-5, Q-4, R-1, S-6 (D) P-3, Q-4, R-1, S-5
Answer: (D) P-3, Q-4, R-1, S-5

Q. 60 Match the AutoCad commands in Group 1 with their functions in Group 2


Group 1 Group 2
P Xline 1 Create solid
Q Bhatch 2 Moves object about a 3-dimensional axis
R Cutclip 3 Creates an infinite line
S Revolve 4 Associative hatch pattern
5 Erase the last drawn line
6 Copies object to the clipboard
7 Non- associative hatch pattern
8 copies object to the clip board and removes the object from the drawing
(A) P-5, Q-4, R-6, S-1 (B) P-3, Q-4, R-8, S-1 (C) P-5, Q-7, R-8, S-2 (D) P-3, Q-7, R-8,
S-2
Answer: (B) P-3, Q-4, R-8, S-1

Q. 61Match the type of land use in Group 1 with their most appropriate colour code in
Group 2.
Group 1 Group 2
P Residential 1 Purple
Q Industrial 2 Yellow
R Recreational 3 Blue
S Transportation 4 Green
5 Gray
6 Red
(A) P-3, Q-6, R-4, S-5 (B) P-2, Q-1, R-4, S-5 (C) P-2, Q-1, R-6, S-3 (D) P-6, Q-1, R-
2, S-4
Answer: (B) P-2, Q-1, R-4, S-5

Q. 62Find out which of the following are TRUE (T) or FALSE (F) and then choose the correct
combination.
Group 1 Group 2
P Housing loan amount is determined by repayment capacity 1 True
Q The period of repayment of housing loans is never more than 7 years 2 False
R In the villages, loans for house buildings is still taken from private sources
S Housing loan packages are available From LIC, HUDCO, HFDC, CPWD &
SBI.
(A) P-1, Q-2, R-1, S-2 (B) P-2, Q-1, R-2, S-2
(C) P-1, Q-2, R-2, S-1 (D) P-2, Q-1, R-1, S-2
Answer: (A) P-1, Q-2, R-1, S-2

Q. 631750 square meters of a water body having as total surface area of 14,000 square
meters is covered with water hyacinth. Assuming the weed grows at GP rate every 24
hour, in how many days the water body will totally covered ?
(A) 16 days (B) 8 days (C) 3 days (D) 1 day
Answer: (C) 3 days
1750 sq.m. will be 3500 in one day, 7000 in two day and 14,000 sq.m. in three days.

Q. 64Match the quotations in Group 1 with their authors in Group 2.


Group 1 Group 2
P Form follows functions 1 Albert Einstein
Q Imagination is more important than knowledge 2 Winston Churchill
R Form follows climate 3 Louis Sullivan
S We shape buildings thereafter they shape us 4 Charles Correa
(A) P-1, Q-2, R-3, S-4 (B) P-3, Q-1, R-4, S-2 (C) P-2, Q-3, R-1, S-4 (D) P-4, Q-2, R-
3, S-1
Answer: (B) P-3, Q-1, R-4, S-2

Q. 65Referring to the Figure below, select the valid combination of symbols representing he
parameters given below.

Group 1 Group 2
P shearing force at point A 1 Maximum value
Q shearing force at point B 2 Positive value
R shearing force at point C 3 Negative value
S bending moment at point A 4 Zero
T bending moment at point B
U bending moment at point C
V Torsion at point A
(A) P-1, S-1, V-1 (B) R-4, U-3, T-2 (C) Q-1, T-3, V-4 (D) S-3, P-4, T-2
Answer: (C) Q-1, T-3, V-4

Q. 66Identify the most appropriate sequence of process used for treatment for waste water.
(A) Skimming tank → Racks and screen → Precipitation tank → Sedimentation tank
→ Biological growth → Disinfection
(B) Biological growth → Racks and screen → Skimming tank→ Sedimentation tank
→ Precipitation tank → Disinfection
(C) Racks and screen → Skimming tank → Sedimentation tank→ Precipitation tank→
Biological growth → Disinfection
(D) Biological growth → Skimming tank → Precipitation → Racks and screen→
Sedimentation tank→ Disinfection
Answer: (C) Racks and screen → Skimming tank → Sedimentation tank→
Precipitation tank→
Biological growth → Disinfection
. 67 A plot of land under development is located in a rocky area without any public sewerage
system. Identify the most appropriate combination for sewage disposal for the
proposed development.
P Septic tank
Q Soak pit
R Anaerobic filter
S Oxidation ditch
(A) P, Q (B) S, R (C) P, R (D) S
Answer: (C) P, R
Soak pit and oxidation ditch will not work in rocky area since ground percolation is low.

Q. 68Match the elements of imageability with their most appropriate characteristics.


Elements Characteristics
P Node 1 Strategic foci into which observer can enter.
Q District 2 Act as lateral reference and often path as well.
R Landmark 3 Area of homogeneous character recognized by clues.
S Edge 4 Singularity, contrast with its context, Observor does not enter in.
(A) P-3, Q-1, R-2, S-4 (B) P-1, Q-3, R-4, S-2 (C) P-4, Q-3, R-1, S-2 (D) P-4, Q-
3, R-2, S-1
Answer: (B) P-1, Q-3, R-4, S-2
From Image of City by Kevin Lynch.

Q. 69 The Bio- climatic chart defines comfort zone in terms of


P DBT
Q Effective Temperature (ET)
R Relative Humidity
S Heat Stress Index
(A) P, Q, R (B) P, R (C) Q, S (D) P, R, S
Answer: (B) P, R

Q. 70 Match the items from Group 1 with the appropriate items in Group 2
Group 1 Group 2
Town Planning Scheme 1 Additional FAR gained to the per if he provides public
amenities / spaces
Q Transferable Development Rights 2 A legal procedure that allows pooling of lands
by owners, preparation
of layout and redistribution of final plots.
R Incentive Zoning 3 Allows the plot owner to transfer the FAR/ FSI on the area of
the
plot, surrendered to the local authority, to another plot.

(A) P-3, Q-1, R-2 (B) P-2, Q-1, R-3 (C) P-2, Q-3, R-1 (D) P-3, Q-2, R-1
Answer: (C) P-2, Q-3, R-1

Q. 71 Match the housing programmes in Group 1 with their key features in Group 2
Group 1 Group 2
P KIP of Indonesia 1 Participatory, stress on young beneficiaries education
Q MHP of Sri Lanka 2 Participatory, beneficiaries contributed land.
R Orangi Project of Pakistan 3 Employment generative, tenureship to beneficiaries,
in situ
S CDP of Hyderabad, India 4 Flexible assistance, tenureship to beneficiaries, in situ
(A) P-2, Q-4, R-1, S-3 (B) P-1, Q-2, R-3, S-4
(C) P-3, Q-1, R-2, S-4 (D) P-4, Q-3, R-4, S-3
Answer: (D) P-4, Q-3, R-4, S-3
These are famous slum upgradation program of Asia.
The innovative Kampung Improvement Programme (KIP) in Jakarta, Indonesia,
launched in 1969 is the world’s first urban slum upgrading project. The densely
populated, unserviced, low-income urban homes scattered around the city that are
home to 60 percent of its 4.8 million inhabitants. The KIP worked to provide basic
urban services, such as roads and footpaths, water, drainage and sanitation, as
well as health and education facilities.
Million Housing Project of Srilanka. In the MHP, the focus is on : (a)very small
loans, the average loan is $ 178 per family ; (b)a large participation by the
households, in the form of work or money, this participation represents 60 to 90 %
of the total value of the accommodation built ; (c)and absence of strict technical
standards ; (d)technical assistance from the administration and a control of how
the loans are used ; (e)a wide range of loans for improvement, building of new
accommodation, sites and services, water supply, sewage and drainage, etc. ; (f)a
policy for improving slums in Colombo and in other towns ; (g)a land policy to
facilitate land access in urban areas, beneficiaries occupying public land acquire
ownership without land expenses ; if the land is privately owned, the State
undertakes to purchase it and only part of the price is counted in the loan granted
to the household ; (h)a subsidy policy, interest rates are below those of the
market, infrastructures are financed without repayment, administrative costs are
not taken into consideration in the cost of the loan. The National Housing
Development Authority (NHDA)is responsible for managing the programme and
granting loans.
Orangi poverty alleviation project (Orangi Pilot Project, OPP) was initiated by
Akhtar Hameed Khan in 1980 in Karachi, Pakistan. The project comprises a
number of programs, including a people’s financed and managed Low-Cost
Sanitation Program; a Housing Program; a Basic Health and Family Planning
Program; a Program of Supervised Credit for Small Family Enterprise Units; an
education Program; and a Rural development Program in the nearby villages.
Community Development Program, Hyderabad.

Q. 72Match the names of the trees in Group 1 with the shape of their leaves in Group 2.
Group 1

P Ficus religiosa
Q Cassia fistula
R Delonix regia
S Polyalthia Longifolia

(A) P-3, Q-2, R-4, S-1 (B) P-4, Q-3, R-1, S-2 (C) P-3, Q-1, R-4, S-2 (D) P-2,
Q-1, R-3, S-4
Answer: (B) P-4, Q-3, R-1, S-2
Ficus religiosa- peepal
Cassia fistula- amaltus (tree with yellow flower)
Delonix regia- gulmohar
Polyalthia Longifolia- false ashok

Q. 73 Match the orders of Architecture in Group 1 with their most appropriate symbolic
expression in Group 2.
Group 1 Group 2
P Doric 1 Floral
Q Ionic 2 Most Decorative
R Corinthian 3 Masculine
S Composite 4 Feminine
(A) P-3, Q-4, R-1, S-2 (B) P-2, Q-3, R-4, S-1 (C) P-4, Q-1, R-2, S-3 (D) P-1, Q-4,
R-3, S-2
Answer: (A) P-3, Q-4, R-1, S-2
Although Corinthian order is regarded as most decorative.
(input from Debanjali Saha, NIT Rourkela, PG SPAD)

Q. 74Referring to the structure shown in figure below, the maximum Shearing force in the
beam is at point
(A) P (B) Q(C) R(D) S
Answer: (C) R

Q. 75Choose the correct set of factors of physical planning to be placed at the vertex of
Geddesian Triangle.
P Development
Q Area
R Function
S Node
T Linkage
(A) P, Q, R (B) Q, R, S (C) R, S, T (D) P, Q, T
Answer: (A) P, Q, R

Q. 76Match the traffic problems in Group 1with their solutions in Group 2.


Group 1 Group 2
P Goods traffic serving city causing 1 Build an elevated road linking
accidents during day time entry and exit to city
Q Traffic jam along a major road 2 Install synchronized signaling
alignment during peak hours during peak hour
R Through traffic constituting major 3 Provide separate lane for slow mode
share of city road space 4 Restrict entry of HTVs to late night hours only
S Slow and fast mode creating much
traffic confusion
(A) P-1, Q-4, R-3, S-2 (B) P-3, Q-2, R-4, S-1 (C) P-4, Q-3, R-1, S-2 (D) P-4, Q-2, R-1,
S-3
Answer: (D) P-4, Q-2, R-1, S-3

Q. 77Arrange the four types of standard urban roads in descending order from the following
Group 1 Group 2
P National Road U Sector Roads
Q District Distributors V Local Distributors
R Slip Roads W Bus Roads
S Primary Distributors X Access Roads
T Neighbourhood Roads Y Dead End Roads
(A) S, Q, V, X (B) P, V, W, Y (C) P, R, T, V (D) S, Q, T, X
Answer: No appropriate answer

Q. 78Match the beneficiaries of housing in Group 1 with type of housing in Group 2.


Group 1 Group 2
P Post disaster homeless families 1 Self financing apartments
Q Slum dwellers under eviction notice 2 Prefabricted portable housing
R Middle income governments employee 3 HIG cooperative apartments S NRIs
wishing to return
4 Site and
services scheme

(A) P-1, Q-3, R-2, S-4 (B) P-2, Q-1, R-4, S-3 (C) P-3, Q-4, R-1, S-2 (D) P-2, Q-4, R-
1, S-3
Answer: (D) P-2, Q-4, R-1, S-3

Q. 79Match the type of floats in Group 1 with their characteristics in Group 2.


Group 1 Group 2
P Total float 1 Excess of minimum available time over the activity time
Q Free float 2 Excess of maximum available time over the activity time
R Independent float 3 Excess of available time over the the activity time when all jobs
start as early as possible.
(A) P-1, Q-3, R-2 (B) P-2, Q-1, R-3 (C) P-3, Q-2, R-1 (D) P-2, Q-3, R-1
Answer: (D) P-2, Q-3, R-1
(Soln provided by Amit Kumar Das, Asst. Manager, Air India)
Floats in Projects are of 4 types:
Total Float: Obtained by deducting the Early start and the activity duration from the
Late finish of the activity. Total Float signifies the total slack time available for any
activity in the Project without delaying the overall Project’s duration.
Safety Float: Total float- Slack on preceeedings nodes.
Free Float: Obtained by deducting the Early start plus the activity duration from
the Early Finish of the Activity. This float does not eat away the slack time
available for the successor activity.
Independent Float: Obtained by deducting the Late start plus the activity duration
from the Early Finish. This float virtually leaves no time since, the predecessor
activity has started late, and the successor activity is scheduled for an early start,
after utilizing the time duration for the activity. Most of the time, the slack time
available is ZERO for this float.
An activity, in general has both predecessors and successors. Each of the four
kinds of float depends on how these accommodate the activity
Q. 80 In Key Indicator Survey, the proxy variables for municipal efficiency are
(P) New construction (Q) Frequency of garbage removal
(R) Electricity Consumption (S) Amount spent on salary / wages of municipal workers
(A) P, Q (B) P, R (C) Q, S (D) Q, R, S
Answer: (C) Q, S
The Key Indicators Survey (KIS) is designed to help meet the monitoring and
evaluation needs of programs involved in population and health activities in
developing countries. The KIS tool includes questionnaires, interviewer’s manuals,
guidelines for sampling and a tabulation plan.

Linked answer Questions: Q. 81a to Q.85b carry two marks each.


.81a The density of a sector is 50 DU per ha. and household size is 5 persons per DU. Find
out the population of a sector having an area of 300 ha.
(A) 15,000 (B) 1,50,000 (C) 75,000 (D) 30,000
Answer: (C) 75,000
Soln: Total no of DU = 300 ×50 = 15000 DU and household size is 5
Total Population = 15,000 ×5 = 75,000

Q.81b If per capita demand of water for various population size is


1. up to 20,000: 100 lit/capita/day
` 2. 20,000 – 50,000 110 lit/capita/day
3. 50,000- 2,00,000 120 lit/capita/day
4. 2,00,000 and above 140 lit/capita/day
Calculate the daily water demand for the sector.
(A) 75 lakhs litre (B) 90 lakhs litre (C) 82.5 lakhs litre (D) 105 lakhs litre
Answer: (B) 90 lakhs litre
Soln: 75,000 population comes under category 3, so the demand will be 120
lit/capita/day
So daily water demand for the sector will be 75,000 × 120= 90,00,000 litres.

.82a The flying height of the plane is 2500 m above MSL and the elevation of the terrain is
500 M. Assuming a camera lens of 210 mm focal length, calculate the scale of the
photograph. (to the nearest ten).
(A) 1: 11900(B) 1: 9520(C) 1: 14290 (D) 1: 2380
Answer: (B) 1: 9520

Q.82b If a tract of land measures 2.5 cm x 4.5 cm in the photograph, calculate its area in
acres.
(A) 27 acres(B) 10 acres (C) 25 acres (D) 18 acres
Answer: (C) 25 acres
2.5 cm equivalent to 2.5 = 23807 cm = 238 m
4.5 cm equivalent to 4.5 = 42853 cm = 428 m
Area= 238

Q.83a An auditorium of size 30m x 20m is 8 m high. Assuming optimum reverberation time of
1.2 seconds and existing absorption power of the hall as 300 m²-sabins, calculate the
extra absorption units required.
(A) 240 m²-sabins (B) 340 m²-sabins (C) 380m²-sabins (D) 450m²-sabins
Answer: (B) 340 m²-sabins
Soln: A standard reverberation time has been defined as the time for the sound to die
away to a level 60 decibels below its original level.

Where RT60 = Reverberation time in sec


V = Volume in m³
Se = Total Absorption
Here Reverberation time = 1.2 sec, Volume = 4800 m³

(Total )
Extra absorption required= 640-300= 340 m²-sabins

Q. 83b Extra absorption units in the above auditorium can be achieved by using indigenous
acoustical materials. If the area available for fixing acoustical materials is 580 sq. m, of
which 40 sq. m. are openings covered with heavy curtains. The absorption coefficients
of curtains is 0.50. Identify a single acoustical material for the remaining area.
(A) Compressed wood particle board- perforated (coefficient of absorption: 0.36)
(B) 18 mm thick compound wood particle board (coefficient of absorption: 0.60)
(C) Wood wool board (coefficient of absorption: 0.20)
(D) Straw board (coefficient of absorption: 0.30)
Answer: (B) 18 mm thick compound wood particle board (coefficient of absorption:
0.60)
Soln: Total area available for fixing = 580 sq m
40 sq m covered with heavy curtain with 0.50 absorption coefficients
Absorption created by curtain = 40 × 0.5 = 20 m²-sabins
Therefore remaining absorption area= 580- 40= 540 sq m and further
absorption to be created = 340- 20= 320.
For particular material : 540 × absorption coefficient = 320

Absorption coefficient= = 0.59= it matches with 18 mm thick compound wood


particle board (coefficient of absorption: 0.60)

. 84a A city had a population of 2,50,000 persons in 2001 with a growth rate of 2% p.a. and
occupied an area of 1500 ha. Estimate the projected population for the year 2010.
Estimate the additional land requirement for the year 2010 assuming an average town
density of 100 ppha.
(A) 300,000 pop, 1500 ha (B) 310,000 pop, 1600 ha
(C) 350,000 pop, 1735 ha (D) 375,000 pop, 1825 ha
Answer: (A) 300,000 pop, 1500 ha

P2010 = P2001(1+ …….where r= 2 % and n= 10 years


P2010 = 2,50,000 (1+ 0.02)10
= 2,50,000 × 1.21= 3,02,500 (closest ans is 3,00,000)
Assuming 100 ppha, total land required = = 3000 ha
1500 ha is already available so additional 1500 ha required.

. 84b If the existing land use under residential and commercial are 970 ha and 105 ha
respectively, and the proposed land use under residential and commercial are 52 %
and 4.5 % respectively, calculate the additional land requirements under these.
(A) 712 ha, 40.5 ha (B) 590 ha, 30 ha (C) 759 ha, 45 ha (D) 642ha, 34.5ha
Answer: (B) 590 ha, 30 ha
Soln: Total area of town is 3000 ha.
Existing land use under residential= 970 ha

Proposed land use= 52% =


Additional land required for residential= 1560- 970= 590 ha
Existing land use under commercial = 105 ha

Proposed land use= 4.5 % =


Additional land required for residential= 135- 105= 30 ha

Q. 85a The pin jointed truss shown in the figure below is :

(A) Stable and statically determinate (B) Stable and statically indeterminate
(C) Unstable and statically indeterminate (D) Unstable and statically determinate
Answer: (A) Stable and statically determinate
To check whether statically determinae or statically indeterminate.
No of members + no of reactions = 2(no of joints) ……. Then it is statically
determinate
(Note : in case of roller support there is one reaction but in case of hinged it is two
reaction)
In the above case total no of members= 25
Total no of Reaction= 3 ( two from hinged and one from roller)
Adding No of members + no of reactions= 25 + 3= 28
This is equal to twice the no. of joints = 14 × 2= 28 hence statically determinate.

To check stability
m =2j-3 where m= no of members and j= no of joints.
Here m= 25 and j= 14, so it satisfies the equation…..so stable.

Q. 85b Calculate the force in the member marked (x ).


(A) 14 kN tensile (B) 28 kN tensile (C) 28 kN compression (D) zero
Answer: (C) 28 kN compression
Since it is supported on hinged support and roller support, so it will have three forces.
Two upward called as reaction A and reaction B and hinged support will also produce
horizontal reaction. So these three reactions will be solved by three equation of
statistics. That’s why they are called as statically determined. Its a howe truss and
truss formula
m =2j-3 so its stable as well so answer for first question is A .
Answer for second question can be calculated by graphical method of analysis of
perfect frames as X is not visible in question but from options it seems that they want
value of middle members so answer would be 28 KN TENSILE.
(with input from Surbhi Jethani, B. Arch, MITS Gwalior and Prashant Anand, NUS
Singapore) .
GATE 2006
Q.1 Autobahn is
(A) An automated mechanized pathway (B) A totally underground high speed freeway
(C) An intercity fast moving freeway (D) A neighbourhood bicycle pathway
Answer: (C) An intercity fast moving freeway
Autobahn is German Highway network.
Other highway’s known by different names in different countries:

Freeway- USA
Motorway- UK
Autobahn- Germany
Auto-estrada- Portugal
Autostrada- Italy
Autopista- Spain

World’s largest highway network.


National Trunk Highway System (NTHS) of China (97,355 km)
Interstate Highway System of the United States (75,932 km)
Highways in Spain (16,204 km).

Autobahn in Germany 12,845 kilometres


Boulevard- large road, usually running through a city with median and
landscape above average.
Cul de sacs- no thoroughfares, dead end.

Q.2 The fire rating of reinforced glass doors is expressed in


(A) kcal (B) hour (C) watt (D) lux
Answer: (B) hour
As per NBC the fire rating of most door used for fire proofing space must be 2
hours.
(adjacent figure is a fire door with push bar)

Q. 3 The ‘Hall of Nations’ in Pragati Maidan at New Delhi is essentially a three dimensional
space frame with the basic unit of
(A) A spheroid (B) A decahedron
(C) An octahedron (D) A tetrahedron
Answer: (C) An octahedron
Raj Rewal used octahedral lattice structure to design ‘Hall of Nations’ in Pragati
Maidan at New Delhi (1972)

Q. 4 In a pitched truss, the two vertical web members set at equal distances from the apex
are called
(A) Joggle post (B) Queen Post (C) King
Post (D) Jack Post
Answer: (B) Queen Post

Q.5 The international charter guiding conservation of historic buildings and areas
is the
(A) Kyoto Protocol (B) Chicago declaration
(C) Agenda 21 (D) Venice Charter
Answer: (D) Venice Charter
Kyoto Protocol – Climate change (reduction of Green House Gases)
Agenda 21- Sustainable Development (Rio de Janeiro, Brazil) 1992
For Historic Buildings and sites
Athens Charter – Restoration of Historic Monuments (1931)
Venice Charter- Historic Building Conservation (1964)
Burra Charter – Historic Building conservation for Australia (1979)
ICOMOS (International Council on Monuments and Sites) was founded in 1965 in
Warsaw as a result of the Venice Charter of 1964, and offers advice to UNESCO on
World Heritage Sites. UNESCO is headquartered in Paris.

Q.6 The command for reading inputs in C programming language is


(A) scanc (B) scanf (C) readf (D) inputf
Answer: (B) scanf

Q.7 The Pritzker Award recipient for the year 2005 is


(A) Thom Mayne (B) Glen Murcutt (C) Rem Koolhass (D) Zaha Hadid
Answer: (A) Thom Mayne

Q. 8 For comfort air-conditioning in Summer, the optimum wet bulb temperature is


(A) Equal to dry bulb temperature (B) Less than dry bulb temperature
(C) More than dry bulb temperature (D) Irrespective of dry bulb temperature
Answer: (B) Less than dry bulb temperature
S. No Condition Effect
1 DBT= WBT 100 % RH, Air is fully
saturated
2 DBT < WBT Not Possible
3 DBT > WBT Air is saturated

Q. 9 The neo-classical movement in the 18th century began as a reaction against


(A) Baroque style (B) Renaissance style (C) Gothic style (D) Romanesque style
Answer: (A) Baroque style
Neoclassicism is the name given to Western movements during 18th century in the
decorative and visual arts, literature, theatre, music, and crchitecture that draw
inspiration from the “classical” art and culture of Ancient Greece or Ancient Rome.

Timeline Architecture Important Structures


Periods
11,600- Prehistoric Stonehenge, Menhir, Pylons
3500 BC Times
3050-900 Ancient Pyramids, Great Bath at
BC Civilization Mohenjodaro, Ziggurat
850BC- Classical Pantheon, Rome; Parthenon-
476AD Athens; Colloseum- Rome
527- 565 Byzantine Hagia Sophia, Istanbul
AD
800-1200 Romanesque Basilica of St. Sernin in Toulouse,
AD France
1100- Gothic Notre Dame Cathedral, Paris
1450 AD
1400- Renaissance Villa Rotonda, Venice, Italy
1600
1600- Baroque Palace of Versailles, France
1830
1650- Rococo Pilgrimage Church of Wies
1790
1730- Neo White House, Washington DC
1925 Classicism
1890- Art Nouveau Secession Building – Vienna
1914
1925- Art Deco Chrysler Building in New York City
1937
1900- Modernism Works of Rem Koolhaas, I.M. Pei,
Le Corbusier, Philip Johnson, and
Mies van der Rohe.
1972- Post Works of Robert Venturi, Philip
Modernism Johnson
1997- Neo Works of Frank Gehry, Zaha
Modernism Hadid, Moshe Shafdie
and
Parametrcism

Q. 10The cubical content of a cement bag of 50.0 kg is generally


(A) 0.25 cu.m (B) 0.034cu.m (C) 0.043 cu.m (D) 0.05 cu.m
Answer: (B) 0.034cu.m

Density=

Volume= = = 0.034 cubic m


Approximately 30 bag (50 kg each) will make one cu.m of cement.
Density of cement is 1.47 g/cm3
Density of dry sand is 1.92 g/cm3.
Density of stone chips 2.5 g/cm3
Density of steel = 7.8 g/cm3

Q.11 Among the following brick paving patterns, the diagonal directional motif is prominent is
(A) Basket weave (B) Stacked bond laid flat (C) Running bond on edge (D) Herring
bone on edge

Answer: (D) Herring bone on edge

Basket weave Staked bond laid flat Running Bond on edge


Herring bone
Q. 12The emission during start up and shut down operations of industrial plants is called
(A) Fugitive emission (B) Emergency emission
(C) Transient emission (D) Steady controlled emission
Answer: (C) Transient emission
Fugitive emissions are emissions of gases or vapors from pressurized equipment due
to leaks and other unintended or irregular releases of gases.

Q. 13The predominant architectural style of Antonio Gaudi’s works is

(A) Art Nouveau (B) Gothic


(C) Renaissance (D) Neo- classicism
Answer: Art Nouveau

Q. 14In barrier free design, non ambulatory disabilities deal with


(A) Impairments confining individuals to wheel chair
(B) Hearing impairments
(C) Total or partial blindness
(D) Spastics or amputees using braces or crutches
Answer:(A) Impairments confining individuals to wheel chair
(input from by Kshitij Gharde, B. Arch VNIT Nagpur)

Q. 15In urban scale, dark objects seen against lighter backgrounds appear to
(A) Advance towards the viewer (B) Recedes from the viewer
(C) Shift laterally (D) Stretch vertically
Answer (A) Advance towards the viewer

Q. 16In an ecosystem, Heterotrophs refers to organism that


(A) Use light to build organic matter from their own bodies
(B) Use chemical energy to transform inorganic matter to organic matter
(C) Change state of matter causing early death
(D) Feed upon other organisms and their waste products
Answer: (D) Feed upon other organisms and their waste products
Use light to build organic matter from their own bodies- phototroph
Use chemical energy to transform inorganic matter to organic matter-
chemotroph
Feed on dead animals- scavengers (vultures)
Saprotroph- take their nutrition from dead and decaying matter by dissolving
them and absorbing through their body surface.
(Refer GATE 2013, Q.39)

Q. 17Cove lighting is a type of indirect lighting focused


(A) Upward from an interior cornice at the edge of a ceiling
(B) Downward from an interior cornice at the edge of a ceiling
(C) Downward from an opal light source at the centre of the ceiling
(D) Simultaneously upward and downward from a front – concealed light
Answer: (A) Upward from an interior cornice at the edge of a ceiling

Q. 18In contour analysis, ‘no cut no fill’ line


(A) joins points with same elevation
(B) Interconnects the points where new contours rejoin the old one
(C) Joins the highest and the lowest elevation
(D) Connects a ridge and a valley
Answer: (A) joins points with same elevation

Q. 19 A sewage treatment unit that works on the principle of anaerobic decomposition of


organic matter is called
(A) Trickling filter (B) Oxidation pond
(C) Septic tank (D) Activated sludge plant
Answer: (C) Septic tank
Trickling filters (TFs) are used to remove organic matter from wastewater. TFs
enable organic material in the wastewater to be adsorbed by a population of
microorganisms (aerobic, anaerobic, and facultative bacteria; fungi; algae; and
protozoa) attached to the medium as a biological film or slime layer (approximately
0.1 to 0.2 mm thick). As the wastewater flows over the medium, microorganisms
already in the water gradually attach themselves to the rock, slag, or plastic surface
and form a film. The organic material is then degraded by the aerobic
microorganisms in the outer part of the slime layer.

Oxidation Ponds are also known as stabilization ponds. They are used for simple
secondary treatment of sewage effluents. Within an oxidation pond heterotrophic
bacteria degrade organic matter in the sewage which results in production of cellular
material and minerals. The production of these supports the growth of algae in the
oxidation pond. Growth of algal populations allows decomposition of the organic
matter by producing oxygen. The production of this oxygen replenishes the oxygen
used by the heterotrophic bacteria.

Septic Tank: The term “septic” refers to the anaerobic bacterial environment that
develops in the tank which decomposes the waste discharged into the tank.

Activated Sludge is a widely used aerobic method of sewage treatment. After


primary settling, the waste stream is brought to an aeration tank. Air is put in and/or
there is mechanical stirring which provides aeration of the waste. Sludge from a
previous run is usually reintroduced to the tanks to provide microorganisms. This is
why it is called activated sludge. During the period in the aeration tank, large
developments of heterotrophic organisms occur. In the activated sludge tank the
bacteria occur in free suspension and as aggregates or flocs. Extensive microbial
metabolism of organic matter in the sewage results in the production of new microbial
biomass. Most of this biomass becomes associated with flocs that can be removed
from suspension by settling. A portion of the settled sewage sludge is recycled and
the remainder must be treated by composting or anaerobic digestion. Combined with
primary settling, activated sludge reduces the BOD by 85% to 90%. It also drastically
reduces the number of intestinal pathogens.

. 20 The science of acquiring information about the earth’s surface without actually being in
contact with it is termed as
(A) Geographical Information Centre (B) Earth Science (C) Remote
Sensing (D) Geomatics
Answer: (C) Remote Sensing

Q. 21 to Q. 75 carry two marks each.


Q. 21The essential components of Buddhist Stupa architecture are
(A) Ratha, Vimana, Harmika (B) Antarala, Medhi, Vimana
(C) Torana, Ratha, Anda (D) Vedika, Pradakshina, Harmika
Answer: (D) Vedika, Pradakshina, Harmika

Q. 22Match the cities in Group I with the planning concepts in Group II


Group I Group II
P. Radburn 1. Linear city
Q. Broadacre 2. Sectoral Plan
R. Chandigarh 3. Town of the motor age
S. Letchworth 4. Garden city
(A) P-3, Q-1, R-2, S-4 (B) P-1, Q-3, R-4, S-2 (C) P-2, Q-4, R-1, S-3 (D) P-4,
Q-2, R-3, S-1
Answer: (A) P-3, Q-1, R-2, S-4

Q. 23The three qualities defining our perception of chromatic light are


(A) Hue, Tint, Shade (B) Brightness, Contrast, Tint (C) Value, Hue, Intensity
(D)Value, Brightness, Shade
Answer: (C) Value, Hue, Intensity

Q. 24Match the items of building works in Group I with their units of measurement in Group
II
Group I Group II
P. Brickwork 1. Square metre
Q. Steel work 2. Running metre
R. Plastering 3. Tonne
S. Nosing 4. Cubic metre
(A) P-4, Q-2, R-3, S-1 (B) P-4, Q-3, R-1, S-2 (C) P-1, Q-3, R-4, S-2 (D) P-2, Q-4, R-
3, S-1

Answer: (B) P-4, Q-3, R-1, S-2

Q. 25The basic elements of Ekistics are


(A) Community, Land, Shell, Network, Image
(B) Society, Nature, Man, Shell, Network
(C) Community, Landscape, Utilities, Society, Transport
(D) Nature, Shell, Network, Administration, Society
Answer: (B) Society, Nature, Man, Shell, Network
Units and Population scale.

Anthropos – 1
room – 2
house – 5
housegroup (hamlet) – 40
small neighborhood(village) – 250
neighborhood – 1,500
small polis (town) – 10,000
polis (city) – 75,000
small metropolis – 500,000
metropolis – 4 million
small megalopolis – 25 million
megalopolis – 150 million
small eperopolis – 750 million
eperopolis – 7,500 million
Ecumenopolis – 50,000 million

Q. 26The de Stijl movement in architecture was characterized by


(A) Rectangular forms (B) Stone Facades (C) Pyramidal Roofs (D) Primary
Colours
Answer: No appropriate answer (both A and D correct)
de Stijl : Propagated by Dutch artist Theo Van Doesburg. They simplified visual
compositions to the vertical and horizontal directions, (squares and rectangles) and
used only primary colors along with black and white. (UPSC 2016)

Q. 27 Match the illustrations in Group I with the types of roof in Group II


Group I

P Q R S
Group II
1. Gable roof 2. Rainbow roof 3. Gambrel roof 4. Hip roof
(A) P-1, Q-3, R-2, S-4 (B) P-4, Q-1, R-2, S-3 (C) P-4, Q-2, R-1, S-3 (D) P-3, Q-2,
R-1, S-4

Answer: (B) P-4, Q-1, R-2, S-3


. 28 A line segment PQ is divided into two parts using the Golden Mean ratio. Segment A is
smaller than segment B. The only relationship that holds true for the line PQ is
(A) A/B = A / (A + B) (B) A/B = B / (A + B)
(C) (A + B) / A = (B – A)/ B (D) (A + B)/ A = (P + Q) / P
Answer: (B) A/B = B / (A + B)
Golden Ratio= 1: 1.618 (GATE 1991)

. 29 A typical fire fighting underground static tank for a 20 m high building should have a
pump capacity of
(A) 2400 lit/hour and pressure of 3.0 N/mm² (B) 2400 gallons/hour and pressure
of 0.3 N/mm²
(C) 2400 gallons/min and pressure of 3.0 N/mm² (D) 2400 lit/min and pressure of 0.3
N/mm²
Answer: (D) 2400 lit/min and pressure of 0.3 N/mm² (Refer IS: 14689)
1 liter= 3.78 gallon

Q. 30 The first four terms of a Fibonacci series are 1, 1, 2, 3, ……The sixth term of the
series will be
(A) 5 (B) 8 (C) 11 (D) 18
Answer: (B) 8

31 Two loading diagrams P and Q are shown. Their corresponding maximum Bending
Moments (P and Q respectively) are

(A) WL/4, WL/8 (B) WL/2, WL/4 (C) W /8, WL/4 (D) WL/4, WL/2
Answer: (C) W /8, WL/4
(Corrected by Chitra Mishra, B. Arch, Sundardeep College of Architecture, Ghaziabad.)

Q. 32 Match the city planners in Group I with the planning theories in Group II
Group I Group II
P. Patric Geddes 1. Mental map theory
Q. Gorden Cullen 2. Cities in transition
R. Kevin Lynch 3. Diagnostic approach to planning
S. Lewis Mumford 4. Serial vision
(A) P-2, Q-3, R-4, S-1 (B) P-4, Q-1, R-2, S-3 (C) P-3, Q-4, R-1, S-2 (D) P-4, Q-2, R-
1, S-3
Answer: (C) P-3, Q-4, R-1, S-2

A cognitive map (sometimes called a


mental map or mental model) is a type of mental representation which serves an
individual to acquire, code, store, recall, and decode information about the relative
locations and attributes of phenomena in their everyday or metaphorical spatial
environment.

Serial Vision of Rashtrapati Bhawan- The avenues of the stees and alignment of the
water bodies with building in backdrop make you feel the serial vision on the
Rastrapati Bhawan. The intention of the mound created just near the North Block
and South Block was to create some sort of visual screening effect to the eyes of the
person who would be approaching it from the Purana Quila. The illusion of nearness
of the Rastrapati Bhawan when you are near Purana Quila will tempt you to march
fast to reach to the Bhawan but suddenly reaching near the North and South Block,
you will realise that Bhawan is still far away. One more effect awaits you. When you
reach near the Bhawan you will realise that how small you are on the scale of the
building. Thus, architects have created a space where the majesty will feel his
eminence among the general public visiting him.

Q. 33 Urban renewal is best explained as


(A) Renewal of expired building permit (B) Renewal of byelaws of a commercial area
(C) Technique to limit the population growth (D) Removal of blight and prevention from
spreading to other parts.
Answer: (D) Removal of blight and prevention from spreading to other parts.

Q. 34In a rectangular room of length 4 m and width 3 m, an electric bulb is to be fixed above
the working plane. If the room index is 1, the ideal mounting height of the bulb from the
floor should be
(A) 2.3 m (B) 1.33 m (C) 1.7 m (D) 2.0 m
Answer: (C) 1.7 m
The room index is a number that describes the ratios of the rooms length, width and
height.

Room Index =

Here 1=

1= h= = 1.71 m
(soln. provided by Jyoti Ahlawat, MBS School of Arch. and Planning, Delhi)

Q. 35Match the architectural terms in Group I with their descriptions in Group II


Group I Group II
P. Trombe wall 1. A black water filled vessel absorbing solar radiation
Q. Brise-soleli 2. Glass enclosed space for sun bathing
R. Drum wall 3. Glass- front masonry absorbing solar radiation
S. Solarium 4. Screen to shield the interiors from sunlight
(A) P-3, Q-4, R-1, S-2 (B) P-4, Q-1, R-3, S-2 (C) P-3, Q-1, R-2, S-4 (D) P-2, Q-1, R-3,
S-4
Answer: (A) P-3, Q-4, R-1, S-2
The term Trombe wall is used to describe a passive solar building technique where a
wall is built on the winter sun side of a building with a glass external layer and a high
mass internal layer separated by a layer of air. Heat in close to UV spectrum passes
through the glass almost unhindered then is absorbed by the high mass wall that then
re-radiates in the far infrared spectrum which does not pass back through the glass
easily hence heating the inside of the building.
Brise-soleli - a variety of permanent sun-shading structures.
Note: This Question was asked in UPSC, 2015, Architect’s recruitment exam.

Q. 36The three components of Christaller’s theories of settlement system are


(A) Production, transport, centrality (B) Population, production, hierarchy
(C) Market, service, hierarchy (D) Production, market, transport
Answer: (D) Production, market, transport

Q. 37Match the valuation terms in Group I with their descriptions in Group II


Group I Group II
P. Scrap value 1. Value at the end of the utility period
Q. Salvage value 2. Value in present transaction
R. Market value 3. Value of dismantled material
S. Book value 4. Value after deducting the depreciation
(A) P-3, Q-1, R-2, S-4 (B) P-2, Q-3, R-4, S-1 (C) P-1, Q-2, R-3, S-4 (D) P-3, Q-
4, R-1, S-2
Answer: (A) P-3, Q-1, R-2, S-4

Q. 38 Match the urban forms Group I with their descriptions in Group II


Group I Group II
P. Satellite 1. Urban centre around large open space
Q. Constellations 2. Urban form around a main centre
R. Ring 3. Similar sized urban centres in close proximity
S. Radio centric 4. Circular form with radial corridors
(A) P-1, Q-3, R-4, S-2 (B) P-2, Q-3, R-1, S-4
(C) P-2, Q-4, R-3, S-1 (D) P-1, Q-2, R-3, S-4
Answer: (B) P-2, Q-3, R-1, S-4
Q. 39Match the arches in Group I with their names in Group II

Group II
1. Segmental 2. Equilateral 3. Flat 4.Semi- circular
(A) P-3, Q-1, R-4, S-2 (B) P-4, Q-1, R-2, S-3 (C) P-3, Q-4, R-2, S-1 (D) P-2, Q-3, R-1,
S-4
Answer: (C) P-3, Q-4, R-2, S-1

Q. 40The correct chronological sequence of the following buildings is


(A) Lingaraj temple → Sanchi Stupa → Padmanabhapuram Palace → Fatehpur Sikri
→ Gol Gumbaz
(B) Lingaraj temple → Sanchi Stupa → Fatehpur Sikri → Padmanabhapuram Palace →
Gol Gumbaz
(C) Sanchi Stupa → Lingaraj temple → Gol Gumbaz → Fatehpur Sikri →
Padmanabhapuram Palace
(D) Sanchi Stupa → Lingaraj temple → Padmanabhapuram Palace → Fatehpur Sikri →
Gol Gumbaz
Answer:(D)Sanchi Stupa→Lingaraj temple →Padmanabhapuram Palace→Fatehpur
Sikri→Gol Gumbaz
Dholavira, Gujarat (Indus Valley)- 2650 BCE
Sanchi Stupa- 3rd century BC
Ajanta Ellora Caves- 2nd century BC to 480- 650 AD
Meenakshi Temple- Mention in 7th Century scripture (AD)
The Shore Temple, Mammalapuram (700–728 AD)
Khajuraho Group – 950-1150 AD
Brihadeshwara Temple- 1010 AD
Lingraj temple- 11th century AD
Qutb Minar- 1193 A D
Konark Sun Temple- 1250 AD
Fatehpur Sikri- (1569-1585) AD
Padmanabhapuram Palace- 1601AD
Buland Darwaja- 1601 AD
Gol Gumbaj- 1626 AD
Taj Mahal- 1632- 1653 A D

Q. 41The components of energy embodied in a building material are


(P) Obtaining government approval for quarrying
(Q) Quarrying of the raw material
(R) Transporting the raw material to the manufacturing unit
(S) Manufacturing the building material
(A) P, R, S (B) P, Q, S (C) Q, R, S (D) P, Q, R
Answer: (C) Q, R, S
Embodied Energy is the sum of all the energy required to produce any goods
or services, considered as if that energy was incorporated or 'embodied' in the
product itself.
Operationa‘ Energy:’All forms of external enegy required for day to day function
of a building.

Q. 42Territoriality refers to a behavioural setting or space that


P. A person will personalize, mark, own and defend
Q. Moves with the person
R. Oscar Newman conceptualized as Defensible Space
S. Deals with encroachment
(A) P, Q, R (B) P, R, S (C) Q, R, S (D) P, Q, S
Answer: (A) P, Q, R
According to author Julia T. Wood, "men go into women's spaces more than women
enter men's spaces’’ (Wood 2007, p. 144). With this in mind, we can understand that
men typically have a stronger sense of ownership and are more likely to challenge
others' boundaries. People respond to invasion of ’territory in different ways
depending on what their comfort norms are. Wood (2007) presents three common
responses:
1. When someone moves too close for comfort, you might step away, giving up
your territory. This reaction is typical of feminine people.
2. When people have to fit into close spaces, they often look down as a
submissive way of showing that they are not trying to invade others' territories.
3. When someone moves too close’ you might refuse to give up your territory.
This reaction is typical of masculine people.
The term stimulated Edward T. Hall to create the word proxemics, which refers to
how people use space, but not necessarily how people communicate ownership.

Q. 43 Match the valves in Group I with their functions in Group II


Group I Group II
P. Gate valve 1. Prevents water flow back in the opposite direction
Q. Air valve 2. Regulates flow of water
R. Scour valve 3. Completely empties the pipe for inspection/ repair
S. Reflex valve 4. Ensures safety of pipe against collapse
(A) P-2, Q-4, R-3, S-1 (B) P-3, Q-1, R-2, S-4 (C) P-1, Q-3, R-4, S-2 (D) P-4, Q-
2, R-1, S-3
Answer: (A) P-2, Q-4, R-3, S-1

Q. 44 The most appropriate hierarchy of roads in an urban area is


(A) Freeway → Arterial → Local → Collector (B) Freeway → Arterial → Collector → Local
(C) Local → Arterial → Freeway → Collector (D) Collector → Local → Freeway → Arterial
Answer: (B) Freeway → Arterial → Collector → Local

Q. 45 Match the structural systems in Group I with the force distribution pattern in Group II
Group I Group II
P. Flying Buttress 1. High tensile strength
Q. Catenary 2. Outward and downward thrust
R. Simply supported truss 3. Comprehensive, tensile and shear stresses acting in the
plane of the surface
S. Shell 4. Axial tension and compression
(A) P-2, Q-1, R-4, S-3 (B) P-3, Q-1, R-2, S-4 (C) P-1, Q-3, R-4, S-2 (D) P-4, Q-2, R-
1, S-3
Answer: (A) P-2, Q-1, R-4, S-3
Catenary: In physics and geometry, a catenary is the curve that an idealized hanging
chain or cable assumes under its own weight when supported only at its ends.

Q. 46 Match the survey questions regarding water supply in Group I with their scaling types in
Group II
Group I Group II
P. Is the water supply sufficient? 1. Ordinal
Q. How is the quality of water ? 2. Nominal
R. What is the quantity of water supply/ day? 3. Interval
(A) P-1, Q-3, R-2 (B) P-2, Q-3, R-1 (C) P-3, Q-1, R-2 (D) P-2, Q-1, R-3
Answer: (D) P-2, Q-1, R-3
Statistical Scales
For qualitative data
i. Nominal: scales are exclusive, no order or overlap eg. What is your
religion: Hindu/ Muslim/ Sikh/……..?
ii. Ordinal: data can be rank ordered and can overlap, eg. Mild, moderate,
severe
iii. Dichotomous: nominal data fall into only two categories. (Yes/No)
For quantitative data
i. Interval: Numeric interval, eg.: height group
ii. Ratio: Interval data with necessary zero

Q. 47Determine the correctness of the following Assertion [a] and Reason [r]
[a] : In cold climate while glazing can act as a heat trap, it can also cause significant
losses at other times.
[r] : Since glass has a very low U- value, conductive heat loss takes place during night.

(A) Both [a] and [r] are True and [r] is the correct reason for [a]
(B) Both [a] and [r] are True but [r] is not the correct reason for [a]
(C) Both [a] and [r] are False
(D) [a] is True but [r] is False
Answer: (A) Both [a] and [r] are True and [r] is the correct reason for [a]

. 48 Two 50 cm cubes of concrete are cast in a Building Materials Testing Laboratory, one of
grade M15 and the other M20. The specified characteristic compressive strength of the
stronger block at 28 days will be
(A) 15 N/mm² (B) 20 N/mm² (C) 1.5 N/mm² (D) 0.88 N/mm²
Answer: (B) 20 N/mm²
Q. 49Match the plants in Group I with their prime uses in Group II
Group I Group II
P. Bignonia magnifica 1. Grass cover
Q. Ixora chinensis 2. Wall climbers
R. Cassia fistula 3. Topiary
S. Wedelia trilobata 4. Arboriculture
(A) P-3, Q-2, R-4, S-1 (B) P-4, Q-1, R-2, S-3 (C) P-4, Q-3, R-1, S-2 (D) P-2, Q-3, R-
4, S-1
Answer: (D) P-2, Q-3, R-4, S-1
Cassia fistula is amaltas with yellow flower covering the whole tree.
Topiary: trimming of plants to give particular form.
Arbour: a leafy gateway shaded by trees, climbers, shrubs, etc., especially when
trimmed to give shape.
Arboriculture: cultivation, management and study of individual trees.
Dendrology: Study of wooded trees.
Silviculture: study of forest.

Q. 50For natural illumination, Daylight factor is defined as the sum of


(A) Sky Component, External Reflected Component, Internal Reflected Component
(B) Sky Component, Reflected Component, Absorption Component
(C) Outdoor Component , Indoor Component, Reflected Component
(D) Azimuth Component, Altitude Component, Sky Component
Answer: (A) Sky Component, External Reflected Component, Internal Reflected
Component
(This question should be “For natural illumination, Daylight is defined as the sum
of…..”) Daylight factor is different from daylight.
Daylight is the sum of Sky Component, External Reflected Component, Internal
Reflected Component while a daylight factor is the ratio of internal light level to
external light level and is defined as follows:

DF = 100%

Q. 51In decision making theory, a mathematical model is represented as Y= a.Xb.Z c where


(A) Y – Independent variable; X, Z – Dependent variables; a, b, c – constants
(B) Y – Dependent variable; X, Z – Independent variables; a, b, c – constants
(C) Y – Independent variable; X, Z – dependent variables; a, b, c – non- constants
(D) Y – Dependent variable; X, Z – Independent variables; a, b, c – non- constants
Answer: (B) Y – Dependent variable; X, Z – Independent variables; a, b, c –
constants

Q. 52Match the components of travel demand in Group I with the forecasting models in
Group II
Group I Group II
P. Trip Generation 1. Multinomial Logit model
Q. Trip Distribution 2. Minimum Tree search
R. Mode choice 3. Gravity Model
S. Trip Assignment 4. Regression model
(A) P-4, Q-3, R-1, S-2 (B) P-4, Q-2, R-1, S-3 (C) P-3, Q-4, R-2, S-1 (D) P-1, Q-4, R-2, S-
3
Answer: (A) P-4, Q-3, R-1, S-2

Q. 53For a two way RCC slab, the Length to Width (L/W) ratio should be
(A) 0.5 ≤ L/W ≤ 2.0 (B) 2.0 < L/W ≤ 3.0
(C) 2.5 ≤ L/W ≤ 4.0 (D) 0.0 ≤ L/W < 0.5
Answer: (A) 0.5 ≤ L/W ≤ 2.0

Q. 54 Match the architects in Group I with their architectural works in Group II


Group I Group II
P. A. P. Kanvinde 1. Gandhi Labour Institute, Ahmedabad
Q. Anant Raje 2. IIFM, Bhopal
R. B V Doshi 3. Balotra City Hall, Balotra
S. U C Jain 4. NDDB Building, New Delhi
(A) P-3, Q-1, R-2, S-4 (B) P-2, Q-3, R-4, S-1 (C) P-1, Q-4, R-3, S-2 (D) P-4, Q-
2, R-1, S-3
Answer: (D) P-4, Q-2, R-1, S-3

Q. 55 Low COD to BOD ratio of an organic pollutant represents


(A) Low biodegradability of the pollutant (B) Presence of free oxygen for aerobic
decomposition
(C) High biodegradability of the pollutant (D) High arsenic in the pollutant
Answer: (A) Low biodegradability of the pollutant

BOD (Biochemical oxygen demand) – The amount of oxygen required by micro-


organisms to degrade the organic matter.
COD (Chemical Oxygen Demand) – In this process , Use of strong chemical
agent (such as potassium dichromate) is used to degrade both the organic as well
as inorganic matter present in the wastewater samples.
COD values are always higher than the BOD values. Because COD includes both
biodegradable and non-biodegradable substances whereas BOD contains only
bio-degradable.

Q. 56 Match the types of pipes in Group I with their functions in Group II


Group I Group II
P. Soil pipe 1. Carries liquid that does not include human excreta
Q. Waste pipe 2. Provides flow of air to or from drainage system and also escape of foul
gases
R. Vent Pipe 3. Preserves the water seal of traps through access to atmospheric air
S. Anti- siphonage pipe 4. Carries liquid waste including human excreta
(A) P-1, Q-4, R-2, S-3 (B) P-4, Q-1, R-2, S-3 (C) P-2, Q-3, R-4, S-1 (D) P-4, Q-1, R-3,
S-2
Answer: (B) P-4, Q-1, R-2, S-3
Q. 57The distance between two points on a map of scale 1: 40,000 is 3.6 cm. The distance
between the same two points in an aerial photograph is 6
cm. The scale of the aerial photograph is
(A) 1: 12000 (B) 1: 24000 (C) 1: 240000 (D) 1: 2400000
Answer: (B) 1: 24000
Actual distance between two points = 3.6 cm × 40,000= 144000 cm

Scale of aerial photograph =

= = 24,000
Therefore scale = 1: 24,000

Q. 58Match the colour schemes in Group I with their positions in the Colour Wheel in
Group II
Group I Group II
P. Complementary colours 1. Forming an equilateral triangle
Q. Analogous colours 2. A radial segment
R. Monochromatic colours 3. Adjacent to each other
S. Secondary colours 4. Opposite sides of each other

(A) P-4, Q-1, R-2, S-3


(B) P-1, Q-2, R-3, S-4
(C) P-4, Q-3, R-2, S-1
(D) P-1, Q-3, R-4, S-2
Answer: (C) P-4, Q-3, R-2, S-1
Colour Composition.
Two colour scheme
(a) Complementary- exactly opposite
(b) Analogous – neighbour
Three colour scheme
(c) Split complimentary- neighbor of complementary
(d) Triadic- three colours evenly spaced on colour wheel
Four colour scheme
(e) Tetrad- four colors arranged into two complementary pairs.

(f) Square – all four colours spaced evenly.

Q. 59The figure shows the Mass Curves of supply and demand for an Elevated Storage
Reservoir (ESR). The minimum required capacity of ESR is given
by
(A) (m + n) (B) (m – n)
(C) (m x n) (D) larger of m or n
Answer: (A) (m + n)

Q. 60Match the architectural styles in Group I with their features in Group II


Group I Group II
P. Greek 1. Hypostyle hall
Q. Buddhist 2. Triumphal Arch
R. Roman 3. Parthenon
S. Egyptian 4. Chaitya hall
(A) P-4, Q-2, R-3, S-1 (B) P-4, Q-3, R-1, S-2 (C) P-3, Q-4, R-2, S-1 (D) P-2, Q-1, R-
4, S-3
Answer: (C) P-3, Q-4, R-2, S-1

Q. 61The correct sequence of stages in the building construction process is


(A)Bidding→Contract documents→Occupancy certificate→Contract award→Post
occupancy evaluation
(B)Contract documents→Bidding→Contract award→Occupancy certificate→Post
occupancy evaluation
(C)Contract documents→Contract award→Bidding→Occupancy certificate→Post
occupancy evaluation
(D)Contract documents→Bidding→Contract award→Post occupancy
evaluation→Occupancy certificate
Answer: (B)Contract documents→Bidding→Contract award→Occupancy
certificate→Post occupancy evaluation

Q. 62A camera is used to shoot an aerial photograph from a flight. The scale of the
photograph is 1: 40000. If the flying height above mean sea level is 7500 m, and the
mean ground level is 1500 m, then the focal length of the camera lens is
(A) 70 mm (B) 150 mm (C) 200 mm (D) 50 mm
Answer: (B) 150 mm
Actual flying height of plane from the terrain= 7500-1500= 6000 m

Q. 63Match the land use in Group I with the prime activities in Group II
Group I Group II
P. Commercial 1. Crematorium and burial grounds
Q. Public and semi- public 2. Truck terminals
R. Recreational 3. Hotels
S. Transport 4. Parks and gardens
(A) P-2, Q-1, R-3, S-4 (B) P-2, Q-3, R-1, S-4 (C) P-3, Q-4, R-1, S-2 (D) P-3, Q-1, R-
4, S-2
Answer: (D) P-3, Q-1, R-4, S-2

Q. 64The Census of India defines ‘Urban Area’s an area with minimum


P. Population of 5000
Q. Population of 10000
R. 75 % of population engaged in non- agricultural activities
S. 75 % of male population engaged in non- agricultural activities
T. Density of 400 persons per sq. km
U. Density of 400 persons per hectare
(A) P, S, T (B) Q, S, T (C) P, R, U (D) Q, R, T
Answer: (A) P, S, T

. 65 In a multipurpose hall, 30 tube lights of 40 W each are switched on for 5 hours. The
electric meter will record a power consumption of
(A) 6 units (B) 12 units (C) 30 units (D) 60 units
Answer: (A) 6 units
Total Watt hour = 30 × 40× 5=6000 Watt hour
1 unit= 1 kWh= 1000Watt hour, So 6000 Watt hour = 6 units

Q. 66Match the architects in Group I with their design philosophies in Group II


Group I Group II
P. Frank Lloyd Wright 1. Less is bore
Q. Robert Venturi 2. House is a machine for living
R. Mies van der Rohe 3. Less is more
S. Le Corbusier 4. Organic Architecture
(A) P-2, Q-1, R-4, S-3 (B) P-4, Q-1, R-2, S-3 (C) P-1, Q-3, R-2, S-4 (D) P-4, Q-
1, R-3, S-2
Answer: (D) P-4, Q-1, R-3, S-2

Q. 67In a linear regression model, R² = 0 indicates


(A) Perfect positive correlation between variables (B) Moderate positive correlation
between variables
(C) No correlation between variables (D) Perfect negative correlation
between variables
Answer: (C) No correlation between variables.
The correlation coefficient ‘r’ tends to lie between -1.0 and +1.0.
If ‘r’ is near +1, it indicates a srong positive association between ‘x’ and ‘y’, i.e.
when ‘x’ increases ‘y’ too increases.
If ‘r’ is near -1, it indicates a srong negative association between ‘x’ and ‘y’, i.e.
when ‘x’ increases ‘y’ decreases.
If ‘r’ =0, it indicates there is no association between ‘x’ and ‘y’.
(soln provided by Dr Anima Ranjan, MBBS (Patna Univ), MMST, IITKgp )

Q. 68Match the properties of illumination in Group I with their units in Group II


Group I Group II
P. Luminous Intensity 1. Lambert
Q. Illumination 2. Candela
R. Luminous flux 3. Lumen
S. Luminance 4. Lux
(A) P-3, Q-4, R-2, S-1 (B) P-2, Q-4, R-3, S-1
(C) P-2, Q-1, R-4, S-3 (D) P-3, Q-1, R-2, S-4
Answer: (B) P-2, Q-4, R-3, S-1
Luminous intensity: luminous flux per unit solid angle, unit candela
Illumination: The lux is the SI unit of illuminance and luminous emittance,
measuring luminous flux per unit area. 1 lx= 1 lm/m2
The SI unit of luminous flux is the lumen
The SI unit for luminance is candela per square metre (cd/m2). Luminance is a
measure for the amount of light emitted from a surface.
Lambert- an American customary unit of luminance named after Johann Heinrich
Lambert
Examples
Illuminance Surfaces
illuminated by:
0.0001 lux Moonless, overcast night
sky (starlight)
0.002 lux Moonless clear night sky
with airglow
0.27–1.0 lux Full moon on a clear night
3.4 lux Dark limit of civil twilight
under a clear sky
50 lux Family living room lights
80 lux Office building
hallway/toilet lighting
100 lux Very dark overcast day
320–500 lux Office lighting
400 lux Sunrise or sunset on a
clear day.
1000 lux Overcast day; typical TV
studio lighting
10000– Full daylight (not direct
25000 lux sun)
32000– Direct sunlight
130000 lux

Q. 69Le Corbusier’s ‘Five Points of a New Architecture’ are


P. Pilotis S. Free facade T. Louvres
Q. Ribbon windows T. Roof gardens W. Free plan
R. Ornamental grills U. Sloping roofs
(A) P, Q, S, T, W (B) P, Q, R, U, W
(C) Q, S, T, V, W (D) Q, R, U, V, W
Answer: (A) P, Q, S, T, W
Visible in Villa Savoy.

Q. 70 Match the architectural styles in Group I with the buildings in Group II


Group I Group II
P. Metabolism 1. Humana Building, Kentucky
Q. Post Modernism 2. Guggenheim Museum, Bilbao
R. Brutalism 3. Sony Tower, Tokyo
S. Deconstructivism 4. United dè Habitation, Paris
(A) P-4, Q-3, R-2, S-1 (B) P-3, Q-1, R-4, S-2
(C) P-4, Q-2, R-1, S-3 (D) P-2, Q-3, R-4, S-1
Answer: (B) P-3, Q-1, R-4, S-2
Metabolism: Nakagin Capsule tower by Kisho Kurokawa.
Humana Building, Kentucky- Michael Graves
Guggenheim Museum, Bilbao- Frank Gehry
Sony Tower- Yoshinobu Ashihara
United dè Habitation, Paris- Le Corbusier

Common Data for Questions 71, 72, 73


A town with an area of 340 hectares has 15, 000 households. The number of
occupied dwelling units is 12,400 of which 15% are dilapidated condition. 5 % of the
households are below poverty line and unable to afford any type of dwelling. (Note:
Dwelling Unit = DU)

Q. 71The present density of the town (in DU/ acre) is approximately


(A) 15 (B) 18 (C) 37 (D) 44
Answer: (A) 15
1 ha= 2.47 acre so total area in acre = 340 × 2.47= 839.8 acre

DU/ acre = ≈ 14.76

Q. 72The housing need of the town is approximately


(A) 1860 DU (B) 2600 DU (C) 3320 DU (D) 4460 DU
Answer: (D) 4460 DU
Housing need= Without house + Dilapidated
Homeless = 15,000- 12,400= 2,600 DU

Dilapidated = × 12,400 = 1,860 DU


Total = 4,460 DU
Need is generally common like Food, Clothing and Shelter.
Demand is the desire to buy a commodity backed by availability of the product,
willingness to buy and ability to pay.

Q. 73The housing demand of the town is approximately


(A) 1240 DU (B) 1860 DU (C) 2470 DU (D) 3710 DU
Answer: (D) 3710 DU
Housing Demand= Dilapidated + homeless who can pay.

Dilapidated = × 12,400 = 1,860 DU

Who cannot pay = × 15,000 = 750 DU


Who can pay= Homeless- who cannot pay
2600- 750 = 1850
Housing Demand= Dilapidated + homeless who can pay.
= 1,860 + 1850 = 3,710 DU
(with input from Mohammad Shibin N, B Arch, SPA Vijaywada)

Common Data for Questions 74, 75:


The population for an urban area for the period 1971- 2001 is given in the following
table:
Year Population in ‘000s Increase in % increase in
Population in ‘000s Population
1971 80
1981 95 15 18.75
1991 115 20 21.05
2001 140 25 21.74
Q. 74The projected population (in thousands) for 2021 using Arithmetic Increase method is
(A) 160 (B) 180 (C) 220 (D) 240
Answer: (B) 180
Avg. increase of population from 1971 to 2001(three decade) is

= 20,000 per decade.


So population in 2021 will be = 140,000 + (20,000×2) = 180,000

Q. 75The projected population (in thousands) for 2021 using Geometric Increase method is
(A) 168.66 (B) 180 (C) 203.18 (D) 253
Answer: (C) 203.18
Geometric mean of % increase of population =
= 20.47 %
Pop in 2021 = P2001 (1+ 0.2047)2
= 140 (1.2047)2= 203.18
(input from Chitra Mishra, B Arch, Sundardeep College of Architecture, Ghaziabad)

Linked Answer Questions: Q. 76 to Q. 85 carry two marks each.


Statement for Linked Answer Questions 76 & 77:
While testing the strength of a steel beam, it is found that
Poisson’s ratio for steel = 0.3 and Young’s modulus of Elasticity = 2.1 x 106

Q. 76If the lateral strain of the beam is 1 unit, the longitudinal strain will be
(A) 7 x 106 units (B) 0.3 units (C) 2.1 x 106 units (D) 3.33 units
Answer: (D) 3.33 units

Poisson’s ratio=

0.3=

Longitudinal strain = =3.33 units

Q.77 The longitudinal stress for the same steel beam will be approximately
(A) 7 x 106 units (B) 0.3 units (C) 2.1 x 106 units (D) 3.33 units
Answer: (A) 7 x 106 units

Young’s modulus of Elasticity=


Longitudinal stress = Young’s modulus of Elasticity × strain
= 2× 106 × 3.33
= 6.66 × 106 units

Statement for Linked Answer Questions 78 & 79:


Every glazing material (e.g. glass) has three properties which define its functioning:
- Reflectivity (coefficient = R)
- Transmissivity (coefficient = T)
- Absorptivity (coefficient = A)

Q. 78The relationship that holds true for the thermal performance of glass is
(A) R + T = A (B) R = T + A(C) R + T + A = 0 (D) R + T + A = 1
Answer: (D) R + T + A = 1

Q. 79If for glass, R = 0.6 and T = 0.1, A will be equal to


(A) – 0.7 (B) 0.3 (C) 0.5 (D) 0.7
Answer: (B) 0.3
Soln: A= 1- (R+T) solving for A we get A= 1- (0.6+0.1)
A =0.3

Statement for Linked Answer Questions 80 & 81:


In a PERT network analysis, an activity is likely to have:
- optimistic duration of 2 days
- pessimistic duration of 7 days
- modal duration of 3 days

Q. 80 The expected time duration that may be allocated in a PERT scheduling would be
(A) 2 days (B) 3 days (C) 3.5 days (D) 4 days
Answer: (C) 3.5 days

Expected duration=

= = 3.5 days

. 81 If the same activity has a free float of 2 days then the maximum time that may be
allocated to it without hampering the start of the succeeding activity is
(A) 1.5 days (B) 2 days (C) 4 days (D) 5.5 days
Answer: (D) 5.5 days

Statement for Linked Answer Questions 82 & 83:


A brick wall 4 m long, 3 m high and 30 cm wide has a door opening of clear span 1 m
and clear height 2.1 m. The door opening has a flat stone arch over it of thickness 20
cm and width 30 cm, subtending an angle of 60º at the centre.

Q. 82The volume of stone work (in cu. M) in the arch is


approximately
(A) 0.047 (B) 0.057 (C) 0.067 (D) 0.077
Answer: (C) 0.067

Q. 83The volume of brick work (in cu. M) in the wall is approximately


(A) 1.9 (B) 2.9 (C) 3.9 (D) 4.9
Answer: (B) 2.9
Statement for Linked Answer Questions 84 & 85:
A transportation engineer notes that a vehicle approaches an intersection at a
velocity of 44 m/s. At time t = 0, it began to decelerate at d = 16 m/s2.

Q. 84The time taken by the vehicle to come to a complete stop will be


(A) 2 seconds (B) 2.75 seconds (C) 2.8 seconds (D) 3 seconds
Answer: (B) 2.75 seconds
v= u + at here final velocity= 0 and initial velocity = 44 m/s
0= 44+ (-16)t
t= 2.75 sec

. 85 The total distance travelled by the vehicle from the initial point of deceleration to the
stopping point is
(A) 10.5 m (B) 30.5 m (C) 50.5 m (D) 60.5 m
Answer: (D) 60.5 m

= 60.5 m
GATE 2009
Q. 1 to Q. 20 carry one mark each.
Q. 1 The essential difference between CPM and PERT is
(A) Critical Path vs. Critical Activity (B) Arrow notation vs. Precedence
notation
(C) Deterministic approach vs. Probabilistic approach (D)Project management vs.
Network Analysis
Answer: (C) Deterministic approach vs. Probabilistic approach
In CPM, (Critical Path Method) activity times are assumed to be predictable.
(“deterministic”).
In PERT, (Project Evaluation and Review Technique) activity times are
assumed to be random, with assumed probability distribution (“probabilistic”)

Q. 2 The Minimum Thickness of wall where single Flemish bond can be used is

(A) Half- Brick thick


(B) One- Brick Thick
(C) One- and- a- Half- Brick thick
(D) Two- Brick thick
Answer: (C) One- and- a- Half- Brick thick
Input from Rohan Maheswari, B. Arch (GCA, Lucknow)

In case of double flemish bond, both faces of the wall have flemish look, i.e.
each course consist of alternate header and stretcher.
In single flemish bond outer faces of walls have flemish look whereas inner
faces have look of English bond.

. 3 On a colour wheel, the combination of ‘Violet- Yellow’ or ‘Orange- Blue’ are best
described as
(A) Complementary (B) Supplementary (C) Analogous (D) Monochromatic
Answer: (A) Complementary
Two colour scheme
a. Complementary- exactly opposite
Three colour scheme
b. Analogous - neighbour
c. Split complimentary- neighbor of complementary
d. Triadic- three colours evenly spaced on colour wheel
Four colour scheme
e. Tetradic- four colors arranged into two complementary pairs.
f. Square – all four colours spaced evenly.

. 4 The sudden stoppage in the flow of water in a close conduit results in a phenomena
called
(A) Cavitation (B) Hydraulic Gradient (C) Stack Pressure (D) Water Hammer
Answer: (D) Water Hammer

Cavitation: Air
bubble formation in liquid.

Hydraulic Gradient: a line joining the points


of highest elevation of water in a series of vertical open pipes rising from a
pipeline in which water flows under pressure.

Stack Pressure: It is due to difference in temperature of air at two different


layers causing air movement.
Water Hammer: sudden stoppage in the flow of water in a close conduit.
Q. 5 The number of intersecting arches that
support Bijapur’s Gol Gumbaj is
(A) 4 (B) 8 (C) 12 (D) 16
Answer: (B) 8
"8 intersecting arches created by two rotated squares that create interlocking
endentives"support the dome.

Q. 6 The 73rd and 74th Constitutional Amendments pertain to


(A) Abolishing Urban Land Ceiling Act
(B) Providing restricted role to rural courts to settle rural disputes
(C) Providing more responsibilities to Municipal and Local bodies for planning and
development.
(D) Providing Right to information for the general people.
Answer : (C) Providing more responsibilities to Municipal and Local bodies for
planning and development

Q. 7 A simply supported beam of length L carries a concentrated load of intensity P at its


centre. The bending moment at the centre of the beam will be
(A) PL/2 (B) PL/4 (C) PL/6 (D) PL/8
Answer: (B) PL/4

Q. 8 ‘Desire Lines’ are associated with


(A) Origin- Destination analysis is Transportation planning
(B) Income- Expenditure analysis in personal Finance
Managemnent
(C) Cut- Fill analysis in landscape planning
(D) Demand- Supply Analysis in economic planning
Answer: (A) Origin- Destination analysis is Transportation planning
Transportation Planning: The path usually represents the shortest or most easily
navigated route between an origin and destination.
Landscape Planning: desire line is a path created as a consequence of foot or bicycle
traffic which is shortcut to original path.

Q. 9 GRIHA is rating for green building given by


(A) The Energy Research Institute
(B) Development Alternatives
(C) Bureau of Energy Efficiency
(D) Ministry of Power
Answer: (A) The Energy Research Institute (TERI)
(It should be The Energy and Resources Institute)

TERI (The Energy and Resources


Institute, New Delhi) was earlier known as Tata Energy Research Institute. It
began its operation from Mumbai in 1974. In 1984 TERI moved to Delhi. As the
scope of its activities widened over a period of time, it was renamed The
Energy and Resources Institute in 2003. It is head quartered at India Habitat
Centre, New Delhi.
Bureau of Energy Efficiency gives star rating for Electrical Appliances.
Q.10 A ‘cul-de-sac’ is a street where
(A) Only two-wheelers are permitted
(B) Thorough Traffic is discouraged
(C) Pedestrians are not permitted
(D) Vehicles are permitted to move in one direction only
Answer: (B) Thorough Traffic is discouraged
Minimum radius of cul-de-sac must be 10 m.

Q. 11 ‘Usonian’ houses were designed by


(A) Mies van der Rohe(B) Alvar Alto
(C) Frank Lloyd Wright (D) Le Corbusier
Answer: (C) Frank Lloyd Wright

Usonian House: Featured low roofs and


open living areas. Abundant use of brick, wood, and other natural material. Wright'’
Usonian homes were small, one-story structures set on concrete slabs with piping
for radiant heat beneath. The kitchens were incorporated into the living areas.
Open car ports took the place of garages.
Prarie House: The style is usually marked by horizontal lines, flat or hipped roofs
with broad overhanging eaves, windows grouped in horizontal bands, integration
with the landscape, solid construction, craftsmanship, and discipline in the use of
ornament. Horizontal lines were thought to evoke and relate to
the native prairie landscape.
Robie House: Located in the campus of Chicago University.
Robie House, Prarie House, Usonian House -–Frank Lloyd Wright

Q. 12 Increase in volume of fine aggregate due to


the presence of moisture is called
(A) Bulking (B) Buckling
(C) Bending (D) Twisting
Answer: (A) Bulking
Q. 13 The Pattern language Theory was propounded by
(A) Christopher Alexander
(B) Patrick Geddes
(C) John Ruskin
(D) Amos Rapoport
Answer: (A) Christopher Alexander
Pattern Language- - Christopher Alexander
Notes on synthesis of Form- Christopher Alexander
Oregon Experiment- Christopher Alexander
Community and Privacy- Christopher Alexander
The city as a mechanism for sustaining Human
contact- Christopher Alexander
Pattern of streets- Christopher Alexander

Q. 14 As per IS 456:2000, the maximum area of tension reinforcement in a RCC beam shall
not exceed x% of its cross- sectional area, where x is equal to
(A) 2 (B) 4 (C) 6 (D) 8
Answer: (B) 4
For Beam
Max percentage of reinforcement(both compression and tension steel) should
be 4% of cross sectional area of beam (cl.26.5.1.2, IS 456 :2000) i.e.
As= 0.04 bD where ‘b’ is breadth and ‘D’ depth of beam.
For column (cl.26.5.3.1 IS 456:2000)
Longitudinal reinforcement shall not be less than 0.8% and shall not be more
than 6% but generally in practice it limit to 4%

Q. 15 ‘No-cut-no-fill’ lines are mostly used in


(A) Land Use Planning (B) Interpretation of stereo- vision- photographs
(C) Earth work computation (D) Interpretation of remotely sensed images
Answer: (C) Earth work computation

Q. 16 The property of concrete measured by slump test is


(A) Durability (B) Hardness (C) Strength (D) Workability

Answer: (D) Workability


Slump cone test is done to measure workability/ consistency of concrete. The
slump test is a means of assessing the consistency of fresh concrete. It is
used, indirectly, as a means of checking that the correct amount of water has
been added to the mix.

Universal Testing Machine (UTM) measures compressive strength of


mortar/concrete blocks.
Vicat’s apparatus is used for consistency test.

Q. 17 The Remote Sensing satellite that gives the highest spatial resolution is
(A) IKONOS 2 (B) IRS 1C/1D (C) Quickbird 2 (D) SPOT 5
Answer: (C) Quickbird 2
India’s Cartosat has a spatial resolution of 1 m
Q. 18 Development that meets the needs of the present generation
without compromising the ability of future generations to meet their own needs is
termed by UNDP as
(A) Comprehensive Development
(B) Equitable Development
(C) Human Development
(D) Sustainable Development
Answer: (D) Sustainable Development
This definition was given by Brutland Commission in 1971 Convention. Brutland was ex Prime Minister of Norway under
whose chairpersonship a committee was formed for Sustainability.

. 19 The parameter that does not appear in Psychrometric chart is


(A) Wind Speed (B) Dry Bulb Temperature (C) Wet Bulb Temperature (D)
Relative Humidity
Answer: (A) Wind Speed

. 20 Allowable stress in the design of a tension member in a steel truss is a function of


(A) Cross-sectional area of the member (B) Yield stress of the material
(C) Slenderness ratio of the member (D) Moment of Inertia of the member’s cross-
section
Answer: Marks awarded to all (Both A and B Correct)
[Ref: Q-69 (2007), Q-42(2003), Q18 (1994)]

. 21 to Q. 26 carry two marks each.

.21 The parameters for determining Human Development Index are


P. Education attainment
Q. Per capita Gross Agricultural Produce
R. Life Expectancy
S. Per Capita Gross Domestic Produce
T. Per Capita State Domestic Produce

(A) P, Q, S (B) P, Q, S, T (C) P, R, S (D) R, S, T


Answer: (C) P, R, S
(Earlier it was GDP per capita and now it has been modified as GNI per capita)
Human Development Index is calculated by UNDP (United Nations Development
Programmes) All three parameters are
calculated on scale of 0-1.
Education attainment: Geometric mean of years of schooling for adults aged
25 years and expected years of schooling for children of school entering age.
Life expectancy: The life expectancy at birth component of the HDI is
calculated using a minimum value of 20 years and maximum value of 83.57
years.
The decent standard of living component is measured by GNI per capita
instead of GDP per capita. The HDI uses the logarithm of income, to reflect the
diminishing importance of income with increasing GNI.
The scores for the three HDI dimension indices are then aggregated into a
composite index using geometric mean.
. 22 Match the individuals in Group I with the works in Group II
Group I Group II
P. Hippodamus 1. Aqueducts
Q.Vitruvius 2. Campidoglio
R. Michelangelo 3. Hagia Sophia
S. Constantine 4. Agora
5. Hanging Garden
(A) P-4, Q-1, R-2, S-3 (B) P-3, Q-1, R-2, S-5 (C) P-4, Q-5, R-1, S-3 (D) P-3, Q-4, R-1,
S-2

Answer: (A) P-4, Q-1, R-2, S-3


Campidoglio: Piazza on Capitoline hill of Rome.
Hippodamus: Father of city planning.

. 23 If the height of facade=h, and the distance of observer from the building = d, then match
the enclosure type in Group I with their corresponding h/d ratio in Group II.
Group I Group II
P. Full Enclosure 1. 1
Q. Threshold of Enclosure 2. 1/2
R. Minimum Enclosure 3. 1/3
S. Loss of Enclosure 4. 1/4
5. 1/5
(A) P-1, Q-2, R-3, S-4 (B) P-4, Q-3, R-2, S-1
(C) P-2, Q-3, R-4, S-1 (D) P-5, Q-1, R-2, S-4
Answer: (A) P-1, Q-2, R-3, S-4

. 24 The correct sequence of activities in Solid Waste Management is


(A) Collection→ Transportation→ Treatment→ Segregation
(B) Segregation→ Collection→ Transportation→ Treatment
(C) Collection→ Segregation→ Treatment → Transportation
(D) Treatment→ Collection→ Segregation→ Transportation
Answer: (B) Segregation→ Collection→ Transportation→ Treatment
. 25 The principles of Universal design include:
(P) Flexibility in use (Q) Tolerance for error
(R) Energy Efficiency (S) Low Physical Effort
(A) P, Q, R (B) Q, R, S
(C) P, R, S (D) P, Q, S
Answer: (D) P, Q, S
Principle 1: Equitable Use.
Principle 2: Flexibility in Use.
Principle 3: Simple and Intuitive Use.
Principle 4: Perceptible Information.
Principle 5: Tolerance for Error.
Principle 6: Low Physical Effort.
Principle 7: Size and Space for Approach and Use.
. 26 Match the Urban Design elements in Group I with their descriptions in Group II
Group I Group II
P. District 1. Recognizable as having some common identifying character
Q. Landmark 2. Centre of activity
R. Node 3. Network of major and minor routes
S. Pathway 4. Prominent visual feature of the city
(A) P-3, Q-4, R-2, S-1 (B) P-1, Q-4, R-2, S-3 (C) P-1, Q-2, R-4, S-3 (D) P-2, Q-4, R-1,
S-3
Answer: (B) P-1, Q-4, R-2, S-3
Taken from The Image of the city by Kevin Lynch.

. 27 A commercial plot measuring 100m x 80 m. If the permissible Floor Space Index is 3.0
and 50 % of the ground is covered, then the maximum number of floors that can be
built is
(A) 3 (B) 4 (C) 6 (D) 12
Answer: (C) 6
Area of plot =100m x 80m = 8000 m2
FSI= 3.0 so total floor area permissible 8000 x 3 = 24,000 m2
Ground coverage 50 % so land area to be utilized for construction = 4000 m2

=> No of floors= =6

Q. 28 Match the elements of a Buddhist Stupa in Group I with their traditional names in
Group II
Group I Group II
P. Hemispherical Dome 1. Vedika
Q. Peripheral Railing 2. Anda
R. Entrance Gateway 3. Harmika
S. Portion above dome 4. Nagara
5. Chaitya
6. Torana
(A) P-2, Q-1, R-6, S-3 (B) P-2, Q-6, R-4, S-3 (C) P-3, Q-1, R-5, S-2 (D) P-5, Q-6, R-1,
S-2
Answer: (A) P-2, Q-1, R-6, S-3

. 29 A microwave oven of 3 kW rating is operated for 30 minutes, a hot water geyser of 1 kW


rating is operated for 15 minutes, and 5 fluorescent lamps of 60 W are operated for 6
hours. The total power consumed in (kWh) will be
(A) 1.80 (B) 3.55 (C) 18.01 (D) 35.50
Answer: (B) 3.55

Microwave= 3 = 1.5 (for half an hour)

Geyser= 1 = 0.25 (for 15 min)

5 fluorescent lamp = 5 6= = 1.8 ( for 6 hours)


Total = 1.5 + 0.25 + 1.8 = 3.55

Q. 30 Match the building projects in Group I with their architects in Group II


Group I Group II
P. National Olympic Stadium, Beijing 1. Rem Koolhas
Q. Glass Pyramid, the Louvre, Paris 2. Richard Rogers
R. Millenium Dome, London 3. Renjo Piano
S. Kansai Airport, Osaka 4. Tadao Ando
5. I M Pei
6. Herzog & de Meuron
(A) P-6, Q-2, R-3, S-4 (B) P-1, Q-6, R-2, S-4 (C) P-6, Q-5, R-2, S-3 (D) P-2, Q-5,
R-1, S-3
Answer: (C) P-6, Q-5, R-2, S-3

. 31 Identify the prehistoric structures in the following:


P. Mastaba Q. Dolmen R. Menhir S. Pylon T. Stonehenge U. Thermae
(A) P, Q, R (B) R, T, U (C) Q, S, T (D) Q, R, T
Answer: (D) Q, R, T
Pre historic period- period from evolution of universe to the period when man started
writing
(evolution of earth to 3000 BC)
Mastaba – Grave of common man in Egypt during the period of Pyramid.
Dolmen-A dolmen, also known as a portal tomb, portal grave, or quoit, is a type
of single-chamber megalithic tomb, usually consisting of three or more upright
stones in Ireland. (Prehistoric)
Menhir- A menhir is a large upright standing stone. (prehistoric)
Cromlech- prehistoric megalithic structure.
Pylon-A monumental gateway in the form of a pair of truncated pyramids serving
as the entrance to an ancient Egyptian temple.
Stonehenge-Stonehenge is a prehistoric monument in England. (prehistoric)
Thermae- Public bath in Rome.
Menhir Dolmen Stonehenge, London

Pylon Mastaba Ruins of Roman Bath

. 32 Match the figures of cut bricks in Group I with their terms in Group II
Group I

Group II
1. King Closer
2. Queen Closer
3. Half Bat
4. Three Quarter Bat

(A) P-2, Q-3, R-1, S-4 (B) P-2, Q-1, R-3, S-4 (C) P-1, Q-2, R-4, S-3 (D) P-3, Q-4, R-
1, S-2
Answer: (A) P-2, Q-3, R-1, S-4
. 33 A site has 6 contour lines and the length of the line joining the midpoints of the highest
contour and the lowest contour is 300 m. If the slope of the line is 1 in 10, then the
contour interval (in m) is
(A) 5 (B) 6 (C) 50 (D) 60
Answer: (B) 6

Soln: = solving for h = 30m

Contour interval = = 6 m ( contour interval is one less than no. of contour line)

Q. 34 Match the plants in Group I with their corresponding examples in Group II


Group I Group II
P. Climber 1. Croton
Q. Shrub 2. Shirish
R. Tree 3. Duranta
S. Hedge 4. Bougainvillea
(A) P-3, Q-1, R-2, S-4 (B) P-2, Q-4, R-1, S-3 (C) P-4, Q-1, R-2, S-3 (D) P-4, Q-3, R-1,
S-2
Answer: (C) P-4, Q-1, R-2, S-3

Q. 35 A neighbourood with a total area of 200 hectares has a gross density of 300 persons
per hectare (pph). If the residential area is 60% of the total area, then net density
(in pph) of the neighbourhood is
(A) 300 (B) 450 (C) 500 (D) 750
Answer: (C) 500

Net Residential Area= × 200 = 120 ha

Net Density (pph) = = = 500 pph

Q. 36 Identify the parameters used in the Hazen and William’s nomogram to calculate pipe
diameter for water supply:
P. flow rate in lit/sec Q. Pipe diameter in mm
R. Population to be served S. Head loss in mm
T. Velocity in m/s
(A) P, Q, S (B) R, S, T (C) P, R, S (D) P, S, T
Answer: (D) P, S, T

Q. 37 Match the domes in Group I with their examples in Group II


Group I Group II
P. Dome with huge central cut out at the top 1. Pisa Cathedral
Q. Dome with slit windows at the springing level 2. St. Peters Cathedral
R. Dome with an elliptical base 3. Pantheon
S. Dome on a drum with a lantern on top 4. Hagia Sophia
(A) P-2, Q-1, R-3, S-4 (B) P-3, Q-1, R-2, S-4 (C) P-3, Q-4, R-2, S-1 (D) P-3, Q-4,
R-1, S-2
Answer: (D) P-3, Q-4, R-1, S-2
Dome with huge central cut out at the top- this central opening known as oculus that is
seen in Pantheon.

. 38 Match the institution in Group I with their architects in Group II:


Group I Group II
P. National Dairy Development Board, New Delhi 1. B. V. Doshi
Q. National Institute of Immunology, New Delhi 2. Charles Correa
R. Indian Institute of Management, Bangalore 3. A. P. Kanvinde
S. Jodhpur University, Jodhpur 4. J. A Stein
5. Raj Rewal
6. U C Jain
(A) P-3, Q-5, R-1, S-6 (B) P-6, Q-3, R-4, S-1 (C) P-3, Q-1, R-4, S-6 (D) P-3, Q-4, R-2,
S-6
Answer: A) P-3, Q-5, R-1, S-6

Q. 39 Identify the urban functions that are included under Social Infrastructure:

P. Schools and Colleges


Q. Hospitals and Clinics
R. Roads and Foot paths
S. Parks and Plazas
T. Malls and Markets
U. Community Centres

(A) P, Q, S, U (B) P, Q, S,T (C) P, R, S, U (D) Q, S, T, U


Answer: (A) P, Q, S, U
Physical Infrastructure: Transportation, Sanitation, Electricity, Water supply

. 40 Match the tombs in Group I with their architectural characteristics in Group II


Group I Group II
P. Tomb of Sher Shah 1. Irregular pentagonal site plan
Q. Tomb of Ghias-ud-din Tughlaq 2. Octagonal plan
R. Humayun’s tomb 3. Gateway with four minarets
S. Akbar’s tomb 4. Persian dome
(A) P-4, Q-1, R-2, S-3 (B) P-2, Q-1, R-4, S-3 (C) P-4, Q-3, R-2, S-1 (D) P-2, Q-3, R-
1, S-4
Answer: (B) P-2, Q-1, R-4, S-3

. 41 Match the high rise tube structural systems in Group I with their corresponding terms in
Group II
Group I
P Q R S

Group II
1. Framed Tube 2. Bundled Tube 3. Braced Tube 4.Perforated shell tube
(A) P-1, Q-3, R-2, S-4 (B) P-4, Q-1, R-3, S-2 (C) P-4, Q-1, R-2, S-3 (D) P-1, Q-4,
R-3, S-2
Answer: (B) P-4, Q-1, R-3, S-2

. 42 A town with a population of 50,000 has an average household size of 5.0. The number of
occupied dwelling units is 8400 of which 10% are in dilapidated condition. The housing
demand of the town is
(A) 760 (B) 1600 (C) 2440 (D) 10840
Answer: (C) 2440

Soln: Total no of household = 10,000 (but only 8400 DU are available, so


there is a backlog)

Out of 8400, 10 % are dilapidated……..so ×8400 = 840 are dilapidated.


Housing demand is backlog + dilapidated
(10,000- 8400) + 840 = 2440

. 43 Match the items in Group I with those in Group II:


Group I Group II
P. Hypostyle Hall 1. Roman Architecture
Q. Ziggurat 2. Egyptian Architecture
R. Acropolis 3. Assyrian Architecture
S. Triumphal Arch 4. Persian dome
(A) P-1, Q-3, R-4, S-2 (B) P-2, Q-3, R-1, S-4 (C) P-1, Q-4, R-2, S-3 (D) P-2, Q-3, R-
4, S-1
Answer: (B) P-2, Q-3, R-1, S-4
Great Hypostyle Hall at Karnak, Egypt: hypostyle hall has a roof which is
supported by columns.
Assyrian empire (2400 B.C. to 612 B.C.), which was located mainly between
Tigris and Euphrates rivers. (Current region of Iran- Iraq)
Acropolis: Citadel located close to Athens, Greece and has ruins of
Parthenon, stoa.

Hypostyle Hall Karnak, Egypt Acropolis, Athens, Greece

. 44 Match the planning model in Group I with their proponents in Group II


Group I Group II
1. Homer Hoyatt
2. Ernest Burgess
3. Von Thunen
4. Harris and Ullman
5. William Reilley
(A) P-1, Q-4, R-5 (B) P-2, Q-1, R-4 (C) P-4, Q-1, R-2 (D) P-3, Q-2, R-1
Answer: (B) P-2, Q-1, R-4

Q. 45 The correct sequence in the four stage model used for transportation planning is
(A) Trip Generation→ Trip distribution→ Modal Split→ Trip Assignment
(B) Trip Generation→ Trip Assignment→ Modal Split→ Trip distribution
(C) Trip distribution→ Modal Split→ Trip Assignment→ Trip Generation
(D) Trip Generation→ Trip distribution→ Trip Assignment→ Modal Split
Answer: (A) Trip Generation→ Trip distribution→ Modal Split→ Trip Assignment
trip generation (travel choice)
trip distribution (destination choice)
modal split (mode choice)
traffic assignment (route choice)
The four steps of the classical urban transportation planning system model are:
Trip generation determines the frequency of origins or destinations of trips in
each zone by trip purpose, as a function of land uses and household
demographics, and other socio-economic factors.
Trip distribution matches origins with destinations, often using a gravity model
function, equivalent to an entropy maximizing model.
Mode choice computes the proportion of trips between each origin and
destination that use a particular transportation mode.
Route assignment allocates trips between an origin and destination by a
particular mode to a route. Often (for highway route assignment) Wardrop’s
principle of user equilibrium is applied (equivalent to a Nash equilibrium),
wherein each driver (or group) chooses the shortest (travel time) path, subject
to every other driver doing the same. The difficulty is that travel times are a
function of demand, while demand is a function of travel time, the so-called bi-
level problem. Another approach is to use the Stackelberg competition model,
where users (“followers”) respond to the actions of a “leader”, in this case for
example a traffic manager. This leader anticipates on the response of the
followers.

Q. 46 Identify the objects with which the explode command in Auto CAD can be used:

P. Polyline
Q. Block
R. Multiline Text
S. Arc
T. 3-D Solid

(A) P, Q, R, T (B) P, R, S, T (C) P, Q, S (D) P, Q, S, T


Answer: (A) P, Q, R, T

. 47 Match the planning terms in Group I with their descriptions in Group II


Group I Group II
P. Eminent Domain 1. Protecting the land by reassigning the rights to
develop
from
one area to another
Q. Police Power 2. Regulatory Behaviour and enforcing order
within the state
teritory
R. Transfer of Development Rights 3. Protecting the individual Development
rights of a citizen
by seeking state protection
4. Inherent power of state
to seize private property without the
owner’s consent
(A) P-4, Q-1, R-2 (B) P-2, Q-3, R-4 (C) P-1, Q-3, R-2 (D) P-4, Q-2, R-1
Answer: (B) P-4, Q-2, R-1
(Input from Prakul Sardana, Apeejay School of Architecture and Planning, Greater Noida)

. 48 A building has a rooftop area of 300 sq. m. If the avg. annual rainfall in the region is 700
mm and the Runoff Coefficient of the roof top is 0.8, then the maximum amount of
rainfall that can be harvested from the roof top (in litres) is
(A) 168 (B) 262 (C) 168000 (D) 2625000
Answer: (C) 168000
Run off= CIA where C= Runoff Coefficient
I= Intensity of rainfall (here annual rainfall is given)
A= Area

= 168,000 litres

. 49 Identify Pozzolana from the following materials:


P. Cement Q. Fly- Ash R. Sand S. Surkhi
(A) Q, S (B) Q, R, S (C) P, Q, S (D) P, R
Answer: (B) Q, R, S (mostly inert material)
Pozzolana, is hydraulic cement discovered by the Romans and is made by grinding
pozzolana (a type of slag that may be either natural—i.e., volcanic—or artificial, from a
blast furnace) with powdered hydrated lime. Roman’s used two parts by weight of
pozzolana mixed with one part of lime to give strength to mortar and concrete in
bridges and other masonry and brickwork. During the 3rd century BC, they used
pozzolana instead of sand in concrete and mortared rubblework, giving extraordinary
strength. Used with an aggregate of brick, or marble, the material contributed to the
evolution of new architectural forms in such monumental constructions as the
Pantheon and the Baths at Rome.

. 50 Match the notations in the given figure in Group A with corresponding names in Group II:
Group II
1. Intrados 2. Extrados 3. Archivolt
4. Spring 5. Rise 6. Keystone
(A) P-6, Q-4, R-1, S-2, T-5 (B) P-6, Q-5, R-2, S-1, T-4
(C) P-6, Q-3, R-2, S-1, T-5 (D) P-6, Q-3, R-1, S-2, T-4
Answer: (B) P-6, Q-5, R-2, S-1, T-4
Common Data Questions
Common data for questions 51 and 52:
A construction project has the following data:
Activity Duration(days) Predecessors
P 4 -
Q 3 P
R 7 P
S 2 P
T 4 Q
U 6 S
V 4 R, T, U

. 51 The normal project duration (in days) is


(A) 14 (B) 15 (C) 16 (D) 17
Answer: (C) 16
The path is P,S,U, V

Q. 52 The critical activities of the project are


(A) P, Q, R, V (B) P, R, S, U
(C) P, Q, T, V (D) P, S, U, V
Answer: (D) P, S, U, V
(courtesy: Shweta Bharti, B Arch, Ganga Institute of Arch and Town Planning, MDU Rohtak )

Common data for questions 53 and 54:


A Seminar hall has a volume of 2,000 cu.m. and total absorption of all acoustic
materials without any audience is 80 m²-sabines.
Q. 53 The reverberation time of the empty hall (in seconds) will be
(A) 1.0 (B) 4.0 (C) 8.0 (D) 12.0
Answer: (B) 4.0
Soln: A standard reverberation time has been defined as the time for the sound to die
away to a level 60 decibels below its original level.

Where RT60 = Reverberation time in sec


V = Volume in m³
Se = Total Absorption

. 54 When the same seminar hall is filled with audience, the reverberation time is recorded as
2.0 seconds. Then the total absorption of all acoustic material (in m²-sabines) will be
(A) 40 (B) 80 (C) 160 (D) 320

Answer: (C) 160


= 160 m²-sabines

ommon data for questions 55 and 56:


An office has an area of 60 sq. m. with floor height of 3 m and occupancy of 5 persons.
The external wall area is 40 sq.m. which includes 4 sq.m. of double glazed windows.
The thermal transmittance rate (U) of external wall is 0.35 and window is 2.00. External
and Internal design temperatures are 34º C and 22º C respectively.
. 55 The heat gain through the external walls and windows (in watts) will be
(A) 151.2 (B) 168.0 (C) 247.2 (D) 264.0
Answer: (C) 247.2
Soln:
?? = A× U × (T1-T2), where
?? = heat transfer in watts
A= Area in sqm.
U= Thermal transmittance rate
T1, T2= Difference in temperature

?? = 36× 0.35 × (34-22) + 4× 2.0 × (34-22)


= 36× 0.35 × 12 + 4 × 2 × 12
= 151.2 + 96= 247.2 watts

. 56 If 20lit/sec/person of air is extracted for the office, calculate the ventilation rate in terms
of air changes/hour.
(A) 0.4 (B) 2.0 (C) 4.0 (D) 20.0
Answer: (B) 2.0

Ventilation rate=

= = = 2 air changes / hour


Standards of ventilation
Cubic space- 3000 cubic feet per person per hour. (range is 300- 3000)
Air change- 2 to 3 times per hour in living room and 4 to 6 times per hour in
work rooms and assemblies.
(Note: For volume of room calculation height above 10-12 ft are not taken into
account)

Linked answers Questions


Statement for linked answers Questions 57 and 58:
A cantilever beam XY of 2.5 m span is supported at P and is subjected to 40 kN point
load at free end Y.

. 57 If self weight of the beam is neglected, bending moment developed at the fixed end (in
kN m) is
(A) 50 (B) 100 (C) 150 (D) 200
Answer: (B) 100
For point load BM at ‘X’ is w× length solving we get 2.5 × 40= 100
kNm
. 58 A Uniformly distributed load (in kN /m) that will result in the same value of bending
moment at the fixed end is
(A) 12 (B) 22 (C) 32 (D) 42
Answer: (C) 32
Let the uniformly distributed load be ‘w’ kN /m

BM for uniformly distributed load =

Here given BM = 100 therefore we get =100


Solving wl2 = 200 (here length of beam is 2.5m)

w= = 32 kN /m

Statement for linked answers Questions 59 and 60:


A semi circular stone arch of thickness 30 cm is provided over an opening in a brick
wall. The wall has length 3.0 m, width 30 cm and height 3.0 m. The opening has span
1.0 m and height 2.0 m.
Q. 59 The quantity of stone work in the semi circular arch (in cu.m) is
(A) 0.141 (B) 0.184 (C) 0.325 (D) 0.613
Answer: (B) 0.184

Quantity of stone work = = 0.184

Q. 60 The quantity of brick work in the wall (in cu.m) is


(A) 1.369 (B) 1.445 (C) 1.629 (D) 1.798
Answer: (D) 1.798
Note: Here the arch is placed over the opening of 1m x 2m. If the total ht of opening
is taken as 2 m including that of arch , answer will not come.

Area of opening = 2.0 ×1 + ( × π × ) since there is semicircular arch


over opening.
= 2.0 + 0.3925
= 2.3925
If we multiply this opening with thickness of wall we get vol. that has to be deducted
from whole vol. of wall as if it was a solid wall.
= 2.3925 × 0.3= 0.718 m3
In this, vol. of stone has to be added = 0.718 + 0.184= 0.902 m3
Total vol of solid wall = 3 × 3 × 0.3= 2.700m3
Subtracting we get 2.700m3 - 0.902 m3 = 1.798 m3
GATE 2007

Q 1- Q 20 One mark each


1. Ramsar list is related to
(A) Wetlands
(B) Heritage Buildings
(C) Seismic Zones
(D) Special Economic Zones
Answer: (A) Wetlands
Ramsar is in Iran and Wetland convention was held in year 1971.

2. Hazen’s- William’s nomogram is used to calculate


(A) size of sanitary pipe lines
(B) size of water supply lines
(C) capacity of overhead water reservoir
(D) capacity of water required for fire fighting
Answer: (B) size of water supply lines

.3 A woonerf is a
(A) pavement pattern (B) sanitation system element (C) speed reducing element (D)
furniture detail
Answer: (A) pavement pattern
A woonerf is a living street where pedestrians and cyclists have legal priority over
motorists as implemented in the Netherlands.
A chicane is an artificial feature creating extra turns in a road, used in motor racing and on streets to slow traffic
for safety. For example, one form of chicane is a short, shallow S-shaped turn, requiring the driver to turn slightly
left and then right again to stay on the road, which slows them down.
A pedestrian chicane is a kind of permanent fence used at a railway crossing to slow pedestrians down and to
force them to observe both directions before crossing the railway tracks. While passing the chicane, one has to
turn to the left and to the right, increasing the probability of seeing an approaching train. A similar arrangement is
sometimes used at the entrances of parks to impede bicycle or car access.

Q.4 In Urban planning, cohort refers to


(A) age and sex classification of population (B) contour levels in slope analysis
(C) land use classification of Public and Semipublic spaces(D) soil layer classification
Answer: (A) age and sex classification of population
Cohorts divides the population into five year age cohorts, usually by gender.
Q. 5 The Project Habitat, Montreal, designed by Moshe Safdie is an example of
(A) high rise apartments
(B) low rise detached dwellings
(C) organic architecture
(D) prefabricated housing
Answer: (D) prefabricated housing

Q.6 The degree of freedom of a joint in a plane truss is


(A) two (B) three (C) four (D) six
Answer: (B) three
In a plane truss members lie in a single plane. A space frame truss is a three-
dimensional framework of members pinned at their ends. Adjacent figure is a space
frame.

Q.7 A brick cut length wise into two pieces so that each piece is half as wide as full brick is
called a
(A) King closer (B) Frog (C) Quion (D) Queen Closer
Answer: (D) Queen Closer
Q. 8 The strength of concrete increases with
(A) increase in water cement ratio (B) decrease in water cement ratio
(C) increase in workability (D) decrease in cement aggregate ratio
Answer: (B) decrease in water cement ratio

Q.9 The point of contraflecture is the point where the


(A) shear force change its sign
(B) deflection is zero
(C) bending moment changes its sign
(D) torque is zero
Answer: (C) bending moment changes its sign
In a bending beam, a point is known as a point of contraflexure if it is a location at
which no bending occurs. In a bending moment diagram,
it is the point at which the bending moment curve
intersects with the zero line. In other words where the bending
moment changes its sign from negative to positive or vice versa.

Q.10 When wind loads are accounted for in the design of structures, the permissible stresses
in the material are increased by
(A) 10 % (B) 16.33 % (C) 33.33 % (D) 50 %
Answer: (C) 33.33 %

Q.11 The term coined by Paolo Soleri that combines ecology with architecture and deals with
habitats maintaining an extremely high population density is
(A) Archaeology (B) Proxemics(C) Arcology (D) Utopia
Answer: (C) Arcology
Proxemics: Study of the spatial requirements of humans and the effects of
population density on behavior, communication, and social interaction.
Utopia: The word was coined in Greek by Sir Thomas More for his 1516 book
‘Utopia’, describing a fictional island society in the Atlantic Ocean. The term
has been used to describe both intentional communities that attempt to create
an ideal society, and imagined societies portrayed in fiction. It has spawned
other concepts, most prominently dystopia.

Q.12 The dislocation of continuity in rock strata as a result of cracking of the earth’s crust is
called
(A) Fissure (B) Fault (C) Eluvium (D) Drift
Answer: (B) Fault
Fissure: a long narrow opening.
Fault: a fault is a planar fracture or discontinuity in a volume of rock, across
which there has been significant displacement along the fractures.
Eluvium: eluvium or eluvial deposits are those geological deposits and soils
that are derived by in situ weathering or weathering plus gravitational
movement or accumulation.
Drift: In geology, drift is the name for all material of glacial origin found
anywhere on land or at sea, including sediment and large rocks.

Q.13 LEED is the internationally accepted rating system for


(A) Green Buildings (B) Fire Resitant Buildings (C) Intelligent Buildings (D)Tall
Buildings
Answer: (A) Green Buildings

Q.14 An architect of Chicago School movement is


(A) Richard Boyle (B) Louis Sullivan (C) Hector Guimard (D) William Morris
Answer: (B) Louis Sullivan

Q.15 Surkhi is obtained by grinding


(A) well burnt clay bricks (B) slag from industry(C) stone aggregate (D) rice husk
Answer: (A) well burnt clay bricks

Q.16 Himadpanthi style of temples belong to


(A) Himalya (B) Deccan (C) Orissa (D) Kerala
Answer: (B) Deccan

Q. 17A building in which the roof is perfectly hemispherical on the inside and a shallow dome
on the outside is
(A) Hagia Sophia (B) Pantheon (C) Parthenon (D) Gol Gumbaz
Answer (B) Pantheon
Q. 18 National Science Centre at Pragati Maidan, New Delhi is designed by
(A) J. A. Stein (B) Anant Raje
(C) Raj Rewal (D) A. P. Kanvinde
Answer: (D) A. P. Kanvinde

Q. 19 In Islamic Architecture, the device used for placing a perfect circular dome over a
square plan is called
(A) Mehrab (B) Scroll (C) Mastaba (D) Squinch
Answer: (D) Squinch
Mehrab/ Mihrab: a semicircular niche in the wall of a
mosque that indicates the qibla; that is, the direction of
the Kaaba. (GATE 1991, GATE 2015)
Scroll: In architecture and furniture design, use of
curved elements suggesting such shapes as a sea
wave, a vine or a scroll of paper partly unrolled
Mastaba: Ordinary man/ Poorman’s grave in Egypt.
Squinch and Pendentive: A squinch in architecture is
a construction filling in the upper angles of a square
room so as to form a base to receive an octagonal or spherical dome. A later
solution of this structural problem was provided by the pendentive.
Pendentives, which are triangular segments of a sphere, taper to points at the
bottom and spread at the top to establish the continuous circular or elliptical
base needed for the dome.

Q. 20Parallel sound rays incident on a convex surface of a fibre board will


(A) converge and reduce in intensity (B) converge and increase in intensity
(C) disperse and reduce in intensity (D) disperse and increase in intensity
Answer: (C) disperse and reduce in intensity

Q 21 to Q 75 carry two marks each.


Q. 21Match the architect-planner in Group I with their contributIons in Group II
Group I Group II
P. Hippodamus 1. City Beautiful
Q. Michelangelo 2. Star- shaped Plan
R. Leon Battista Alberti 3. Grid Iron Plan
S. Daniel Burnham 4. Campidoglio
5. St. Peters Square
(A) P-3, Q-4, R-2, S-1 (B) P-3, Q-5, R-2, S-4 (C) P-4, Q-1, R-5, S-3 (D) P-3, Q-
2, R-1, S-5
Answer: (A) P-3, Q-4, R-2, S-1
Priene- Hippodamus
Alexandria- Dinocrates
Daniel Burnham- Town planner of Chicago who incorporated city beautiful
movement.
Campidoglio: The existing design of the Piazza del Campidoglio and the
surrounding palazzi was created by Renaissan architect Michelangelo in
1536–1546.

Q. 22The characteristics of Japanese gardens are


P. Stepping stones S. Miniature symbolic elements

Q. Stone lanterns T. Stone water basins


R. Octagonal Geometry U. Monumental scale
(A) P, Q, S, T (B) P, Q, U (C) R, S, T (D) Q, R, S, T
Answer: (A) P, Q, S, T

Q. 23Match the styles of Architecture in Group I with the elements in Group II


Group I Group II
P. Khajuraho 1. Star shaped garbh griha

Q. Dravidian 2. Gopuram
R. Hoysala 3. Pyramidal roof
S. Himalyan 4. Urushringa
(A) P-1, Q-2, R-4, S-3
(B) P-4, Q-2, R-1, S-3
(C) P-2, Q-4, R-3, S-1
(D) P-3, Q-4, R-2, S-1
Answer: (B) P-4, Q-2, R-1, S-3

Q.24 A site has a uniform slope of 6 %. The site map has


seven contour lines with the elevation of highest contour as + 53 meters. If the distance
between the mid point of highest and lowest contours is 700 metres, then the contour
interval in metres is
(A) 6 (B) 7 (C) 11 (D) 42
Answer: (B) 7
Soln: Slope is 6 % (for every 100 m there is drop of 6 m)
For 700 m, drop is 42 m ( it means lowest contour is at 53- 42= + 11m)
42 m is to be divided in 6 interval (note there are seven contour lines but six intervals)
so contour interval = 7 m.
Contour lines are at 53, 46, 39, 32, 25, 18,11 meters.
Q. 25Match the statement about thermal comfort in Group I with True/ False in Group II
Group I Group II
P. Low capacitance material should be used to store heat gain 1. True
Q. Stack effect depends on temperature difference between indoor 2. False
and outdoor air
R. Venturi effect is a passive cooling technique
S. Wind breaks are used to maximize winter wind turbulance
(A) P-1, Q-2, R-2, S-2 (B) P-1, Q-2, R-2, S-1 (C) P-2, Q-1, R-1, S-2 (D) P-2,
Q-1, R-1, S-1
Answer: (C) P-2, Q-1, R-1, S-2

Q. 26A person standing at a point in a public plaza is observing a façade of height 40


metres from a distance of 120 metres. The sense of enclosure
experienced by the person is equivalent to the limits of
(A) Loss of enclosure (B) Minimal enclosure (C) Full enclosure (D) Threshold of
enclosure.
Answer: (B) Minimal enclosure
(input from Surbhi Jethani, B. Arch, MITS Gwalior, BEM ,SPA Delhi)
1:1 is full enclosure, 1:2 is threshold of enclosure, 1:3 is minimal enclosure, 1:4 is loss of enclosure.

Q. 27Match the Urban Planning Theories in Group I with their proponents in Group II
Group I Group II
P. Sector Theory 1. Walter Christaller
Q. Multiple Nuclei Theory 2. Clarence Perry
R. Neighbourhood theory 3. Ebenezar Howard
S. Central Place Theory 4. Harris and Ullman
5. Homer Hoytt
(A) P-1, Q-4, R-5, S-3 (B) P-4, Q-2, R-3, S-1 (C) P-5, Q-1, R-2, S-3 (D) P-5, Q-4, R-
2, S-1
Answer: (D) P-5, Q-4, R-2, S-1

Q. 28The plan of residential area with small houses on small plot has an uniform fabric with
(A) fine grain and uniform texture (B) coarse grain and uniform texture
(C) fine grain and uneven texture (D) coarse grain and uneven texture
Answer: (A) fine grain and uniform texture
Q. 29Match the Change Properties command in AutoCad (Group I) with the actions (Group
II) it can perform on a given dashed lines.
Group I Group II
P. Elev 1. Change the dashed line to a non dashed line
Q. Ltype 2. Changes the size and spacing of dashes
R. Thickness 3. Changes the position along th Z axis
S. LtScale 4. Change the width of line on screen
5. Changes the height along th Z axis
6. Changes the position along th Y axis
(A) P-6, Q-1, R-4, S-2 (B) P-5, Q-2, R-6, S-4 (C) P-3, Q-1, R-5, S-2 (D) P-6,
Q-4, R-3, S-1
Answer: (A) P-6, Q-1, R-4, S-2

Q. 30Match the statements on Intelligent Buildings in Group I with True/ False in Group II
Group I Group II
P. All intelligent buildings are example of high tech architecture 1. True
Q. An intelligent building is synonymous with a smart building 2. False
R. An intelligent building need not deploy a building automation system
S. High tech architecture always result in intelligent buildings
(A) P-1, Q-1, R-2, S-2 (B) P-1, Q-2, R-2, S-2
(C) P-2, Q-2, R-1, S-1 (D) P-2, Q-1, R-1, S-1
Answer: (A) P-1, Q-1, R-2, S-2

Q. 31The correct sequence of various components of a house water connection from the
municipal water main is
(A) Stop cock → Water Meter → Goose Neck → Service Pipe → Ferrule connection
(B) Ferrule connection → Stop cock → Goose Neck → Service Pipe → Water Meter
(C) Goose Neck → Ferrule connection → Service Pipe → Water Meter → Stop cock
(D) Ferrule connection → Goose Neck → Service Pipe → Stop cock→ Water Meter

Answer: (D) Ferrule connection → Goose Neck → Service Pipe → Stop cock→
Water Meter

Q. 32The figure that will be generated by the following sequence of commands in AutoCAD
is
Command: pline
Specify start point: 0, 0
Specify next point: @50, 0
Specify next point: @0, -25
Specify next point: @25<180
Specify next point: c

Answer: B
Q. 33A sector has a gross density of 250 persons per hectare and a net density of 400
persons per hectare. If the area of sector is 120
hectares, then the percentage of non- residential area is
(A) 30 (B) 35.5 (C) 37.5 (D) 40
Answer: (C) 37.5
Population of Sector = 250 ×120 = 30000
Let the Residential area be ‘y’, so non residential area = 120 – y
400 × y = 30000 therefore ‘y’= 75 ha (Residential Area)

Percentage of non residential area= ×100= 37.5 %

Q. 34Match the systems of plumbing for building drainage in Group I with their descriptions
in Group II
Group I Group II
P. One- pipe system 1. Minimum two pipes, one for soil and other for sullage
Q. Two-pipe system 2. Single pipe for soil and sullage, and serving as vent for all
traps
R. Single stack system 3. Minimum two pipes one for soil and sullage and the other
for vent
4. Single pipe for soil and sullage, and serving as vent for soil
traps only
(A) P-4, Q-3, R-2 (B) P-3, Q-2, R-1 (C) P-2, Q-3, R-4 (D) P-3, Q-1, R-2

Answer:
(D) P-3, Q-1, R-2
Q. 35In a plane truss, the equation in terms of m and j is used to check its determinacy and
stability, where m= no of members and j = no of joints. The truss is deficient and
unstable when
(A) m < 2j – 3 (B) m = 2j - 3
(C) m > 2j- 3 (D) both A and B –re correct
Answer: (A) m < 2j – 3
Internal Stability: m < 2j – 3 ⇒ truss is internally unstable
m ≥ 2j – 3 ⇒ truss is internally stable provided it is geometrically
stable
m ≡ total number of members, j ≡ total number of joints
Geometric stability in the second condition requires that the members be properly
arranged.

Q. 36Match the functions in Group I with the numbers shown in the figure of Concentric
zone Theory of Burgees
Group I Group II

P. Central Business Disrict


Q. Commuters zone
R. Workingmen’s Home
S. Zone of better residences
T. Zone of Transitions
(A) P-1, Q-2, R-5, S-4, T-3 (B) P-1, Q-5, R-3, S-4, T-2
(C) P-2, Q-4, R-5, S-3, T-1 (D) P-3, Q-5, R-1, S-4, T-2

Answer: (B) P-1, Q-5, R-3, S-4, T-2


The eventual model from centre to periphery contained: the core or central business
district, the zone in transition, the zone of workingmen’s
homes, the zone of better residences, and the commuter zone.
‘Garden City’ movement was propagated by Ebenezar Howard in 1896.
Burgees (US based) ‘Concentric Zone theory’ came in the year 1925 in his
book ‘The City’.
Homer Hoytt in 1939 presented ‘Residential Sector Theory’.
Harris and Ullman in 1945 brought ‘Multiple Nuclei Model’.
Chandigarh was planned in 1950 based on Sector theory.

. 37 For a PERT activity, the optimistic time, the most likely time and pessimistic time are 1, 2
and 9 days respectively. The expected time for the activity (in days) is
(A) 9 (B) 6 (C) 4 (D) 3
Answer: (D) 3

PERT Weighted Average (expected time) (mean) =

= days

Q. 38Zoning regulations deal with


P. Density S. Minimum areas of room
Q. Land Use T. Height
R. Building Materials U. Reserved Land areas
(A) P, Q, T (B) P, Q, R, U (C) Q, S, U (D) Q, R, S, T
Answer: (A) P, Q, T

Q. 39Match the temples in Group I with their distinguishing feature in Group II


Group I Group II
P. Konark 1. Golden Lily Pond
Q. Madurai 2. Sculpted marble ceiling
R. Dilwara 3. Twin Vimanas
S. Mamallapuram 4. Chariot
5. Torana
(A) P-3, Q-1, R-2, S-5 (B) P-4, Q-1, R-2, S-3 (C) P-2, Q-3, R-5, S-1 (D) P-3, Q-4,
R-1, S-2
Answer: (B) P-4, Q-1, R-2, S-3

Q. 40The correct sequence of generic elements in a classical order arranged from top to
bottom is
(A) Architrave→Frieze→Capital→Cornice→Shaft → Pedestal → Base
(B) Architrave → Capital → Cornice → Frieze → Base → Shaft → Pedestal
(C) Cornice → Frieze → Architrave → Capital → Shaft → Base → Pedestal
(D) Cornice → Capital → Frieze → Architrave→ Shaft → Pedestal → Base
Answer: (C) Cornice → Frieze → Architrave → Capital → Shaft → Base → Pedestal
Q. 41Match the tree forms in Group I with their common examples in Group II

Group I Group II
P. Broad 1. False Acacia
Q. Tapering 2. Holly
R. Conical 3. Lombardy Poplar
S. Columnar 4. Oak
5. Silver Maple
(A) P-1, Q-5, R-4, S-2 (B) P-1, Q-3, R-4, S-5
(C) P-4, Q-1, R-2, S-3 (D) P-4, Q-5, R-2, S-1
Answer: (C) P-4, Q-1, R-2, S-3

Broad/Layered Conical/ Columnar Weeping


Pyramidal
Oak Holly Ashoka Birch
Peepal Blue Spur Lombardy Hamlock
Poplar
Banyan Fir Cherry Willow
Mango Lindem Red Maple Bottle
Brush
Neem Sweet gum Tulip tree Q. 42A town has 16,000 existing
Jack fruit Cedar dwelling units of which 10% is
dilapidated. If the house need is 8,700 dwelling units and the average household size is
4.5, then the population of the town is
(A) 64,800 (B) 1,03,950 (C) 1,11,150 (D) 1,18,350
Answer: (B) 1,03,950
No of dilapidated units= 10/100 x 16,000= 1600 units.
House Need= dilapidated + homeless household
so homeless household = 8700- 1600
= 7100
Total household= 16,000 + 7,100= 23,100 household
Tot Pop= No of household × family size
= 23,100 × 4.5 = 1,03,950

Q. 43Match the descriptions in Group I with the elements of Ornamentation in Group II


Group I Group II
P. Painting on a freshly spread moist plaster surface with powdered pigments 1.
Chiaroscuro
Q. Figure incised into a stone surface or a metal plate yielding an impression 2.
Emboss in relief

3. Filigree R. Delicate or
intricate design on lattice work allowing through openings
4. Fresco S. Artistic
composition consisting of motifs borrowed from different sources
5. Intaglio
6. Pastiche
(A) P-1, Q-2, R-3, S-5 (B) P-1, Q-5, R-4, S-6
(C) P-4, Q-2, R-3, S-1 (D) P-4, Q-5, R-3, S-6
Answer: (D) P-4, Q-5, R-3, S-6
Chiaroscuro: use of strong contrasts between light and dark,in a painting
Emboss- To write or decorate with a raised character or pattern from the level
Filigree- a delicate kind of jewellery metalwork, usually of gold and silver, made
with tiny beads or twisted threads.
Fresco- mural painting executed upon freshly laid lime plaster.
Intaglio- Intaglio is the family of printing and printmaking techniques in which
the image is scratched into a surface, and the scratched line or sunken area
holds the ink.
Pastiche: A pastiche is a work of visual art, literature, or music that imitates the
style or character of the work of one or more other artists.
Glyph: an individual mark that contributes to the meaning of what is written.
Q. 44 Match the city plans in Group I with their designers in Group II
Group I Group II
P. London 1. Eliel Saarinen
Q. Berlin 2. Kenzo Tange
R. Helsinki 3. Alvar Aalto
S. Tokyo 4. Tadao Ando
5. Martin Machler
6. Patrick Abercrombie
(A) P-6, Q-5, R-1, S-2 (B) P-1, Q-3, R-5, S-2 (C) P-6, Q-3, R-1, S-4 (D) P-5, Q-
6, R-3, S-4
Answer: (A) P-6, Q-5, R-1, S-2
Patrick Abercrombie – British architect and town planner who redesigned
London after it was devastated by enemy bombardment in World War II.
(Greater London Plan of 1941)
Cristopher Wren: Designed London in 1666.
Kenzo Tange designed Tokyo according to the philosophy of Metabolism.

Q. 45On a door opening with effective span L, the total weight (W) of an equilateral triangle on
the base L is considered as a uniformly distributed load over the span. The bending
moment for the door opening is given by
(A) WL/2 (B) WL/4 (C) WL/6 (D) WL/8
Answer: (A) WL/2

Total weight= ‘W’ acting at distance from end.

Bending Moment= W× =

Q. 46Match the descriptions in Group I with the traffic terminology in Group II


Group I Group II
P. The length of the road ahead of the vehicle which should be visible 1. Visibility
distance to enable a driver to
stop in case of obstruction on the road. 2. Sighting
distance
Q. Distance covered by a vehicle from the instant a driver sees an 3. Overtaking sight
distance
obstruction ahead and brings the vehicle to a stop 4. Cross over distance
R. Distance required for a vehicle to overtake and safely pass another 5. Stopping
distance vehicle moving in the same
direction but at a lower speed.
(A) P-1, Q-3, R-4 (B) P-4, Q-3, R-5 (C) P-2, Q-5, R-4 (D) P-2, Q-5, R-3
Answer: (D) P-2, Q-5, R-3

Q. 47Match the labels on a door in Group I with their names in Group II


Group I Group II
1. Jamb
2. Muntin
3. Panel
4. Rail
5. Saddle
6. Stile

(A) P-1, Q-6, R-5, S-4, T-2 (B) P-1, Q-6, R-2, S-4, T-3
(C) P-5, Q-3, R-1, S-6, T-2 (D) P-5, Q-6, R-1, S-4, T-3
Answer: (B) P-1, Q-6, R-2, S-4, T-3

Q. 48A house was constructed 20 years ago at a cost of Rs 1,00,000/-. The estimated life of
the building is 50 years, at the end of which it will have a 15 % scrap value equal of its
cost of construction. Its present value in Rupees is
(A) 36, 000 (B) 66,000 (C) 75,000 (D) 85,000
Answer: (B) 66,000
Current cost of building = 15,000 Rs

Annual depreciation = = 1700 Rs /year


Depreciation in 20 years= 1700 × 20= 34,000 Rs
Present Value= 1,00,000 - 34,000= 66,000 Rs

Q.49 A typical roof top Rain Water Harvesting System essentially comprises of
P. Roof Catchment
Q. Down Pipes
R. Rain gauge
S. Filter chamber
(A) P, R (B) P,R,S (C) Q,R,S (D) P,Q,S
Answer: (D) P,Q,S

Q. 50Match the architects in Group I with their works in Group II


Group I Group II
P. Norman Foster 1. Petronas Towers
Q. Cesar Pelli 2. Kansai Airport
R. Richard Meier 3. HSBC, Hongkong
S. Renzo Piano 4. The Athenum
5. Sydney Opera House
(A) P-3, Q-1, R-4, S-2 (B) P-4, Q-1, R-2, S-3 (C) P-3, Q-2, R-5, S-1 (D) P-5, Q-3, R-1, S-
2
Answer: (A) P-3, Q-1, R-4, S-2

Q. 51A single room of 3 metres x 5 metres enclosed by 20 cm thick walls has to be


constructed. The required foundation trench is 80 cm wide and 80 cm deep. The
quantity of earthwork in excavation in cubic metres is
(A) 10.75 (B) 12.80 (C) 18.70 (D) 20.24
Answer: (A) 10.75

Q. 52Match the parts of a tree log in Group I with their descriptions in Group II
Group I Group II
P. Heartwood 1. Outer Annual rings of the tree
Q. Sapwood 2. Thin horizontal rays radiating from the pith towards the bark
R. Cambium Layer 3. Outermost protective covering of the log
S. Medullary Rays 4. Innermost ring surrounding the pith
5. Outermost one ring between the bark and sap wood
(A) P-4, Q-2, R-5, S-3 (B) P-3, Q-5, R-4, S-1 (C) P-4, Q-1, R-5, S-2 (D) P-5, Q-1,
R-4, S-2
Answer: (C) P-4, Q-1, R-5, S-2

Q. 53The quantity of plastering in sq m required for both sides of a wall 5.0m x 0.3m x 3.0m (L
x B x H) with a window opening 2.0m x 0.30m x 1.2m is
(A) 25.2 (B) 27.6 (C) 30.0 (D) 34.8
Answer: (A) 25.2

Q. 54Match the Urban Theorist in group I with the Planning Concepts in group II
Group I Group II
P. Patrick Geddes 1. Cities in evolution and their relationship with man
Q. Charles Abrams 2. Judicious use of technological power
R. Constantine Doxiadis 3. Role of housing in Urban development
S. Lewis Mumford 4. The Science of Human settlements called Ekistics
(A) P-1, Q-3, R-4, S-2 (B) P-4, Q-2, R-3, S-1 (C) P-3, Q-4, R-1, S-2 (D) P-2, Q-
1, R-4, S-3
Answer: (A) P-1, Q-3, R-4, S-2

Q. 55If the reinforcement steel provided for a RCC slab of volume 15 .0 cu. M. is @ 1%,
then the quantity of steel required in kg is
(A) 655.5 (B) 1000.0 (C) 1,177.5 (D) 1,500.0
Answer: (C) 1,177.5
Density of steel is 7850 kg/m³

cu. M.

Q. 56The Prairie house design of Frank Lloyd Wright is characterized by


P. Horizontal Planes Q. Extended roofs R. Focal Fire Place
S. Steel Columns T. Vertical Screen windows
(A) P, R, S (B) P, Q, S (C) Q, R, S, T (D) P, Q, R, T
Answer: (D) P, Q, R, T
Wright’s Prairie Style house typically features a large, centrally-placed fireplace, a
hearth that “grounds” the house, that becomes its focus. Frequently he designed
benches on either side of it, in effect creating a room within a room.

Q. 57Match the window types in Group I with their descriptions in Group II


Group I Group II
P. Bay Window 1. Horizontal louvers pivoting simultaneously in a common frame
Q. Pivoted Window 2. A sash that rotates 90 or 180 about vertical or horizontal
axis at or near its
centre
R. Dormer window 3. Projecting outward from the main wall of a building forming
an alcove
within the room
4. Vertical window projecting out of a sloping roof
(A) P-3, Q-2, R-4, (B) P-2, Q-3, R-1 (C) P-1, Q-4, R-2 (D) P-4, Q-2, R-3
Answer: (A) P-3, Q-2, R-4

Q. 58Match the housing projects in Group I with the architects in Group II


Group I Group II
P. Tara Group housing, New Delhi 1. Bal Krishna Doshi
Q. Marine Front Housing, Cochin 2. Charles Correa
R. Aranya Community Housing, Indore 3. Hasmukh Patel
S. Asiad Village, New Delhi 4. Kuldip Singh
5. Laurie Baker
6. Raj Rewal
(A) P-2, Q-4, R-1, S-6 (B) P-3, Q-4, R-2, S-6 (C) P-2, Q-5, R-6, S-1 (D) P-1, Q-5, R-
3, S-6
Answer: (A) P-2, Q-4, R-1, S-6

Q. 59A beam of 50 mm diameter is simply supported at both ends and has an effective span
of 6 meters. It carries two loads of 50kN each at one-third span. The section modules (in
cm ³) of the beam at the quarter span is
(A) 11.17 (B) 12.27 (C) 13.37 (D) 14.47
Answer: (B) 12.27
Soln:
Section modulus (Z) is independent of loads

Z= where I = for circular section, and y=

Z= upon solving we get Z =

Z=
= 12.27
(with input from Tabassum Ansari,B Arch, Integral University, Lucknow)

Q. 60Match the Earth Quake related terms in Group I with the definitions in Group II
Group I Group II
P. Focus 1. The geographical point on Earth’s surface vertically above the
originating source
Q. Epicenter 2. The originating source of the seismic waves inside the
Earth
R. Centre of Mass 3. The point corresponding to Centre of Gravity of a Structural
system
S. Centre of Stiffness 4.The point through which the resultant of the restoring forces of
a
structural system act
(A) P-1, Q-2, R-3, S-4 (B) P-1, Q-2, R-4, S-3 (C) P-2, Q-1, R-3, S-4 (D) P-2, Q-1, R-
4, S-3
Answer: (C) P-2, Q-1, R-3, S-4
(with input from Aparna Shaw, NIT Hamirpur)

Q. 61Match the Architectural Styles in group I with the Construction systems in Group II
Group I Group II
P. Greek 1. Semi circular arch
Q. Roman 2. Trabeation
R. Indian 3. Corbelling
S. Gothic 4. Pointed Arch
(A) P-2, Q-4, R-3, S-1 (B) P-1, Q-2, R-4, S-3
(C) P-2, Q-1, R-3, S-4 (D) P-3, Q-1, R-2, S-4
Answer: (D) P-3, Q-1, R-2, S-4
Trabeation:Having horizontal beams or lintels rather than arches.

Q. 62For incandescent lamps the distribution of total energy emission is


(A) 5 % light and 95 % heat (B) 25 % light and 75 % heat
(C) 50 % light and 50 % heat (D) 75 % light and 25 % heat
Answer: (A) 5 % light and 95 % heat (incandescent lamps)
Fluorescent Lamps: 25 % light and 75 % heat
Energy Efficacy: The luminous efficacy of lamps is the number of lumens produced for
each watt of electrical power used.
Incandescent lamps: 10-17 lumens/watt (1.5%-2.5 %)
Fluorescent Lamps: 50- 70 lumens/watt (7 % - 10%)
(Note: For 100% efficacy = 680 lumes/watt.)

Q. 63Match the characteristics in Group I with the Climate elements in Group II


Group I Group II
P. High humidity accelerates rusting and rotting 1. Composite or monsoon
Q. High day time temperature and rapid cooling 2. Hot dry desert
at night causes materials to crack 3. Hot dry maritime
R. Seasonal changes in relative humidity cause 4. Tropical Upland
rapid weakening of building material 5. Warm humid
(A) P-5, Q-2, R-1 (B) P-4, Q-1, R-3 (C) P-5, Q-3, R-4 (D) P-4, Q-3, R-5
Answer: (A) P-5, Q-2, R-1

Q. 64The architectural projects of the International Style are


P. Aurora house by Aldo Rossi Q. Schroder house by Gerrit Reitveld
R. Thematic house by Jencks and Farrell S. Tugendhat House by Mies van der Rohe
T. Villa Savoye by Le Corbusier
(A) P, Q, R, T (B) P, S (C) Q, S, T (D) Q, R, T
Answer: (C) Q, S, T
Three principles of International Style (1920-30)
the expression of volume rather than mass.
the emphasis on balance rather than preconceived symmetry.
the expulsion of applied ornament.

Q. 65Tactile flooring with guiding blocks, an element of Barrier Free Design is used to aid
P. ambulant disabled Q. non ambulant disabled R. partially sighted S. totally
blind
(A) P, Q, S (B) P,Q, R (C) R, S (D) Q, S
Answer: (C) R, S

Q.66 Match the characteristics of vaults in Group I with their names in Group II
Group I Group II
P. Uniform semi circular cross section 1. Barrel
Q. Semi circular cross section larger at one end than the other 2. Cloister
R. Compound vault formed by perpendicular intersection of two vaults 3. Conical
S. Compound vault formed by four coves meeting along diagonal vertical planes 4.
Groin

5. Rampant
6. Stilted
(A) P-1, Q-6, R-5, S-2 (B) P-6, Q-3, R-4, S-2
(C) P-4, Q-5, R-2, S-6 (D) P-1, Q-3, R-4, S-2

Answer: (D) P-1, Q-3, R-4, S-2

Cloister Vault

Rampant Arch
Cloister Vault: A domelike vault having a square or polygonal base from which
curved segments rise to a central point. Also called domical vault.
A Barrel vault where one springing in point is higher than the opposite
springing point.

. 67 A 60º segmental arch is provided over a door of 1.0 m width. The wall thickness is 30
cm and the arch thickness is 20 cm. the mean length of arch in meters is
(A) 1.00 (B) 1.15 (C) 1.20 (D) 1.30
Answer: (B) 1.15

Radius of arch = 1 m +

Mean Length= (Since it is 60


Q.68 Match the statement about elevators and escalators in Group I with True/ False in
Group II
Group I Group II
P. Handling capacity of elevators for residential buildings as per 1. True
Indian standards is 7.5 % 2. False
Q. Minimum height from the top floor to the bottom of the lift
machine room should be 3,000 mm
R. Minimum width of escalator as per Indian standards is 1000 mm
S. Recommended angle with the horizontal for escalators is 30º
(A) P-1, Q-2, R-1, S-2 (B) P-2, Q-2, R-2, S-1 (C) P-2, Q-1, R-1, S-1 (D) P-1, Q-2,
R-2, S-1
Answer: (C) P-2, Q-1, R-1, S-1

. 69 The slenderness ratio for a cantilever prismatic column of length L with a circular cross
section having radius r is
(A) L / r (B) 2L / r (C) 3L / r (D) 4L / r
Answer: (D) 4L / r

The ratio of the effective length of a


column to the least radius of gyration of its cross section is called the slenderness
ratio.
Soln:

Slenderness ratio =
Where I= Ak² (where I is moment of inertia and ‘A’ is cross sectional area of column)
and = 2L (since it is cantilevered, see adjoining fig.)
k= radius of gyration

I= = Ak²

and ‘A’ is cross sectional area = π( Solving for k =( k²

k= or

Slenderness ratio = =
(corrected by Nischay Kumar, BVCoA, Navi Mumbai) [Ref: Q-69 (2007), Q-42(2003), Q18 (1994)]
S. No Type of column Moment of Inertia of
cross section
1 Circular solid column with
dia= D
I=
2 Rectangular cross section
column with breadth= b
and h= other dimension of I=
rectangle.

Q. 70Match the designers in Group I with the terms in Group II


Group I Group II
P. Max Dubious 1. Prefabrication
Q. Josephh Paxton 2. Domino System
R. Victor Horta 3. Minimalism
4. Vegetal ornamentation

(A) P-2, Q-1, R-4 (B) P-4, Q-1, R-3


(C) P-2, Q-4, R-3 (D) P-1, Q-3, R-4
Answer: (A) P-2, Q-1, R-4
Max Dubious was concrete expert
Domino system : Le Corbusier envisaged it as an affordable, prefabricated system
for the construction of new housing in the wake of World War I’s
destruction. Developed with the help of Max Dubois and Perret, the
system differed from the then standard Hennibique frame in its idealization of floors
as flat slabs without exposed beams. Its columns were perfectly straight
posts without capitals, set in from the edge of the slab. This system freed
both exterior and interior walls from all structural constraints.

Joseph Paxton- Crystal Palace design of London


exhibition (1851) using Glass and structural steel.
Victor Horta- Belgian Architect famous for Art Nouveau. Horta is one of the most
important names in Art Nouveau architecture; the construction of his Hôtel Tassel in
Brussels in 1892-3 means that he is sometimes credited as the first to introduce the
style to architecture from the decorative arts. The French architect Hector Guimard
was deeply influenced by Horta and further spread the “whiplash” style in France and
abroad.
Art Nouveau Elements
Flowing Lines: Art Nouveau is characterized by graceful lines. The lines are
rarely angular.
Violent Curves: Some artists referred to the curves in Art Nouveau works as
whiplash curves. Rhythmic patterns of curvy lines are characteristic of this art
style.
Organic Subject Matter: Plenty of flowers, leaves, vines, grass, seaweed,
insects and other organic images in Art Nouveau jewelry, hardware, windows
and architecture. Examples include images of birds etched into window frames
or curled around each other on fabric for upholstery, or abstract lilies drifting
around and connecting to each other on dinnerware.

Resistance of Classical Restrictions:


Instead of limiting art to painting on a canvas or sculpting out of marble, Art
Nouveau artists and architects looked for ways to make everyday objects into
pieces of art. A doorknocker might be molded to look like a dragonfly; an
entranceway might be graced by vine-like lines in the molding. You can find a
classic example of this by studying the entrances designed for the Paris Metro
by Hector Guimard.

François Hennebique (26 April 1842 – 7 March 1921) was a French engineer and
self-educated builder who patented his pioneering reinforced-concrete construction
system in 1892, integrating separate elements of construction, such as the column
and the beam, into a single monolithic element. The Hennebique system was one of
the first appearances of the modern reinforced-concrete method of construction.

Common data for Questions 71, 72, 73:


The continuous utility data for a construction project is as follows:
Activity Duration (days) Immediate
Normal Crash Predecessors
P 3 3 -
Q 4 4 P
R 2 1 P
S 3 3 P
T 0 0 Q
U 6 5 R, T
V 4 2 S
Q. 71The normal project time for the given network is
(A) 11 (B) 12 (C) 13 (D) 14
Answer: (C) 13

Q. 72For the all- normal solution, the total float and free float for the activity S are
(A) 1, 1 (B) 0, 3 (C) 3, 3 (D) 3, 0
Answer: (D) 3, 0
Floats in Projects are of 4 types:
i. Total Float: Obtained by deducting the Early start and the activity duration from
the Late finish of the activity. That means, we can get both the slacks obtained
from Late start / Early Start, as well as from Late Finish / Early Finish. Thus, Total
Float signifies the total slack time available for any activity in the Project without
delaying the overall Project’s duration.
ii. Safety Float: Obtained by deducting Late start plus the activity duration from the
Late Finish of activity. That means, we have the slack time available due to time
of Late finish / Early early finish of the activity. This Float does not interfere with
the float time available by the Predecessor activity.
iii. Free Float: Obtained by deducting the Early start plus the activity duration from
the Early Finish of the Activity. We have the slack time available due to time of
Early Start / Late Start of the activity. However, this float does not eat away the
slack time available for the successor activity.
iv. Independent Float: Obtained by deducting the Late start plus the activity duration
from the Early Finish. This float virtually leaves no time for breathing since, the
predecessor activity has started late, and the successor activity is scheduled for
an early start, after utilizing the time duration for the activity. Most of the time, the
slack time available is ZERO for this float

Q. 73While crashing the project, the first step of compression would involve the activity
(A) R (B) U (C) T (D) V
Answer: (B) U

Common data for Questions 74, 75


A room measuring 10 m x 10 m has to be illuminated to a level of 200 lux by a single
electrical lamp. The coefficient of utilization is 0.75 and the maintenance factor is 0.8.

Q. 74The lumen output required for the above lamp is


(A) 12,000 (B) 16,666 (C) 30,000 (D) 33,333
Answer: (D) 33,333
Soln: Total Lumens required for the room 200 × 10 × 10 = 20,000 lumens
Let the lumen output of lamp be ‘a’
Therefore ‘a’ × 0.75 × 0.8 = 20,000
Solving ‘a’ = 33,333 lumen
The Coefficient of Utilization is the ratio of luminous flux from a luminaire
incident upon a work plane to that emitted by the lamps within the luminaire.
Maintenance factor is the product of: Lamp lumen maintenance factor (LLMF)
× Lamp survival factor (LSF) × Luminaire maintenance factor (LMF)

Q. 75The depreciation factor for the above lamp is


(A) 0.6 (B) 1.25 (C) 1.33 (D) 1.66

Answer: (B) 1.25

Soln: Depreciation factor =


= 1.25

Linked Answers Questions: Q.76 to Q.85 carry two marks each.


Statement for Linked Answer Questions 76 & 77:
The following data is related to the design of a septic tank for a housing complex:
Population of the housing complex = 150
Water supply/ person / day =130 litres
Waste water flow= 80 % of water supply
Detention period = 1 day
Sludge production = 0.045 cum/ person/ year
Storage capacity of sludge = 1/3rd of septic tank capacity.

Q.76 Total capacity of septic tank in cubic metres is


(A) 31.70 (B) 23.40 (C) 20.80 (D) 15.60
Answer: (B) 23.40
Let the total capacity of tank be ‘x’ m3

Maximum Sludge storage capacity = and liquid storage will be =


Given that 80% of water supply goes in waste water , which finally enters the septic tank.
Therefore ×130 × 150 = solving for ‘x’ we get x= 23,400 litres= 23.40 m3

Q.77 De-sludging interval (to the nearest year) is


(A) 1 (B) 2 (C) 3 (D) 4
Answer: (A) 1

Maximum permissible sludge is 1/3rd the capacity of septic tank. × 23.40 = 7.8 m3
Sludge produced per year by 150 person 0.045 × 150 = 6.75 m 3

Desludging = = 1.15 year (say 1 year)

Statement for Linked Answer Questions 78 & 79:


A residential plot measuring 12 meters x 15 metres about a road on its smaller side.
Permissible ground coverage = 50 %, Floor Space Index (FSI) = 2.5 and maximum
permissible floors= 4.

Q.78 Maximum total buildable area in sq.m is


(A) 180 (B) 225 (C) 360 (D) 450
Answer: (C) 360
If there is no ht. restriction, total buildable area is 12× 15 × 2.5= 450 sq.m (if 50 %
ground coverage then 5 floors will be build)
Since permissible Ground coverage is 50 % and ht. restriction is up to 4 floors, total
buildable area

= 12 × 15× × 4= 360 sq.m

Q.79 As per revised building bye-laws, if the required setbacks are - Front 3m, each Side 2m
and, Rear 2 metres, then the maximum total buildable area will
(A) increase by 248 sq.m (B) increase by 40 sq.m
(C) decrease by 30 sq.m (D) decrease by 40 sq.m
Answer: (D) decrease by 40 sq.m
Effective area of plot for construction= 8m x 10m
If 4 floors are constructed then total buildable area= 80× 4= 320 sq.m (loss of 40
sq.m from previous one)

Statement for Linked Answer Questions 80 & 81:


An aerial photograph is taken from a plane with a camera lens of focal length 305
mm. the desired scale of the photography is 1:25,000 and the height of the terrain
above mean sea level is 300 metres.
Q.80 The flying height of plane above mean sea level (in meters) is
(A) 7,625 (B) 7,925 (C) 8,562 (D) 8,965
Answer: (B) 7,925

Solving we get flying height from terrain = 25,000 x 305 =7625,000 mm= 7625 m
Question asked is flying height of plane above mean sea level= so 300 meters to be
added and we get 7,925m.
Q.81 If the above photograph is taken by a camera lens of focal length 210 mm from the same
flying height, then the scale of the photograph will be
(A) 1:45,000 (B) 1: 37,740 (C) 1: 36,310 (D) 1: 19,050
Answer: (C) 1: 36,310

Scale of photograph=

= making common units we get =


= 1: 36,310

Statement for Linked Answer Questions 82 & 83:


A beam of cross-section 300 mm x 400 mm has overhangs at both the ends. The
beam has a simple support of 10 metres and an overhang of 5 metres each at both
ends and carrying a load of 10 kN on both the free ends.
Q.82 The maximum value of shear force and bending moment in the beam are

(A) 5 kN, 50kN-m (B) 20kN, 80 kN-m


(C) 15 kN, 45kN-m (D)10 kN, 50kN-m
Answer: (D)10 kN, 50kN-m
RB+RC= 20
Taking moment about B, (RC × 10) + (10 × 5)= 10 ×15
10 RC = 150 -50
RC= 10 kN

Therefore RC = 10kN
From SF diagram max. SF= 10 kN
From BM diagram max BM= 50kNm

Q.83 The maximum values of bending stress and shear


stress developed in the beam in N/mm2 is
(A) 5.15, 0.1 (B) 6.25, 0.125
(C) 7.35, 0.15 (D)8.45, 0.175
Answer: (B) 6.25, 0.125
Max. bending stresses occur at the top and the bottom of the beam, where c = 200
mm, and is given by:

σ=

= = 6.25 N/mm2

Max. shear stress=

τ=
τ= = 0.125 N/mm2

Statement for Linked Answer Questions 84 & 85:


An auditorium has a volume of 3000 m3 with optimum reverberation time of 0.8
seconds.
Q.84 The sound absorption power required in the auditorium in m2-sabines is
(A) 250 (B) 400 (C) 600 (D) 800
Answer: (C) 600

Where RT60 = Reverberation time in sec


V = Volume in m³
Se = Total Absorption
Here Volume = 3000 m³ and = 0.8 sec

0.8 = solving this we get m2-sabines

Q.85 During a convocation programme in the same auditorium, the absorption power
increases by 200 m2-sabines. The reverberation time in seconds will now be
(A) 0.4 (B) 0.6 (C) 0.8 (D) 1.2
Answer: (B) 0.6

Here V= 3000 m3 and Se= 800 m2-sabines

Solving this we get


GATE 2008
Q.1- Q.20 carry one mark each

Q.1 Function of Air Handling unit in a building is to


(A) purify and re-circulate the cool air
(B) supply purified bulk of air from outside to the window air-conditioner
(C) collect the stale air from the room and throw it outside the building
(D) act as a container in which air is carried from one place to the another
Answer: No appropriate answer.
Air handling unit (AHU), is a device used to condition and circulate air as part of a
heating/cooling, ventilating, and air-conditioning (HVAC) system.
A larger air handler that conditions 100% outside air, and no recirculated air, is
known as a makeup air unit (MAU). It is used in OT of hospitals. DOAS
(Dedicated Outdoor Air System) provides 100% outdoor air and no air is
recirculated.

. 2 The KYOTO Protocol -2003 addressed the issue of


(A) Bio-diversity (B) Green House Gases (C) Wetlands (D) Rainwater Harvesting
Answer: (B) Green House Gases
Ramsar Convention- Wetlands (Ramsar is in Iran and Convention was held in
1971)
Kyoto Protocol- Green House Gases (Kyoto is in Japan)
Agenda 21- Sustainable Development

. 3 The principle of Solid Waste Management involves


(A) Reproduce, Reuse, Recycle (B) Recycle, Replenish, Reuse
(C) Reduce, Reuse, Reproduce (D) Reduce, Reuse, Recycle

Answer: (D) Reduce, Reuse, Recycle

. 4 The correct diagram for mirror stereoscope is


Answer: (A)
Wrong representation of mirror in (B) and (C).
In (D), light passing through Prism is wrong.

.5 Which of the following is not included in the UDPFI Guidelines for urban development?
(A) Perspective Plan (B) Development Plans (C) City Development Plans (D) Annual
Plans
Answer: (C) City Development Plans
(Courtsey Tejasvi Madeti, Vaishnavi School of Architecture and Planning, Hyderabad)

.6 A system of art- appreciation characterized by an unorthodox experimental approach to


appreciate visual, literary, and musical aspects of a design process, is called
(A) Avante- garde (B) Post- modernism (C) Neo- Impressionism (D) Proto-
Deconstruction
Answer: (A) Avante- garde
. 7 An applied science of Design concerning Universal human characters and
configurations aiming at effective utility and safety is called
(A) Anthropometry (B) Cognitive Behavioural Mapping (C) Universal design (D)
Ergonomics
Answer: (C) Universal design
Anthropometry: Anthropometry refers to the measurement of the human
individual.
Ergonomics: Design factors, as for the workplace, intended to maximize
productivity by minimizing operator fatigue and discomfort.

. 8 ‘Entasis’ is a visual correction for end columns by providing


(A) a slight convexity to the columns (B) a slight concavity to the columns
(C) a major convexity to the columns (D) a major concavity to the columns
Answer: (A) a slight convexity to the columns.
Entasis is the application of a convex curve to a surface for optical correction. The
Classical columns were slightly bulged out a bit above the base.
. 9 The first group of people to influence the architecture of South-east Asia and the
Amravati School of Art was
(A) Sakas and Pallavas (B) Satavahanas and Pandayans
(C) Pallavas and Guptas (D) Rashtrakutas and Chalukyans
Answer: (B) Satavahanas and Pandayans

. 10 A linear regression model involving one independent and one dependent variable
requires at least
(A) one pair of data (B) two pairs of data (C) three pair of data (D) four pair of data
Answer: (C) three pair of data

. 11 Identify the FALSE statement


(A) Susceptibility to non structural elements, damage in any building would be high
even in moderate level earthquake
(B) For important non structural elements, no structural analysis is required to access
vulnerability.
(C) Earthquake damage to non structural elements results in loss of critical functions.
(D) The non- structural elements can be retrofitted appropriately.
Answer: (B) For important non structural elements, no structural analysis is required to
access vulnerability

. 12 Under which category the percentage of landuse decreases with an increase in city size
(A) Residential (B) Commercial (C) Recreational (D) Transportation and
Communication
Answer: (A) Residential

. 13 The instrument that provides standards for land development by indicating lot sizes and
layouts is
(A) Zoning Regulation (B) Landuse Control
(C) Building Bylaws (D) Sub division Regulations
Answer: (D) Sub division Regulations

. 14 Identify the group containing only GIS packages


P. Total Station

Q. Sat Guide
R. GPS
S. ILWIS
T. Corel Draw
U. Geo Media
V. ArcInfo
(A) P, Q, U (B) Q, R, V (C) S, U, V (D) R, T, V
Answer: (B) Q, R, V
(Input from Sangamjeet, GNDU, Amritsar)

. 15 Organizations namely STACO, UNSCC and ISO are associated with


A) Environmental Planning (B) Landscape Architecture
(C) Modular Coordination (D) Urban Design
nswer: (C) Modular Coordination
he ISO (International Organization of Standardization) connects 157 national standards
institutes from around the world — each member country being represented by one
institute. The ISO is managed by a Central Secretariat, headquartered in Geneva,
Switzerland. It is a non-government agency, although many of the participating
institutions are government agencies, and others are private

Q. 16 ‘Inflorescence’ in a tree- structure refers to


(A) Flowering Character (B) Fragrance of the flowers
(C) Spread characteristics of the branches(D) Depth of root structure
Answer: (A) Flowering Character
rown- Crown spread is a measure of the footprint or plan area of the crown (spread
characteristics of Branch) of the tree expressed as a diameter.
anopy- The uppermost layer in a forest, formed by the crowns of the trees. Also called crown
canopy.
irth is a measurement of the distance around the trunk of a tree measured perpendicular to
the axis of the trunk.

. 17 Income inequalities across population is expressed through


(A) Cohort Pyramid (B) Lorenz Curve (C) Indifference Curve (D) Inverted U-Curve
Answer: (B) Lorenz Curve
Cohort Pyramid- graph showing population variation (gender) for particular age group.

. 18 The Columbian Exposition in North America is synonymous with


(A) City Beautiful Movement (B) Urbana Lake front Development
(C) CIAM (D) Broad-acre city
Answer: (A) City Beautiful Movement
. 19 The ideal cross section of a combined sewerage system for significant variation in flow
is
(A) Circular (B) Egg shaped (C) Semi elliptical (D) Horse shoe shaped
Answer: (B) Egg shaped

. 20 The international guideline for conservation and restoration of monuments and sites
recommended by ICOMOS, is known as
(A) Venice Charter (B)Amsterdam Charter (C) Granada Convention (D) Burra
Charter
Answer: (A) Venice Charter
ICOMOS- International Council on Monuments and Sites. (Headquartered: Paris)
Athens Charter – Restoration of Historic Monuments (1931) – manifesto with 7 points.
Venice Charter- Conservation and restoration of monuments and sites. Total 16 articles.
(1964)
Burra Charter: conservation and restoration of monuments and sites in Australia.
Florence Charter- Historic Gardens.(1982)
Granada Convention: Convention for the Protection of the Architectural Heritage of
Europe.(1985)
History of Architectural Conservation: Architectural Conservation started during the period
of Modernism when sentimental attachment for old buildings grew due to rapid
advancement in building construction system.
John Ruskin’s (1819-1900, British writer) ideas on, preservation of open spaces and
conservation of historic buildings. The Society for the Protection of Ancient Buildings
(SPAB) was founded by William Morris (textile designer), Philip Webb and others, in
1877, to oppose what they saw as destructive ‘Restoration’ of ancient buildings then
occurring in Victorian England.
Two School of Thoughts
Preservation/Conservation were used interchangeably to refer to the
architectural school of thought that either encouraged measures that would
protect and maintain buildings in their current state, or would prevent further
damage and deterioration to them. Main proponent’s art critic John Ruskin and
artist William Morris.
Restoration was the conservationist school of thought that believed historic
buildings could be improved, and sometimes even completed, using current
day materials, design, and techniques. Main proponent French architect
Eugène Viollet-le-Duc.
Archaeological Survey of India is an Indian government agency in the Department of
Culture that is responsible for archaeological studies and the
preservation of cultural monuments. ASI’s function is to “explore, excavate, conserve,
preserve and protect the monuments and sites of National & International Importance.”
The Archaeological Survey of India (ASI) is the successor of the Asiatic Society of
British archaeologist founded in January 15, 1784 in Bengal. ASI was founded in 1861.
After independence an Indian act was passed known as ‘Ancient Monuments and
Archaeological Sites and Remains Act of 1958’.

.21- Q.75 carry two marks each


. 21 Heating, cooling and ventilation in passive systems designs are dependent on
(A) differences in standards of active energy systems and amount of light.
(B) quality of insulation and quality of glazing.
(C) mechanical ventilation and floor height o the building.
(D) daylight factor and energy from mechanical systems
Answer: (B) quality of insulation and quality of glazing

. 22 Which pair, out of the following options, is used in more than one computer languages
listed below
(A) ; /n (B) , ? (C) ? /n (D) , ;
Answer: No appropriate answer.

. 23 Match labels in the diagram with items in the table :

Activity
Dummy Activity
Duration
Event starting/ finishing
(A) P-1, Q-2, R-4, S-3 (B) P-4, Q-1, R-3, S-2 (C) P-3, Q-2, R-1, S-4 (D) P-1, Q-3, R-4,
S-2
Answer: (D) P-1, Q-3, R-4, S-2

. 24 Select the valid combination of shear force and bending moment diagrams fro the
loading shown below

(A) P-3 (B) Q-2 (C) R-1 (D) S-4


Answer: (A) P-3

. 25 Recommended temperature and fresh air flow for HVAC systems in office building in
India are
(A) 21º C with maximum of 30º C in summer and 25 º C in winter, with fresh air provision
of 18-22 litres per second per person.
(B) 29º C with maximum of 32º C in summer and 36 º C in winter, with fresh air provision
of 28-32 litres per second per person.
(C) 30º C with maximum of 36º C in summer and 32 º C in winter, with fresh air provision
of 38-42 litres per second per person.
(D) 21º C with maximum of 24º C in summer and 22 º C in winter, with fresh air provision
of 8-12 litres per second per person.
Answer: (D) 21º C with maximum of 24º C in summer and 22 º C in winter, with fresh air
provision of 8-12 litres per second per person.

. 26 Match the architects/ city planners from Group I with the design movements listed in
Group II
GROUP I GROUP II
P. Violett- le- duc 1. Post Modernism
Q. William Morris 2. Arts and Crafts movement
R. Robert Venturi 3. Ekistics
S. C A Doxiadis 4. French Rationalism
(A) P-4, Q-2, R-1, S-3 (B) P-3, Q-1, R-4, S-2 (C) P-2, Q-3, R-1, S-4 (D) P-1, Q-4, R-
2, S-3
Answer: (A) P-4, Q-2, R-1, S-3
Violett- le- duc – He was the French architect hired to design the internal structure
of the Statue of Liberty.
William Morris- Arts and Crafts was an international design movement that
flourished between 1860 and 1910 It was led by the artist and writer William
Morris (1834–1896) during the 1860s, and was inspired by the writings of John
Ruskin (1819–1900) and Augustus Pugin (1812–1852)
C A Doxiadis famous for – Ekistics, Dynapolis and design of Islamabad.
Robert Venturi- Pritzker prize winner of 1991. (Less is bore)
. 27 Structural adjustments between two regions with respect to supply and demand of
labourers and their wages is explained by
(A) Input- Output Analyses by W. Leontiff.
(B) Export Base Model by Douglas C North.
(C) Backwash effect based Economic Growth Model by Gunner Myrdal.
(D) Economic Base theory by Hans Blumenfield.
Answer: (C) Backwash effect based Economic Growth Model by Gunner Myrdal.
This Question is of Urban and Regional Economics.
Input–Output Analysis is a quantitative economic model that represents the
interdependencies between different industries or sectors of a national
economy.
Spread effect and Backwash effect- Gunner Myrdal

Spread Effect Backwash Effect


Spread refers to the situation Backwash occurs if the adverse
where the positive impacts on effects dominate and the level of
nearby localities and labor economic activity in the
markets exceed the adverse peripheral communities declines.
impacts.
Export Base Model: Growth of a region depends upon the growth of its export
industries and that expansion in demand external to the region is the main
determinant of regional growth.
The theory of economic base is one of the standard and the most well-
known theories of regional development, or theories trying to explain why
different regions have different economic levels. The basic idea, on which the
theory is based, is that the basic income of the region is export.

. 28 Match the surfaces in Group- I with the respective range of albedo values in Group- II

GROUP I GROUP II
P. Close ground crops 1. 0.45- 0.95
Q. Bare lands 2. 0.05- 0.055
R. Water surface 3. 0.05- 0.45
S. Snow 4. 0.15- 0.25
(A) P-3, Q-4, R-2, S-1 (B) P-4, Q-3, R-2, S-1 (C) P-4, Q-3, R-1, S-2 (D) P-4, Q-2, R-
3, S-1
Answer: (A) P-3, Q-4, R-2, S-1

. 29 Which of the following is NOT a criterion for defining urban area in India?
(A) Population size.
(B) Percentage of male working population engaged in non-agricultural pursuits.
(C) Density of Population.
(D) Percentage of pucca houses.
Answer: (D) Percentage of pucca houses.
Criterion for defining urban area in India.
Population size of 5000 and above.
Percentage of male working population engaged in non-agricultural pursuits
should be more than 75 %
Density of Population should be more than 400 persons/ sq. km
Should have a Municipal Corporation, Municipality, Nagar Panchayat or
Cantonment board or should be a notified town.
There are about 8000 urban settlements and about 5,00,000 village settlements in
India.
According to 2011 census the level of Urbanization in India is 31.16 %
. 30 Signal phasing in transportation system refers to
(A) the number of combinations of traffic movements served through a signalized
intersection.
(B) the distance between signalized intersections.
(C) phase of electric power required to make the signals operational.
(D) relative placements of red, green and amber lights on a signal post.
Answer: (A) the number of combinations of traffic movements served through a
signalized intersection

. 31 Pair the groups correctly


GROUP I GROUP II
P. Solar Constants 1. W/m deg C
Q. Air to air transmittance, U- value 2. 1.4kW/m²
R. Volumetric specific heat 3. W/m² deg C
S. Conductivity, K -value 4. Kcal/m³ deg C
(A) P-2, Q-3, R-4, S-1 (B) P-2, Q-1, R-4, S-3 (C) P-1, Q-2, R-3, S-4 (D) P-4, Q-3,
R-1, S-2
Answer: (A) P-2, Q-3, R-4, S-1

. 32 Identify the right network representing the following statement,


‘S controls X, Y and Z; T controls Y & Z and U controls Y’.

Answer: B

. 33 Architectural projects designed by Laurie Baker are generally characterized by


P. Appropriate technology
Q. Human scale
R. Interpretation of Nine- square model
S. Use of locally available material
(A) P, R, S (B) P, Q, S (C) Q, R, S (D) P, Q, R
Answer: (B) P, Q, S
Definition of appropriate technology: It is defined as ‘technology that is scientifically
sound, adaptable to local needs, and acceptable to those who apply it and those for
whom it is used, and that can be maintained by people themselves in keeping with the
principle of self reliance with the resources like the community and country can afford.’
Nine fold plan: adopted in design of Taj Mahal.
. 34 Match the glasses listed in Group I, with the appropriate descriptions in Group II
GROUP I GROUP II
P. Liquid crystal laminated glass 1. Promotes absorption of both visible light and infra
red radiation
Q. Electro-chromic glass 2. Improves thermal performance of the glass by
reflecting
visible light and
infra red radiation
R. Coated glass 3. Requires continuous supply of electricity to change from

translucent to transparent state


S. Tinted glass 4. Requires electrical pulses to change from transparent to

opaque state
(A) P-4, Q-1, R-3, S-2 (B) P-1, Q-3, R-2, S-4
(C) P-3, Q-4, R-2, S-1 (D) P-2, Q-3, R-4, S-1
Answer: (C) P-3, Q-4, R-2, S-1

. 35 Which of the following commands in AutoCad is used to extract one or more elements
from a list?
(A) Filter (B) Boundary (C) Explode (D) Eattext
Answer: (D) Eattext

. 36 Identify the satellite that provides useful information for physical planning.
P. IKONOS Q. IRS- 1D R. CartoSAT S. INSAT 1B
T. PSLV U. Google Earth V. Apple W. Quick Bird
(A) P,Q,R,W (B) R,S,T,V (C) P,Q,R,S (D) S,T,U,V
Answer: (A) P,Q,R,W

. 37 Stack effect is
(A) the process of supplying fresh air by electro mechanical means both vertically and
horizontally.
(B) the tendency of hot air in a shaft to rise and create a draft of cool air intake.
(C) the air- supply to a motor- driven louvered opening in basement .
(D) the circulation of fresh air through windows from the plenum level.
Answer: (B) the tendency of hot air in a shaft to rise and create a draft of cool air intake.

. 38 Match the equipments with their use


GROUP I GROUP II
P. Power shovel 1. Spreading
Q. Front end loader 2. Drilling
R. Drop hammer 3. Excavation
S. Earth-auger 4. Piling
(A) P-1, Q-3, R-2, S-4 (B) P-2, Q-3, R-4, S-1 (C) P-4, Q-2, R-1, S-3 (D) P-3, Q-1, R-
4, S-2
Answer: (D) P-3, Q-1, R-4, S-2

. 39 ‘Contemporary Architecture has made a shift from machine-based modernist approach


to passive energy- sensitive approach.’
Which of the following group of architect’s best represent this shift?
(A) Paul Rudolph, Mies van der Rohe, Arato Isozaki
(B) Norman Foster, James Carpenter, Richard Rogers
(C) James Sterling, Philip Johnson, Ralph Rapson
(D) Arthus Erikson, Frei Otto, Rem Koolhas
Answer: (B) Norman Foster, James Carpenter, Richard Rogers
. 40 ‘Park le de Villete’, Paris designed by Bernard Tschumi, is characterized by
(A) continuous sequence along a zig-zag lines.
(B) beast like benches embedded with fragments of coloured tiles and stepped terraces.
(C) point grid, super imposition and agglomeration of activities.
(D) semi underground cave like galleries for the display of art work.
Answer: (C) point grid, super imposition and agglomeration of activities.

. 41 The predominant characteristics of spatial organizational principles found in the works of


Le Corbusier and Frank Lloyd Wright are characterized respectively by
(A) Grid organization and Linear planar organization.
(B) Centralized clustered organization and Grid organization.
(C) Radial organization and Grid-radial organization.
(D) Centralized organization and multi Grid organization.
Answer: (A) Grid organization and Linear planar organization.

S. Type of Explanation Examples


No Organization
1 Centralized A Central Ideal Church- Da Vinci
Dominant San Lorenzo Maggiore, Milan
space about Italy
which a S. Ivo della Sapienza, Rome,
number of Francesco Borromini
secondary National Assembly Building,
spaces are Dhaka, Lou I Kahn
grouped Villa Rotunda, Venice, Andrea
Palladio
Pantheon, Rome
Hagia Sophia, Istanbul, Turkey
2 Linear A linear Unite’d’ Habitation, Marseilles,
sequence of Le Corbusier
repetitive Pearson House, Robert Venturi
space Lloyd Lewis House, F L Wright
Baker House, MIT, Alvar Alto
3 Radial A Central Secretariat Building, UNESCO
space from Head quarter, Paris, Marcel
which Breuer
organization Herbert F Johnson House, F L
of space Wright
extend in a Kaufman Desert House,
radial manner California, Richard Neutra
4 Clustered Spaces Fateh pur Sikri
grouped by Meeting House, Salk Institute
proximity or of Biological Studies, California,
the sharing of Louis Kahn
a common Friedman House, F L Wright
visual trait or Kauffman House, Falling Water,
relationship Pennsylvenia, F L Wright
5 Grid Space Shodhan House, Le Corbusier
organized Kimble Art Museum, Louis
within the field Kahn
of a structural Snyderman House, Michael
grid or other Graves
three Adler House, Philadelphia,
Louis Kahn
dimensional
framework
(Source: Architcture Form, Space and Order, Francis D. K. Chiang, Chapter 4, pp 178-220)

. 42 The ratios represented by the two number series 70:113:183 and 86:140:226 stand
respectively for
(A) the blue and red series of Le Modular
(B) the vertical and horizontal proportions found in Leonardo da Vinci’s pentagram
(C) the horizontal and vertical proportions found in Leonardo da Vinci’s pentagram
(D) the red and blue series of Le Modular
Answer: (D) the red and blue series of Le Modular
(Ref: page 318, “Architecture: Form, Space and Order”, Francis D. K. Ching, John Wiley
and Sons, 3rd edition)

. 43 The difference between an axonometric projection and an isometric projection of an


object with respect to a picture plane is in terms of
(A) height or breadth of cross-sectional views generated by the picture plane.
(B) measurements in the angle of faces with respect to the aspect ratio.
(C) obliqueness in the projection of faces of the object on the vertical plane.
(D) foreshortened angular measurements in the three principle axis.
Answer: (D) foreshortened angular measurements in the three principle axis.

. 44 A squinch system is a method of constructing an arch across a square base by erecting


(A) Pendentives and Cul-de-four (B) Intra-domes and tension ring.
(C) Saucer domes and transverse vaults (D) Cross bandages and hoop lines
Answer: (A) Pendentives and Cul-de-four
Cul-de-four: A half-dome or quarter-sphere vault, as over an apse or niche. In
architecture, the apse is a semicircular recess covered with a hemispherical
vault or semi-dome.

. 45 Match the following


P. Nile valley civilization 1. Shang
Q. Indus valley civilization 2. Harappa
R. Euphrates and Tigris valley civilization 3. Akhetaton
S. Yellow River Civilization 4. Babylon
(A) P-1, Q-2, R-3, S-4 (B) P-3, Q-2, R-4, S-1 (C) P-4, Q-3, R-2, S-1 (D) P-4, Q-2, R-3,
S-1
Answer: (B) P-3, Q-2, R-4, S-1
Nile Valley cities:
East of Nile- Akhetaton, Tel-el Amarna, Karnak, Thebes, Dendera, Syene, Memphis,
Heliopolis, Pi Hapy.
West of Nile – Elephantine, Edfu, Abydos, Medum, Saqqara, Giza.
Port City- Alexandria (famous for library at Alexandria) designed by Dinocrates and city of
Alexander.
Kahun- slaves village close to Pyramid site. (abandoned after completion of Pyramid)
Indus Valley cities: Mohenjo-daro, Harappa, Lothal, Dholavira, Mehrgarh, Kalibagan,
Rakhigarhi.
Lothal was a port city (now in Gujarat)
Asserian cities: Babylon

Chinese Civilization: Shang

. 46 Match the appropriate arches with types listed below


1. Equilateral Arch 2. Lancet Arch 3. Drop Arch
4. Surbased Arch 5. Tudor Arch 6. Roman Arch
(A) P-1, Q-2, R-5, S-3 (B) P-1, Q-2, R-3, S-5
(C) P-1, Q-2, R-3, S-4 (D) P-6, Q-2, R-5, S-3
Answer: (C) P-1, Q-2, R-3, S-4
(Input from Satarupa Roy, Priyadarshini Institute of Arch. And Design Studies. Nagpur)

Q. 47 The study of varying population size of urban centres in a


region is assessed by
(A) Multiplier effect (B) Rank size rule
(C) Shift-share analysis(D) Bulk share of workforce
Answer: (B) Rank size rule
According to rank size rule a rank 2 city would have half the population of a country’s
largest city, a rank 3 city would have one-third the population of a country’s largest city,
a rank 4 city would have one-fourth the population of the largest city.

. 48 Identify the correct hierarchy of traditional Indian Settlements expressed in an ascending


order.
(A) Kharvata- Khetaka- Nagara- Durg (B) Durg- Vidambaka- Pura- Rajdhani
(C) Grama- Khetaka- Kharvata- Nagara (D) Nigam- Agrahara- Pura- Kharvata
Answer: (C) Grama- Khetaka- Kharvata- Nagara
Hiearchy of ancient settlement.
Griha-house
Grama-village
Palli- settlement of wild tribes
kheta- a place fortified by a mud wall or by a river or hill surrounding it,
khetaka- hamlet
kharvata- market town, a poor town surrounded by a low wall, while according
to kautilya-centre of union of 200 villages
dronamukha- end of the valley ...centre of union of 400 villages,
durg- citadel.
pattana,pura- a large town or centre of trade/a mart for precious metals or a
mining centre.
matamba- open town associated with a cluster of 10,000 villages/fortified place
in which produce from the field was deposited for safe custody.
nagara- a town exempted from paying any of the 18 taxes levied on a village.
rajadhani-capital.
nigama-settlement of merchants
samvatta-kotta- a fortified place or refuge

. 49 Formal regions and Functional regions are determined respectively by their


(A) ‘Natural resources; physiography’ and ‘Economic Linkages.’
(B) ‘Economic Linkages’ and ‘Natural resources; ‘physiography.’
(C) ‘Industrial Location’ and ‘Transportation; communication.’
(D) ‘Transportation; ‘communication’ and ‘Industrial Location.’
Answer: (A) ‘Natural resources; Physiography’ and ‘Economic Linkages.’
This Question is of Regional Planning. In this Question keep note of the word
‘respectively’. Answer ‘A’ signifies that Formal Region is determined by Natural
Resources and physiography while Functional regions are determined by Economics.

Regions
Formal Region Functional Region Planning Region
It is an area It is a product of A planning region is a
designated by official modern economy, segment of territory over
boundaries. industrial which economic
A formal region is development and decisions apply. Planning
created on the basis urbanization. It is regions are priorities
of at least one or based on the area delineated by
more of the following principle of scholars/ organizations
characteristics: interdependence to start the development
Common and interlinking. based on certain
language These regions show parameters.
Religion flows related to First phase had industrial
Nationality people, commodity, (economic)planning
Political affiliation capital etc. that bind regions, focused on
Culture the various sub sectoral industrial
Geographical parts of a region in a growth, these were
features (climate, coherent whole. mineral and capital
vegetation, For delineation of industry based regions.
elevation, functional regions • Second phase focused
landform, etc.) Quantitative on equitable
methods are used, development. Planning
such as: regions were special
Zipf’s-principle of problem areas (DPAP,
‘distance decay’ Hill Areas, TribalAreas,
Stewart Wastelands etc.
‘s–‘Gravity Model’
Chistaller’s and
Losche’s-‘Central
Place’ theory.
Thiessen Polygon
method
L.S BHAT AND V.L.S.
PRAKASARAO
REGIONS
NATH REGIONS
SEN GUPTA REGIONS
C.S.CHANDRASEKHA
R REGIONS
TOWN AND
COUNTRY PLANNING
ORGANISATION
REGIONS
Examples of Planning Region in India as proposed by various Regional Planners

L.S BHAT AND V.L.S. PRAKASARAO


REGIONS
Bhat and Rao (1964) proposed a regional framework for resource development.
Delineation was done with the help of qualitative maps of distribution of important natural
resources. The major regions cut across the state boundaries. However, administrative
convenience was not ignored. The scheme included 7 major and 51minor regions. Seven
major regions include:
(1) South India
(2) Western India
(3) Eastern Central India
(4) North-Eastern India
(5) Middle Ganga Plain
(6) North-Western India, and
(7) Northern India.

NATH REGIONS
Nath (1965) prepared a scheme of Resource Development Regions and Division of
India based at the homogeneity in physical factors, and agricultural land use and cropping
pattern. Although the regions cut across the state boundaries, the division is kept within
the state limit. Thus the entire country has been divided into 15 main and 48 sub regions.
These major resource development regions include:
(1) Western Himalaya,
(2) Eastern Himalaya
(3) Lower Ganga Plain,
(4) Middle Ganga Plain
(5) L Upper Ganga Plain
(6) Trans- Ganga Plain
(7) Eastern Plateaus and Hills
(8) Central Plateaus and; I Hills
(9) Western Plateaus and Hills
(10) Southern Plateaus and Hills
(11) Eastern Coastal Plains and Hills,
(12) Western Coastal Plains and Ghats,
(13) Gujarat Plains and Hills
(14) Western Arid Region, and
(15) Island Region.

SEN GUPTA REGIONS


Following the Soviet concept of economic regions and production specialisation, P.Sen
Gupta (1968) presented a framework of economic regions of different order. She started
with the discovery of planning units of the lowest order and then grouped and regrouped
them to achieve planning regions at meso and macro levels. In her scheme of economic
regions, Sen Gupta gave much importance to natural regions and used modality,
production specialization and utilization of power resources as bases of delineation. Her
7macro regions are further divided into 42meso regions. These 7 regions include :
(1) North Eastern Region
(2) Eastern Region
(3) Northern Central Region
(4) Central Region
(5) North-West-ern Region
(6) Western Region, and
(7) Southern Region

C.S. CHANDRASEKHAR REGIONS


C.S. Chandrasekhar proposed a scheme of planning regions . He divided India into 13
micro and 35 meso planning regions. He used the criteria of physical economic and
ecological factors to demarcate the macro planning regions. These regions include:
(1) South peninsular region
(2) Central peninsular region
(3) Western peninsular region
(4) Eastern peninsular region
(5) Central Deccan region,
(6) Gujrat region
(7) Western Rajasthan region
(8) Aravali region
(9) Jammu & Kashmir and the Ladakh region
(10) Trans Gangetic region & the hill regions
(11) Ganga- Yamuna plain region
(12) The lower Ganga plain region,
(13) North-Eastern region ,

TOWN AND COUNTRY PLANNING ORGANISATION REGIONS


In 1968, the Town and Country Planning Organisation suggested a scheme of planning
regions delineated on the principle of economic viability, self-sufficiency and ecological
balance at the macro and meso levels. The emphasis of the scheme was to introduce
regional factor in economic development. This approach would complement the macro
planning at the national level, with a component of regional policies, aimed at reducing
regional disparities in the development. The macro- regionalization sought to link a set of
areas, rich in one type of resources with areas having complementary resources or even
resource poor areas, so that the benefits of economic activity in the former may flow into the
latter. These planning regions cut across the State boundaries, but do not completely ignore
the basic administrative units. The 13 macro- regions proposed under the scheme include:
(1)South Peninsular (Kerala and Tamil Nadu)
(2)Central Peninsular (Karnataka, Goa,Andhra Pradesh
(3) Western Peninsular (western Maharashtra coastal and interior districts)
(4) Central Deccan (eastern Maharashtra,central and southern Madhya Pradesh)
(5) Eastern Peninsular (Orissa, Jharkhand north-eastern Andhra Pradesh and
Chattisgarh
(6) Gujarat
(7) Western Rajasthan
(8) Aravalli Region (Eastern Rajasthan and western Madhya Pradesh),
(9) Jammu, Kashmir and Laddkh
(10) Trans Indo-Genetic Plains and Hills (Haryana, Himachal Pradesh, West Uttar
Pradesh and Uttaranchal)
(11) Ganga-Yamuna Plains (central and eastern Uttar Pradesh, and northern Madhya
Pradesh)
(12) Lower Ganga Plains (Bihar and West Bengal Plains)
(13) North-Eastern Region (Assam and north-eastern states including Sikkim and north
Bengal).

. 50 Nagar Panchayats and District Planning in India were introduced as a result of


(A) National Urbanization Policy (B) Jawahar Lal Nehru National Urbanization Mission
(C) Electoral Reforms (D) Constitution (73rd and 74th Amendment) Acts

Answer: (D) Constitution (73rd and 74th


Amendment) Acts

. 51 The concept of ‘Slum- networking’ aims to promote


(A) social and physical improvement of Slums.
(B) holistic development in conformity with the infrastructure of the entire city.
(C) improvement of physical networks within the slum areas.
(D) rehabilitation of slum dwellers.
Answer: (A) social and physical improvement of Slums.

. 52 Shells and Space Frames are examples of


(A) modular Bulk- active and Form-active systems respectively.
(B) modular Surface- active and Vector-active systems respectively.
(C) modular Vector- active and Form-active systems respectively.
(D) modular Bulk- active and Surface-active systems respectively.
Answer: (B) modular Surface- active and Vector-active systems respectively.
Classification of Structural Systems
Section Vector active Surface Form active
active or active
Bulk
active
They Individual They are in These structural
transfer members are surface systems are
loads subject to only stress exposed to single
mainly to coactive condition stress condition.
through stresses (compression (compression or
Bending (compression or tension) tension)
Moments. and tension)
eg Beam, Flat trusses, Plate Cable structures,
Rigid Curved trusses, structures, Tent structures,
frame, Transmitted flat folded plate, Pneumatic
beam trusses, space shell structures, Arch
grid, slab trusses structures structures
structure

. 53 The Law of Vicinity states that


(A) the objects of similar form situated close enough together are perceived as one.
(B) the objects of similar form situated at a distance are perceived as one.
(C) the objects of different forms situated close enough together are perceived as one.
(D) the objects situated close together enough are perceived as confusing.
Answer: (B) the objects of similar form situated at a distance are perceived as one.

. 54 Luminaire efficiency is defined as the


(A) sum of the light output of the lamps operating inside the luminaire to the ratio of the
sum of the light output of the luminaire operating outside the luminaire.
(B) sum of the individual lights outputs of the lamps operating outside the luminaire to
the ratio of the of the light output of the luminaire.
(C) ratio of the light output the luminaire sum of the individual light outputs of the lamps
operating outside the luminaire.
(D) ratio of the light output of the luminaire to the individual light output of the lamp
operting outside the luminaire.
Answer: (B) sum of the individual lights outputs of the lamps operating outside the
luminaire to the ratio of the of the light output of the luminaire.
The ratio of the luminous flux of the luminaire and the luminous flux of the light
source defines the luminaire efficiency.
A luminaire is a complete set of light fixture including light source, light holder,
reflector, casing etc.

Q. 55 Match the following


GROUP I GROUP II
P. Dumbwaiter 1. Opening
Q. Comb-plate 2. Escalator
R. Co-axial cable 3. Elevator
S. Transom 4. Data Signal
(A) P-1, Q-2, R-3, S-4 (B) P-3, Q-2, R-4, S-1 (C) P-2, Q-1, R-4, S-3 (D) P-4, Q-
3, R-1, S-2
Answer: (B) P-3, Q-2, R-4, S-1
Transom is opening above door or window.Comb-plate is riser and tread of
accelerator.
Dumbwaiter- it is a small service lift for carrying service objects but not
passenger.
Q. 56 Negative and positive correlations between Price and Quantity of a
commodity are respectively represented by
(A) Demand and supply curves.
(B) Supply and demand curves.
(C) Indifference curves and scattered matrix.
(D) Scattered matrix and indifference curves.
Answer: (A) Demand and supply curves.
Demand shows a negative correlation between Price and quantity.

. 57 Traditional Indian settlement patterns, based on orthogonal grid are represented by:
(A) Padmaka, Kurmaka and Swastika
(B) Mandala, Kurmaka and Angula
(C) Dandaka, Vidambaka, and Dhanurmusti
(D) Sarvatabhadra, Prastara and Chaturmukha
Answer: (D) Sarvatabhadra, Prastara and Chaturmukha
Padmaka and kurmaka are circular in form.
Angula and Dhanurmusti are measurements.

. 58 Plans of Mohenjodaro and medieval Jaipur are based on


(A) grid pattern and sectoral allocation of zoning.
(B) radial pattern and grid allocation of zoning
(C) clustered pattern and segregated allocation of zoning.
(D) centralized pattern and composite allocation of zoning.
Answer: (A) grid pattern and sectoral allocation of zoning.
(adjacent fig. is plan of Jaipur old city)

Q. 59 Match the following


GROUP I GROUP II
P. Frank Gehry 1. Pyramide du Louvre
Q. Norman Foster 2. Bilbao Guggenheim Museum
R. I M Pei 3. HongKong and Sanghai Bank
S. James Sterling 4. Neu Staatsgalerie
(A) P-1, Q-3, R-4, S-2 (B) P-2, Q-3, R-1, S-4 (C) P-2, Q-1, R-3, S-4 (D) P-3, Q-
2, R-4, S-1
Answer: (B) P-2, Q-3, R-1, S-4

. 60 The most appropriate criteria to be considered for delineating backward regions are
(A) density of population (B) amount of sales tax collection
(C) infant mortality (D) per capita income and its distribution
Answer: (D) per capita income and its distribution

. 61 The relationship between headway (h) and flow (q) in a traffic stream is represented by:
(A) h= q² (B) h= q (C) h= 1/q² (D) h= 1/q
Answer: (D) h= 1/q
Flow (q) = the rate at which vehicles pass a fixed point (vehicles per hour)
Space headway (s) = difference in position between the front of a vehicle and
the front of the next vehicle (in meters)
Time headway (ht) = difference between the time when the front of a vehicle
arrives at a point on the highway and the time the front of the next vehicle
arrives at the same point (in seconds)
Q. 62 Match the diagram of Age Sex
pyramids from the descriptions of the population growth, given below:
1. Rapid growth 2. Slow growth 3. Zero growth 4.Negative growth
(A) P-1, Q-2, R-3, S-4
(B) P-4, Q-3, R-2, S-1
(C) P-3, Q-2, R-4, S-1
(D) P-1, Q-2, R-4, S-3
Answer: (B) P-4, Q-3, R-2, S-1

Q. 63 ‘Ecological Footprints’ corresponds to


(A) the land area required to preserve as forests to ensure sufficient levels of oxygen for
a community.
(B) the land area necessary to supply natural resources to a community and disposal of
its wastes.
(C) the land area required to take care of solid wastes and sewage of a community.
(D) the land area per person per year, from which forests are cut.
Answer: (B) the land area necessary to supply natural resources to a community and
disposal of its wastes

Q. 64 Match the following with their area of application


P. Potometer 1. Area measurement
Q. Histogram 2. Soil moisture measurement
R. Electrostatic precipitator 3. Transpiration
S. Planimeter 4. Suspended particles
T. Potentiometer 5. Statistics
(A) P-1, Q-1, R-2, S-3, T-1 (B) P-4, Q-5, R-5, S-4, T-1
(C) P-3, Q-5, R-4, S-1, T-2 (D) P-2, Q-3, R-2, S-5, T-5
Answer: (C) P-3, Q-5, R-4, S-1, T-2

Q. 65 Select the appropriate word from the list given below that fits in all the blank
1. The aim of conservation is to retain or recover the _________ significance of a
place.
2. Preservation is appropriate where the existing state of the fabric itself constitutes
of specific ______________ significance.
3. Restoration is appropriate only if there is sufficient evidence of an earlier state of
the fabric and only if returning the fabric to that state recovers the _________
significance of the place.
4. Reconstruction is appropriate where a place is incomplete through damage or
alteration and where it is necessary for its survival, or where it recovers the
__________ significance of the place as a whole.
(A) historical (B) cultural (C) architectural (D) aesthetic
Answer: No appropriate answer.

Q. 66 The rule for generating Fibonacci series is


(A) Fi = Fi-1 +2 for i>1 given Fi and F0 (B) Fi = Fi-1 +1 for i>1 given Fi and F0

(C) Fi = Fi-1 +Fi-2 for i>1 given Fi and F0


(D) Fi = (Fi-1)2 for i>1 given Fi and F
Answer: (C) Fi = Fi-1 +Fi-2 for i>1 given Fi and F0
Fibonacci sequence are the numbers in the following integer sequence:
0, 1, 1, 2, 3, 5, 8, 13, 21, 34, 55, 89, 144………….
The first two numbers in the Fibonacci sequence are 0 and 1, and each subsequent
number is the sum of the previous two.

= Golden Ratio when n is sufficiently large.

Q. 67 Two names associated with planning of Paris and Philadelphia are respectively:
(A) Geroges- Eugene Hausmann and William Penn (B) Patrick Geddess and Louis
Wirth
(C) Albert Perry and Oswald Spangler (D) Le Corbusier and John Friedman
Answer: (A) Geroges- Eugene Hausmann and William Penn

Q. 68 Which of the following statements is valid for a saddled surface shell structure?
P. regions of downward curvature exhibit arch like action
R. regions of upward curvature behave as a cable structure
(A) P is true and R is false (B) R is true and P is false
(C) Both P and R are true (D) Both P and R are false
Answer: (C) Both P and R are true
A tensile structure is a construction of elements carrying only tension and no
compression or bending.
Tensegrity, which is a structural form with both tension and compression elements

Q. 69 Which of the following statements describe the advantage of A.C. supply over D.C.
supply?
(A) Electroplating process (B) Noise reduction in motors
(C) Facility of transforming from one voltage to another (D) Charging of storage batteries
Answer: (C) Facility of transforming from one voltage to another.

Q. 70 For which application software the following expression is valid?


(* 2.5 (+ (/ a 2) (-5x)))
(A) Qbasic (B) AutoLISP (C) Java (D) C++
Answer:

Common Data Questions. Common data for questions 71, 72 and 73:
For a building, the gross rent fetched is Rs 22,500/- per month; municipal tax is Rs
8,000/- per quarter; repair and maintenance charges are @ 10% of gross rent and
other expenses borne by the owner are Rs 16,000/- per annum.
. 71 What would be the total outgoings in Rs?
(A) 60,000/- (B) 70,000/- (C) 75,000/- (D) 80,000/-
Answer: (C) 75,000/-
Annual Municipal Tax= 8000

Repair and Maintenance (10 % of Gross Rent) = 22500


Other annual expenses= 16000
Total Expenses= 32000 + 27000 + 16000 = 75,000/-

. 72 What would be the annual rent in Rs?


(A) 1, 90,000/- (B) 1, 95,000/- (C) 2, 00,000/- (D) 2, 05,000/-
Answer: (B) 1, 95,000/-
Annual rent= (22,500 ×12) – 75000=

. 73 If the years purchase in perpetuity comes out to be 12.5, what would be the capitalized
value, in Rs of the above building?
(A) 24, 00,000/- (B) 24, 37,000/- (C) 24, 37,500/- (D) 25, 00,000/-
Answer: (C) 24, 37,500/-
Capitalized value= 1,95,000 × 12.5= 24, 37,500/-

Common data for Questions 74 and 75:


The following table provides total population and urban population of India in various
years.
Year Total Urban
Population population
(millions) (millions)
1901 238.40 25.85
1911 252.09 25.94
1921 251.32 28.09
1931 278.98 33.46 Q. 74 Level of Urbanization in the year 1921 was:
(A) 10.29 (B) 10.84 (C) 11.18 (D) 11.99
Answer: (C) 11.18

Level of urbanization= × 100

= × 100= 11.18

. 75 As per the table given in Q. no. 74, the annual growth rate of urban population of India
during 1921-1931 was:
(A) 0.03 (B) 0.79 (C) 1.76 (D) 1.91
Answer: (C) 1.76
For annual growth rate
R= [( -1] ×100
Where R= annual growth rate (%)
= Population after n years(1931) and is the population in the base year
(1921)
n= no of years (interval)
Solving we get

R= [( -1] ×100
[( -1] ×100
(1.0176-1) ×100= 1.76

Linked Answer Questions: Questions 76 to Questions 85 carry two marks


each
Linked answer question 76 and 77:
. 76 In professional practice when there are disputes among the architects, clients and
contractors regarding the building constructions or contract, then to resolve the issues
the Expert/ Experts appointed for the same is termed as
(A) Arbitrator (B) Lawyer (C) Solicitor (D) Valuer
Answer: (A) Arbitrator

. 77 When there is dispute among the above Experts, then another expert is appointed to
resolve the issues who is known as
(A) Mediator (B) Referee (C) Judge (D) Umpire
Answer: (D) Umpire
Linked answer question 78 and 79:
. 78 Identify the formula for calculating the reverberation time (t) of a hall of volume V cu. M.,
where S represents sound absorption area
(A) t= 16V/S (B) t= 016V²/S (C) t= 0.16V/S (D) t= 16V/S²
Answer: (C) t= 0.16V/S

. 79 A school auditorium has a capacity of 800 persons. Considering 3.5 cu. M. of volume per
person and reverberation time of 1.25 sec, the total sound absorption area required
would be
(A) 348 sq.m. (B) 358 sq.m. (C) 368 sq.m. (D) 378 sq.m.
Answer: (B) 358 sq.m.
Soln: A standard reverberation time has been defined as the time for the sound to die
away to a level 60 decibels below its original level.

Where RT60 = Reverberation time in sec V = Volume in m³


Se = Total Absorption (Sabines- m )

Linked answer question 80 and 81:


. 80 If a bed room of 3m x 3m x 3m requires 3 air changes per hour, and difference in
temperature between inside and outside (∆T) = 12 deg C, then ventilation heat flow
rate (Qv) will be:
(A) 0.12 kW (B) 0.35 kW (C) 0.70 kW (D) 1.17 kW
Answer: (B) 0.35 kW
Soln : Qv= 1300 × V × ∆T
Where 1300= Volumetric specific heat of air (J/m3°C)
V= Ventillation rate (m3/sec)
∆T= Temperature difference (°C)

Qv= 1300 × × 12 = 351 Watt = 0.351 kW


Note: For conduction heat flow Q = U × A × ∆T, where U= transmittance value of
material (W/m2K)
For convection heat flow Qv= 1300 × V × ∆T, where 1300= Volumetric specific heat
of air (J/m3°C) and V= Ventillation rate (m3/sec)
For solar radiation on opaque surface Qs= A× I × θ, where I= radiation heat flow
density (W/m2)

. 81 For a given air velocity of 2m/s, the necessary cross sectional area of supply duct will
be:
(A) 0.0375 sq.m. (B) 0.0225 sq.m. (C) 0.0113 sq.m. (D) 0.0037 sq.m.
Answer: (C) 0.0113 sq.m.

Soln: Rate of ventilation = = 0.0225 m3/s


Air velocity = 2m/s

Cross sectional area of supply duct = = 0.0113 sq.m.

Linked answer question 82 and 83:


. 82 The number of senior secondary schools required for a city of population 1,00,000
persons is about
(A) 8-10 (B) 14-15 (C) 18-20 (D) 25-28
Answer: (B) 14-15

No of Senior Secondary School= = 13.33 (say 14)


Senior Secondary School (VI to XII) – 1 for 7,500 populattion
Strength of the school- 1000 students, Area per school – 1.8 ha
School Building area- 0.6 ha

Senior Secondary School (Class 6 to 12) (1 for every 7500 population)


Strength of school- 1000 students
Area per school 1.8 ha
(a) School Building area 0.60
ha
(b) Play field area (with a minimum of 68 1.0 ha
x 126 m to be ensured for effective
play)
(c) Parking area 0.20
ha
(Source: UDPFI guidelines prepared by ITPI.)

. 83 The total land requirement for senior secondary schools for a city of population of
1,00,000 persons is
(A) 8 ha (B) 15 ha (C) 25 ha (D) 40 ha
Answer: (C) 25 ha
Total area = no of schools × 1.6
= 14 × 1.8 = 25.2 ha
Linked answer question 84 and 85:
Q. 84 Identify the relationship governing the cost of land (C) based on the following factors:
Net density in plots per hectare = p
Land use percentage allocation in net housing = q
Price of land in Rs. Per sq. m. =s
(A) C= (10,000/p x 100/q) s (B) C= (10,000/p x q/100) s
(C) C= (10,000/p x 100/s) q (D) C= (10,000/s x 100/q) p
Answer: (B) C= (10,000/p x q/100) s
Let the area of whole land be Á' ha
given plots per ha = p therefore total no of plots = A × p

percantage area alloted for housing is 'q' so total area for housing = × A=

Area of one plot = = × = = ha

= sqm

Cost of one plot = = (10,000/p x q/100) s

. 85 If for a housing development, p= 30, q= 45 and s= 500, then the cost of land per dwelling
unit is:
(A) Rs 1,333/- (B) Rs 3,000/- (C) Rs 75,000/- (D) Rs 3,70, 370/-
Answer: (C) Rs 75,000/-
Cost of land per dwelling unit = (10,000/p x q/100) s

( × ) × 500 = 75,000/-
GATE 2010
Q. 1 Natural granite used for cladding in Buildings belongs to the category of
(A) Igneous Rock (B) Acid Rock (C) Sedimentary (D) Metamorphic rock
Answer: (A) Igneous Rock
Marble- Metamorphic rock
Sandstone- Sedimentary

Kota Stone- Sedimentary

Q. 2 ‘Flying Buttress’ is an architectural element of


(A) Indian Architecture (B) Greek Architecture (C) Gothic Architecture (D)
Byzantine Architecture
Answer : C (Gothic Architecture)
Notre Dame, Paris (1163-1240) - Designer is Eugène Viollet-le-Duc, Jean-Bapt–ste-
Antoine Lassus. It is the finest example of French Gothic Architecture. It is famous for
Rose Window and flying buttress. A Rose window is a circular window with
mullions and traceries generally radiating from centre and filled with stained glass. A
buttress is an exterior support, to retain the tall structures.
Example of Byzantine Architecture: Hagia Sophia, Constantine
Q. 3 A major hole in the ozone layer has been identified above the
(A) Amazon Forest (B) Arctic Region (C) Savannah Grassland (D) Sahara Desert
Answer: (B) Arctic Region
The ozone hole is an annual thinning of the ozone layer over Antarctica, caused by
stratospheric chlorine. (CFC- Chloroflurocarbon). In 1987, the Montreal Protocol
was signed, controlling the emissions of CFC which is being replaced by HCFC
(Hydro Chloroflurocarbon)
No Ozone depletion at North Pole/Arctic Region: Unlike Antarctica, which is a
continent surrounded by oceans, the Arctic is an ocean surrounded by mountainous
continents. This means that the stratospheric circulation is much more irregular.
Because it is not as cold as the Antarctic, stratospheric clouds are less common. So a
deep ozone hole over the North Pole is unlikely, but limited ozone depletion can occur
above parts of the Arctic, though it lasts only a few days at a time.

Q. 4 A flat arch at the skewback should NOT have angle less than
(A) 30° (B) 45° (C) 60° (D) 90 °
Answer : (C) 60°

Q. 5 Primary colours of natural light are


(A) Red, Blue, Yellow (B) Red, Green, Blue (C) Red, Violet, Yellow (D) Red,
Green, Yellow
Answer : B (Red, Green, Blue)
Note: There is a different set of Primary colour for Natural Light and Pigment.
Television and other computer and video displays are a common example of
the use of
additive primaries and the RGB (Red, Green, Blue) color model.
Primary Colour for pigment is RYB (Red, Yellow, Blue)

Q. 6 Horizontal member of a shutter that subdivides a window is termed as

(A) Mullion (B) Transom


(C) Reveal (D) Purlin
Answer: (B) Transom
Mullion: it is vertical element.
Transom is a transverse horizontal structural beam or bar, or a crosspiece
separating a door from a window above it.
Reveal is linear cutting in frame of door and windows where the shutter fits.
Purlin is horizontal member in truss system which connects various trusses.

Q.7 If the temperature of a composite bar made of copper and steel is raised, then the
copper bar will be under
(A) Tension (B) Compression (C) Shear (D)Torsion
Answer: B (Compression)
The thermal coefficient of linear expansion of copper is more so its expansion
is suppressed so compressed.

Q. 8 E. I. A. stands for
(A) East India Association (B) Environment Impact Audit
(C) Environment Impact in Asia (D) Environment Impact Assessment
Answer: D (Environment Impact Assessment)

Q. 9 A steel truss with parallel upper and lower chords and inclined connecting
members forming a series of equilateral triangle is known as
(A) Bowstring Truss (B) Warren Truss (C) Kingpost truss (D) Scissors Truss
Answer: (B) Warren Truss

Q. 10 In water supply systems, the reflux valve allows water to flow


(A) in one direction only (B) in both directions
(C) through air locked joints (D) only under low pressure
Answer: (A) in one direction only
Reflux valve is used in the suction line of the pumping system to prevent the
backflow, when the pump is at the rest.
Reflux valve: a valve that closes by fluid pressure to prevent return flow. Also
called: nonreturn valve.
Sluice valve: also known as a gate valve, a valve that opens by lifting a round
or rectangular gate/wedge out of the path of the fluid.
Relief valve is a type of valve used to control or limit the pressure in a system
or vessel which can build up for a process upset, instrument or equipment
failure, or fire. The pressure is relieved by allowing the pressurised fluid to flow
from an auxiliary passage out of the system. (safety valve of a pressure cooker
is a relief valve)
Scour Valve: also known as wash out valve. These valves are used at the
valley point to clean the pipe line.

Q. 11 In Islamic Architecture, the circular dome was


constructed over a square configuration through
(A) Grid iron coffered slab
(B) Pendentives and Squinch Arches
(C) Double Barrel Vault and Jack Arches
(D) Horizontal Cross Tie Members
Answer is B. (Pendentives and Squinch Arches)
Pendentive: A triangular segment of a spherical surface.

Q. 12 With respect to energy conservation and cost efficiency, the nature of an ideal
built form should be
(A) High Rise Low density (B) Medium Rise High Density
(C) Low Rise High Density (D) Low Rise Low Density
Answer is B (Medium Rise High Density)

Q. 13 Two default sequences P and Q are given below


P: Specify height of Extrusion or [Path]: 50
Specify Angle of taper for extrusion <0>

Q: Specify height of Extrusion or [Path]: p


Select extrusion path or [Taper Angle]:
The above mentioned sequences P and Q respectively belong to
(A) 2D AutoCAD and 2D AutoCAD (B) 2D and 3D AutoCAD
(C) 3D and 2D AutoCAD (D) 3D and 3D AutoCAD
Answer:

Q. 14 When shear stress exceeds the permissible limit in a RCC Slab, then this
problem is solved by
(A) Increasing the slab depth (B) Providing shear reinforcement
(C) Using High Strength Steel (D) Using thinner bar but more in numbers
Answer: (B) Providing shear reinforcement
Shear reinforcement provides restraint against cracks formed under shear force by
running perpendicular reinforced to the direction of crack.
examples are –stirrups
Courtesy: Souradeep Gupta ,Phd scholar, NUS Singapore.

Q. 15 Considering the total heat loss from all fluorescent lamps to be 79 % the heating
load (Btu/hr) due to office illumination with 48 ceiling mounted luminaries, each
containing four, 40 W fluorescent lamps and flat surface diffusers will be
(A) 10,000 Btu/hr (B) 15000 Btu/hr (C) 17,500 Btu/hr (D) 21,000 Btu/hr
Answer: (D) 21,000 Btu/hr
Total watt= 48x4x40= 7680 W
79 % heating load= 7680x 79/100= 6067.2 W= 6067.2 Joules/sec
(1055 Joules= 1Btu)
6067.2 Joules/sec= 5.75Btu/sec
In 1 sec, 5.75 Btu so in 3600 sec, 5.75x 3600= 20,700 Btu/hr. (say 21,000Btu)

Q.16. Prime resultant forces that develop in a structure due to an earthquake depend
on
(A) Mass and Surface Area of Structure (B) Surface Area and Stiffness of
Structure
(C) Stiffness and Mass of Structure (D) Surface Area and Volume of Structure
Answer: (C) Stiffness and Mass of Structure

Q.17. Concept of Serial Vision has been applied to the approach layout of
(A) Victoria Memorial Complex, Kolkata (B) Umaid Bhawan Palace Jodhpur
(C) Vidhan Soudha Precinct, Bangalore (D) Rashtrapati Bhawan Complex, New Delhi
Answer: (D) Rashtrapati Bhawan Complex, New Delhi
Serial Vision: Gorden Cullen

Q.18. Advance traffic lane Information is an important feature of


(A) Para Transit System (B) Intelligent Transportation System
(C) High Level Cable Car System (D) Pedestrian Travellator System
Answer: (B) Intelligent Transportation System

Q.19. A local Authority can go for Urban Development through


(A) Land Acquisition (B) Land Pooling (C) Transferable Development Rights (D)
All the above.
Answer :( D) All the above.
Land Acquisition is the process through which the Govt. can acquire the Private land
for Public Purpose. Land Acquisition Act of 1894 was replaced by Right to Fair
Compensation and Transparency in Land Acquisition, Rehabilitation and
Resettlement Act, 2013. Salient Features of the Act.
Public Purpose Defined
For strategic purposes relating to naval, military, air force, and armed forces of the
Union, including central paramilitary forces or any work vital to national security or
defence of India or State police, safety of the people; or
For infrastructure projects, which includes the following, namely:
All activities or items listed in the notification of the Government of India
in the Department of Economic excluding private hospitals, private
educational institutions and private hotels;
Projects involving agro-processing, supply of inputs to agriculture,
warehousing, cold storage facilities, marketing infrastructure for
agriculture and allied activities such as dairy, fisheries, and meat
processing, set up or owned by the appropriate Government or by a
farmers' cooperative or by an institution ’et up under a statute;
Project for industrial corridors or mining activities, national investment
and manufacturing zones, as designated in the National Manufacturing
Policy;
Project for water harvesting and water conservation structures, sanitation;
Project for Government administered, Government aided educational and
research schemes or institutions;
Project for sports, heath care, tourism, transportation of space
programme;
Any infrastructure facility as may be notified in this regard by the Central
Government and after tabling of such notification in Parliament;
Project for housing, or such income groups, as may be specified from time to
time by the appropriate Government;
Project for planned development or the improvement of village sites or any site
in the urban areas or provision of land for residential purposes for the weaker
sections in rural and urban areas;
Project for residential purposes to the poor or landless or to persons residing in
areas affected by natural calamities, or to persons displaced or affected by
reason of the implementation of any scheme undertaken by the Government,
any local authority or a corporation owned or controlled by the State.
Protection of Agricultural Land
Act requires that wherever multi crop irrigated land is acquired an equivalent area of
culturable wasteland shall be developed by the state for agricultural purposes.
Compensation
At least four times the market value for land acquired in rural areas;
At least two times the market value for land acquired in urban areas
Rehabilitation
Both Economic and Social Rehabilitation to be covered.
Land Pooling: Through the land pooling policy, landowners — including farmers — can
form consortiums and tie up with private builders or banks to consolidate their land
parcels and then develop the land.
TDR- Transferable Development Rights: In areas in which building heights are limited,
the right to sell air rights for purposes of business expansion and construction; i.e., a
company not using all of its height expansion allowance can transfer the right to build
in the air space to another company.
Q.20. The planning document submitted for selected cities under JNNURM is
(A) Master Plan (B) Basic Development Plan(C) City Development Plan (D) Outline
Development Plan
Answer: (C) City Development Plan
JNNURM is a massive city-modernisation scheme launched by the Government of
India under Ministry of Urban Development initially for a
seven year period from 2005 to 2012 and is now extended for a period of 2
years up to Mar 2014. AMRUT is new program initiated by Modi Govt.

Q.21. Excessive tilt of the leaning tower of Pisa has been checked by
(A) Pumping Cement Concrete under the dipping foundation
(B) Relocation heavier furniture to the rising side of the tower
(C) Raising the dipping side by massive jack screws
(D) Pumping out mud and slurry from the foundation base of the rising side.
Answer: (A) Pumping Cement Concrete under the dipping foundation

Q.22. The Pritzker prize 2009 has been awarded to


(A) Zaha Hadid (B) Peter Zumthor (C) Jean Nouveal (D) Norman Foster
Answer: (B) Peter Zumthor

Q.23. The age of a tree is determined by


(A) Counting the number of rings in stem cross-section
(B) Counting the number of leaves on main branch
(C) Measuring the height of tree from the root ball
(D) Measuring the canopy circumference of the tree
Answer : (A) Counting the number of rings in stem cross-section

Q.24. Nakagin capsule tower, Tokyo famous for its spatial modular approach was
designed by

(A) Arata Isozaki (B) Tadao Ando


(C) Kisho Kurokawa (D) Minoru Yamasaki
Answer: (C) Kisho Kurokawa
The Nakagin Capsule Tower is a mixed-use residential and office tower designed by
architect Kisho Kurokawa and located in Tokyo, Japan. The tower is an example of
Japanese Metabolism. The building is actually composed of two interconnected
concrete towers, respectively eleven and thirteen floors, which house 140
prefabricated modules (or “capsules”) which are each self-contained units. Each
capsule measures 2.3 m (7.5 ft) × 3.8 m (12 ft) × 2.1 m (6.9 ft) and functions as a small
living or office space. Capsules can be connected and combined to create larger
spaces. Each capsule is connected to one of the two main shafts only by four high-
tension bolts and is designed to be replaceable.

Q. 25 Proportioning system used in the layout of Mughal Gardens is derived from


(A) Rational Number System (B) Constants of Equilateral Triangle
(C) Irrational Number System (D) Constants of Right angled Isosceles Triangle
Answer: (A) Rational Number System

Q 26-55 carry two marks each


26. Match the cities in Group I with their form in Group II
Group I Group II
P. Detroit 1. Star Form
Q. Copenhagen 2. Poly Centered Net
R. Stalingrad 3. Linear City
S. San Francisco 4. Ring Form
5. Galaxy
(A) P-1, Q-4, R-3, S-2 (B) P-2, Q-1, R-3, S-4 (C) P-5, Q-1, R-2, S-3 (D) P-4,
Q-3, R-1, S-5
Answer : (B) P-2, Q-1, R-3, S-4
Detroit is also known as Motor city because of large automobile industry.
Radburn is known as town for motor age.

Q.27. Match the visionaries in Group I with their concepts in Group II


Group I Group II
P. Clarence A Perry 1. Post Modernism
Q. Constantinos Doxiadis 2. Bauhaus
R. Paul Davidoff 3. Advocacy Planning
S. Walter Gropius 4. Dynapolis
5. Neighbourhood unit
(A) P-5, Q-4, R-3, S-2 (B) P-5, Q-4, R-2, S-1 (C) P-1, Q-3, R-2, S-1 (D) P-2, Q-4,
R-2, S-5
Answer : (A) P-5, Q-4, R-3, S-2

Neighbourhood Unit by
Clarence A Perry. (Basic Concept)
Centre the school in the neighbourhood so that a child’s walk to school was
only about one-quarter of a mile and no more than one half mile and could be
achieved without crossing a major arterial street.
Size the neighbourhood to sufficiently support a school, between 5,000 to
6,000 residents, approximately 160 acres at a density of ten units per acre.
Implement a wider use of the school facilities for neighbourhood meetings and
activities, constructing a large play area around the building for use by the
entire community.
Place arterial streets along the perimeter so that they define and distinguish
the “place” of the neighborhood and by design eliminate unwanted through-
traffic from the neighborhood. In this way, major arterials define the
neighborhood, rather than divide it through its heart.
Design internal streets using a hierarchy that easily distinguishes local streets
from arterial streets, using curvilinear street design for both safety and
aesthetic purposes. Streets, by design, would discourage unwanted through
traffic and enhance the safety of pedestrians.Neighbourhood unit by Clarence Perry
Restrict local shopping areas to the perimeter or perhaps to the main
entrance of the neighborhood, thus excluding nonlocal traffic destined for these
commercial uses that might intrude on the neighborhood.
Dedicate at least 10 percent of the neighborhood land area to parks and open
space, creating places for play and community interaction”.

Note: Radburn Plan of Neighbourhood was conceived by Clarence Stein.


(Clarence Perry and Clarence Stein are two different Sociologist/ Planners)
Constantinos Doxiadis known for Ekistics and Dynapolis

“Advocacy and Community Planning”


(book) by Paul Davidoff who was a lawyer.
Bauhaus- Walter Gropius (started in Germany)

Engelhardt’s diagram of neighbourhoods (adjacent figure)


Grade School Distance
Category (mile) .
Kindergarten Play
School/Nursery
1-6 Elementary
School

7-10 Middle School 1 Q.28. Match the trees in Group I with their
11-14 Upper School botanical names in Group II
1-1 Group I Group II
P. Neem 1. Castia Fistula
Q. Amaltas 2. Azadirachta Indica
R. Pipal 3. Ficus Bengalensia
S. Asoka 4. Ficus Religiosa
5. Saraca Indica
(A) P-1, Q-2, R-3, S-4 (B) P-3, Q-4, R-5, S-1 (C) P-2, Q-1, R-4, S-5 (D) P-2, Q-3,
R-5, S-4

Answer: C. (P-2, Q-1, R-4, S-5)

Q.29. Following graphs represent the relationship between city size (in terms of
Population) on X- axis and area under residential use (in percent) on Y
axis. Identify the correct graph.
Answer: A
A large city has large number of slum population with high occupancy rate.

Q.30. Global Climate change is expected to bring about a combination of following


changes. Identify the correct combination.
P. Increase in Biodiversity Q. Emergence of new Disease
R. Loss of Biodiversity S. Loss of all Rocky outcrops
T. Sea Level Rise U. Extinction of Polar Bear
V. Emergence of New Islands
(A) P, Q, R, S (B) Q, R, T, U (C) R, T, U, V (D) Q, R, U, V
Answer : (B) Q, R, T, U

Q. 31 Annual housing demand of a Metropolitan city is estimated through the


combination of the following components. Identify the correct combination.
P. New entrants in the city Q. Elderly Population living in the city
R. New Relocated Slum Dwellers S. Slum Squatter Dwellers
T. Unauthorized Dwelling Units U. Dilapidated Houses

V. Part of Backlog W. Any other Houses


(A) P, R, T (B) U, S, Q (C) W, Q, T (D) P, U, V
Answer: D. (P, U, V)
Q. 32 A Square pin jointed truss is subjected to a load P, acting in the direction of
member US at joint U. The force in member UR is
(A) 1.414 P (B) 1.000P (C) 0.707P (D) 0.000P
Answer: (D) 0.000P

Q. 33 Given below is the sketch plan of a site


showing contours. The broken lines show valleys and ridges. Identify the
ridges and valleys.
(A) Ridges: P, Q, R
Valleys: S, T

(B) Ridges: T, V
Valleys: R, T, U
(C) Ridges: S, U
Valleys: Q, T, V

(D) Ridges: R, V, U
Valleys: P, T,
Answer: (C)

. 34 From the following, identify the factors which influence the loudness of sound to a
listener in an enclosure
P. Loudness of sound at Source
Q. Directivity Factor
R. Length/ Width Ratio of enclosure
S. Distance between Sound Source and Listener
U. Sound absorption co-efficient of all enclosing surface
V. Surface Area of enclosing surfaces
Y. Inside Temperature level of enclosure
(A) P, Q, S, V, Y (B) Q, R, S, U, Y (C) P, R, S, U, V (D) P, Q, S, U, V
Answer: (A) P, Q, S, V, Y
Directivity is a measure of the directional characteristic of a sound source. If you have
a limited amount of sound energy available (like that produced by the human voice),
you can increase the distance that your voice will cover by increasing the directivity of
your voice. You have probably experienced this calling to someone outdoors, when
you cup your hands around your mouth to increase the directivity.

Q. 35 Match the lamps in Group I with their Colour Rendering Index (CRI) in Group II
Group I Group II
P. Mercury Vapour 1. 65-70
Q. Metal Halide 2. 40- 55
R. High- Pressure Sodium 3. 20- 25
S. Low-Pressure Sodium 4. 60-64
(A) P-4, Q-2, R-1, S- 3 (B) P-3, Q-2, R-4, S- 1 (C) P-2, Q-1, R-4, S- 3 (D) P-4,
Q-3, R-2, S- 1
Answer : (C) P-2, Q-1, R-4, S- 3
Colour Rendering Index (CRI) is a quantitative measure of the ability of a light
source to reproduce the colors of various objects faithfully in comparison with
natural light source.
S Light source CRI Luminous Uses
No efficacies
Low-pressure sodium 200 Street Light/
1 (LPS/SOX) -44 lm/W Security Light
35-65 Replaced by
2 Clear mercury-vapour 17 lm/W metal halide
High-pressure sodium 100-150 Street Light/
3 (HPS/SON) 24 lm/W Security Light
100 Interior
4 Coated mercury-vapour 49 lm/W
Halophosphate warm- 100 Interior
5 white fluorescent 51 lm/W
Halophosphate cool-white 100 Interior
6 fluorescent 64 lm/W
Tri-phosphor warm-white 100 Interior
7 fluorescent 73 lm/W
Halophosphate cool- 100 Interior
8 daylight fluorescent 76 lm/W
9 “White” SON 82 Interior
75-100 Interior
10 Quartz metal halide 85 lm/W
Tri-phosphor cool-white Interior
11 fluorescent 89
12 Ceramic metal halide 96 75-100 Automobile head
lm/W light/
photography/
stage
lighting/flood
lights
Incubators,
13 Incandescent bulb 100 16 lm/W Home lighting
Low Pressure Sodium Light – When the lamp is turned on it emits a dim red/pink
light to warm the sodium metal and within a few minutes it turns into the common
bright yellow as the sodium metal vaporizes.LPS lamps are the most efficient
electrically powered light source when measured for lighting conditions—up to 200
lm/W, As a result they are widely used for outdoor lighting such as street lights and
security lighting where CRI is unimportant.
Flourescent Lamps – These are a low pressure mercury-vapor gas-discharge lamp
that uses fluorescence to produce visible light. An electric current in the gas excites
mercury vapor which produces short-wave ultraviolet light that then causes a
phosphor coating on the inside of the bulb to fluoresce.

Q. 36Match the terms in Group I with the Architectural Elements in Group II


Group I Group II
P. Tympanum 1. Auditorium Stage
Q. Proscenium 2. Door or Window Bands
R. Campanile 3. Circular House
S. Dymaxion 4. Church Tower
5. Horizontal Space for Services
(A) P-1, Q-3, R-2, S- 4 (B) P-2, Q-3, R-4, S- 1 (C) P-2, Q-1, R-4, S- 3 (D) P-1, Q-
2, R-3, S- 5
Answer : (C) P-2, Q-1, R-4, S- 3
Tympanum- element located between arch or pediment.
Proscenium- The rectangular frame around the stage is the proscenium.
Campanile- A bell tower, especially one near but not attached to a church or
other public building.
Dymaxion- A term attached with Buckminster Fuller for his entire circular –
dome (Geodesic Dome), Circular Car, Circular house etc. It is a portmanteau of
the words dynamic, maximum, and tension.

. 37 Identify the most representative percentage distribution of landuse for a medium


urban centre, according to UDPFI guidelines, where Residential= R,
Commercial= C, Transport= T, Industry= I
(A) R= 30%, C=20%, T= 12%, I= 10% (B) R= 45%, C=4%, T= 14%, I= 8%
(C) R= 30%, C=4%, T= 14%, I= 15% (D) R= 45%, C=10%, T= 12%, I= 10%
Answer: (B) R= 45%, C=4%, T= 14%, I= 8%

.38 If the area of a plot is 1000 sq.m, area of its adjoining roads is 500 sq.m, maximum
permissible FAR is 150 and maximum permissible ground coverage is 50%, then
utilizing fullest ground coverage and assuming floors of equal area, the number
of storey that can be built on the plot is
(A) 6 (B) 4 (C) 3 (D) 2
Answer : (C) 3
Area of plot = 1000 sq m FAR= 150 (it means 1.5)
Total floor area = 100
Permissible ground coverage = 50 % = 500 sq m

So no of floors= =3
(Answer corrected as informed by Pappal Suneja, GNDU Amritsar)

Q. 39 Match the buildings in Group I with their Architects in Group II


Group I Group II
P. British Council Library, New Delhi 1. Hasmukh C Patel
Q. Osho Commune Centre, Pune 2. Charles Correa
R. CII Sohrabji Godrej Green Business Centre, Hyderabad 3. Hafeez Contractor
S. IIM New Campus, Ahmedabad 4. Karan Grover
5. Balkrishna V Doshi
(A) P-1, Q-2, R-3, S- 4 (B) P-2, Q-3, R-4, S- 1 (C) P-2, Q-3, R-5, S- 1 (D) P-5,
Q-4, R-3, S- 2
Answer: B (P-2, Q-3, R-4, S- 1)
CII Sohrabji Godrej Green Business Centre, Hyderabad- First Platinum rated green
Building (LEED) of India by Karan Grover.

. 40 The age sex compositions of three communities are represented by diagrams P, Q


and R as shown below.
Each of these implies a strong socio economic characteristic as indicated below.
1. Aging Community 2. Economically Vibrant Community
3. Multi Ethnic Community 4. Young Community with High Birth Rate
Identify the correct set out of the following
(A) P-4, Q-3, R-1 (B) P-2, Q-4, R-3 (C) P-2, Q-4, R-1 (D) P-4, Q-2, R-3
Answer: (C) P-2, Q-4, R-1

. 41. The correct requirements provided to seek permission from the local authority for
constructing small residential buildings are
P- Key Plan Q- Site Plan
R- Zonal Plan S- Building Plan
T – Power of Attorney U- Ownership Title
V – Transport Plan W – Drainage/ Sewarage/ Water Supply Plan
X – Solid Waste Disposal Plan
(A) P, Q, R, S, W (B) P, Q, S, U, W (C) P, S, V, W, X (D) Q, S, T, V, X
Answer is B. (P, Q, S, U, W)

Q. 42 Two commands P and Q in AutoCAD are given below.


P: Current Settings: Mode = TRIM, Radius=0.0000
Select First Object or [Polyline/ Radius/ Trim/mUltiple]:
Q: {Trim Mode) Current Chamfer Dist1= 0.0000, Dist 2= 0.0000
Select First Line or [Polyline/ Distance/ Angle/ Trim/ Method/mUltiple]:
The above mentioned commands are used for
(A) P: Trim and Q: Trim (B) P: Fillet and Q: Trim
(C) P: Fillet and Q: Chamfer (D) P: Trim and Q: Chamfer
Answer: (D) P: Trim and Q : Chamfer

. 43 A T- Beam slab is cast and cured. The Shuttering has to be removed. The right
sequence of removal of shuttering is
(A) Base of Beam→ Sides of Beam→ Base of Slab→ Vertical support under beam
(B) Base of Slab→ Sides of Beam→ Base of Beam→ Vertical support under beam
(C) Base of Slab→ Sides of Beam→ Vertical support under beam→ Base of Beam
(D) Base of Beam→ Base of Slab→ Sides of Beam→ Vertical support under beam
Answer is (C) Base of Slab→ Sides of Beam→ Vertical support under beam→ Base
of Beam

Q.44 Bio- climatic chart developed by Victor Olgyay shows the relationship between
(A) Temperature and Precipitation (B) Relative Humidity and Precipitation
(C) Air Movement and temperature (D) Temperature and Relative humidity
Answer is D. (Temperature and Relative humidity)
(Refer page 50, Manual of Tropical Housing and Building, Koenigsberger)

. 45 In a display window of height H= 8.66m, of a retail store, a luminaire of intensity I


is mounted at a distance L= 5m from the rear. Its light beam is cast at an angle of
45°, from the ceiling as shown in the figure alongside.
The ratio of illumination at Points P1 and P2 is
(A) 1:√3 (B) √3:2 (C) √2:1 (D) 1:2
Answer: (C) √2:1
Intensity at P1 is greater than P2 (only one such option there !).
Since it’s a beam of light intensity will not vary with respect to short distances. Angle of
incidence will make difference.
Illuminance at P1= I × Cos 45
Illuminance at P2= I × Cos 60

= = × = :1

CASE II
Let us consider ‘I’ to be a point source then the soln will be as follows

Illuminance at P1= × Cos 45º

= ×

Illuminance at P2= × Cos 60º

= ×

= = × =

Q. 46. Following figure shows network for a particular project consisting of four
activities
Normal duration and crash time for each activity are given below
Activity Normal Duration Crash time
(in days) (in days)
1-2 3 2
2-3 4 2
2-4 5 4
3-4 7 5
The minimum time required for completion of project is
(A) 9 days (B) 13 days (C) 14 days (D) 19 days
Answer: (A) 9 days
Minimum time required for completion of project is through path 1-2-3-4.

. 47 Pick the ODD one from the figures given below with respect to reflection and
transmission of Light

(A) P (B) Q (C) R (D) S

Answer : (B) Q

Common Data Questions


Common data for questions 48 and 49:

A simply supported beam PQ is subjected to a load


of 100 kN through a rigid link at the centre of the beam as shown in the figure below

Q. 48 Correct shear force diagram for the beam is


Answer: (A)
Q. 49 Bending moment diagram for the above beam is
Answer: (D)

Common data for questions 50 and 51:


A plot of land is to be developed as a residential neighbourhood. The key
development conditions and project requirements are given below.
Plot Area: 1.25 Hectare Maximum Permissible FAR: 350
Maximum Permissible plot coverage: 30% Maximum Permissible Height: 45 m
Density of Population = 800 ppHa Average Household size: 3.55
Building Type Percentage of Dwelling units Total Built up
area (in sq.m)
L.I.G. 20 4480
M.I.G. 55 18600
H.I.G. 25 14200

Q. 50 The total number of units under L.I.G., M.I.G. and H.I.G. respectively are
(A) P (B) S (C) Q (D) R L.I.G. M.I.G. H.I.G.
Answer: (A) P P 56 155 71
Land area= 1.25 ha= 12500sqm Q 60 142 71
Pop density = 800 per hectare =0.08 per sqm.
R 56 150 63
Total Population= 0.08 × 12500 = 1000
Average household size = 3.55 S 59 155 63

So total no of house = = 281.69 (say 282)

So LIG= 20 % therefore × 282= 56


L.I.G. M.I.G. H.I.G.
So MIG= 55 % therefore × 282= 155 P 70 130 200
Q 80 120 200
So HIG= 25 % therefore × 282= 70
R 70 130 220
. 51 With the above data, the covered area of each flat (in S 80 120 220
sq. m) under L.I.G., M.I.G. and H.I.G. respectively are
(A) Q (B) S (C) R (D) P
Answer: (A) Q

LIG = = 80 sqm MIG = = 120 sqm HIG = = 203


sqm

Linked Answers Questions


Statement for Linked Answer Questions 52 and 53:
A person has purchased an old building at a cost of Rs 2,50,000/-, excluding the cost
of land. The scrap value of the building is 10% of the cost of purchase and the future
life of the building is 20 years.

Q. 52 The total amount of sinking fund at the end of 20 years will be


(A) Rs. 1,35,000/- (B) Rs. 1,90,000/- (C) Rs. 2,25,000/- (D)Rs. 2,30,000/-
Answer: (C) Rs. 2,25,000/-

Scrap value of the building = 10 % of the cost of purchase= 2,50,000 × =


25,000/-
So total amount of sinking fund= 2,50,000 – 25,000= Rs. 2,25,000/-

Q. 53 If the rate of Interest is 7%, then the annual installment of Sinking fund will be
(A) Rs. 4,583/- (B) Rs. 4,855/- (C) Rs. 5,507/- (D) Rs. 5,640/-
Answer: (C) Rs. 5,507/-
Sinking fund formula:

Annual Installment (A) = where S is sinking fund, ‘i’ is interest rate and
‘n’ is no of years.

A
A= Rs 5,507/-

Statement for Linked Answer Questions 54 and 55:


A standpipe system in a 39 m tall building has a rooftop reservoir for Fire Fighting
containing ½ hour water supply.

. 54 Assuming accommodate each floor has one hose, the delivery rate (in litre per
second) of the fire hose at greatest pressure is
(A) 75 (B) 78 (C) 81 (D) 84
Answer: Data Missing (Marks awarded to all)
Alternative Solution
Given data:
1. Building Height : 39 meters
2. water supply for 30 mins fire fighting is available
Assumptions:
1. Only 1 hose is open (greatest pressure) including additional standpipes (if any)
The maximum total permissible flow rate of standpipe system is 1250 USgallon per minute.
Which is equal to 4731 litre/min = 78.85 litre/sec
(Reference: 2007 Edition of NFPA 14 Chapter 7)

Q. 55 The volume of water (in cubic metre) required for the reservoir is
(A) 105 (B) 110 (C) 120 (D) 130
Answer: Data Missing (Marks awarded to all)
According to THE LPC DESIGN GUIDE the answer should be (D) 130
you can also calculate it by above flow rate (since the above flow rate is at greatest
pressure, therefore the average flow
rate will be less than it)
Calculation based on the formula ( tank emptying time = volume /rate of delivery)
(note: the above formula will not give exact data since the rate of delivery will vary due to
decreasing water pressure due to water
drainage)
we have tank emptying time = 30 mins (water supply for 30 mins fire fighting is
available)
rate of delivery = 78 litre/sec
So, the volume will be (30*60)seconds * 78litre/sec = 140400 lite = 140.4 cubic meter
if you take Related rate of delivery ( http://study.com/.../related-rates-the-draining-tank...)
the answer will be approximately 130 cubic meter.
The soln was provided by Rewati Raman (B Arch,NIT Patna, MURP, PhD IITR) on FB page

General Aptitude (GA) Questions


Q 56 to Q 60 carry one marks each
. 56 25 Persons are in a room. 15 of them play hockey, 17 of them play football, and 10
of them play both hockey and football. Then the number of persons playing
neither hockey nor football is
(A) 2 (B) 17 (C) 13 (D) 3
Answer: (D) 3

. 57 Choose the most appropriate word from the options given below to complete the
following sentence:
If we manage to __________our natural resources, we would leave a better
planet for our children.
(A) uphold (B) restrain (C) cherish (D) conserve
Answer: (D) conserve

. 58 The question below consists of a pair of related words followed by four pair of words.
Select the pair that best expresses the relation in the original pair.
Unemployed: Worker
(A) fallow : land (B) unaware: sleeper (C) wit: jester (D) renovated: house
Answer: (A) fallow : land

Q. 59 Which of the following options is the closest in meaning to the word below:
Circuitous
(A) cyclic (B) indirect (C) confusing (D) crooked
Answer: (B) indirect

. 60 Choose the most appropriate words from the options given below to complete the
following sentence.
His rather casual remarks on Politics ______________ his lack of seriousness
about the subject.
(A) masked (B) belied (C) betrayed (D) suppressed
Answer: (C) betrayed

Q 61 – Q 65 carry two marks each


. 61 Hari (H), Gita (G), Irfan (I) and Saira (S) are siblings (i.e. brothers and sisters). All were
born on 1st January. The age difference between any two successive siblings
(accommodate is born one after another) is less than 3 years. Given the following
facts.
i. Hari’s age + Gita’s age > Irfan’s age + Saira’s age
ii. The age difference between Gita and Saira is 1 year. However, Gita is
not the oldest and Saira is not the youngest.
iii. There are no twins
In what order were they born (oldest First)
(A) HSIG (B) SGHI (C) IGSH (D) IHSG
Answer: (C) IGSH

. 62 5 skilled workers can build a wall in 20 days; 8 semi skilled workers can build a
wall in 25 days; 10 unskilled workers can build a wall in 30 days. If a team has
two skilled, 6 semi skilled and 5 unskilled workers how long will it take to build
the wall?
(A) 20 days (B) 18 days (C) 16 days (D) 15 days
Answer: (D) 15 days

. 63 Modern warfare has changed from large scale clashes of armies to suppression of
civilian populations. Chemical agents that do their work silently appear to be suited to
such warfare; and regretfully there exist people in military establishment who think that
chemical agents are useful tools for their cause.
Which of the following statements best sums up the meaning of the above
passage.
(A) Modern warfare has resulted in civil strife.
(B) Chemical agents are used in modern warfare
(C) Use of chemical agents in warfare would be undesirable
(D) People in military establishments like to use chemical agents in war.
Answer: (D) People in military establishments like to use chemical agents in war.

64. Given digits 2,2,3,3,3,4,4,4,4 how many distinct 4 digit numbers greater than 3000
can be formed ?
(A) 50 (B) 51 (C) 52 (D) 54
Answer: (B) 51

Q 65. If 137 +276 =435 how much is 731 +672 ?


(A) 534 (B) 1403 (C) 1623 (D) 1513
Answer: (C) 1623
This problem is of base 8. Normal calculation we use base 10.
GATE 2011
Q. 1 Capital Town of Gandhi Nagar has been designed by
(A) Norman Foster (B) B V Doshi (C) H K Mewada (D) Le Corbusier
Answer: C (H K Mewada).
In 1971 the city of Gandhi Nagar was planned for a population of 1,50,000, by H K
Mewada and Prakash M Apte.
Norman Foster is planning Abu Dhabi, Zero Carbon + Zero Waste City which is to
be completed by 2015. B V Doshi had planned Vidyadhar Nagar, near Jaipur, while
Le Corbusier planned Chandigarh.

Q. 2 Rajiv Awas Yojana of Ministry of Housing, Government of India addresses housing for
(A) Middle Income Group (B) Low income Group (C) High Income Group (D)
Slum Dwellers
Answer is D (Slum Dwellers).
Rajiv Awas Yojana (RAY) was announced in 2009, which addresses the needs of
Urban Slum dwellers. It is running parallel to the program of JNNURM (Jawahar Lal
Nehru National Urban Renewal Mission). JNNURM was started in 2005 with an
objective to provide Basic Services to Urban Poor.Initial period was 2005-2012, which
ws extended to two more years up to 2014.
Indira Awas Yojana (IAY) provides financial assistance for Rural Housing to the BPL
(Below Poverty Line) families. This program is operated by Ministry of Rural
Development, Govt. of India. Under IAY financial grant: Rs 45,000/- provided per
dwelling unit for plain areas and Rs 48,500/- provided per dwelling unit for
hilly/difficult areas

Q. 3 The triangular space formed between two consecutive arches is


(A) Tympanum (B) Spandreal (C) Regula (D) Extrados
Answer is B. (Spandreal)

Tympanum is the area within arch or pediment.


.4 Rose Window is an iconic feature of
(A) Notre Dame, Paris (B) Hagia Sophia,
Istanbul
(C) St Peters, Rome (D) Victoria Memorial,
Kolkata
Answer is A. (Notre Dame, Paris)
Notre Dame, Paris (1163-1240) – Designer is
Eugène Viollet-le-Duc, Jean-Baptiste-Antoine
Lassus. It is the finest example of French
Gothic Architecture. It is famous for Rose
Window and flying buttress. A Rose window is a
circular window with mullions and traceries
generally radiating from centre and filled with
stained glass. A buttress is an exterior support,
to retain the tall structures.
Hagia Sophia, Istanbul, Turkey (532-537 AD) – This building served as Basilica,
Mosque and now a secular Museum. It is famous for massive dome and is finest
example of Byzantine Architecture. It was designed by the Greek scientists Isidore of
Miletus, a physicist, and Anthemius of Tralles, a mathematician.
St Peters Cathedral, Rome- Build over a period of 120 years (1506-1626 AD). The
Architects associated with this building are Donato Bramante, Antonio da
Sangallo the Younger, Michelangelo, Jacopo Barozzi da Vignola, Giacomo della
Porta, Carlo Maderno, Gianlorenzo Bernini. Cathedral has the tallest dome in the
world.
Victoria Memorial, Kolkata- (completed in 1921)
Architect- Sir William Emerson
Style- Indo-Saracenic
Indo-Saracenic architecture represents a synthesis of Muslim designs and Indian
materials developed by Mughals/ British architects in India during the late nineteenth
and twentieth century.eg: Structures of Leutyens Delhi i.e Parliamaent House,
Rashtrapati Bhawan, Victoria Memorial, Kolkata.

Q. 5 Purity of colour is defined by


(A) Hue (B) Value (C) Chroma (D) Tone
Answer is C. (Chroma)
Hue refers just pure spectrum colour. (Black, white, grey are colours and not Hue).
Value is relative lightness or darkness of colour.
Chroma is a measure of color purity in the Munsell color system. When grey is
added to a colour the mixture is a diffgrent tone.If a color is made lighter by adding
white, the result is called a tint. If black is added, the darker version is called a
shade. And if grey is added, the result is a different tone.

. 6 A slab simply supported on all its edges with a ratio of longer side to shorter side greater
or equal to 2.0 is designed as
(A) One way slab (B) Two Way Slab (C) Flat Slab (D) Coffered Slab
Answer is A. (One way slab)
In one way slab main reinforcement is along shorter side of the slab. When ratio is
less than 2.0 two way slab is designed.

Q. 7 Entablature consists of
(A) Architrave, Tenia, Cornice (B) Architrave, Frieze, Cornice
(C) Frieze, Cornice, Triglyphs (D) Cornice, Guttae, Tympanum
Answer is B. (Architrave, Frieze, Cornice)
An entablature refers to the superstructure of moldings and bands which lie
horizontally above columns and below Pediment i.e. sandwiched between column
and pediment. A pediment is triangular section above entablature. Entablature has
three layers namely Architrave, Frieze and Cornice.
Note: Questions are set every year on this topic because of the various architectural elements available in
Classical Architecture. (Refer: History of Architecture,Bannister Fletcher)

Q. 8 Town planned for ‘Motor Age’ refers to


(A) Toranto, Ontario (B) Nassan Shore, Long Island (C) Radburn, New Jersey (D)
Green Belt, Meryland
Answer is C. (Radburn, New Jersey)
Radburn was designed by Clarence Stein and Henry Wright which is known as
‘Motor Age’ city. Radburn is unique because it was envisioned as a town for better
living, and it was the first example of city planning which recognized the
importance of the automobile in modern life without permitting it to dominate the
environment.
Philadelphia designed by Louis Kahn is based on Stop and Go Roads, with
emphasis on Parking tower.
Detroit is known as ‘Motor city’ because of huge number of automobile industry.

Q. 9 The minimum road curb length required for parking 10 cars perpendicular to road is
(A) 15 m (B) 25 m (C) 35 m (D) 40 m
Answer is B. (25 m)
For perpendicular parking road curb length required is 2.3 m per car. For 10 car it
comes 23 m. The closest answer is 25 m hence correct answer is
B. (Refer Architect’s Data, Neufert, page 248, 2nd ed)
Standard Parking area = 2.5 m × 5.4 m
No of cars that can be accommodated in a stretch of 400 m
400 m Perpendicular 60 º 45 º 30 º Parallel
stretch to kerb to kerb
parking
2.5 m × 160 cars 138 110 cars 80 74 cars
5.4 m cars cars

Q. 10 Which of the following generate heat islands?


(A) Urban Areas (B) Coastal Areas (C) Wet Lands (D) Forest Areas

Answer is A. (Urban Areas)


Urban areas generate heat while rest has cooling effect.

Q. 11 The most suitable earthquake resistant built form is


(A) (B)
(C) (D)
Answer : C.

Q.12. Transfer of Development Right (TDR) is a tool used for


(A)Human Development (B) Land Development (C) Economic Development (D)
Infrastructure Development
Answer is B. (Land Development)
What is Transfer of Development Right (TDR)?
Where any land within a local planning area is required by a planning authority for a
public purpose and the owner of any site or land which comprises such area
surrenders it free of cost and hands over possession of the same to the planning
authority free of encumbrances, the Government, permit development rights in the
form of additional floor area which shall be equal to one and half times of the area of
land surrendered (or as decided by authority). The development right so permitted may
be utilized either at the remaining portion of the area after the surrender or anywhere in
the local planning area, either by himself or by transfer to any other person, as may be
prescribed.
It deals with development of land, hence answer is B.

Q.13. Dandaka form of settlement layout is basically a


(A) Grid Iron Pattern (B) Ring Radial Pattern (C) Radial Pattern (D) Informal Pattern
Answer is A. (Grid Iron Pattern)

Q.14. Maximum horizontal angle from the speaker in a seating area of a lecture theatre
should be
(A) 70º (B) 90º (C) 120º (D) 140º
Answer : (D) 140º (Refer: Time Saver Standards for Building Types, 3rd ed. Page
274)
Viewing range of chalkboard: The maximum distance of a person from the chalk
board should not exceed 400 times the size of the smallest letter or digit being
written. Suppose the smallest letter is 2.5 cm the maximum distance of the student
should not exceed 400x2.5 = 1000cm i. e. 10m
Viewing range for projection screen: Distance from screen should not be less than
twice nor more than six times the width of screen image. Optimum viewing angle
should not be more than 30° from eye level. Suppose the screen image width is 1.5
m then the closest viewer should be at 3 m and farthest at 9 m.

Q.15. U- value refers to


(A) Utility functions for convenient heat transfer (B) Thermal transmittance of
building component
(C) Energy transfer between thermal bridges (D) Measure of area related
heating and cooling load
Answer is B. (Thermal transmittance of building component)
Thermal transmittance, also known as U-value is measured as the amount of heat lost
through a one square meter of the material for every degree difference in temperature
either side of the material. It is indicated in units of Watts per Meter Square per Degree
Kelvin W/m²K or W/m²degC.

Q.16 Consistency of cement is measured by


(A) Pycometer (B) Slump Cone (C) Universal Testing Machine(D) Vicat’s
Apparatus
Answer: D. (Vicat’s Apparatus)
A cement paste is said to be of normal consistency when a 300 gram, 10-mm-
diameter Vicat needle penetrates 10 + 1 mm below the surface in 30 seconds.
Slump cone test is done to measure workability of concrete.
Universal Testing Machine (UTM) measures compressive strength of
mortar/concrete blocks.

Q.17. The appropriate material for flooring of an external ramp of a building would be
(A) Polished Granite (B) Wax polished marble (C) Glazed Ceramic Tile (D) Rough
Finished sandstone.
Answer is D. (Rough Finished sandstone)

Q.18. Which of the following is not a member of a Steel Truss?


(A) Gusset Plate (B) Wall Plate (C) Fish Plate (D) Anchor Bolt
Answer is C. (Fish Plate)
Fish Plate is used to tie the rails of Railway track.

Q.19. Identify the odd one among the following


(A) Security Deposit (B) Professional Tax (C) Performance Bank Guarantee (D)
Earnest Money
Answer is B. (Professional Tax)
Except the Professional Tax all are paid in order to execute the work as per terms and
conditions.

Q.20. Weep hole is a term used to describe


(A) Perforations in cast iron pipe used for boring (B) Holes in retaining wall for
draining water
(C) Holes in cover plate of floor trap (D) Holes dug in earth to recharge ground water
Answer is B. (Holes in retaining wall for draining water)

Q.21. Busway, Busduct, and Raceway are component of


(A) Security Systems (B) Air Conditioning System (C) Electrical Systems (D) Water
Supply Systems
Answer: C (Electrical Systems)

Q.22. The difference between Wet Bulb temperature and dry bulb temperature is called
(A) Dry Bulb Depression (B) Wet Bulb Depression (C) Variable Depression (D)
Atmospheric depression
Answer is B. (Wet Bulb Depression)
Due to evaporative cooling Wet Bulb Thermometer always shows a lower temperature
than a Dry Bulb Thermometer, except at 100% RH.
The Atmospheric Depression refers an area of low barometric pressure.

Q.23. In India, one of the Slum Improvement initiatives is


(A) Special Residential Zone (B) Valmiki Ambedkar Malin Basti Awas Yojna
(C) Indira Awas Yojna (D) Eco Housing
Answer is B. (Valmiki Ambedkar Malin Basti Awas Yojna)
VAMBAY, (Valmiki Ambedkar Malin Basti Awas Yojna): Ministry of Urban Development,
Government of India has recently launched (VAMBAY), which will be operated through
HUDCO. The objective of the new scheme is to provide shelter and to upgrade existing
shelter for Below Poverty Line (BPL) families in urban areas. Malin Basti is the hindi
term for slum.

Q.24. Suspended floors is a structural system used in


(A) Lloyds Building, London (B) Jin Mao Building, Sanghai
(C) Petronas Tower, Kualalampur (D) HongKong Sanghai Bank, Hongkong
Answer: (D) HongKong Sanghai Bank, Hongkong
Lloyds Building London (1978-86) – Ar Richard Rogers
Jin Mao Building, Sanghai- Skidmore, Owings & Merrill
Petronas Tower, Kualalampur- Ar Cesar Pelli
HongKong Sanghai Bank, Hongkong- Norman Foster

Q.25. Residual method of valuation is used to determine


(A) Public Private Partnership deal (B) Rent (C) Property Tax (D) Selling
Price
Answer: (D) Selling price
Residual Land value= Final selling price of land – (purchase+ development cost of
land)
: with input from Anjali Pathak, CEPT University

Q 26 to Q 55 carries two marks each.

Q.26. Match the buildings in Group I with their Architects in Group II


Group I Group II
P. Bibliotheca Alexandria, Alexandria 1. I M Pei
Q. Institute de Monde 2. Jean Nouvel
R. Bank of China, Hongkong 3. Daniel Lebiskind
S. Jewish Museum, Berlin 4. Renzo Piano
5. Snohetta
(A) P-5, Q-2, R-1, S-4 (B) P-5, Q-4, R-1, S-3
(C) P-4, Q-2, R-5, S-3 (D) P-5, Q-2, R-1, S-3
Answer is D. (P-5, Q-2, R-1, S-3)
In 2004 Aga Khan Award of Architecture was won by the design team Snohetta, for
the project Bibliotheca Alexandria, also known as Library of Alexandria. Egypt.
Institute de Monde was designed by Jean Nouvel, recipient of Pritzker Prize in
2008.
Bank of China, Hongkong – I M Pei.
Jewish Museum, Berlin – Daniel Lebiskind

.27. A room measuring 5mx3.5 m enclosed by brick wall has a ceiling of 3 m height. The
room has a door and window opening of 1 m x 2 m and 1 m x1 m respectively. The
quantity of plastering required in the interior wall (in sqm) is
(A) 46.5 m (B) 48 (C) 51 (D) 68.5
Answer is B (48 m²)

Q.28. One cubic meter of ordinary cement yields a volume of M15 concrete in the range of
(A) 2-3 cum (B) 4-5 cum (C) 7-8 cum (D) 8-9 cum
Answer (B) 4-5 cum
The ratio of M 15 concrete is 1:2:4 (cement: sand: stone chips). As such if one cubic
meter of ordinary cement is taken we need 2 cu m of stone chips and 4 cu m of sand
along with water to make concrete adding to final concrete less than 7 cum. (Vol of
concrete reduces by a factor of 1.54).

So acual vol will be = 4.54


M 15= 1:2:4
M 20= 1: 1.5: 3
M 25= 1: 1: 2
M 15 means compressive strength of concrete after 28 days = 15 N/mm².
Note: One Cubic metre of cement= 1500 kg of cement= 30 bag of 50 kg cement each.

Q.29.Match the CAD command in Group I with their functions in Group II


Group I Group II
P. LAYISO 1. Blends selected object to destination layer
Q. LAYMCH 2. Freezes layer of selected object
R. LAYMRG 3. Hides or locks layers other than those of selected objects
S. LAYLCK 4. Assign selected object to destination layer
5. locks object of destination layer
(A) P-2, Q-4, R-1, S- 5 (B) P-3, Q-2, R-1, S- 5
(C) P-4, Q-2, R-3, S- 5 (D) P-3, Q-4, R-1, S- 5
Answer: No appropriate answer

.30. Match the buildings in Group I with their corresponding structural forms in Group II
Group I Group II
P. Hall of Nations, New Delhi 1. Spherical Structure
Q. Salvacao Church, Mumbai 2. Folded Plates
R. State Trading Corporation Building, New Delhi 3. Octahedral Lattice Structure
S. Matri Mandir, Auroville 4. Vierendeel Girders
5. Shell Roof Structure
(A) P-3, Q-5, R-4, S-1 (B) P-2, Q-5, R-4, S-1
(C) P-3, Q-5, R-4, S-2 (D) P-3, Q-5, R-2, S-1
Answer is A. (P-3, Q-5, R-4, S-1)
Hall of Nations (1972) – commonly known as Pragati Maidan is exhibition space in
New Delhi. It is the most famous project of Ar Raj Rewal incorporating Octahedral
Lattice Structure. The depth of the structural system is utilized as a Sun breaker
and conceived of in terms of the traditional ‘jali’, a geometrical pattern of perforation
that serves to obstruct directs rays of the harsh Sun while permitting air circulation.
Salvacao Church, Mumbai (1977) –Designed by Charles Correa. The shell roof
structure act as giant flues where the hot air rises and exits through a vent at the top,
thus drawing in fresh air from the courtyards around.
The Vierendeel girder is a truss where the members are not triangulated but form
rectangular openings. This girder is used in State Trading Corporation Building,
New Delhi designed by Raj Rewal in 1989.
Matri Mandir, Auroville having spherical structure was conceptualized by Mirra
Alfassa, (1878 –1973), also known as The Mother, was the spiritual collaborator of
Sri Aurobindo. When Sri Aurobindo retired into seclusion, she founded his ashram
(Sri Aurobindo Ashram), with a handful of disciples living around the Master. She
along with Ar Roger Anger founded the Auroville.

Q.31. Identify the incorrect statement.


(A) Guggenheim, Bilbao is an example of deconstructivism.
(B) Silver Abstraction is a term used for metal clad modern high rise building.
(C) Spiral Building in Tokyo has a curvilinear built form.
(D) Free building plan form is a concept given by Le Corbusier.
Answer: Marks to all
Guggenheim, Bilbao, Spain – Architect – Frank Gehry. Other Guggenheim Museum
is in New York, by F L Wright. Guggenheim, Bilbao is an example of
Deconstructivism.
Features of Deconstructivism.
(a) Fragmentation, geometrical blocks is fused together.
(b) Non-Rectilinear shapes, eg. Tilted cuboids
(c) Manipulating surface eg. Metallic cladding
The attempt in deconstructivism throughout is to move architecture away from what
its practitioners see as the constricting ‘rules’ of modernism such as “form follows
function,” “purity of form,” and “truth to materials.” Frank Gehry, Daniel Libeskind,
Rem Koolhaas, Peter Eisenman, Zaha Hadid, Coop Himmelbau, and Bernard
Tschumi practice deconstructivism.
Spiral Building in Tokyo – Ar Fumihiko Maki. This building houses gallery space,
multipurpose hall, cafe, restaurant and bar, salon, and shops. Spiral is a nexus of
cultural life in Aoyama, presenting music,
art, film, and theater events. Aoyama is in Tokyo, Japan. The main feature of the
building is a floating spiral ramp (15m in dia) that encircles the rear gallery space and
climbs to the second floor.
Five points of Architecture, Le Corbusier.
1. Building supported by pilotis – reinforced concrete stilts.
2. Free facade
3. An open floor plan (Free Building plan)
4. Long strips of ribbon windows
5. Roof Garden.

Q.32. Match the terms in Group I with their description in Group II


Group I Group II
P. Quoin 1. Geometric description of Universe.
Q. Stucco 2. Small Dome
R. Mandala 3. Triangular form above an opening
S. Cupola 4. Corner Stone at the angle of Buildings
5. Plaster
(A) P-4, Q-3, R-2, S-1 (B) P-3, Q-5, R-1, S-4
(C) P-4, Q-5, R-1, S-2 (D) P-3, Q-1, R-5, S-4
Answer is C. (P-4, Q-5, R-1, S-2)
Quoin-Cornerstone of brick or stone wall generally provided at corner to give
impression of firmness.
Stucco is decorative plaster.
Mandala is the universe
Cupola a small dome often known as chhatri in Hindu Architecture.

Q.33. Match the Architectural style in Group I with their features in Group II
Group I Group II
P. West Asiatic 1. Arches and Pendentives.
Q. Greeks 2. Pagodas
R. Byzantine 3. Flying Buttresses
S. Japanese 4. Orders and Pediments
5. Hanging Gardens
(A) P-3, Q-2, R-1, S-4 (B) P-5, Q-4, R-1, S-2
(C) P-5, Q-4, R-1, S-3 (D) P-4, Q-3, R-5, S-2
Answer is B. (P-5, Q-4, R-1, S-2)
West Asiatic Civilization: (Ancient Mesopotamia: The Sumerians, Babylonians, and
Assyrians) Ziggurat and Hanging Gardens of Babylon are most famous.
Greek and Roman- The feature was classical Architecture with Orders, Entablature
and Pediment giving a complete building form.
Byzantine:period when Hagia Sophia was built.
Squinch and Pendentive: A squinch in architecture is a construction filling in the
upper angles of a square room so as to form a base to receive an octagonal or
spherical dome. A later solution of this structural problem was provided by the
pendentive. Pendentives, which are triangular segments of a sphere, taper to points
at the bottom and spread at the top to establish the continuous circular or elliptical
base needed for the dome.

Q.34.Gestalt’s law of Visual Perception Do Not relate to


(A) Aesthetics of form are a function of Golden Section.
(B) Things are perceived as a whole.
(C) Whole is greater than the sum total of its parts.
(D) Elements with continuity are perceived together.
Answer: A. Aesthetics of form are a function of Golden Section.
The gestalt effect is the form-generating capability of our senses, particularly with
respect to the visual recognition of figures and whole forms instead of just a collection
of simple lines and curves.
Law of Proximity- Closer objects are perceived as groups.
Law of Symmetry- Objects must be balanced or symmetrical to be perceived
as a whole.
Law of Similarity- Similar objects are more likely to be organized together.
Law of common fate- Objects with common movement, moving in same
direction with same pace, at the same time are organized as group.

Q.35. A site in a map drawn to a scale of 1: 16000 measures 75sqcm. The actual area of
the site is
(A) 120 (B) 162 (C) 192 (D) 256
Answer: C. (192)
Soln: Area on the map is= 75sqcm.
Suppose it is a square then each side will be √75= 8.66 cm. on map.
Actual length of the square= 8.66x16000= 138564 sqcm
So the actual area of square= 138564x138564=192 x108 sqcm.
1 Hectare = 100x100 sqm = 10000x10000 sqcm = 108 sqcm
10 sqcm= 1 hectare 192 x10 sqcm= 192 hectare.
8 8

Q.36. Identify the CORRECT CAD Statements


P. SPLINE connects sequence of line segments into a single object
Q. SPLINE is a smooth curve passing through or near a given set of points.
R. PLINE creates straight line segment, Arc segment or both.
S. PLINE can be closed only when its start and end points are coincident and
tangent.
T. PLINE allows adjusting the width and curvature of its multiline segment
U. SPLINE can be exploded into smaller segments.
V. PLINE can be converted into a continuous curve segment.
(A) P, R, S, U (B) Q, R, T, V (C) R, S, T, V (D) S, T, U, V
Answer: B

Q.37. Match the eminent personalities in GROUP I with their books and Statements in
GROUP II
GROUP I GROUP II
P. Kevin Lynch 1. The Fountainhead
Q. Ayn Rand 2. Small is Beautiful
R. Paul D. Spreiregen 3. Site Planning
S. E F Schumacher 4. Urban Design: Architecture of Town and Cities
5. Design of Cities
(A) P-4, Q-2, R-5, S-3 (B) P-3, Q-1, R-2, S-5
(C) P-5, Q-1, R-4, S-2 (D) P-3, Q-1, R-4, S-2
Answer: (D) P-3, Q-1, R-4, S-2
Less is more – Meis van der Rohe
Less is bore- Robert Venturi
Small is beautiful- E F Schumacher

Q.38. Match the Urban form listed in Group I with the towns listed in Group II
Group I Group II
P. Grid Iron 1. New Delhi
Q. Radial 2. Washington D.C.
R. Linear 3. Copenhagen
S. Finger Plan 4. Mumbai
5. Canberra
(A) P-2, Q-1, R-4, S-3 (B) P-3, Q-1, R-2, S-5
(C) P-3, Q-1, R-4, S-2 (D) P-2, Q-1, R-4, S-5
Answer: (A) P-2, Q-1, R-4, S-3
The Finger Plan is an urban plan from 1947 which provides a strategy for the
development of the Copenhagen metropolitan area, Denmark. According to
the plan, Copenhagen is to develop along five ‘fingers’, centred on S-train
commuter rail lines, which extend from the ‘palm’, that is the dense urban
fabric of central Copenhagen. In between the fingers, green wedges are
supposed to provide land for agriculture and recreational purposes.

Q.39. Consider the following features


1. Length finely proportioned to its width.
2. Statues as silhouettes against the sky above cornice lines.
3. Fountains signifying fine vintage points
4. Series of different shapes connected by traditional narrow streets, column
screen or arches.
(A) Vista (B) Piazza (C) Rond Point (D) Bosque
Answer: B. Piazza
Vista: A distant view or prospect, especially one seen through an opening, as
between rows of buildings or trees. An avenue or other passage affording such a
view.
Piazza: an open public square, especially in Italian towns, surrounded by
buildings, and usually the center of public life. In other words, it’s still defined by
its historic heritage as a pedestrian-oriented, often pedestrian-only destination,
designed to be the heart of a neighborhood or town, filled with cafes, restaurants
and bars and host to a series of local events and public markets.
Plaza: A public space at the intersection of important streets set aside for civic
purposes and commercial activities. A plaza is circumscribed by frontages; its
landscape consists of durable pavement for parking and trees requiring little
maintenance. All parking lots on frontages should be designed as plazas with
the paving not marked or detailed as parking lots.
Rond Point: Round about in France.
Bosque: Bosque is the name for areas of gallery forest found along the
riparian flood plains of stream and river banks in the southwestern United
States.
Vintage Point: A position that affords a broad overall view or perspective, as of
a place or
situation.

Q.40. Match the instruments in Group I with their corresponding functions in Group
II
Group I Group II
P. Hygrometer 1. Precipitation
Q. Disdrometer 2. Vapour Pressure
R. Anemometer 3. Solar Radiation
S. Manometer 4. Relative Humidity
5. Velocity of Air
(A) P-4, Q-1, R-2, S-3 (B) P-4, Q-3, R-2, S-5
(C) P-1, Q-2, R-5, S-4 (D) P-4, Q-1, R-5, S-2
Answer is D (P-4, Q-1, R-5, S-2)
Thermometers are installed in Stevenson Screen, a wooden louvered box kept at a
height of 1.2-1.8m above ground.
Q.41. Match the features in Group I with the corresponding types of garden in Group
II
Group I Group II
P. Symmetrical layout, water cascades, entombment 1. French gardens
Q. Radial Layout, symmetrical sculpture, boulevards 2. English gardens
R. Occult Symmetry, pontoon bridges, stepping stones 3. Chinese gardens
S. Hierarchy of Courts, hierarchy of gates, zoomorphic forms 4. Mughal gardens
5. Japanese gardens

(A) P-2, Q-1, R-4, S-3 (B) P-4, Q-1, R-5, S-3
(C) P-4, Q-3, R-5, S-1 (D) P-5, Q-1, R-2, S-3
Answer is B. (P-4, Q-1, R-5, S-3)
The occult is “knowledge of the hidden”.
In occult symmetry an optical axis or centre of gravity is implied and opposing
elements may be symmetrical or assymetrical. An example of occult balance would
be trees appearing to balance a hill on an applied visual axis.

Q.42. Arrange the following sense of enclosures in a hierarchy of decreasing order.

(A) S>Q>U>P>T>R (B) U>S>Q>R>P>T (C) P>Q>R>S>T>U (D) T>P>S>Q>U>R


Answer: (B) U>S>Q>R>P>T

Q.43. Match the elements of Group I with their corresponding type in Group II
Group I Group II
P. Fire Hydrant 1. Street Furniture
Q. Planter Beds 2. Street Hardware
R. Letter Box
S. Traffic Signs
T. Lamp Posts
(A) P-2, Q-1, R-1, S-2, T-2 (B) P-1, Q-1, R-2, S-1, T-1
(C) P-1, Q-1, R-2, S-2, T-2 (D) P-2, Q-1, R-2, S-2, T-2
Answer: A

.44. In a Construction project Schedule A is the first activity. Activities B and C follow A.
Activity D follows B & C. Activity E Follows C. Activity F Follows D & E.
Activity A B C D E F
Duration
(in 3 2 5 6 5 3
Days)
The critical time to complete the project will be
(A) 14 days (B) 16 days (C) 17 days (D) 20 days
Answer: (C) 17 days

5. The maintenance cost of a building will be Rs 2 lacs after 10 years. The annual sinking
fund required for such maintenance @ rate of 6 % interest per annum will be
(A) Rs 17,200/- (B) Rs 15,200/- (C) Rs 13,200/- (D) Rs 11,200/-
Answer: (B) Rs 15,200/-

R= where R= annual fund required to deposit


i= rate of interest
n= no of installments

R=

= = = 15189

Q. 46. Match the figures in Group I with the fixtures in Group II

Group II
1. Sink Cock 2. Bib Cock 3. Pillar Cock 4. Stop Cock
(A) P-1, Q-4, R-2, S-3 (B) P-2, Q-3, R-1, S-4
(C) P-3, Q-1, R-2, S-4 (D) P-2, Q-4, R-3, S-1
Answer: (A)

Q.47. Match the joints in Group I with the corresponding figures in Group II
Group I
P. Butt Joint Q. Rebated JointR. Table Joint S. Tongue and Groove Joint

(A) P-3, Q-4, R-1, S-2 (B) P-4, Q-1, R-3, S-2
(C) P-3, Q-1, R-2, S-4 (D) P-3, Q-4, R-2, S-1

Answer A. (P-3, Q-4, R-1, S-2)


Butt joint is the weakest joint and remains on glue.
Rebate is a recess or groove cut into the edge of a piece of wood.
Common Data Questions
Common Data for Questions 48 & 49:
A Beam of span L is simply supported at two ends. One half span of the beam weighs W
and the remaining half span weighs 2W.

Q.48 Maximum shear force in the Beam will be


(A) W (B) 1.25 W (C) 1.75 W (D) 3 W
Answer: C. (1.75 W)

Q.49. Maximum Bending Moment will occur at


(A) L/16 from midpoint of beam (B) Midpoint of the beam
(C) L/7 from midpoint of beam (D) One of the endpoints of the beam.
Answer: (A) L/16 from midpoint of beam

Common Data for Questions 50 & 51:


A building site has a plot of 500 sqm.
Maximum allowable height- G+7 Area to be utilized for paved access road- 10%
Maximum Ground Coverage- 40% Runoff co-efficient for paved surface- 0.9
Maximum allowable FAR- 2 Runoff co-efficient for unpaved surface- 0.3

Q.50. If maximum allowable FAR is utilized, the minimum ground coverage would be
(A) 20% (B) 25% (C) 30% (D) 35%
Answer: B. (25%)
Plot size = 500 sqm. And Max FAR= 2 so total built up area= 500× 2= 1000 sqm.

1000 sqm is divided on G+7 i.e. 8 floors so ground coverage = = 125 sqm. This is
the minimum ground coverage that can be achieved or else as given in Question max

ground cover= × 500= 200 sqm.

Therefore Min ground coverage = × 100 = 25%

Q. 51 If it rains for 30 min, with an intensity of 10 cm/ hour, minimum volume of rain water
that can be collected will be
(A) 12.75 cum (B) 14 cum (C) 15 cum (D) 16 cum
Answer: A. (12.75 cum)
Total open area = 500 sqm- 125 sqm= 375 sqm and Road area = 50 sqm.

Total unpaved surface = Deduct 10% for road = 375- ( × 500)= 325 sqm.
Total rainfall = area of surface × total rainfall × run off co- efficient
(open ground , road and roof top area is considered)

( 325 × 0.1 x × 0.3 ) + ( 50 × 0.1 x × 0.9 ) ( 125 × 0.1 x × 0.9)


= 4.875 + 2.250 + 5.625= 12.75 cum

Linked Answer Questions


Statement for Linked Answer Questions 52 and 53:
An auditorium having volume of 4500 cum and total absorption of all acoustic material
is 480m² sabine.
Q.52 The reverberation time of the auditorium is
(A) 1.0 sec (B) 1.5 sec (C) 2.0 sec (D) 2.5 sec
Answer is B. (1.5 sec)
Soln: A standard reverberation time has been defined as the time for the sound to die
away to a level 60 decibels below its original level.
Where RT60 = Reverberation time in sec
V = Volume in m³
Se = Total Absorption

= 1.5 sec
Q. 53 To reduce reverberation time by 0.5 second, additional absorption (m² sabine)
required would be
(A) 120 (B) 160 (C) 240 (D) 720
Answer is C. (240)
Here RT60 = 1 sec

Se = 0.16 × 4500= 720


Additional absorption required = 720- 480= 240 m² sabine

Statement for Linked Answer Questions 54 and 55:


A Residential sector planned over an area of 100 hectares has been divided into
various plots, each having one dwelling unit with an average household size of 5
persons. Remaining area is devoted for schools, roads, parks, shops
etc.
Plot Size No
500 sqm 500
300 sqm 500
200 sqm 1000
Q. 54 The gross density of the residential sector in person per hectare would be
(A) 100 (B) 150 (C) 200 (D) 250
Answer is A. (100)

Q. 55 Assuming 20% of total population being higher secondary school going children and
expected enrolment being 80 % with per capita floor space
requirement of 5.0 sqm, then minimum land required for school
building with 40% ground coverage and FAR 0.5 would be
(A) 1.0 hectare(B) 1.6 hectare (C) 2.2 hectare (D) 2.8 hectare
Answer is B. (1.6 hectare)

General Aptitude (GA) Questions


Q 56- Q60 carry one marks each.

Q.56 Choose the words from the options given below that is most nearly opposite in meaning
to the given word:
Amalgamate
(A) merge (B) split (C) collect (D) separate
Answer: (D) separate

Q.57 Choose the most appropriate word from the options given below to complete the
following sentence.
If you are trying to make a strong impression on your audience, you cannot do
so by being understated, tentative or _______________.
(A) hyperbolic (B) restrained (C) (D) indifferent
Answer : (B) restrained
Q.58 Choose the most appropriate word(s) from the options given below to complete the
following sentence.
I contemplated _______________ Singapore for my vacation but decided against it.
(A) to visit (B) having to visit (C) visiting (D) for a visit
Answer : (C) visiting

Q. 59 If Log (P) = (1/2) Log (Q) = 1/3 Log (R) then which of the following options is TRUE.
(A) P²= Q3 R2 (B) Q2=PR (C) Q2=R3P (D) R=P2Q2
Answer : (B) Q =PR
2

Q.60 Which of the following options is the closest in meaning to the word below:
Inexplicable
(A) Incomprehensible (B) Indelible (C) Inextricable (D) Infallible
Answer: A. (Incomprehensible)

Q 61- Q 65 carry two marks each.


. 61. A container originally contains 10 litres of pure spirit. From this container 1 litre of spirit is
replaced with 1 litre of water. Subsequently, one litre of the mixture is again replaced
with 1 litre of water and this process is repeated one more time. How much spirit is
now left in the container?
(A) 7.58 litre (B) 7.84 litre (C) 7 litre (D) 7.29 litre
Answer: D (7.29 litre)

Soln: 10 × × × = = 7.29 litre

. 62 A transporter receives the same number of order each day. Currently he has some
pending orders (backlog) to be shipped. If he uses 7 trucks, then at the end of the 4th
day he can clear all the orders. Alternatively, if he uses only 3 trucks, then all
the orders are cleared at the end of the 10th day. What is the minimum numbers of
trucks required so that there will be no pending orders at the end of the fifth day?
(A) 4 (B) 5 (C) 6 (D) 7
Answer: (C) 6

. 63 The variable cost (V) of manufacturing a product varies according to the equation V= 4q,
where q is the quantity produced. The fixed cost (F) of production of same
product reduces with q according to the equation F = 100/q. How many units should be
produced to minimize the total cost (V+F)?
(A) 5 (B) 4 (C) 7 (D) 6
Answer: (A) 5

. 64 P, Q, R, and S are four types of dangerous microbes recently found in a human habitat.
The area of each circle with its diameter printed in brackets represents the growth of
single microbe surviving human immunity system within 24 hours of entering the body.
The danger on human beings varies proportionately with the toxicity, potency and
growth attributed to a microbe shown in figure below.
A pharmaceutical company is contemplating the development of a vaccine against the
most dangerous microbe. Which microbe should the
company target in its first attempt?
(A) P (B) Q (C) R (D) S
Answer: (D) S

. 65 Few School curricula include a unit on how to deal with bereavement and grief,
and yet all students at some point in their lives suffer from losses through death
and parting.
Based on the above passage which topic would not be included in a unit on bereavement?
(A) how to write a letter of condolence
(B) what emotional stages are passed through in the healing process
(C) what the leading causes of death are
(D) how to give support to a grieving friend

Answer: (C) what the leading causes of death are.


GATE 2012
Q. 1 ‘Agora’ was provided in Greek towns as a place of
(A) Worship (B) Drama (C) Sports (D) Meeting
Answer: D (Meeting)
Agora was the meeting place in Greek towns while Forum was the market place in
Rome. Later on Agora also served as a market place.
Stoa consisted of two rows of columns supporting a roof with a wall on one side.
Stoas were both ornamental and practical. They served as promenades sheltered
from the heat of summer and the cold winds of winter, as judicial and shopping
centers, and as boundary markers.

Q. 2 The hue at the centre of the Munsell Colour Solid is


(A) Black (B) Grey (C) Sepia (D) White

Answer: (B) Grey


(Actual Question would be “The Value at the centre of the Munsell Colour Solid is”
Answer: Grey
Munsell developed three dimensional colour concepts, depicting hue, value and chroma.
Munsel five Principle hues are red, yellow, green, blue, and purple.
Munsel Colour representation: eg. 5.3R6.1/14. Here 5.3R is Hue representing a value of
5.3 on Hue Band (0-10) for Red. 6.1/ represents value i.e. brightness and 14 is the
purity i.e. chroma. (Refer page 136. Manual of Tropical Housing and Building,
Koenigsberger)
The first circular colour diagram was designed by Newton in 1666. It is two
dimensional colour scheme depicting variables of hue. Colour wheel is
helpful in suggesting colour scheme. Six prominent colour schemes are
a. Complementary- exactly opposite
b. Analogous - neighbour
c. Split complimentary- neighbour of complementary colours on colour wheel
d. Tetradic- four colors arranged into two complementary pairs.
e. Square – all four colours spaced evenly.
Hue- 1st Dimension. By definition it is pure colour, containing no white, black or grey. Hue
is displayed on Horizontal ring in Munsell Colour solid. There are ten hues. Five
principle hues are Red, Yellow, Green, Blue, Purple and five others are Y-R, G-Y, B-G,
P-B, R-P.
Value: 2nd Dimension. Degree of colous luminosity i.e. brightness. Displayed on vertical
bar, ranging from 0 (black) at bottom to 10 (white) at top. 5 is grey.
Chroma: 3rd Dimension. Measure of quality of relative colourfulness or greyness. Also
known as intensity, purity or saturation. It starts from 0 and there is no
intrinsic upper limit to chroma.

Eg : 10 PB 7/12
Here PB represents Purple- Blue
Number 10 before PB represents purity….. it could be 2.5PB, 5PB (purest) ,7.5PB or
10PB.

CIELAB and
CIECAM02 are other colour models.
Q. 3 Which one of the following is NOT a traffic calming measure?
(A) Rumble strips (B) Roundabouts (C) Pedestrian crossings (D) Roadside trees

Answer: D (Road side Trees)


Rumble strips, also known as sleeper lines, audible lines, and growlers are a
road safety feature that alert inattentive drivers to potential danger by causing a
tactile vibration and audible rumbling, transmitted through the wheels into the
vehicle body. Speed breakers, blinkers are commonly used in India.

Q. 4 ECBC stands for


(A) Electrical Conduit in Building Construction (B) Energy Conservation Building Code
(C) Electrical Credit in Building Code (D) Energy Credit in Building Construction

Answer: (B) Energy Conservation Building Code


The Indian Parliament passed the Energy Conservation Act 2001, in the year 2001. ECBC
(Energy Conservation Building Code) was launched in 2007 by Ministry of
Power, Govt. of India. The building code has provisions for:
(a) Building envelopes, except for unconditioned storage spaces or warehouses,
(b) Mechanical systems and equipment, including heating, ventilating, and air
conditioning,
(c) Service hot water heating,
(d) Interior and exterior lighting, and
(e) Electrical power and motors.

Q. 5 Age-Sex cohort for a state in India is obtained from


(A) Census of India (B) Election Commission of India
(C) Indian Statistical Institute (D) Survey of India
Answer: A (Census of India)

Q. 6 ‘Cover block’ is used as a building


construction component in
(A) Brick wall (B) Curtain wall
(C) Steel truss (D) RC beam
Answer is D. (RC beam)
Cover block is placed below the steel (Reinforcement) so that steel is not exposed to
outer surface and cover is provided to the reinforcement.

Q. 7 ‘Villa Savoye’, Paris is an example of


(A) Modernism (B) Post Modernism
(C) Deconstructivism (D) Eclecticism
Answer: A (Modernism)
The Villa Savoye in Paris is among Le Corbusier’s most famous buildings. It is based
on his “Five Points” of Architecture Design and started the era of Modernism in
Architecture.
Post Modernist architects are Philip Johnson, Robert Venturi.
Deconstructivism ws propogated by Frank Gehry, Zaha Hadid, Lebiskind.
Eclecticism: the combination, in a single work, of elements from different historical
styles.
Le Corbusier’s “Five Points” of Architecture.
1. Support of ground-level pilotis, elevating the
building from the earth. This allowed an extended continuity of the garden beneath.
2. Functional roof, serving as a garden and terrace, reclaiming for nature the land
occupied by the building. (Roof Gardening)
3. Free floor plan, relieved of load-bearing walls, allowing walls to be placed freely and
only where aesthetically needed.
4. Long horizontal windows, providing illumination and ventilation.
5. Freely-designed facades, serving only as a skin of the wall and windows and
unconstrained by load-bearing considerations.
Modernism in Architecture is a period from 1900-1970. Notable architects of modernist
movement were Frank Lloyd Wright (1867-1959), Ludwig Mies van der Rohe (1886-
1969), Le Corbusier (1887-1965), Oscar Niemeyer (1907-2012 ), Alvar Aalto (1898-
1976), Walter Gropius (1883-1969) and Louis I Kahn (1901-1974).

Q. 8 The least important measure for reducing cost of site development is


(A) Clustering the units (B) Eliminating landscaping costs
(C) Reducing road lengths (D) Narrowing road widths without reducing the right of
way
All the options are correct. (Marks were awarded to all the students)

Q. 9 The role of a plasticizer in concrete is to improve


(A) Compressive strength (B) Permeability (C) Workability (D) Tensile
strength
Answer is C. (Workability)

Q.10 Which one of the following causes seismic irregularity in a building?


(A) Rectangular plan shape (B) Vertical setback (C) Increase in height (D)
Seismic joint
Answer is B. (Vertical setback)

.11 The respective lengths of shadows generated for a free standing wall of given
height L by sunlight incident at angles of 30°, 45°, 60°, 90° to the horizontal are

(A) L √3, L, L/√3, 0 (B) L, L √3, /√3, 0


(C) L/√3, L, L√3, 0 (D) 0, L/√3, L, L √3
Answer is A. (L √3, L, L/√3, 0)

Q. 12 Which one of the following mode is NOT categorized as a public transit?


(A) Bus (B) Ferry (C) Taxi (D) Tram
Answer is C. (Taxi)
In public transit, people share the system by strangers. By definition taxicab, car
pooling or hired buses are not categorized as a public transit.

Q. 13 Aerial photography is a useful tool to obtain


(A) Land contour data (B) Land cover data (C) Land ownership data (D) Land
use data
Answer is B. (Land cover data)

Q. 14 Which one of the following is biodegradable?


(A) Detergent (B) Leather (C) Recycled plastic (D) Aluminum foil
Answer is B. (Leather)

Q.15 The volume of surface runoff is least influenced by


(A) existing storm water drainage system (B) amount of rainfall
(C) site slope (D) size of watershed
Answer: None of the option is suitable (Marks awarded to all candidates)

Q. 16 The best location for laying the main sewer line on a flat land is
(A) under the road (B) under the sidewalk
(C) under the central verge (D) under the open space along sidewalk
Answer is A. (under the road)

Q. 17 Ponding is associated with


(A) RC column (B) Steel column (C) RC slab (D) Steel truss
Answer is C. (RC Slab)
Ponding is the process of curing the concrete by creating pool of water above
horizontal concrete slab. Other methods of curing are fogging, spraying and saturated
wet covering, using jute or cotton fabric.

Q.18 Among the following, the urban open space known for its human scale is
(A) Piazza del Campo, Sienna (B) Piazza del Popolo, Rome
(C) St. Peter’s Square, Rome (D) Place de la Concorde, Paris
Answer is A. (Piazza del Campo, Sienna)

Q. 19 The most appropriate tree for designing a smell sensory pathway is


(A) Delonix Regia (B) Casia Fistula (C) Bougainvillea (D) Jasminum
Augustifolium
Answer is D. (Jasminum Augustifolium)
Jasmine (chameli) has strong odour. Delonix Regia is Gulmohar. Castia Fistula is
Amaltas. Bougainvillea is odourless.

Q.20 Load from a slab to beam is primarily transferred through


(A) axial force (B) bending moment (C) shearing force (D) torsion
No appropriate answer. (Marks were awarded to all the candidates)

Q.21 Geographic Information System (GIS) combines maps with


(A) computer automation, statistics and topology
(B) computer graphics, databases and analytical tools
(C) computer graphics, informatics and quantitative tools
(D) computer informatics, databases and qualitative tools
Answer is B. (computer graphics, databases and analytical tools)

Q.22 ‘Glazing stop’ is used


(A) as a bearing support for glass (B) to seal the glass against water and air
infiltration
(C) to provide lateral support to glass (D) to provide a cushion between the glass
and the glazing pocket
Answer is C. (to provide lateral support to glass)

Q.23 Which one of the following is NOT a basic principle of designing people-friendly
urban built environment?
(A) Diversity (B) Monotony (C) Adaptability (D) Legibility
Answer is B. (Monotony)

Q.24 SWOT analysis is used for examining a situation’s inherent


(A) Strength, Wastefulness, Opportunity and Tactfulness
(B) Sanction, Weakness, Ownership and Threat
(C) Strength, Weakness, Opportunity and Threat
(D) Support, Wastefulness, Ownership and Transparency
Answer is C. (Strength, Weakness, Opportunity and Threat)

Q.25 The deflection of a two way slab is primarily a function


(A) of the long span (B) of the short span
(C) mostly of the long span, sometimes short span (D) independent of the long or short
span
Answer: No appropriate answer. (Marks were awarded to all the candidates)

Q. 26 to Q. 55 carry two marks each.


Q.26 A painting in a living room needs to be lighted at 250 lux. If the light is incident
normally on the painting from a distance of 6 m, then the intensity of light source
required (in candela) is
(A) 1500 (B) 3000 (C) 6000 (D) 9000
Answer is D. (9000)
Solution: Lux= Intensity (candela) /distance²
250= Intensity /6²
250= Intensity /36
Intensity = 250x36
= 9000 Candela

Q.27 Total land area for a plotted housing development project is 50.0 hectare, of which,
area under community facilities, common open spaces and roads are 9%,
10 %, and 16 % respectively. Total number of saleable plots of 250 m² each will
be
(A) 1100 (B) 1300 (C) 1500 (D) 1700
Answer : (B) 1300
Solution: Total land area = 50.0 hectare= 50x10,000 m²= 5,00,000 m²
Area under community facilities, common open spaces and roads= 9+10+16= 35 %
35 % of 5,00,000 m²= 1,75,000 m²
Area left for saleble plot = 5,00,000 - 1,75,–00 =3,25,000 m²
Total number of saleable plots of 250 m²= 3,25,000/250= 1300

Q.28 A 6m long beam is fixed at its left end and is free at its right end. If a
concentrated load of 25kN acts downwards at 4m from the left end, then the
bending moment (in kNm) at the left end is
(A) 0.0 (B) 25.0 (C) 50.0 (D) 100.0
Answer is D. (100.0)

Solution: For concentrated load BM= load x distance= 25x4= 100 kNm.
Q.29 A classroom of 12m x 25m x 4m requires 3 air changes per hour. At an air velocity
of 2 m/s, the necessary duct cross section (in m²) is
(A) 0.50 (B) 0.75 (C) 1.00 (D) 1.25
Answer is A. (0.50)
Soln: Classroom needs 3 air changes per hour i.e. Air change every 20 min.
Volume of air required every 20 min= 12m x 25m x 4m= 1200 m³
Let the cross section of duct be ‘y’ m²
Since air flow is 2 m/s, Volume of air flow will be 2y m³/s
In 20 min total volume of air flow will be 2y 60 20=2400y m³
2400y m³ =1200 m³
y = 0.50 m²

Q. 30 If the original cost of a building is Rs. 25,00,000/- and its scrap value after 55
years is Rs.25,000/-, then its annual depreciation (in Rs.) using straight line
method is
(A) 25,000 (B) 35,000 (C) 45,000 (D) 55,000
Answer is C. (45,000)
Solution: Original cost of Building = Rs 25,00,000/-
Scrap value( 55 Years) = Rs 25,000/-
Total Depreciation in 55 Years= 25,00,000- 25,000= Rs 24,75,000/-

Annual Depreciation = = Rs 45,000/-

Q.31 For a pin jointed steel truss system, which of the following statements is TRUE?
(A) Bending moment resisting capacity at any section is zero as members carry axial
forces only.
(B) Forces in members at any section align in appropriate combination of tension and
compression to develop moment resisting capacity.
(C) Forces in members at any section align in such a manner as to develop zero
moment resisting capacity.
(D) Shear resisting capacity at any section is zero as members carry axial forces only
Answer B. (Forces in members at any section align in appropriate combination of
tension and compression to develop moment resisting capacity.)

Q. 32 Match the books in Group I with their authors in Group II


Group I Group II

P. Space, Time and Architecture 1. Jane Jacob


Q. Form, Space and Order 2. Kevin Lynch
R. The Death and Life of Great 3. Ian McHarg
American Cities
S. The Image of the City 4. Francis D K
Ching
5. Gideon S
(A) P-5, Q-4, R-3, S-2 (B) P-5, Q-4, R-1, S-2
(C) P-4, Q-3, R-2, S-1 (D) P-4, Q-1, R-3, S-5
Answer : B. (P-5, Q-4, R-1, S-2)

Q.33 Match the schemes in Group I with their specific targets in Group II
Group I Group II
P. JNNURM 1. Urban Amenities for
Rural Areas
Q. IAY 2. Infrastructure and Slum
Upgradation
R. PURA 3. Rural Employment
S. NREGA 4. Land Acquisition
5. Housing for BPL
families
(A) P-2, Q-5, R-1, S-3 (B) P-5, Q-2, R-1, S-3
(C) P-5, Q-2, R-3, S-4 (D) P-2, Q-5, R-3, S-4
Answer : A. (P-2, Q-5, R-1, S-3)
JNNURM- Jawahar Lal Nehru National Urban Renewal Mission, aims at creating
‘economically productive, efficient, equitable and responsive Cities’ by a strategy of
upgrading the social and economic infrastructure in cities. This project runs under
Ministry of Urban Development, Govt. of India.
IAY – Indira Awas Yojna runs under Ministry of Rural Development, Govt. of India. The
financial assistance provided for new construction in the form of full grant is
Rs.45,000/- per unit for plain areas & Rs.48,500/- for hilly/difficult areas. Further, an
IAY beneficiary can avail top-up loan upto Rs 20,000/- under the Differential Rate of
Interest (DRI Scheme) from any Nationalized Bank at an interest rate of 4% per
annum.
The assistance for up-gradation of unserviceable kutcha house to pucca/semi pucca
house is Rs.15,000/- . The assistance for credit-cum-subsidy scheme is Rs.12,500/-
per unit. As a part of Indira Awaas Yojana, this Ministry is implementing since 1999-
2000 the Credit-cum-Subsidy Scheme for rural housing in which credit upto
Rs.50,000/- and subsidy upto Rs. 12,500 is provided to rural households having an
annual income upto Rs. 32,000/- for house construction. The credit component of the
scheme is being disbursed by various housing financing institutions and through
scheduled commercial banks.
5% of the total allocated funds under IAY are kept apart to meet the exigencies arising
out of natural calamities and other emergent situations like riot, arson, fire,
rehabilitation under exceptional circumstances etc.
PURA- Provision of Urban Amenities in Rural Area. The scheme envisages clubbing of
rural infrastructure development with economic re-generation activities in Rural Areas.
NREGA- now known as MNREGA (Mahatma Gandhi National Rural Employment
Guarantee Act) The Mahatma Gandhi National Rural Employment Guarantee Act aims
at enhancing the livelihood security of people in rural areas by assuring hundred days
of wage-employment (rate fixed by state Govt) in a financial year to a rural household
whose adult members volunteer to do unskilled manual work.

Q.34 Match the concepts in Group I with the personalities in Group II


Group I Group II
P. Linear City 1. Le Corbusier
Q. Radiant City 2. Paolo Soleri
R. Garden City 3. Louis Kahn
S. Arcology 4. Soria Y Mata
5. Ebenezer
Howard
(A) P-4, Q-3, R-5, S-1 (B) P-3, Q-1, R-4, S-2 (C) P-4, Q-1, R-5, S-2 (D)
P-1, Q-5, R-2, S-4
Answer : C. (P-4, Q-1, R-5, S-2)
Concept City
Concept Propagated Population Area Year
by
1 Plato’s city Plato 5040 - 400
BC
2 Linear City Soria Y 1895
Mata
3 Garden City Ebenezer 32,000 6000 acre 1899
Howard
4 Neighbourhood Clarence 5000-6000 160 acre 1929
Perry 10 house
per acre
5 Radburn Plan Clarence 25000 149 acre 1929
Stein/Henry
Wright
6 Radiant City Le - - 1930
Corbusier
7 Broadacre City F L Wright - 1 acre per 1932
household
8 Arcosanti Paolo Soleri 5000 25 acre 1970
Garden City concept was given by Ebenezer Howard. Letchworth, UK was
the first garden city built in 1909 planned by Raymond Unwin.
Arcology: Architecture + Ecology first coined by Paolo Soleri. It is
architectural design principles for enormous habitats (hyperstructures) of
extremely high human population density

Q.35 Match the map in Group I with the scale in Group II


Group I Group II
P. Site Plan 1. 1: 4000
Q. Perspective 2. 1: 1000000
Plan
R. Master Plan 3. 1: 500
S. Zonal Plan 4. 1: 20000
5. 1: 2000
(A) P-5, Q-1, R-4, S-2 (B) P-3, Q-2, R-5, S-4 (C) P-1, Q-4, R-5, S-3
(D) P-3, Q-2, R-4, S-1
Answer: D (P-3, Q-2, R-4, S-1)
Scales of map for planning
S. Types of map/ Size of planning Area
No Planning Exercise
1 Perspective Plan 1: 2,50,000 – 1: 1,00,000 (for large
regions)
1: 1,00,000 – 1: 50,000 (for
smaller regions)
2 Regional Plan 1: 50,000 - 1: 25,000 (District
Development Plan)
1: 25,000 - 1:10,000 (Metropolitan
Region Plan)
3 Development Plan 1:10,000 - 1:8,000 (as per State
Provision)
4 Local Area Plan/ 1:5000 - 1:1,000 (as per the
Special Purpose Plan requirement of the project)
source: URDPFI guidelines 2014, Vol I, page 263
Q.36 Match the terminologies of Group I with their corresponding meanings in Group
II
Group I Group II

.
1.
P. Antarala 1. Antechamber
Q. Mandapa 2. Palace hall
R. Gopuram 3. Womb chamber
S. Prasada 4. Gateway
5. Pillared sanctuary
(A) P-3, Q-2, R-4, S-5 (B) P-1, Q-5, R-4, S-2 (C) P-5, Q-3, R-1, S-2 (D) P-
1, Q-2, R-3, S-5
Answer is B. (P-1, Q-5, R-4, S-2)
Antarala- (Antechamber) An antechamber is a smaller room or vestibule serving
as an entryway into a larger one. Antarala is a small space between the garbha
griha (shrine) and the Mandapa.
Mandapa- Mandapa is a pillared hall for public rituals like marriage, mundan
(Head shaving) etc.
Gopuram- It is the entrance Gateway to the Dravidian style temple.
Prasada- Prasada as palace hall.

Q. 37 Match the standard safety colour codes of Group I with their corresponding
usage in Group II
Group I Group II

P. Blue 1. Biodegradable
waste
Q. Green 2. Fire protection
equipment
R. Red 3. Recyclable
waste
S. Yellow 4. Stumbling
against hazards
5. Radiation
standards
(A) P-3, Q-1, R-2, S-4 (B) P-3, Q-4, R-5, S-1 (C) P-1, Q-3, R-2, S-5 (D) P-1, Q-5,
R-2, S-4
Answer : A. (P-3, Q-1, R-2, S-4)
Blue – Recyclable waste, metal, plastic
Green – Biodegradable waste, food, paper, leather
Black- domestic hazard like cfl, battery, sanitary napkin
Red- Fire Fighting Equipment
Yellow- Physical dangers such as Slipping, Tripping, Falling
Purple- Radiation hazard
Bio-Medical Waste Segregation
Colour Waste Description
Code of Bin
Yellow Human tissue organs, body parts, items
contaminated by blood, body fluids, soiled
cotton & dressing, soiled plaster.
Red Catheters, tubes, cannulae, syringe, plastic, IV
bottles and sets, used gloves, infected plastics,
specimen container, lab waste, microbiology
culture, used and discarded blood bags,
vaccines
Blue Glass items, needle syringe, scalpels, blades,
used and unused sharps
Black Discarded medicines, discarded cytotoxic drugs
Green General waste, non infected plastic,
disposables, cardboard, metal container, office
waste, food waste.

Q.38 Match the alignment of the rotating prisms in wall sections in Group I with their
corresponding acoustic function in Group II.

Group I Group II
P 1. Moderate Diffusion

Q 2. Moderate Absorption

R 3. Specular diffusion

S 4. Specular reflection

(A) P–4, Q–1, R–2, S-3 (B) P–4, Q–3, R–1, S–2

(C) P–2, Q–3, R–


4, S- 1 (D) P–2, Q–1, R–3, S- 4

Answer is A. (P–4, Q–1, R–2, S-3)


This problem is a bit confusing.
In the wall section direction of acoustic wave is unknown.
There is no term like Specular Diffusion.
Q.39 Match the projects in Group I with their architects in Group II
Group I Group II

P. 1.
P. Milwaukee Art Museum, 1. Bernard
Wisconsin Tschumi
Q. Kimbell Art Museum, 2. Richard Meier
Fortworth
R. Getty Center, Los Angeles 3. Daniel
Libeskind
S. Freedom Tower, New York 4. Tadao Ando
5. Santiago
Calatrava
(A) P-5, Q-4, R-2, S-3 (B) P-5, Q-1, R-2, S-4 (C) P-2, Q-1, R-4, S-3 (D)
P-2, Q-5, R-4, S-3
Answer is A. (P-5, Q-4, R-2, S-3)
Milwaukee Art Museum, Wisconsin-Santiago
Kimbell Art Museum, Fortworth- Tadao Ando
Getty Center, Los Angeles- Richard Meier
Freedom Tower, New York- Daniel Libeskind on the site of Twin Tower, New
York.

.40 Match the AutoCAD commands in Group I with their corresponding program in
Group II
Group I Group II
one 1. Specify centre point of base or [3P/2P/Ttr/Elliptical]: Specify
base radius or [Diameter] < > :
Specify height or [2Point/Axis endpoint] < >:
Q. Cylinder 2. Specify centre point or [3P/2P/Ttr]:
Specify radius or [Diameter] < > :
Specify tube radius or [2Point/Diameter] < > :
phere 3. Specify centre point of base or [3P/2P/Ttr/Elliptical]:
Specify base radius or [Diameter] < > :
Specify height or [2Point/Axis endpoint/Top radius]:
orus 4. Specify centre point of base or [3P/2P/Ttr/Elliptical]: Specify radius
or [Diameter] < > :
Specify tube radius or [2Point/Diameter] < > :
5. Specify centre point or [3P/2P/Ttr]:

(A) P-1, Q-2, R-4, S-5 (B) P-3, Q-1, R-5, S-2 (C) P-1, Q-3, R-2, S-5 (D)
P-3, Q-5, R-1, S-2
Answer is B. (P-3, Q-1, R-5, S-2)

.41 Identify the hierarchy, from highest to lowest, of the number of potential
conflict points at the unmanaged traffic intersections given below.

P Q R S
(A) P, Q, R, S
(B) R, Q, S,
(C) R, S, Q, P (D) P,
R, S, Q
Answer: (C) R, S, Q, P

P- 0 conflict point
Q- 9 conflict points (3 diverging, 3 merging and 3 crossing)
R- 32 conflict points ( 8 diverging, 8 merging, 16 crossing)
S- 18 conflict points (its basically two T- points)

Q.42 Match the architects in Group I with the terms in Group II


Group I Group II

P.
1.
P Kisho 1 Paper Tubes
Kurokawa
Q. Ken Yeang 2. Deconstructivism
R. Shigeru Ban 3. Metabolism
S. Mies van der 4. Eco Skyscrapers
Rohe
5. Minimalism
(A) P-2, Q-3, R-1, S-5 (B) P-3, Q-5, R-1,S-4
(C) P-2, Q-1, R-5, S-3 (D) P-3, Q-4, R-1, S-5

Answer: D. (P-3, Q-4, R-1, S-5)


Paper Tubes- Recycled cardboard paper tubes used to quickly and efficiently house
disaster victims in time of emergencies. This was propagated by Shigeru Ban,
Japanese architect. Origami is the Japanese art of folding papers.
Deconstructivism.( Features)
a. Fragmentation, geometrical blocks is fused together.
b. Non-Rectilinear shapes, eg. Tilted cuboids
c. Manipulating surface eg. Metallic cladding
The attempt in deconstructivism throughout is to move architecture away from what
Architect’s see as the constricting ‘rules’ of modernism such as “form follows
function,” “purity of form,” and “truth to materials.” Frank Gehry, Daniel Libeskind,
Rem Koolhaas, Peter Eisenman, Zaha Hadid, Coop Himmelbau, and Bernard
Tschumi practice deconstructivism.

Metabolism: Started in 1950’s by Japanese Architects. Metabolist designs relied


heavily on advanced technology, and they often consist of adaptable plug-in mega
structures. The Architects were Noboru Kawazoe, Kiyonori Kikutake, Fumihiko Maki,
Masato Otaka, Kisho Kurokawa, and Kiyoshi Awazu. Nakagin Capsule tower by
Kisho Kurokawa is an example of Metabolosm. Nagakin capsule tower has plug in
pre fabricated units that is attached to the main core. The city of Tokyo by Kenze
Tango is example of Metabolism. Sony Tower
Eco Skyscrapers- Book and concept of Ken Yeang.

Minimalism- Minimalist architecture simplifies living space to reveal the essential


quality of buildings and conveys simplicity in attitudes toward life as opposed to over
decorated elements. Minimalist architects not only consider the physical qualities of
the building, they look deeply into the spiritual dimension. Mies van der Rohe
propagated “Less is more” in his design. His Seagram Building at New York City
made of bronze and glass is America’s first glass skyscraper.

Less is bore- Robert Venturi

Q.43 Match the terms in Group I with their meanings in Group II


Group I Group II

P. Mimbar 1. Pillared assembly hall


Q. Qibla 2. Covered passage around
central court
R. Liwan 3.Pulpit
S. Baradari 4.Parapet between wall
openings
5. Direction of Mecca
(A) P-5, Q-1, R-4, S-2 (B) P-3, Q-5, R-1, S-2
(C) P-2, Q-3, R-4, S-5 (D) P-4, Q-5, R-2, S-1
Answer is B. (P-3, Q-5, R-1, S-2 )
Mimbar- A raised platform (pulpit) in a mosque where the imam stands to
deliver sermons.
Qibla- The Qibla is the direction that should be faced when a Muslim prays.
Liwan- Pillared assembly hall.
Baradari- Covered passage around central court.

Q. 44 Match the locations in Group I with the corresponding traps in Group II


Group I Group II
P. Inspection 1. P-Trap
chamber
Q. Wash basin 2. Gully Trap
R. Bathing space 3. S-Trap
S. European water 4. Bottle Trap
closet
5. Floor Trap
(A) P-2, Q-1, R-4, S-2 (B) P-4, Q-5, R-2, S-3 (C) P-2, Q-4, R-5, S-3 (D) P-
2, Q-3, R-4, S-1
Ans: (C) P-2, Q-4, R-5, S-3
(Input from Satarupa Roy, Priyadarshini Institute of Arch. and Desig Studies, Nagpur)

. 45 Match the building construction components in Group I with their application


areas in Group II
Group I Group II
P. Bracket Plate 1. Steel Column
Q. Kick Plate 2. Curtain Wall
R. Pressure Plate 3. Rolling
Shutter
S. Base Plate 4. Stone Wall
5. Toilet Door
(A) P-2, Q-3, R-4, S-1 (B) P-2, Q-5, R-1, S-4
(C) P-2, Q-5, R-3, S-1 (D) P-3, Q-5, R-2, S-1
Answer is D. (P-3, Q-5, R-2, S-1)

Q.46 Match the historical buildings in Group I with their styles in Group II
Group I Group II
P. Pantheon, Rome 1. Baroque
Q. St. Paul’s Cathedral, 2. Roman
London
R. St. Peter’s Basilica, 3. Romanesque
Rome
S. Notre Dame, Paris 4. Renaissance
5. Gothic
(A) P-3, Q-2, R-1, S-4 (B) P-2, Q-5, R-3, S-1
(C) P-2, Q-1, R-4, S-5 (D) P-3, Q-4, R-2, S-1

Answer: C. (P-2, Q-1, R-4, S-5)


Pantheon, Rome (Roman) St Peter’s Basilica (Renaissance)

Notre dame Paris (Gothic) St Paul’s Cathedral, London


(Baroque)

Q. 47 Corrected Effective Temperature is an index which combines the effect of


P. Climatic zone
Q. Temperature
R. Wind velocity
S. Vegetation
T. Humidity
U. Solar radiation
(A) P, Q, R, S (B) Q, R, T, U (C) Q, S, T, U (D) Q, R, U, P
Answer is B. (Q, R, T, U)
While the ET (Effective Temperature) integrates the effect of three variables-
temperature, humidity, air movement, the CET (Corrected Effective Temperature)
includes radiation effect too. (Page 49-2.2.3, Manual of Tropical Housing and
Building, Koenigsberger)
Effective Temperature Corrected Effective
(ET) Temperature
Temperature Temperature
Humidity Humidity
Wind Velocity Wind Velocity
Radiation

Common data for Questions 48 and 49:


An aircraft flying at an altitude of 5000 m above mean sea level takes aerial
photographs of a flat terrain having an average elevation of 1000m above mean sea
level. The scale of photographs is 1:20000.
Q. 48 The focal length of the camera (in cm) is
(A) 15 (B) 20 (C) 25 (D) 30
Answer: (B) 20
The elevation of terrain is 1000m above mean sea level. i.e. the effective height at
which the
aircraft is flying is 5000-1000= 4000m from which photograph is taken.
Solution:

Q. 49 The area covered on ground (in hectare) by each photo format of 18 cm x 18


cm is
(A) 1266 (B) 1276 (C) 1286 (D) 1296
Answer is D.
Solution: Scale of the photograph is 1:20,000
18cm x18 cm on photograph = (18x 20,000) cm x (18x 20,000) cm on terrain
= 3600m x 3600m
1 hectare= 10,000 sq m

= 1296 hectare

Common data for Questions 50 and 51:


2 ton of cement is used to make 10 ton of hardened concrete having a water
cement ratio of 0.45. One cubic meter of this concrete has a mass of 2.5 ton.
Q.50 Quantity of water used (in kg) is
(A) 90 (B) 450 (C) 900 (D) 945
Answer: C (900 kg)\
Soln: water/cement= 0.45/1
Water= 0.45x cement
= 0.45x 2= 0.9 ton= 900 kg.

Q. 51 The area of 200 mm thick slab (in sq. m) that can be cast using the entire 10
ton of concrete will be
(A) 10 (B) 15 (C) 20 (D) 25
Answer is C (20 m²)
Soln: Total weight of concrete is 10 ton.
Density of concrete=mass/volume= 2.5/1 = 2.5
Volume of concrete= mass/density= 10/2.5= 4 m³
Area of 200mm (0.2m) thick slab= 4/0.2 =20 m²

Linked Answer Questions


Statement for Linked Answer Questions 52 and 53:
A group housing project spread over 20 hectares with FAR 1.5 has to accommodate
different housing units in the following manner:
Categor Percentage Built up Average household
y distribution of area per size(including
built up area unit in sq. common areas)
m
P 10 120 4.0
Q 20 80 4.6
R 40 60 5.5
S 30 45 6.0
Q. 52 The respective number of housing units to be built for each category are
(A) P – 25, Q – 75, R – 200, S – 200
(B) P – 75, Q – 25, R – 200, S – 200
(C) P – 25, Q – 25, R – 75, S – 200
(D) P – 25, Q – 75, R – 200, S – 75
Answer: (A) P – 25, Q – 75, R – 200, S – 200
NOTE: This Question is not matching with set of Options
We take area of land as 2 ha instead of 20 ha to get the ansers.
Area of land= 2.0 hectare= 2x 10,000= 20,000 m²
Since FAR is 1.5 built up area= 20,000 x 1.5= 30,000 m²
Category P (10%) = 0.1 (30,000) / 120 = 25 (Total units of P type)
Category Q (20%) = 0.2 (30,000) / 80 = 75 (Total units of
Q type)
Category R (40%) = 0.4 (30,000) / 60 = 200 (Total units of R type)
Category S (30%) = 0.3 (30,000) / 45 = 200 (Total units of S
type)

Q. 53 The gross density of the above housing complex (in persons per hectare) is
(A) 125 – 130 (B) 130 – 135 (C) 135 – 140 (D) 140 – 145
Answer : C (135 – 140)
In this case we take area as 20 ha (Rediculous)
Total Population (P type) 25× 4= 100
Total Population (Q type) 75× 4.6 = 345
Total Population (R type) 200× 5.5= 1100
Total Population (S type) 200× 6= 1200
TOTAL POPULATION = 2745

Gross Density = 137.25 pph

Statement for Linked Answer Questions 54 and 55:


An Unplastered brickwork has a resistivity of 0.83 mdegC/W. For a 230 mm thick
brick wall, the inside and outside surface resistances are 0.123 and 0.1 m2deg C/W
respectively.

Q. 54. The R-value of the given wall section (in m2deg C/W) is
(A) 0.41 (B) 0.5 (C) 0.67 (D) 1.05
Answer is A. (0.41)

R-value can directly be added. Here resistivity ( of wall is given which is 0.83 m
degC/W.
From resistivity of wall we need the R-value which is = Resistivity × wall thickness
R value of wall = 0.83 m degC/W × 0.23 m (here wall thickness is taken in
m)
= 0.19 m2deg C/W
Now adding all the R-value = 0.19 + 0.123 + 0.100= 0.413 m2deg C/W

Q. 55.The brick wall has an overall dimension of 6m x 3m with a 1m x 2m window


assembly (U-value =2.5 W/m²degC) and a 1m x 2.1m door assembly (U-value =
1.25 W/m²degC). The overall U-value of the wall (UO) (in W/m² degC) is
(A) 1.8 (B) 2.3 (C) 2.6 (D) 3.7
Answer is B. (2.3)
Area window= 1 x2 = 2m2
Area of door = 1x 2.1 = 2.1 m2
Remaining area of wall = 18- 4.1 = 13.9 m2

From Q. 54 U-value of wall = = = 2.43 W/m2degC


Heat loss from wall per deg C = 2.43 x 13.9 = 33.7
W
Heat loss from window per deg C = 2.5 x 2 = 5.0 W
Heat loss from door per deg C = 2.1 x 1.25 = 2.6
W
Total Heat Loss = 41.3 W
Total area of wall including door + Window = 18 m2

The overall U-value of the wall = = 2.29 W/m² degC

General Aptitude (GA) Questions


Q. 56 Which one of the following options is the closest in meaning to the word given
below?
Pacify
(A) Excite (B) Soothe (C) Deplete (D) Tire
Answer is B. (Soothe)
Q. 57 Choose the most appropriate pair of words from the options given below to
complete the following sentence:
The high level of of the questions in the test was
by an increase in the period of time allotted for answering them.

(A) difficulty, compensated (B) exactitude, magnified


(C) aptitude, decreased (D) attitude, mitigated
Answer: A. (difficulty, compensated)

Q. 58 Choose the grammatically CORRECT sentence:


(A) He laid in bed till 8 o’clock in the morning.
(B) He layed in bed till 8 o’clock in the morning.
(C) He lain in bed till 8 o’clock in the morning.
(D) He lay in bed till 8 o’clock in the morning.
Answer: D. (He lay in bed till 8 o’clock in the morning)

Q. 59 Which one of the parts (A, B, C, D) in the sentence contains an ERROR?


No sooner had the doctor seen the results of the blood test, than he suggested
the patient to see the specialist.
(A) no sooner had (B) results of the blood test (C) suggested the patient (D)
see the specialist
Answer C. (suggested the patient)

Q. 60 Ten teams participate in a tournament. Every team plays each of the other
teams twice. The total number of matches to be played is
(A) 20 (B) 45 (C) 60 (D) 90
Answer is D. (90)
Q. 61 – Q. 65 carry two marks each.
Q. 61 A value of x that satisfies the equation log x + log (x – 7) = log (x + 11) + log 2 is
(A) 1 (B) 2 (C) 7 (D) 11
Answer D. (11)
Q. 62. Let f(x) = x – [x], where x ≥ 0 and [x] is the greatest integer not larger than x.
Then f(x) is a
(A) monotonically increasing function
(B) monotonically decreasing function
(C) linearly increasing function between two integers
(D) linearly decreasing function between two integers
Answer C. (linearly increasing function between two integers)

Q.63 Ravi is taller than Arun but shorter than Iqbal. Sam is shorter than Ravi. Mohan is
shorter than Arun. Balu is taller than Mohan and Sam. The tallest person
can be
(A) Mohan (B) Ravi (C) Balu (D) Arun
Answer: C. (Balu)

Q.64 A smuggler has 10 capsules in which five are filled with narcotic drugs and the
rest contain the original medicine. All the 10 capsules are mixed in a single
box, from which the customs officials picked two capsules at random and
tested for the presence of narcotic drugs. The probability that the
smuggler will be caught is
(A) 0.50 (B) 0.67 (C) 0.78 (D) 0.82
Answer: C (0.78)

Q. 65 The documents expose the cynicism of the government officials – and yet as
the media website reflects, not a single newspaper
has reported on their existence.
Which one of the following inferences may be drawn with the greatest accuracy
from the above passage?
(A) Nobody other than the government officials knew about the existence of the
documents.
(B) Newspapers did report about the documents but nobody cared.
(C) Media reports did not show the existence of the documents.
(D) The documents reveal the attitude of the government officials.
Answer: D. (The documents reveal the attitude of the government officials)
GATE 2013
Q. 1 – Q. 25 carry one mark each.
.1 In case of residential apartments, the effective floor area available for use within an
apartment, is known as
(A) Carpet Area (B) Built-up Area (C) Plinth Area (D) Super Built-up Area
Answer: (A) Carpet Area
As per IS: 3861-2002, Carpet area is full area of house/ office unit excluding area
of all walls, kitchen, toilet and passage. Only 50% of balcony area is to be added
in Carpet area.
Built-up area is the carpet area plus the thickness of outer walls, balcony (full
area), kitchen and toilet.
Plinth area is ‘built up area’ plus common areas.
Super built-up area is the ‘built up area’ plus proportionate area of common
areas such as the lobby, lifts shaft, stairs, etc.
Suggested Reading- IS: 3861 – 2002; titled “Method of measurement of plinth, carpet and rentable areas of
buildings.”

Q.2 Star Rating of an Air Conditioner is determined by its


(A) Power Consumption (B) Energy Efficiency Ratio (C) Cooling Capacity (D)
Power of Compressor
Answer: (B) Energy Efficiency Ratio

Energy Efficiency Ratio=


For air conditioners, the measure of energy efficiency is the Energy Efficiency
Ratio (EER) for cooling and the Coefficient of Performance (COP) for
heating. The EER and COP are defined as the capacity output divided by the
power input.
Cooling capacity is the amount of heat energy removed by the Air conditioner
from a space for a given time. It is generally measured in British Thermal Unit
(BTU) per hour (BTU/Hr). In SI units it is measured by KJ / Sec= KW.
Both cooling capacity and power consumption is measured in watt.
A refrigeration ton is approximately equivalent to 12,000 BTU/h or 3,517 W.
1 Btu= 1055 Joules
EER (W/W) of 3.40 gives a 5 – Star Rating which means for every watt of power
consumed 3.40 W (3.4 Joules/sec) of heat energy is removed by the air conditioner.
Bureau of Energy Efficiency under Ministry of Power, Govt. of India was formed
in the year 2002, under the provision of Energy Conservation Act, 2001. All
electrical appliances should have star rating that determines its Energy
Efficiency.
The Energy Conservation Building Code (ECBC) was launched in 2007, which
promotes Energy Efficiency in Building Sector.

Star Rating of Geyser is determined by its ‘Standing Loss’ (kWh/ 24 hr)


Practice Question.
Q: A two ton AC has EER of 3.5. Calculate its power consumption.
Soln.EER= Cooling capacity/ Power consumption
Given EER= 3.5
Cooling capacity = 2 ton
= 2x3517= 7034 Watt (since 1 ton ≈3517 W)

3.5=

Power consumption= = 2009.7 W (say 2 KW)


(Note: In the above question 2 KW of power consumption transfers 7034 W of power from inside to outside)
The adjacent table shows Indian Seasonal EER for Window AC.
Q.3 V7 concept given by Le Corbusier refers to
(A) Neighbourhood Planning (B) Housing Typologies
(C) Architecture Design Principle (D) Hierarchy of Roads
Answer: (D) Hierarchy of Roads
The “Vs” are pathways hierarchically organized according to the intensity of the traffic
flow that they support.
V1: National Highways (connecting one city to another)
V2: Special facilities Road (urban city Road)
V3: High speed avenues that cross the city (vehicular road around sector)
V4 and V5: Neighborhood ways, (Road through Shopping Street and Sector)
V6: Domestic paths
V7: Pedestrian paths
V8: Cycle tracks

.4 In AUTOCAD, a line of infinite length in the direction defined by starting point and
through point, is known as
(A) RAY (B) LINE (C) PLINE (D) XLINE
Answer: (A) RAY
Q.5 Orbit Tower built at the London Olympic Park has been designed by
(A) Foster & Partners (B) Anish Kapoor & Cecil Balmond
(C) Zaha Hadid & Antony Gormley (D) Richard Rogers & Renzo Piano
Answer: (B) Anish Kapoor & Cecil Balmond
Orbit tower is built, close to main Olympic stadium in London. Sir Anish Kapoor, is an
Indian-born British sculptor. Born in Mumbai, Kapoor has lived and worked in London
since the early 1970s when he moved to study art in London.
Cecil Balmond is a Sri Lankan – British designer, artist, architect, and writer.
Ram Sutar (Padmshree) is an Indian Sculptor making life size statues.
Richard Rogers & Renzo Piano: Designed Pompidou Centre in Paris which is a
high tech. architecture building.

Q.6 As per National Building Code 2005, the minimum size of a habitable room in m² is
(A) 9.5 (B) 10.5 (C) 11.5 (D) 12.5
Answer: (A) 9.5 m²
Habitable Room (definition): A room occupied or designed for occupancy by one or
more persons for study, living, sleeping, eating; kitchen if it is used as a living room,
but not including bathrooms, water-closet, compartments, laundries, serving and store
pantries, corridors, cellars, attics, and places that are not used frequently or during
extended periods.

Minimum Size and Width of Different Components of Residential Premises (NBC)

Sl. Component Min. requirement Min. Area and width of habitable


No. of Building for plots upto 50 requirement for rooms: The area of
sq m. plots above 50 habitable room shall not be
sq m. less than 9.5 m2, where
1 Habitable Area 7.50 sq m. Area 9.50 sq there is only one room with
Room m. a minimum width of 2.4 m.
Width 2.10 m. Width 2.40 m. Where there are two rooms,
one of these shall not be
Height 2.75 m. Height 2.75 m.
less than 9.5 m2 and the
2 Kitchen Area 3.30 sq m. Area 4.50 sq other not less than 7.5 m2,
m. with a minimum width of 2.1
Width 1.50 m. Width 1.50 m. m.
Height 2.75 m. Height 2.75 m. Height of habitable rooms :
3 Pantry Area Not Area 3.00 sq The height of all rooms for
applicable m. human habitation shall not
Width Not Width 1.40 m. be less than 2.75 m
applicable measured from the surface
Height Not Height 2.75 m. of the floor to the lowest
applicable point of the ceiling (bottom
4 Bathroom Area 1.20 sq m. Area 1.80 sq of slab). In the case of
m. pitched roof, the average
height of rooms shall not be
Width 1.00 m. Width 1.20 m.
less than 2.75 m. The
Height 2.20 m. Height 2.20 m. minimum clear head room
5 W.C. Area 1.00 sq m. Area
1.10 sq under a beam, folded plates
m. or eaves shall be 2.4 m. In
Width 0.90 m. Width 0.90 m. the case of air-conditioned
Height 2.20 m. Height 2.20 m. rooms, a height of not less
than 2.4 m measured from
6 Combined Area 1.80 sq m. Area 2.80 sq
the surface of the floor to
Bath & W.C. m.
the lowest point of air-
(Toilet) Width 1.00 m. Width 1.20 m. conditioning duct or the
Height 2.20 m. Height 2.20 m. false ceiling shall be
7 Garage Area 14.85 provided.
sqm
Width 2.75 m Space standards
Length 5.40 m, for Parking
ht= 2.4
m
S No Type of parking Area in sqm
per ECS
1 Open 23
2 Ground Floor 28
Covered
3 Multi level with 30
Ramps
4 Basement 32
5 Automated 16
multilevel with lifts
source: URDPFI guidelines 2014, Vol I (page 292)

Q.7 The urban form of Srirangam town in Tamil Nadu refers to


(A) Dandaka (B) Swastika (C) Nandyavarta (D) Sarvotabhadra
Answer: (D) Sarvotabhadra
According to Manasara there are eight types of plans for designing towns/ villages.
They are Dandaka,

Sarvatobhadra, Nandyavartha, Padmaka, Swastika, Prastara, Karmuka and


Chaturmukha.
Note: Karmuka was the plan suitable for settlement on waterfront i.e. river or sea.
.

(Ref: Town Planning in Ancient India by Binod Bihari Dutta)


Srirangam: (Sarvotabhadra) A river (Cauvery) island city famous for Vishnu temple is
located in Tiruchirappalli, Tamil Nadu.
Jaipur: Prastara (City was built by King Sawai Jai Singh (1700-1743). The architect for
conceiving the plan of Jaipur was Vidyadhar Bhattacharya.
Banaras Hindu University campus plan is based on Karmuka.

Q.8 PMGSY, a programme of Government of India, deals with


(A) Urban Employment Generation (B) Rural Employment Generation
(C) Rural Electrification (D) Rural Road Development
Answer: (D) Pradhan Mantri Gram Sadak Yojna (PMGSY)
Urban Employment Generation: Swarn Jayanti Sahri Rojgar Yojna(SJSRY)
Rural Employment Generation: Mahatma Gandhi Rural Employment
Guarantee Scheme (MNREGS)
Rural Electrification: Rajeev Gandhi Grameen Vidyutikaran Yojana.

Q.9 Beam or lowest division of the entablature which extends from column to column, is
known as
(A) Arabesque (B) Arcade (C) Architrave (D) Arbour
Answer: (C) Architrave
Entablature has three sections- {remember AFC} Architrave, Frieze and
Cornice
Arabesque: surface decorations based on rhythmic linear patterns of scrolling
and interlacing foliage, tendrils or plain lines.
Arcade: a series of adjoining arches.
Arbour: a leafy gateway shaded by trees, climbers, shrubs, etc., especially
when trimmed to give shape. Topiary is trimming of plant to give some
geometric shape. When the shape is in form of animals it is zoomorphic.

ther Islamic terms in Architecture.


Arabesque- interlacing plant motifs like decoration.
Arcade- series of arches.
Drum- A cylindrical wall that supports the dome.
Hammam- public bath (In Rome it was thermae)
Hareem- women folk
Mashrabiya- wooden lattice work placed infront of building for ventilation and
privacy.
Mihrab- a semicircular niche in the wall of a mosque that indicates the qibla; that is,
the direction of the Kaaba. (GATE 1991)
Mimbar- a pulpit placed on the right of mihrab used by imam for Jumma lecture.
Muqranna- decorative pattern consisting of small niche arranged like honey comb
and made mostly from plaster for internal treatment of curved surface especially in
transitional zone between dome and their support i.e. pendentive and squinch.
Nave- main central aisle leading to the Mihrab.
Squinch- arch placed at corners of square base to support the dome.
Stucco- plaster used for coating and making decorative casts.

(Mihrab)
(Mihrab and Mimbar) (Mimbar)

Q.10 The information that is NOT essential to be submitted for sanction of any building plan
is
(A) Site Plan (B) Floor Plans (C) Title Deed (D) Land Cost
Answer: (D) Land Cost
A title deed is a legal document used to prove ownership of a piece of property.
Owner Book is a legal document used to prove ownership of a vehicle.
Cadastal map shows the land ownership.
MVR (Minimum value register ) is the document issued by State Govt. which fixes
the minimum value of land for registration purpose.
Market Value of land is prevailing value of land at particular location, often more
than Minimum value fixed by Govt.
One decimel of land is equivalent to 1/100 of acre i.e. 40.46 sq.m in India and
Bangladesh.

Q.11 The tendency of an ecosystem to maintain its balance by regulatory mechanisms when
disrupted, is known as
(A) Homeostasis (B) Entropy (C) Succession (D) Evolution
Answer (A): Homeostasis
Homeostasis: The tendency of an ecosystem to maintain its balance by
regulatory mechanisms when disrupted.
Entropy: Nature tends from order to disorder in isolated systems.
Ecological succession: The observed process of change in the species
structure of an ecological community over time.
Evolution: The change in the inherited characteristics of biological populations
over successive generations.
Ecotone: Interface of two ecological zones.

Q.12 Gantt Chart DOES NOT provide information about


(A) List of Jobs (B) Duration of Jobs (C) Interdependency of Jobs (D)
Progress of Work
Answer (C): Interdependency of Jobs

.13 If threshold of hearing has a sound level of zero decibels and the sound level in
broadcasting studio is 100 times the threshold of hearing, its value in decibels would
be
(A) 0 (B) 10 (C) 20 (D) 100
Answer (C): 20
Decibels measure the ratio of a given intensity ‘I’ to the threshold of hearing intensity
I0.

Where I0 is threshold of hearing.= 10-12 Watt/ m2


In this problem I / Io is 100, so I (dB) = 10 x log 10 (100) = 20 dB since {log 10 (100)=
2}

Addition of Decibels. (Very important to understand) –Intensities can be added


directly.

L∑ = Output decibel
L1 , L2, L3……….Ln are individual decibels.
Note: Every doubling the distance, reduces the intensity of sound by 6 dB.

Q.14 The width to height ratio of the front facade of Parthenon (without the pediment) is
(A) 9:4 (B) 4:9 (C)
1:1.618 (D) 1.618:1
Answer (A): 9:4
If height of Parthenon is taken as
1.000
Column= 0.618
Entablature= 0.146
Pediment= 0.236
Width of temple = 1.618 and
height without
pediment = 0.618 + 0.146=
0.764

= = 2.11 and the closest ratio is 9:4

Golden Ratio= 1: 1.618 (GATE 1991)


Note: Pediment is the triangular element, and Entablature is the element between
pediment and column.
Parthenon is dedicated to Goddess Athena who is known for Wisdom and
Architecture.
Entasis and Golden Mean Ratio is the main attribute of Parthenon.
[Refer Q. 28 (2006)]

Q.15 The face of an Icosahedron is


(A) Equilateral Triangle (B) Isosceles Triangle (C) Square (D) Pentagon
Answer (A): Equilateral Triangle
Icosahedron is a 20 faced polyhydron, with 30 edges and 12 vertices.

Difference between Polygon and Polyhedron- A


polygon is a 2-dimensional plane figure with multiple faces. A polyhedron is a 3-
Dimensional object composed of polygons.
There are only five regular polyhedron. (platonic solids)
Tetrahedron- Four equilateral triangular faces
Cube- Six squre faces
Octahedron- Eight equilateral Triangular faces
Dodecagon- Twelve Pentagonal faces
Icosahedron- Twenty equilateral Triangular faces

Note: A cube is also known as hexahedron. Euclid proved that there are only five
regular polyhedron.
Dual of Tetrahedron is a tetrahedron, dual of Dodecahedron is Icosahedron and dual
of Icosahedron is a dodecahedron.
Archemedian Solid: Soild with two or more regular polygonal face with identical
vertex.There are 12 Archemedian solids.

Q.16 The term ‘Zeitgist’, used in contemporary architecture, refers to


(A) Iconicity (B) Spirit of Times (C) Kinesthetics (D) Semantic Associations
Answer (B): Spirit of Times
The German word ‘Zeitgeist’ signifies spirit of the age or spirit of the time.

Q.17 Alhambra, a UNESCO world heritage site, is classified as an example of


(A) Moorish Architecture (B) Mudejar Architecture
(C) Mozarabic Architecture (D) Tudor Architecture
Answer (A): Moorish Architecture
Alhambra: Fortress located in Spain.

Q.18 Wythenshawe and Becontree are examples of


(A) Factory Town (B) Satellite Town
(C) Garden City (D) Vertical Neighborhood
Answer (B): Satellite Town
Wythenshawe (1920) and Becontree (1921) are satellite town of London.
Garden city- Wellwen and Letchworth in UK.
Q.19 National Ceremonial Plaza at Thimpu in Bhutan has been designed by
(A) Christopher Charles Benninger (B) Charles Correa (C) Karan Grover (D) I.
M. Pei
Answer (A): Christopher Charles Benninger
Christopher Charles Benninger is a Pune based Architect with his office at Thimpu,
Bhutan. His projects are listed below. He also planned the new city of Thimpu, capital
of Bhutan.
1980-1982, Development Plan for the city and region of Jaffna, Sri lanka
1986-1989, 1999-2000, Center for Development Studies and Activities, Pune,
Maharashtra
1996-1998, Institute of Social Science, New Delhi
1998-1999, Indian Institute of Management, Calicut, Kerala (Competition)
2000, The Coptic Orthodox Church, near Newyork, USA
2000, Ministry of Foreign affairs, Videsh bhawan, New Delhi (Competition)
2001-2004, Center for Neurological Disorders and Complimentary Medicine,
Pune, Maharashtra
2001, Duxton Housing, Singapore (Competition)
2001-2002, Thimpu Structure Plan, Bhutan
2001-2004, Tain Square, Pune, Maharashtra
2003-todate, Urban design of the Dzong Precinct, Thimpu, Bhutan
2003-todate, The National Secretariat Complex, Thimpu, Bhutan
2004, Symbiosis University, Pune, Maharashtra (Competition)
2006-2011, Indian Institute of Management, Calcutta (Phase-1)
2009,The Directorate of Town and Country Planning, Hyderabad, Andhra
Pradesh
2010 Azim Premji University, Bengaluru
2011 Indian Institute of Management, Calcutta (Phase-2)
2013 - Indian Institute of Technology, Hyderabad, Telangana (Phase 1B)

.20 Physiochemical process of removing micro-organisms, colour and turbidity from sullage
and sewage is known as
(A) Putrefaction (B) Clarification (C) Liquefaction (D) Infiltration
Answer: (B) Clarification

Putrefaction: Decomposition
of organic matter, especially
protein, by microorganisms, resulting in production of foul-smelling matter.
Clarifiers consist of tanks or basins which hold water or wastewater for a
period sufficient to allow the floc and other suspended materials to settle
to the bottom. The clarification process makes the water clear by
removing all kinds of particles, sediments, oil, natural organic matter and
color.

Liquefaction is a phenomenon in which the strength and stiffness of a soil is


reduced by earthquake shaking or other rapid loading.
Infiltration is the process by which water on the ground surface enters the soil.

Q.21 Identify which is NOT a green building rating system


(A) LEED (B) CASBEE (C) ENERGY BUILD (D) BREEAM
Answer (C): ENERGY BUILD
LEED: Leadership in Energy and Environmental Design, developed by United States
Green Building Council. Four types of rating are given based on points earned.
Certified: 40–49 points
Silver: 50–59 points
Gold: 60–79 points
Platinum: 80 points and above
CASBEE: Comprehensive Assessment System for Built Environment Efficiency
developed in Japan.
BREEAM: BRE Environmental Assessment Method, UK.
GRIHA: Indian Model of Green Building rating is GRIHA (Green Rating for
Integrated Habitat Assessment). Developed by TERI (The Energy and
Resources Institute) it gives star rating from 1 to 5 Star.

Q.22 In 3DS Max, smooth 3D surfaces, by blending a series of selected shape curves, can be
created by
(A) Lofting (B) Sweeping (C) Filleting (D) Extruding
Answer: (A) Lofting

Q.23 Travel behavior characteristics of an urban area can be derived from


(A) Parking Survey (B) Demographic Survey (C) Socio Economic Survey (D) Origin &
Destination Survey
Answer (D): Origin & Destination Survey

Q.24 In GIS, the set of entities representing vector data type is


(A) Point, Line, Polygon, TIN (B) Pixel, Voxel (C) DEM, DSM, DTM (D) Coordinates,
Elevation, Slope

Answer (A): Point, Line, Polygon, TIN

Q.25 A common flowering shrub is


(A) Tectona grandis (B) Mimusops elengi
(C) Dalbergia sisso (D) Ixora coccinea
Answer: (D) Ixora coccinea
Tectona grandis (teak) is a tall tree from Southeast Asia and is widely cultivated for
its durable wood. (sagwan)
Mimusops elengi: Bakul, Indian Medicinal Plant This tree has been mentioned in the
‘Sangam’ literature, in the plays of the great poet Kalidasa and more recently by
Abul Fazl in Ain-in-Akbari.
Dalbergia sisso: Sheesham
Ixora coccinea: It is a common flowering shrub native to Southern India and
Sri Lanka. (adjacent image )
For writing botanical name first word is genus and second is species, both written in
italic form.

Q.26 The correct arrangement of the height of towers given below in descending order is

P. Burj Khalifa, Dubai Q. Petronas Tower, Kuala Lumpur


R. Taipei 101, Taiwan S. Bank of China Tower, Hong Kong
(A) P, Q, R, S (B) P, Q, S, R
(C) P, R, S, Q (D) P, R, Q, S
Answer: (D) Burj Khalifa > Taipei 101> Petronas Tower> Bank of China Tower
Under Construction
Kingdom Tower, Jeddah, Saudi Arabia (Under Construction)
Total height 1000 m.
Architect : Adrian Smith (Adrian Smith + Gordon Gill architecture, AS + GG)

Q.27 Match the buildings in Group I with their corresponding architects in Group II
Group I Group II
P. Khalsa Heritage Complex, Anandpur Sahib 1. Philip Johnson
Q. Lisbon Ismaili Centre, Lisbon 2. Charles Correa
R. Neuroscience Centre, Cambridge, USA 3. Raj Rewal
S. National Centre for Performing Arts, Mumbai 4. B. V. Doshi
5. Moshe Safdie
(A) P-2, Q-5, R-1, S-4 (B) P-5, Q-3, R-2, S-1 (C) P-4, Q-2, R-1, S-3 (D) P-5, Q-2, R-1,
S-4
Answer: (B) P-5, Q-3, R-2, S-1
Ismili Centre, Toronto, Canada- Charles Correa (Ismili Centre at Lisbon, Portugal
is designed by Raj Rewal)
Champaulimaud Centre, Lisbon, Portugal- Charles Correa.
( Both Raj Rewal and Charles Correa has worked in Lisbon, Portugal)
Moshe Safdie- known for Habitat, Montrel Canada.

Khalsa Heritage Complex, Punjab Centre for Performing Arts, Mumbai

Lisbon Ismili Centre Neuroscience Centre, Cambridge,


USA

Q.28 The term ‘Working head’ in context of water supply system means
(A) Height of a body of water falling freely under the force of gravity to acquire a certain
velocity
(B) Rate of increase of velocity with respect to distance normal to the direction of flow
(C) Total head with deduction for velocity head or losses
(D) Difference between supply and delivery water levels
Answer: (D) Difference between supply and delivery water levels

Q.29 In a theoretical traffic flow relationship, as shown in the figure given below, the slope of
line of joining point F on the curve and the origin O represents
(A) Corresponding space mean speed
(B) Speed at maximum flow
(C) Travel time at corresponding density
(D) Average headway at corresponding flow
Answer: (A) Corresponding space mean speed
We know that a slope is tangent of the line and in this case tangent will show the
relationship between x axis and y axis. And this relationship will be relationship
between time and distance. Relationship between time and distance is generally
known as speed, velocity etc. Since this graph is a traffic flow diagram means
this will show a mean or average of speed. Hence option A.
There are mainly two types of “mean speeds” in a traffic flow:
Time mean speed- Time mean speed is the average of all vehicles passing a
point over a duration of time. It is the simple average of spot speed.
Space mean speed- Space mean speed is simply the harmonic mean of the
spot speed.
: with input from Rewati Raman (Research scholar IIT Roorkee, B Arch(NIT Patna)

Q.30 Match the CAD terms in Group I with their corresponding functions in Group II
Group I Group II
P. Tiled viewport 1. Boolean operator
Q. UCS 2. Solid model
R. DXF 3. Coordinate system
S. Extrude 4. Drawing interchange format
5. Model space
(A) P-4, Q-3, R-2, S-1 (B) P-2, Q-5, R-2, S-1
(C) P-5, Q-3, R-4, S-2 (D) P-3, Q-5, R-4, S-2
Answer: (C) P-5, Q-3, R-4, S-2

Q.31 Match the historic periods in Group I with their corresponding examples of towns in
Group II
Group I Group II
P. Egyptian 1. Miletus
Q. Greek 2. Montpazier
R. Medieval 3. Kahun
S. Renaissance 4. Versailles
5. Timgad
(A) P-3, Q-1, R-2, S-4 (B) P-3, Q-1, R-4, S-5 (C) P-4, Q-1, R-5, S-2 (D) P-5, Q-1, R-
3, S-2
Answer: (A) P-3, Q-1, R-2, S-4
Miletus: Ancient Greek Town. Miletus is the birthplace of the Hagia Sophia’s architect
(and inventor of the flying buttress) Isidore of Miletus and Thales. It is also the birth
place of Hippodamus. Hippodamus (498- 408 BC) is known as father of Urban
planning and designed Piraeus, Olynthus, Priene, and Miletus.
Montpagier: French town
Kahun: Egyptian village where slaves lived during pyramid construction
Versailles Garden- Andre Le Notre was a French landscape architect and the
principal gardener of King Louis XIV of France who designed Versailles Garden. (This
is the most famous garden ever designed).
Timgad: Ancient Roman town

Q.32 Match the components of an Indian urban land use map in Group I with their
corresponding colour codes as per UDPFI guidelines in Group II
Group I Group II
P. Public / Semipublic 1. Violet
Q. Industry 2. Grey
R. Transportation 3. Red
S. Commercial 4. Blue
5. Yellow
(A) P-1, Q-3, R-2, S-5 (B) P-2, Q-1, R-3, S-4 (C) P-3, Q-4, R-5, S-2 (D) P-3, Q-1, R-
2, S-4
Answer: D P-3, Q-1, R-2, S-4

Q.33 Match the books in Group I with their corresponding authors in Group II
Group I Group II
P. Design of Cities 1. Amos Rapoport
Q. On the Cultural Origin of Settlements 2. Leo Jacobson and Ved Prakash
R. Urbanization and National Development 3. Edmond Bacon
S. Planning Theory 4. Christopher Alexander
5. Andreas Faludi
(A) P-3, Q-4, R-1, S-5 (B) P-3, Q-1, R-2, S-5 (C) P-4, Q-3, R-5, S-2 (D) P-3,
Q-4, R-1, S-2
Answer: (B) P-3, Q-1, R-2, S-5

Q.34 Match the temples in Group I with their corresponding historical periods in Group II
Group I Group II
P. Vaikuntha Perumal Temple, Kancheepuram 1. Vijaynagara
Q. Meenakshi Temple, Madurai 2. Chalukya
R. Durga Temple, Aihole 3. Chola
S. Brihadeshwara Temple, Thanjavur 4. Pandya
5. Pallava
(A) P-2, Q-3, R-5, S-1 (B) P-5, Q-1, R-4, S-3 (C) P-3, Q-5, R-2, S-1 (D) P-5,
Q-4, R-2, S-3
Answer : (D) P-5, Q-4, R-2, S-3
Vaikuntha Perumal Temple, Kancheepuram: (Pallava) Temple is believed to have been
built by the Pallava king Nandivarman II (720-96 CE), constructed in the Dravidian style
of architecture.
Meenakshi Temple, Madurai: (Pandya)The complex houses 14 gateway towers called
gopurams, ranging from 45-50m in height, the tallest being the southern tower, 51.9
metres (170 ft) high, and two golden sculptured vimana, the shrine over sanctum of the
main deities. The temple forms the heart and lifeline of the 2500 year old city of
Madurai.
Durga Temple, Aihole: (Chalukya) The plan of the temple is oblong and apsidal. It
means that the corridor with pillars between the porch and the heart of the
shrine encompasses the heart of shrine and allows to run the parikrama
(circumambulation ritual). This apse gives outward through openings between the
pillars.
Brihadeshwara Temple, Thanjavur: (Chola) The temple is part of the UNESCO World
Heritage Site “Great Living Chola Temples”. This is the largest temple in India. The
temple stands amidst fortified walls that were probably added in the 16th century. The
vimana or (temple tower) is 216 ft (66 m) high and is among the tallest of its kind in the
world. The kalash on vimana (sikhar) of the temple weighs around 80 tons.

Q.35 Match the theories in Group I with their corresponding propagators in Group II
Group I Group II
P. Choice theory of planning 1. Paul Davidoff and T.A. Reiner
Q. Connurbation 2. Patrick Geddes
R. Classical theory of land use 3. Homer Hoyt
S. Central place theory 4. Richard L. Meier
5. Walter Christaller
(A) P-2, Q-3, R-5, S-1 (B) P-1, Q-2, R-4, S-5 (C) P-4, Q-3, R-5, S-2 (D) P-5,
Q-4, R-3, S-2

Answer: (B) P-1, Q-2, R-4, S-5


Soln : Courtesy Kavinaz Kaur, B. Arch, GNDU, Amritsar.

Q.36 Match the buildings in Group I with their corresponding structural feature in Group II
Group I Group II
P. Yokohama Port Terminal 1. Geodesic Dome
Q. Stanstead Airport, London 2. Shell Structure
R. TWA Terminal, New York 3. Space Frame
S. Montreal Biosphere, Montreal 4. Folded Steel Plate Structure
5. Pneumatic Structure
(A) P-4, Q-3, R-2, S-1 (B) P-2, Q-1, R-3, S-4 (C) P-4, Q-3, R-5, S-2 (D) P-5,
Q-3, R-4, S-2
Answer: (A) P-4, Q-3, R-2, S-1
Geodesic Dome- Buckminster Fuller.
Buckminster fuller also known for Dymaxion.

Q.37 Match the Five Year Plans listed under Group I with their corresponding feature from
Group II
Group I Group II
P. First Five Year Plan 1. Formation of HUDCO
Q. Fourth Five Year Plan 2. Establishment of TCPO
R. Seventh Five Year Plan 3. Introduction of JNNURM
S. Tenth Five Year Plan 4. Announcement of National Housing Policy
5. Passing of Urban Land Ceiling and Regulation Act
(A) P-5, Q-2, R-4, S-3 (B) P-2, Q-1, R-4, S-3
(C) P-4, Q-1, R-2, S-5 (D) P-1, Q-2, R-3, S-5

Answer: (B) P-2, Q-1, R-4, S-3

1st Five Year Plan - (1951-56)


2nd Five Year Plan- (1956- 61)
3rd Five Year Plan- (1961-66)
4th Five Year Plan- (1969-74)
5th Five Year Plan- (1974-79)
Rolling Plan- (1978-80)
6th Five Year Plan- (1980-85)
7th Five Year Plan- (1985-90)
Annual Plans- (1990-92)
8th Five Year Plan- (1992-97)
9th Five Year Plan- (1997-2002)
10th Five Year Plan- (2002-2007)
11th Five Year Plan- (2007-2012)
12th Five Year Plan- (2012-17)

1955 Town and Country Planning Organization under Ministry of Urban Development.
-1st FYP
1970- Formation of HUDCO - 4th FYP
1976- Passing of Urban Land Ceiling and Regulation Act -5th FYP
1988- Announcement of National Housing Policy – 7th FYP
2005- Introduction of JNNURM - 10th FYP
2013- Right to Fair Compensation and Transparency in Land Acquisition, Rehabilitation
and Resettlement Act. (This is new act in place of Land
Acquisition Act 1893)
Note: ITPI (Institution of Town Planners of India) was established in 1952

Q.38 Match the landscape designers listed under Group I with their appropriate contribution
from Group II
Group I Group II
P. Lancelot ‘Capability’ Brown 1. The Well-tempered Garden
Q. Andre Le Notre 2. Kew Garden
R. Joseph Paxton 3. Versailles Garden
S. Frederick Law Olmstead 4. Crystal Palace
5. Central Park
(A) P-3,Q-1, R-4, S-2 (B) P-5, Q-3, R-4, S-2 (C) P-3,Q-1, R-2, S-5 (D) P-2,Q-3, R-4,
S-5
Answer: (D) P-2, Q-3, R-4, S-5
The Well-tempered Garden is a book by Christopher Hamilton Lloyd, (1921 –2006)
who was a British gardener and author. Other books by him are
The Mixed Border
Clematis
Dear Friend and Gardener
Color for adventurous gardeners
The cottage garden
Christopher Lloyd’s flower garden
Exotic Planting for Adventurous Garden.
Kew Garden- The Royal Botanic Gardens, Kew, usually referred to as Kew Gardens,
comprises 121 hectares of gardens and botanical glasshouses between Richmond and
Kew in Richmond upon Thames in southwest London, England. It was designed by
Lancelot ‘Capability’ Brown. It’s a World Heritage Site.
English Gardeners (Landscape planners): William Kent → Lancelot ‘Capability’
Brown → Humphrey Repton
English Landscape Gardens
Lines were no longer straight.
Paths curve and wander.
Parterres are replaced by grass. (Parterre: An ornamental flower garden having
the beds and paths arranged to form a pattern)
Trees were planted in clusters rather than in straight lines.
Rounded lakes replaced the rectangular ponds of the earlier style.
The garden became open, a park joining the house to the outside world rather
than a carefully nurtured refuge from it.
This natural style begun by Kent evolved into the “landscape garden” under Kent’s
pupil and son-in-law, Lancelot “Capability” Brown. Brown, whose curious nickname
came from his habit of telling prospective clients that their gardens showed “great
capabilities”, had an enormous effect upon the course of English gardening and
architectural style. The greatest surviving landscape gardens by Capability Brown are
at Longleat (Wiltshire), Burghley House (Lincolnshire), Petworth House (West Sussex),
and Blenheim Palace (Oxon).

Versailles Garden- (15,000 ha)Rennaissance Period- Andre Le Notre was a French


landscape architect and the principal gardener of King Louis XIV of France who
designed Versailles Garden. (This is the most famous garden ever designed)

Crystal Palace- Exhibition hall designed by Joseph Paxton for Great London
Exhibition of 1851 in Hyde Park, London. Charles Fox was the structural Engineer.
Central Park – Central Park is a public park at the center of Manhattan in New York
City, USA, designed by Frederick Law Olmstead.
The World’s biggest natural flower garden- Dubai Miracle Garden, Dubai.

Rock Garden, Chandigarh designed by Nek Chand.


Vrindavan Garden, Mysore by G.H. Krumbigal
Q.39 Match the organism type from Group I with the appropriate example from Group II
Group I Group II
P. Autotroph 1. Nitrifying Bacteria
Q. Heterotroph 2. Grasshopper
R. Chemotroph 3. Grass
S. Saprophyte 4. Vulture
5. Fungus
(A) P-5, Q-4, R-1, S-2 (B) P-2, Q-1, R-5, S-4 (C) P-1, Q-2, R-4, S-5 (D) P-3, Q-2,
R-1, S-5
Answer (D) P-3, Q-2, R-1, S-5
Autotroph- they produce their food of their own with the help of light(plants and
algae) or inorganic chemical reaction
Heterotroph- organisms that cannot make their own food . (animals)
Chemotroph- any organism that oxidizes inorganic or organic compounds as its
principal energy source. They are not dependent on solar energy. They are found on
ocean floor, soil and
Saprophyte- Saprotrophs is the correct word instead of saprophytes because phytes
means plants. Saparotrophs take their nutrition from dead and decaying matter by
dissolving them and absorbing through their body surface.Xerophyte- plants that
grow in desert.
Vultures are scavengers.

Photosynthesis Chemosynthesis
Using light energy for Obtain energy by the
synthesis of organic oxidation of electron
material donars in their
environment

Q.40 Match the concepts in Group I with their corresponding authors in Group II
Group I Group II
P. Proxemics Theory 1. Gordon Cullen
Q. Serial Vision 2. Edward T. Hall
R. Urban Imageability 3. Oscar Newman
S. Defensible Space 4. Paul Zucker
5. Kevin Lynch
(A) P-2, Q-1, R-5, S-3 (B) P-2, Q-1, R-3, S-4 (C) P-4, Q-1, R-5, S-2 (D) P-3, Q-
5, R-2, S-1
Answer (A): P-2, Q-1, R-5, S-3
Proxemic theory: Edward T. Hall, the cultural anthropologist who coined the
term in 1963, is a subcategory of the study of nonverbal communication.
Proxemics
Intimate distance for embracing, touching or whispering:
Close phase – less than 6 inches (15 cm)
Far phase – 6 to 18 inches (15 to 46 cm)
Personal distance for interactions among good friends or family members:
Close phase – 1.5 to 2.5 feet (46 to 76 cm)
Far phase – 2.5 to 4 feet (76 to 120 cm)
Social distance for interactions among acquaintances
Close phase – 4 to 7 feet (1.2 to 2.1 m)
Far phase – 7 to 12 feet (2.1 to 3.7 m)
Public distance used for public speaking
Close phase – 12 to 25 feet (3.7 to 7.6 m)
Far phase – 25 feet (7.6 m) or more.
Haptics (touch) - Haptics is the study of touching as nonverbal communication,
and haptic communication refers to how people and other animals communicate
via touching.
Kinesics (body movement)- Kinesics is the area of nonverbal communication
related to movements of the body, including gestures, posture, and facial
expressions, and the study of that area. The word was first coined by Ray
Birdwhistell

Serial Vision: English architect and urban designer Gordon Cullen developed the term
serial vision to describe what a pedestrian experiences when moving through a built
environment. (Townscape)
Occupied territory: « Shade, shelter, amenity and convenience are the usual causes of
possession. The furniture of possession includes floorscape, posts, canopies,
enclaves, focal points and enclosures ». (Cullen, 1971, p.23)
Viscosity: « Where there is a mixture of static possession and possession in
movement, we find what may be termed viscosity: the formation of groups
chatting, of slow window-shoppers, people selling newspapers and so on. »
(Cullen, 1971, p.24)
Enclave: « The enclave or interior open to the exterior and having free and
direct access from one to the other is seen here as an accessible place or room
out of the main directional stream » (Cullen, 1971, p.25)
Enclosure: « It is the basic unit of the precinctual pattern ; outside, the noise
and speed of impersonal communication which comes and goes but is not of
any place. Inside, the quietness and human scale of the square, quad or
courtyard ». (Cullen, 1971, p.25)
Focal point: « Coupled with enclosure (the hollow object) is the focal point, the
vertical symbol of congregation. In the fertile streets and market places of town
and village it is the focal point (be it column or cross) which crystallizes the
situation, which confirms ‘this is the spot. Stop looking, it is here ». Cullen, 1971,
p .26)
Urban Imageability: “Image of City”
by Kevin Lynch
1. District: Area of homogeneous character recognized by clues.
2. Node: Strategic foci into which observer can enter.
3. Landmark: Important location.
4. Pathway: Act as lateral reference and often path as well.
5. Edge: perceived boundaries such as walls, buildings, and shorelines.

Defensible Space: The defensible space theory of architect and city planner Oscar
Newman encompasses ideas about crime prevention and neighborhood safety. The
theory developed in the early 1970s, and he wrote his first book on the topic,
‘Defensible Space’ in 1972. Newman focused on explaining his ideas on social
control, crime prevention, and public health in relation to community design.

Q.41 If the area coverage of one sprinkler is 20 m², with a maximum and minimum spacing of
4.6 m and 1.8 m respectively, the minimum number of sprinklers required to be
arranged in a regular orthogonal grid to cover the area of a 15 m x 20 m room would be
_______.
Answer: 20
Sprinkler has a circular coverage, so a sprinkler covering 20 m² will have a radius of
π r²= 20

r= (solving for ‘r’ we get r = 2.52 m)

So the sprinkler will be placed at a distance of m


= 1.75 m
We place the sprinkler at all the corner of the room at a distance of 1.75 m from each
wall.
Now at the longer side (20 m) space remaining is 20 m – (1.75 ×2)
= 16.5 m

16.5 is now divided in 4 spaces , so we get = 4.125 m

If it is divided by 3 the spacing comes = 5.5 ( it exceeds the mandatory maximum


spacing of 4.6 m)
For 4 spaces we need 3 sprinklers, and two at corner is already placed in the longer
side, so there will be 5 sprinkler on the longer side.
Similarly for shorter side (15m) space remaining is 15m - (1.75 ×2)
= 12.5 m

12.5 is now divided in 3 spaces , so we get = 4.17 m


For 3 spaces we need 2 sprinklers, and two at corner is already placed in the longer
side, so there will be 4 sprinkler on the shorter side
Total no of sprinkler 5 × 4= 20 sprinkler
Q.42 If the slope of a hipped roof is 60 degrees and height of the roof is 3 m, span of the
room, in m, would be ……………….. ?
Ans: 3.46 m
Q.43 Volume of coarse aggregate in m3 present in 1.0 m³ of 1 : 1.5 : 3 concrete mix made by
volume batching is _______.
(Answer= 0.5 to 0.6 m³)
Method I
In concrete mixing the void of coarse aggregate (stone chips) is filled by fine aggregate
(sand) and void created by fine aggeegate is filled by cement, so the volume of
concrete is the volume of coarse aggregate (stone chips) taken.
1 : 1.5 : 3 represent cement: sand: stone chips

So, x 1= = 0.5 m3
Method II
Suppose we take 1 m3 of cement, 1.5 m3 of sand and 3 m3 of coarse aggregate, total
dry volume makes 5.5 m3.

Wet volume = = 3.57


3.57 has 3 of coarse aggregate

1 has of coarse aggregate (unitary method)


= 0.84 of coarse aggregate

Q.44 A tank of internal dimension 3 m x 5 m x 4 m (Length x Breadth x Height) has 200 mm


thick brick wall on all sides. Volume of brickwork in m3 would be _______.
Answer: 13.44 m3
External dim= 3.4m x 5.4m x 4m
Internal dim = 3 m x 5 m x 4 m
Ext. volume – Int. volume = (3.4 x5.4 x 4 – 3x5 x4 ) = 13.44 m3

Q.45 Flux emitted from a 1 cd light source in all directions, in lumens, would be _______.
Ans: 12.57 lumens
1 lumen = 1 cd.sr
A full sphere has a solid angle of 4 π steradian, so a light source that uniformly radiates
one candela in all direction has a total luminous flux of = 1 cd . 4 π sr
= 4 π cd. Sr
= 12.57 lumens

Q.46 50 Hectare of residential sector has 65% buildable area. The FAR of the buildable area
is 1.5. Within the residential sector, 60% of dwelling units are of area 100 m² each and
40% of the dwelling units are of area 80 m² each. The gross residential density, in
dwelling units per Hectare, would be _______.
Answer: (105 to 107 dwelling units/ hectare )

Total buildable area is 60 % of 50 Ha = 50 x = 32.5 ha.


Total area of all the floors = 32.5 x 1.5
= 48.75 ha = 48.75 x 10,000 = 4,87,500 m2
Let total no. of dwelling units be ‘d’
60 % of dwelling units are of 100 m2 each = 0.6d x 100 = 60d
40 % of dwelling units are of 80 m2 each = 0.4d x 80 = 32d
92d = 4,87,500 m2
Solving for ‘d’ we get d= 5299 units

Gross residential density, in dwelling units per Hectare = = 105 dwellings units/ ha
Q.47 In the given project network diagram, the total slack for job A in days would be _______.
Answer: 1 day

Common Data Questions


Common Data for Questions 48 and 49:
The scale of a contour map is 1:10,000 and the contour interval is 5 m. Distance between
two given points on the map is 2 cm and the elevation difference between the two given
points is 10 m.

Q.48 The actual distance between the two given points in ‘m’ would be
(A) 2 (B) 20 (C) 200 (D) 2000
Answer: (C) 200
Soln: scale of the map 1: 10,000
Distance on map = 2cm , so actual distance 2cm × 10,000= 20,000 cm
= 200 m
Q.49 The slope between two given points in percentage is
(A) 5 (B) 10 (C) 15 (D) 20
Answer: (A) 5

Slope = × 100 = 5 %

Common Data for Questions 50 and 51:

A point load of 3kN acts at mid-span of a 4 m long cantilever beam as shown in figure
below.

Q.50 Shearing force at free end in kN is


(A) 0 (B) 3 (C) 6 (D) 12
Answer: (A) 0

Q.51 Bending moment at mid-span in kNm is


(A) 0 (B) 2 (C) 4 (D) 6
Answer: (A) 0
Linked Answer Questions
Statement for Linked Answer Questions 52 and 53:
Cost of a new building is Rs 10,00,000 and its scrap value after 50 years is Rs. 1,00,000.
Using straight line method
Q.52 The annual depreciation of the building in Rs. Would be
(A) 10,000 (B) 15,000 (C) 18,000 (D) 20,000
Answer: (C) 18,000

D= where C is original cost, s is salvage or scrap value and n is life of property


in years.

= = 18,000

Q.53 The book value after 10 years in Rs. would be


(A) 1,80,000 (B) 3,60,000 (C) 6,00,000 (D) 8,20,000
Answer: (D) 8,20,000
The building is depreciating @ Rs 18,000 per year, and in 10 years it will deprecate Rs
18,000 × 10 = 1,80,000
So Book Value 10,00,000- 1,80,000= Rs 8,20,000
Book Value- For buildings, book value is the original cost of the building, minus
depreciation.

Statement for Linked Answer Questions 54 and 55:


A room of size 100 is illuminated by 10 lamps of 40 W having a luminous efficacy
of 50 lm/W.
Q.54 Total flux emitted by the lamps in lumens would be
(A) 2,000 (B) 5,000 (C) 10,000 (D) 20,000
Answer: (D) 20,000
Total Lumens emitted by one lamp= 40 × 50= 2,000 Lumens
For 10 lamps total flux= 2,000× 10= 20,000 lumens

Q.55 If utilization factor is 0.5, at a working height of 90 cm above the floor level, the
illumination in lux would be
(A) 100 (B) 200 (C) 500 (D) 1000
Answer: (A) 100
Utilization factor = 0.5 , so total effective lumens will be 20,000 × 0.5= 10,000 lumens

Illumination in Lux = = = 100 Lux.


(Note: It is not a point source. If it was appoint source and a single lamp then
calculation would be different)

General Aptitude (GA) Questions


Q. 56 – Q. 60 carry one mark each.
Q.56 A number is as much greater than 75 as it is smaller than 117. The number is:
(A) 91 (B) 93 (C) 89 (D) 96
Answer: (D) 96

Q.57 The professor ordered to the students to go out of the class.


I II III IV
Which of the above underlined parts of the sentence is grammatically incorrect?
(A) I (B) II (C) III (D) IV
Answer: (B) II

Q.58 Which of the following options is the closest in meaning to the word given below:
Primeval
(A) Modern (B) Historic (C) Primitive (D) Antique
Answer: (C) Primitive

Q.59 Friendship, no matter how _________it is, has its limitations.


(A) cordial (B) intimate (C) secret (D) pleasant
Answer: (B) intimate

Q.60 Select the pair that best expresses a relationship similar to that expressed in the pair:
Medicine: Health
(A) Science: Experiment (B) Wealth: Peace (C) Education: Knowledge (D) Money:
Happiness
Answer: (C) Education: Knowledge

Q. 61 to Q. 65 carry two marks each.


Q.61 X and Y are two positive real numbers such that 2X+Y≤6 and X+2Y≤8 . For which of the
following values of (X,Y)the function f(X,Y)=3X+6Y will give maximum value?
(A) (4/3, 10/3) (B) (8/3, 20/3) (C) (8/3, 10/3) (D) (4/3, 20/3)
Answer: (A) (4/3, 10/3)

Q.62 If |4X-7|=5 then the values of 2|X|-|-X| is:


(A) 2, 1/3 (B) 1/2, 3 (C) 3/2, 9 (D) 2/3, 9
Answer: (B) 1/2, 3

Q.63 Following table provides figures (in rupees) on annual expenditure of a firm for two years
– 2010 and 2011.

In 2011, which of the following two categories have registered increase by same
percentage?
(A) Raw material and Salary & wages (B) Salary & wages and Advertising
(C) Power & fuel and Advertising (D) Raw material and Research & Development
Answer: (D) Raw material and Research & Development
Raw Material= 6240- 5200= 1040

% Increase=

Power and Fuel = 9450- 7000= 2450

% Increase=

Salary and Wages= 12600-9000= 3600

% Increase=

Plant & Machinery= 25000- 20000= 5000

% Increase=
Advertising= 19500- 15000= 4500

% Increase=

Research and Development= 26400-22000= 4400

% Increase=

Q.64 A firm is selling its product at Rs. 60 per unit. The total cost of production is Rs. 100 and
firm is earning total profit of Rs. 500. Later, the total cost increased by 30%. By what
percentage the price should be increased to maintain the same profit level.
(A) 5 (B) 10 (C) 15 (D) 30
Answer: (A) 5

Q.65 Abhishek is elder to Savar. Savar is younger to Anshul.


Which of the given conclusions is logically valid and is inferred from the above
statements?
(A) Abhishek is elder to Anshul (B) Anshul is elder to Abhishek
(C) Abhishek and Anshul are of the same age (D) No conclusion follows
Answer: (D) No conclusion follows
GATE 2014
General Aptitude
Q. 1 – Q. 5 carry one mark each.

.1 A student is required to demonstrate a high level of comprehension of the subject,


especially in the social sciences. The word closest in meaning to
comprehension is
(A) understanding (B) meaning (C) concentration (D) stability
Answer: (A) understanding

.2 Choose the most appropriate word from the options given below to complete the
following sentence. One of his biggest ______ was his ability to forgive.
(A) vice (B) virtues (C) choices (D) strength
Answer: (B) virtues

.3 Rajan was not happy that Sajan decided to do the project on his own. On
observing his unhappiness, Sajan explained to Rajan that he preferred to work
independently. Which one of the statements below is logically valid and can be
inferred from the above sentences?
(A) Rajan has decided to work only in a group.
(B) Rajan and Sajan were formed into a group against their wishes.
(C) Sajan had decided to give in to Rajan’s request to work with him.
(D) Rajan had believed that Sajan and he would be working together.
Answer: (D) Rajan had believed that Sajan and he would be working together.

Q.4 If y = 5x2 + 3, then the tangent at x = 0, y = 3


(A) passes through x = 0, y = 0 (B) has a slope of +1
(C) is parallel to the x-axis (D) has a slope of −1
Answer: (C) is parallel to the x-axis

.5 A foundry has a fixed daily cost of Rs 50,000 whenever it operates and a variable
cost of Rs 800Q, where Q is the daily production in tonnes. What is the cost of
production in Rs per ton for a daily production of 100 tonnes?
Answer: 1300
Q. 6 – Q. 10 carry two marks each.

Q.6 Find the odd one in the following group: ALRVX, EPVZB, ITZDF, OYEIK
(A) ALRVX (B) EPVZB (C) ITZDF (D) OYEIK
Answer: (D) OYEIK

.7 Anuj, Bhola, Chandan, Dilip, Eswar and Faisal live on different floors in a six-storeyed
building (the ground floor is numbered 1, the floor above it 2, and so on). Anuj lives on
an even-numbered floor. Bhola does not live on an odd numbered floor. Chandan does
not live on any of the floors below Faisal’s floor. Dilip does not live on floor number 2.
Eswar does not live on a floor immediately above or immediately below Bhola. Faisal
lives three floors above Dilip. Which of the following floor-person combinations is
correct?
(A) (B) (C) (D)
Anuj 6 2 4 2
Bhola 2 6 2 4
Chandan 5 5 6 6
Dilip 3 1 3 1
Eswar 4 3 1 3
Faisal 1 4 5 5
Answer: (B)

.8 The smallest angle of a triangle is equal to two thirds of the smallest angle of a
quadrilateral. The ratio between the angles of the quadrilateral is 3:4:5:6. The
largest angle of the triangle is twice its smallest angle. What is the sum, in
degrees, of the second largest angle of the triangle and the largest angle of the
quadrilateral?
Answer: 180

.9 One percent of the people of country X are taller than 6 ft. Two percent of the
people of country Y are taller than 6 ft. There are thrice as many people in
country X as in country Y. Taking both countries together, what is the percentage
of people taller than 6 ft?
(A) 3.0 (B) 2.5 (C) 1.5 (D) 1.25
Answer: (D) 1.25

Q.10 The monthly rainfall chart based


on 50 years of rainfall in Agra is shown in the following figure. Which of the
following are true? (k percentile is the value such that k percent of the data fall
below that value)
(i) On average, it rains more in July than in December
(ii) Every year, the amount of rainfall in August is more than that in January
(iii) July rainfall can be estimated with better confidence than February rainfall
(iv) In August, there is at least 500 mm of rainfall

(A) (i) and (ii) (B) (i) and (iii)


(C) (ii) and (iii) (D) (iii) and (iv)

Answer: (B) (i) and (iii)

Q. 1 – Q. 25 carry one mark each.

Q.1 Toothing is a construction technique used in


(A) Wood construction (B) Steel construction
(C) Reinforced cement concrete construction (D) Brick masonry
Answer: (D) Brick masonry
Toothing: Cutting or leaving out of alternate masonry units in a wall to provide a bond
for new work.

Q.2 ‘Skeleton and Skin’ concept in building design and construction evolved
during the
(A) Roman period (B) Renaissance period (C) Gothic period (D)
Greek period
Answer: (C) Gothic period

.3 As per the IRC standards, the minimum width (in m) of a two lane urban
carriageway without a raised kerb is
(A) 6.0 (B) 6.5 (C) 7.0 (D) 8.0
Answer: (C) 7.0
The roads are grouped as Single lane (3.5 m), intermediate lane (5.5 m),
Double lane (7m with kerb or 7.5 m without kerb) and multiple lanes (addl. 3.5
m for each lane) depending upon the width of Carriageway. On village roads,
the width may be restricted to 3 m.
Government Roads
NH: Main highways running through the length and breadth of the Country
connecting major ports, State Capitals, large industrial and tourist center.
SH: Arterial routes of a State linking District Headquarters and important cities
within the State and connecting them with NH or Highways of the neighbouring
States.
MDR: Major District Road, Important roads within a district serving areas of
production and markets and connecting those with each other or with main
highways
ODR: Other District Road, Roads serving rural areas of production and
providing them with outlet to market centers, taluk headquarters, Block
Development headquarters or other main roads.
Village Road

Q.4 Pritzker Architecture Prize 2013 has been awarded to


(A) Mario Botta (B) Toyo Ito (C) Rem Koolhaas (D) Arata
Isozaki
Answer: (B) Toyo Ito

Q.5 Hip roof is formed by surfaces sloping in


(A) One direction (B) Two directions (C) Three directions (D)
Four directions

Answer : (D) Four directions


Q.6 Hiroshima Peace Memorial Museum in Japan has been designed by
(A) Kenzo Tange (B) Kisho Kurokawa (C) Tadao Ando (D) I M Pei
Answer: (A) Kenzo Tange
The Hiroshima Peace
Memorial Museum is museum located in Hiroshima Peace Memorial Park, in central
Hiroshima, Japan dedicated to documenting the atomic bombing that occurred with the
additional aim of world peace. The museum was established in August 1955 with the
Hiroshima Peace Memorial Hall (now the International Conference Center Hiroshima)
Raj Ghat, New Delhi: Vanu G Bhuta
One World Trade Centre (Freedom Tower) New York- Daniel Libeskind

.7 In AutoCAD, the maximum number of points which can be snapped in a circle


using OSNAP command is
(A) 1 (B) 3 (C) 4 (D) 5
Answer: (D) 5

Q.8 Development authorities in India are established under the provision of


(A) Municipal Act (B) 74th Constitutional Amendment Act
(C) Town and Country Planning Act (D) Land Acquisition Act
Answer: (C) Town and Country Planning Act
74th Constitutional Amendment Act, 1992 has been enacted to accord constitutional
recognition to the Urban Local Bodies as third tier of Government.
Land Acquisition is the process through which the Govt. can acquire the Private
land for Public Purpose. Land Acquisition Act of 1894 was replaced by Right to Fair
Compensation and Transparency in Land Acquisition, Rehabilitation and
Resettlement Act, 2013.
Salient features of this act:
Public Purpose Defined
For strategic purposes relating to naval, military, air force, and armed forces of
the Union, including central paramilitary forces or any work vital to national
security or defence of India or State police, safety of the people; or For
infrastructure projects, which includes the following, namely: All activities or
items listed in the notification of the Government of India in the Department of
Economic excluding private hospitals, private educational institutions and
private hotels;
Projects involving agro-processing, supply of inputs to agriculture,
warehousing, cold storage facilities, marketing infrastructure for agriculture and
allied activities such as dairy, fisheries, and meat processing, set up or owned
by the appropriate Government or by a farmers’ cooperative or by an institution
set up under a statute;
Project for industrial corridors or mining activities, national investment and
manufacturing zones, as designated in the National Manufacturing Policy;
Project for water harvesting and water conservation structures, sanitation;
Project for Government administered, Government aided educational and
research schemes or institutions;
Project for sports, heath care, tourism, transportation of space programme;
Any infrastructure facility as may be notified in this regard by the Central
Government and after tabling of such notification in Parliament;
Project for housing, or such income groups, as may be specified from time to
time by the appropriate Government;
Project for planned development or the improvement of village sites or any site
in the urban areas or provision of land for residential purposes for the weaker
sections in rural and urban areas;
Project for residential purposes to the poor or landless or to persons residing in
areas affected by natural calamities, or to persons displaced or affected by
reason of the implementation of any scheme undertaken by the Government,
any local authority or a corporation owned or controlled by the State.
Protection of Agricultural Land
Act requires that wherever multi crop irrigated land is acquired an equivalent area of
cultivable wasteland shall be developed by the state for agricultural purposes.
Compensation
At least four times the market value for land acquired in rural areas;
At least two times the market value for land acquired in urban areas
Rehabilitation
Both Economic and Social Rehabilitation to be covered.

Q.9 In escalators, the angle of inclination with the horizontal plane should be in the
range of
(A) 10°- 20° (B) 20°- 30° (C) 30°- 35° (D) 35°- 45°
Answer: (C) 30°- 35°

.10 As per the Census of India 2011, ‘Metropolitan Urban Agglomeration’ is a


contiguous spread of several urban settlements where the minimum population
size (in Lakh) is
(A) One (B) Five (C) Ten (D) Fifty
Answer : (C) Ten
India Percentage Urbanization (2011) = 32.16 %
Maharashtra has the highest no of people living in Urban Areas= 5.08 crore
Delhi is highest urbanized state = 97.50 %
Town (Definition): For the Census of India 2011, the definition of urban area is
as follows;
All places with a municipality, corporation, cantonment board or notified town area
committee, etc.
All other places which satisfied the following criteria:
a. A minimum population of 5,000;
b. At least 75 per cent of the male main working population engaged in non-
agricultural pursuits; and
c. A density of population of at least 400 persons per sq. km.
Urban Agglomeration (UA): An urban agglomeration is a continuous urban spread
constituting a town and its adjoining outgrowths (Ogs), or two or more physically
contiguous towns together with or without outgrowths of such towns. An Urban
Agglomeration must consist of at least a statutory town and its total population (i.e. all
the constituents put together) should not be less than 20,000 as per the 2001
Census.
Class I Uas/Towns: The Uas/Towns are grouped on the basis their population in
Census. The Uas/Towns which have at least 1,00,000 persons as population are
categorised as Class I UA/Town. At the Census 2011, there are 468 such
Uas/Towns. 264.9 million persons, constituting 70% of the total urban population, live
in these Class I Uas/Towns. The proportion has increased considerable over the last
Census. In the remaining classes of towns the growth has been nominal.
Million Plus Uas/Towns: Out of 468 Uas/Towns belonging to Class I category, 53
Uas/Towns each has a population of one million or above each. Known as Million
Plus Uas/Cities, these are the major urban centres in the country. 160.7 million
persons (or 42.6% of the urban population) live in these Million Plus Uas/Cities.18
new Uas/Towns have been added to this list since the last Census.
Mega Cities: Among the Million Plus Uas/Cities, there are three very large Uas with
more than 10 million persons in the country, known as Mega Cities. These are
Greater Mumbai UA (18.4 million), Delhi UA (16.3 million) and Kolkata UA (14.1
million).

Q.11 BEES is an acronym for


(A) Building for Environmental and Economic Sustainability
(B) Built Environment and Ecological Society
(C) Building for Energy and Environment Sustainability
(D) Built Environment and Engineering Services
Answer: (A) Building for Environmental and Economic Sustainability
Developed by the Building and Fire Research Laboratory of the National Institute of
Standards and Technology (NIST), USA.

Building for Environmental and Economic Sustainability (BEES) is an automated


approach for measuring the life-cycle environmental and economic performance of a
building product. The BEES software analyzes all stages in the life of a product,
including raw-material acquisition, manufacture, transportation, installation, use,
recycling and waste management. Economic performance is measured using the
ASTM standard life-cycle cost method, including initial investment, replacement,
operation, maintenance and repair, and disposal; and environmental and economic
performance are combined into an overall performance measurement.

Q.12 In a single-stack system of plumbing


(A) All the appliances and traps are fully ventilated
(B) Only WC branches are connected with anti- siphonage pipes
(C) Anti-siphonage pipes are omitted
(D) Only the stack is vented above the branch
connection at each floor level
Answer : (C) Anti-siphonage pipes are omitted

Q.13 The maximum bending moment (kNm) in a simply supported


beam of 8 m span subjected to a uniformly distributed load of 20 kN/m (inclusive
of its self-weight) over the entire span is
(A) 40 (B) 160 (C) 240 (D) 380
Answer : (B) 160

BM is maximum at centre = = = 160 kN

SF at dist ‘x’ Vx= w ( -x )


In this case Shear force at centre, i.e. x= is 0.

Q.14 Criteria for background noise (in NC) in


hospital
and apartments is
(A) 10 – 20 (B) 20 – 30
(C) 30 – 40 (D) 40 – 50
Answer : (B) 20 – 30
Noise Criterion – NC – were established in U.S. for rating indoor noise, noise from
air-conditioning equipment etc. It is different from dB. Consider the adjacent graph,
the lowest curve is for NC 15 and at different frequency different dB is obtained.
Maximum acceptable noise level as per Central Pollution Control Board (CPCB)
norms in India.

Day time Night time


Industrial area 75 70
Commercial area 65 55
Residential area 55 45
Silence Zone 50 40
Silence zone is referred as areas upto 100 meters around such premises as
hospitals,educational institutions and courts. Recommended NC (Noise Criterion) and
Equivalent Sound Level.

.15 As per the National Building Code, the minimum width (in m) of a staircase flight in
an educational building above 24 m height should be
(A) 1.0 (B) 1.5 (C) 2.0 (D) 2.5
Answer: (C) 2.0
S. Types of Building Minimum clear
No. width of stair case
1 Row Housing with 0.75 m
two storeys
2 Residential (A-2) 1.0 m
3 Residential (A-1), (A- 1.25 m
3) and (A-4)
4 Residential Hotel (A- 1.5 m
5) and (A-6)
5 Asssembly (up to 1.5 m
150 person)
6 Asssembly (more 2.0 m
than 150 person)
7 Educational Building 1.5 m
8 Institutional Building 2.0 m
9 All other occupancies 1.5 m
source : NBC 2016 (Part I) page 49

Q.16 Among the following, the one that is NOT a land assembly technique is
(A) Land Use Zoning (B) Accommodation Reservation(C) Town Planning Scheme (D)
Transfer of Development Right
Answer: (A) Land Use Zoning

Q.17 The Grand Gallery in Egyptian Architecture is provided only at


(A) Great Pyramid (B) Temple
(C) Mastaba (D) Bent Pyramid
Answer: (A) Great Pyramid
Mastbas were used for ordinary people while Pyramid was used for the Royals.
First European Pyramid discovered in Bosnia is considered the largest man made
pyramid on Earth. Bosnian Pyramid of the Sun or used to be called Visocica is
standing 220 meters high or 1/3 taller than the Giza. On the top of it, researchers
found medieval walled wreckages once belong to a Bosnian King Tvrtko of
Kotromanic who ruled the complex from 1338-1391.

.18 In the Taipei 101 building, the steel sphere as TMD (Tuned Mass Damper) is
suspended to reduce horizontal sway due to
(A) Settlement & Wind Load (B) Wind & Geothermal Load
(C) Seismic & Geothermal Load (D) Seismic & Wind Load
Answer : (D) Seismic & Wind Load
Suspended from the 92nd to the 87th floor, the pendulum sways to offset movements in
the building caused by strong gusts. Its sphere, the largest damper sphere in the
world, consists of 41 circular steel plates, Two additional tuned mass dampers, each
weighing 6 tonnes, are installed at the tip of the spire which help prevent damage to
the structure due to strong wind loads.
Q.19 ‘Finger Plan’ concept of urban planning was

initially adopted in
(A) Canberra (B) Paris (C) Copenhagen (D) Tokyo
Answer: (C) Copenhagen
Star Shaped City- Moscow, Washington D.C.
Linear City- Mumbai, Leningrad, Kolkata, Stalingrad
Ring form- Delhi, San Francisco
Sheet form- Tokyo
Poly Centred net- Detroit
Finger Plan- CopenhagenCopenhagen is the capital of Denmark. The Finger Plan is
Denmark’s town planning showcase, because it involves comprehensive concepts.
The infrastructure of roads and commuter railways forms the backbone of the Finger
Plan, but it covers the overall metropolitan development, i.e. where people will work,
live and have their service requirements met. The Finger Plan is historic – presented
in 1947 – but its key principles still apply to Denmark’s metropolis, Greater
Copenhagen. Greater Copenhagen would form a hand with the existing city as the
palm, while the fingers grew around commuter railways and highways. Shops,
schools and service organisations were placed around the local stations. Near them,
people lived close together in multi-storey blocks, while there was space for one-
family houses further away and open country was left between the fingers. Industry
was located at the transition between palm and finger and the industrial areas were
linked by ring railways and ring motorway.

Q.20 The most important property of concrete in its fresh state is


(A) Compressive strength (B) Tensile strength(C) Elastic modulus(D) Workability
Answer: (D) Workability
workability represents the amount of work which is to be done to compact the
concrete in a given mould. The desired workability for a particular mix depends
upon the type of compaction adopted and the complicated nature of
reinforcement used in reinforced concrete. A workable mix should not
segregate.
Compressive strength is checked using cube test on Universal Testing
Machine.
.21 An element constructed at intervals along the length of a wall to stabilize it
against overturning is
(A) Barrel Vault (B) Pilaster (C) Squinch Arch (D) Buttress
Answer: (D) Buttress
Flying Buttress was used in Gothic Architecture, Notredame Cathedral, Paris
to support the high standing walls.
The pilaster is an architectural element in classical architecture used to give
the appearance of a supporting column and to articulate an extent of wall, with
only an ornamental function.

Q.22 Landscape design of Shakti Sthal, the ‘Samadhi’ of late


Prime Minister Smt. Indira Gandhi, was done by architect
(A) Ram Sharma (B) Mohammad Shaheer
(C) Ravindra Bhan (D) Raj Rewal
Answer : (C) Ravindra Bhan
Ar. Ravindra Bhan, Landscape Architect and Ecological Planner studied Architecture
in London and later in America where he also completed his formal education in
Landscape Architecture and Ecological Planning. He was also awarded the
prestigious Agha Khan award for Mughal Sheraton Hotel Project in Agra India for his
outstanding contribution to Architecture and Landscape Architecture.
Raj Ghat Landscaping: Alick Percy- Lancaster
Rashtrapati Bhawan- Edward Leutyens

Q.23 Horizontally Wedge-shaped


Treads in stairways are termed as
(A) Stringers
(B) Winders
(C) Scotia
(D) Newel
Answer: (B) Winders
Q.24 The sequence of development in a Site-and- Services scheme is
(A) Land – Service – House – Occupant
(B) Occupant – Land – House – Service
(C) Occupant – Land – Service – House
(D) Land – Occupant – House – Service
Answer : (A) Land – Service – House – Occupant
SITES-AND-SERVICES: THE BASIC PRINCIPLES
The key components of a housing scheme are the plot of land, infrastructure (like
roads, water supply, drainage, electricity or a sanitary network), and the house itself.
Thus, the sites-and-services approach advocated the role of government agencies
only in the preparation of land parcels or plots with certain basic infrastructure,
which was to be sold or leased to the intended beneficiaries. The next step of actual
house building was left to the beneficiaries themselves to use their own resources,
such as informal finance or family labour and various other types of community
participation modes to build their house. The beneficiaries could also build the house at
their own phase, depending on the availability of financial and other resources.

Q.25 Which of the following is NOT a classical spatial theory of land use planning?
(A) Concentric Zone theory (B) Multiple Nuclei Theory (C) Centripetal Theory (D)
Sector Theory
Answer: (C) Centripetal Theory

THEORIES OF URBAN
STRUCTURE: Models to describe the development of cities.
ZONE THEORY (Earnest Burgess)
city grows outward in concentric rings
city has a single centre, otherwise known as the CBD (Central Business District)
Around the business centre is an area of older industry and beyond that are
residential areas
it is assumed that the poor cannot afford to commute long distances, and also that
they must live in the older and cheaper houses near the centre, so low class
residential are near the CBD
the wealthy live in a commuter zone outside the city proper
best describes the pre-automobile (pre 1920) pattern of North American cities but is
still useful today in describing patterns in the older parts of our towns
SECTOR MODEL (Homer Hoyt)
recognizes the existence of land use zones, but suggests that there are sectors or
wedges of land uses in the city.
due to the emergence of star-shaped transportation routes, such as bus lines and
streetcar lines.
as such, the industrial would lie in a sector along the rail lines coming into the city
centre poorer people live adjacent to industrial near their jobs
Rich live on the opposite side of town far from the industry and poor middle income in
between
MULTIPLE NUCLEI MODEL (Chauncy Harris and Edward Ullman)
refinement of first two, but incorporates outlying shopping malls, industrial areas and
large residential suburbs.
developed only with the use of automobiles, mostly since 1945.
CBD no longer has a monopoly on retail and commercial activities since outlying
malls and industrial parks compete with it.
industry also moves to the edge of the city where land is cheaper.
Q. 26 – Q. 55 carry two marks each.

.26 A housing project is proposed to be designed in a plot of 2 hectare. Maximum


permissible FAR is 2. The share of the numbers of dwelling units (DU) for MIG,
LIG and EWS is 1:2:3 having sizes of 55, 35 and 25 sq.m respectively. The
maximum number of DU which can be accommodated in the plot is
_______________
Answer: 1200
Soln: Since FAR is 2, maximum built up area for 2 ha plot will be 20,000 × 2=
40,000 m2
Let there be ‘x’ MIG, ‘y’ LIG, and ‘z’ EWS no of dwelling units.
55x +35y + 25z = 40,000 and x: y: z = 1: 2: 3

= and = therefore

x= and z= y

55 ) + 35 y + 25 ( y) = 40,000 solving this we get y = 400


Therefore MIG= 200, LIG= 400 and EWS = 600 ……………………..Total DW=
1200

Alternative Method
Let the number of MIG by ‘a’ therefore LIG will be ‘2a’ and EWS will be ‘3a’
Now equating the area 55 (a) + 35 (2a) + 25 (3a)
= 55a + 70a + 75a
200 a = 40,000
a= 200 (MIG) , 400 (LIG) and 600 (EWS)
Total Dwelling units = 1200

Q.27 Arrange the following in ascending order of width


P. Collector Street
Q. Arterial Road
R. Local Street
S. Sub-Arterial Road
(A) P, Q, S, R (B) R, P, S, Q (C) Q, S, R, P (D) Q, S, P, R
Answer : (B) R,
P, S, Q

Indian Road classification according to Indian Road Congress.


Non- Urban Urban Rural
Roads Roads Roads
Expresswys Arterial Other
district
roads
National Sub Village
Highways arterial Roads
State Collector
Highwys street
Major Local
District street
Roads

Min width of free walkway


Residential/Mixed landuse area= 1.8 m.(as per NBC and IRC)
Commercial/Mixed landuse area= 2.5 m
Shopping frontage= 3.5 to 4 m
Bus stop= 3 m
High Density Commercial= 4m

.28 Out of the following, the maximum points in the LEED (New construction) rating
system can be earned through
(A) Sustainable Sites (B) Water Efficiency (C) Materials and Resources (D)
Energy and Atmosphere
Answer: (D) Energy and Atmosphere
The LEED certification system is organized into five environmental categories:

1. Sustainable Sites (SS)


2. Water Efficiency (WE)
3. Energy and Atmosphere (EA)
4. Materials and Resources (MR)
5. Indoor Environmental Quality (IEQ).

An additional category, Innovation in Design (ID), addresses sustainable building


expertise as well as design measures not covered under the five environmental
categories.

Q.29 Which of the following is NOT a mechanism of bond resistance in reinforced


concrete?
(A) Chemical adhesion (B) Friction (C) Mechanical interlock (D)
Aggregate interlock
Answer : (D) Aggregate interlock

.30 A neighborhood has 250 units of 80 sq.m each and 200 units of 100 sq.m each. If
the mandatory parking requirement is one per 100 sq.m of built space then, the
total area(sq.m) required for parking considering 30 percent additional area for
circulation is ________________
Answer : 6240 to 6760 sq m (any value in between will be considered correct)
Total Built up space= (250 × 80) + (200 × 100) = 40,000 sq m

One parking space per 100 sq m , so no of parking space required= = 400


@ 12 sq m per car , total area = 12 ×400= 4800 and add 30 % additional circulation
area

4800 + (4800 × )= 6240 sq m

.31 A brick wall 19 cm thick has a thermal conductivity 0.811 W/m °C. The outside and
inside surface conductance of the wall are 16 W/m2 °C and 8 W/m2 °C
respectively, then the U-value of the wall in W/m2 °C is ____________
Answer: 2.37

U- value of wall = = = 4.26 W/m2 °C

R- value is inverse of U value, so R value of wall = m2 °C/W

Now adding all the R- value = + + (R value of surface conductance is


added to R value of wall)
= 0.0625 + 0.234 + 0.125
= 0.4215 m2 °C/W

U- value of composite wall= = = 2.37 W/m2 °C

Thermal conductivity (also known as Lambda) is the rate at which heat passes
through a material, measured in watts per square metre of surface area for a
temperature gradient of one kelvin for every metre thickness.This is expressed
as W/mK. Thermal conductivity is not affected by the thickness of the product.
(It is material specific)
Thermal transmittance, (known as the U-value) is a measure of the rate of heat
loss of a building component. The U-value is the sum of the combined thermal
resistances of all the elements in a construction, including surfaces, air
spaces, and the effects of any thermal bridges, air gaps and fixings. It is
expressed as W/m2K
Thermal resistance (known as R value) is the ability of a material to prevent
the passage of heat. It’s the thickness of the material (in metres) divided by its
conductivity. This is expressed as m2K/W.
( solution provided by VK Vidyarthee, B Arch (NIT Patna), M. Tech (Const. Management, IIT D))

Q.32 Match the contemporary buildings in Group I with their architects in Group II
Group I Group II
P. Vitra Design Museum, Basel 1. Adrian Smith
Q. Turning Torso, Malmö 2. Jean Nouvel
R. Burj Khalifa, Dubai 3. Herzog de Meuron
S. Tate Modern, London 4. Santiago Calatrava
5. Frank O Gehry
(A) P-3, Q-4, R-2, S-5 (B) P-5, Q-4, R-1, S-3 (C) P-5, Q-3, R-1, S-2 (D) P-5,
Q-3, R-2, S-1
Answer : (B) P-5, Q-4, R-1, S-3

Q.33 From the following cost components of a building construction project which is not a
direct cost combination?

P. Labour cost
Q. Equipment cost
R. Material cost
S. Establishment cost
T. Supervision cost

(A) P and Q (B) Q and R (C) P and R (D) S and T


Answer : (D) S and T

.34 A house located in Delhi has 111 m2 of flat terrace area (runoff coefficient = 0.85) and 55
m2 of ground area covered with grass (runoff coefficient = 0.15). If annual average
rainfall is 611.8 mm, then rain water harvesting potential (L/year) from runoff will be
______________
Answer: 62700 to 62800
Soln: Avg. annual rainfall= 611.8 mm= 0.6118 m
Total water harvested= 0.6118{(111× 0.85) + (55× 0.15)}
= 0.618{94.35 + 8.25}
= 0.618 (102.60)
= 63.4068 m3 = 63406 L/year
Q.35 Match the elements in Group I with the structures in Group II
Group I Group II
P. Harmika 1. Dilwara Temple, Mount Abu
Q. Sixteen Vidyadevis 2. Sun Temple, Modhera
R. Lat pillar 3.Stupa of Sanchi
S. Urushringa 4. Lauriya, Nandangarh
5.Great Kailash Temple, Ellora
(A) P-5, Q-1, R-4, S-3(B) P-1, Q-2, R-4, S-3 (C) P-3, Q-5, R-4, S-2 (D) P-3, Q-1,
R-4, S-2
Answer : (D) P-3, Q-1, R-4, S-2

.36 At a site, based on percolation test, the allowable rate of treated sewage
application was determined as 65 L/m2/day. The effective depth (m) of a soak pit
with a diameter of 2.5 m for the disposal of 1020 L/day of septic tank effluent is
______________
Answer: 1.98 to 2.02
Soln:
Let the depth of cylindrical pit be ‘h’ and radius = 1.25 m given
Total surface area for absorption= 2πrh
Total absorbed effluent= 2πrh ×65
Total effluent to be absorbed = 1200 L (in one day)

2πrh = , where r= 1.25 , solving for h we get it as 1.99 m.

Q.37 Match the AutoCAD command in Group I with their functions in Group II
Group I Group II
P. OOPS 1.Creates solid lines
Q. RAY 2. Restores an erased drawing
R. TRACE 3. Manages customized user interface elements
S. CUI 4.Creates semi-infinite line
5. Creates a five sided 3D solid with a sloped face tapering
along the X axis
(A) P-1, Q-3, R-2, S-5 (B) P-2, Q-4, R-1, S-3 (C) P-2, Q-1, R-3, S-4 (D) P-1,
Q-2, R-3, S-4
Answer: (B) P-2, Q-4, R-1, S-3

Q.38 Match the bridges in Group I with their structure type in Group II
Group I Group II
P.Harbour Bridge, Sydney 1. Simply Supported
Q.Golden Gate Bridge, San Francisco 2.Cable Stayed
R. Howrah Bridge, Kolkata 3.Arch
S. Millau Viaduct, Millau 4.Suspension
5.Cantilever
(A) P-3, Q-4, R-5, S-2 (B) P-5, Q-1, R-4, S-3 (C) P-2, Q-3, R-4, S-5 (D) P-1, Q-
2, R-3, S-4
Answer : (A) P-3, Q-4, R-5, S-2

Structure Image Examples


Type
Simply Rajendra Setu -
supported The road cum rail
bridge near
Mokama in Patna
Cable Bandra- Worli sea
Stayed link, Mumbai
Millau Viaduct,
France
Arch Bridge Sydney harbor
Bridge
Suspension The Akashi Kaikyō
Bridge in Japan,
world's longest
mainspan.
Cantilever Pont de Québec,
Canada
Howrah Bridge,
India
Minato Bridge,
Osaka, Japan
.39 The arithmetic average value of the sound absorption coefficient for a specific
material and particular mounting condition for four frequencies is
(A) Transmission coefficient (B) Noise reduction coefficient
(C) Absorption coefficient (D) Reflection coefficient
Answer: (B) Noise reduction coefficient

.40 A single span simply supported reinforced concrete beam (250 mm wide and 480
mm effective depth) is subjected to a concentrated load of 120 kN at its mid-
span. Neglecting self-weight of the beam, the nominal shear stress (Mpa) at the
support section is ______________
120kN
Answer: 0.5

R1= 60 kN R2= 60kN

Nominal shear stress at support = = = 0.5 Mpa


( Nominal shear: It is simply the shear force generated on the structure due to the imposed
force(vertical/horizontal) on a given cross sectional area. It is represented by Tv. The way of calculating the
nominal shear is Tv=V/bd.)
Maximum shear stress= 1.5 times the avg shear stress.

.41 The optimistic time, the pessimistic time and the most likely time of a job are 6, 13
and 8 days respectively. The variance for this job is _______________
Answer: 1.34 to 1.37

Variance = [

=(

= = 1.36
Optimistic time estimate: it is the smallest time taken to complete the activity if
everything is fine
Most likely time estimate: It refers to the estimate of the normal time

Pessimistic time estimate: it is the longest time that an activity would take
The square root of variance is called the standard deviation.

Expected time =

.42 A refuse collection system consisting of two chutes is to be provided in a 20 storied


residential building with 2 flats/floor (average family size = 5) and with each chute
serving one flat on each floor. Average quantity of refuse and its density are 880
g/person/day and 240 kg/m3 respectively. If the cleaning interval is two days, then the
minimum size of the refuse container (litre) at the bottom of each chute is
_______________
Answer: 730 to 740
Soln: For each chute 20 family will use, so total no of people = 20× 5= 100 person
Total amount of garbage generated = 100× 0.880 kg= 88 kg/day
Frequency of cleaning is 2 days so total garbage collected= 88 × 2= 176 kg

Density = here density given = 240 kg/m3

So, volume = = solving we get 0.733 m3 = 733 litre


Q.43 Match the features in Group I with their architectural periods in Group II
Group I Group II
P. Caryatids 1. Roman
Q. Hypocaust 2. Gothic
R. Pylons 3.Greek
S. Lofty Pinnacles 4. Egyptian
5.Romanesque
(A) P-1, Q-5, R-4, S-2 (B) P-5, Q-1, R-3, S-2 (C) P-3, Q-2, R-5, S-4 (D) P-3, Q-
1, R-4, S-2
Answer: (D) P-3, Q-1, R-4, S-2

A caryatid is a sculpted female figure


serving as an architectural support taking the place of a column or a pillar supporting
an entablature on her head. Some of the earliest known examples were found in the
treasuries of Delphi, dating to about the 6th century BC, but their use as supports in
the form of women can be traced back even earlier, to ritual basins, ivory mirror
handles from Phoenicia, and draped figures from archaic Greece.

A hypocaust (Latin hypocaustum) was an ancient Roman system of


underfloor heating, used to heat houses with hot air. Hypocausts were used for
heating hot baths (thermae), houses and other buildings, whether public or private.
The floor was raised above the ground by pillars, called pilae stacks, with a layer of
tiles, then a layer of concrete then another of tiles on top; and spaces were left inside
the walls so that hot air and smoke from the furnace would pass through these
enclosed areas and out of flues in the roof, thereby heating but not polluting the
interior of the room.
Pylon is the Greek term for a
monumental gateway of an Egyptian temple. It consists of two tapering towers, each
surmounted by a cornice, joined by a less elevated section which enclosed the
entrance between them.
Entrance nomenclature:
Japanese: torii
Chinese: paifang
Korean: hongsalmun
Egyptian: pylons
Buddhist Stupas: torana
South Indian temple: gopuram
Pinnacles: Decorative elements above the buttress.

Q.44 Match the following terminologies in Group I with their descriptions in Group II
Group I Group II
P. Pruning 1. Useful in reproducing plants that would not breed true if propogated
by seed
Q. Topiary 2. A live bud from a desired plant inserted into a host plant
R. Grafting 3. Cutting of evergreen shrubs into abstract or geometric shapes
S. Budding 4. Trimming and cutting of lawns
5. Selective cutting of plant branches for better growth
(A) P-5, Q-3, R-4, S-2 (B) P-2, Q-3, R-1, S-5 (C) P-3, Q-4, R-1, S-2 (D) P-5, Q-3,
R-1, S-2
Answer : (D) P-5, Q-3, R-1, S-2

.45 In a dance hall the indoor and outdoor temperatures are 28°C and 18°C respectively.
There is an internal heat gain of 5 kW and the specific heat of air (on volume basis) is
1300 J/m3 °C, then the necessary cross sectional area (m2) of a duct with an air
velocity of 2 m/s required for cooling by ventilation is ____________
Answer : 0.190 to 0.195
Soln: 5 kW of heat has to be extracted= 5000 W
Specific heat of air = 1300 J/m3 °C …….. it means for every m3 of air 1300 Joule is
extracted for 1 dec C temp difference.
Temp difference is 10 degree so for every m3 …………..1300× 10= 13,000 Joules of
energy is extracted.

In order to extract only 5000 Joules amount of air to be removed will be = 0.38
m3 per sec.
So necessary Cross section for 2m/sec air flow= a × 2 = 0.38
Solving ‘a’= 0.19 m2

Q.46 Assuming full compaction, strength of concrete is inversely proportional to


(A) Water – cement ratio (B) Water – sand ratio
(C) Water – coarse aggregate ratio (D) Water – fine aggregate ratio
Answer : (A) Water – cement ratio

Q.47 Match the terms in Group I with their examples in Group II


pI Group II
entive zoning 1.Boardwalk, Atlantic City
ersal design 2.Minneapolis, USA
menading 3. Broadway Theatre District, New York
way system 4.Pruitt Igoe Housing, St. Louis, Missouri
5.Curitiba, Brazil
(A) P-5, Q-3, R-2, S-1 (B) P-4, Q-5, R-1, S-3 (C) P-3, Q-5, R-4, S-2 (D) P-3, Q-
5, R-1, S-2
Answer: (D) P-3, Q-5, R-1, S-2
Incentive zoning allows developers more density in exchange for community
improvements. An increase in density encourages high density development
supportive of compact development. In exchange, the developer would be
encouraged to include some community improvements in their projects. Community
improvements may include additional open space, affordable housing, special
building features, or public art.

The term “universal design” was coined by the


architect Ronald L. Mace to describe the concept of designing all products and the
built environment to be aesthetic and usable to the greatest extent possible by
everyone, regardless of their age, ability, or status in life. Curitiba, Brazil with
approximately two million people has most accessible transport system, due to the
adoption of an Integrated Transport Network. Public transportation in general and
especially the solutions aimed at the disabled have been a priority in the city’s public
transportation system which is done through an Integrated Transportation Network.
Promenading is a stroll or walk, especially in a public place, as for pleasure or
display.
The Minneapolis Skyway System is an interlinked collection of enclosed pedestrian
footbridges that connects various buildings in Downtown Minneapolis enabling
people to walk in a climate-controlled environment.
St. Louis’s Pruitt-Igoe housing (a failed project) project is arguably the most
infamous public housing project ever built in the United States. A product of the
postwar federal public-housing program, this mammoth high-rise development was
completed in 1956. Only a few years later, disrepair, vandalism, and crime plagued
Pruitt-Igoe. The project’s recreational galleries and skip-stop elevators, once
heralded as architectural innovations, had become nuisances and danger zones.
Large numbers of vacancies indicated that even poor people preferred to live
anywhere but Pruitt-Igoe. In 1972, after spending more than $5 million in vain to cure
the problems at Pruitt-Igoe, the St. Louis Housing Authority, in a highly publicized
event, demolished three of the high-rise buildings. A year later, in concert with the
U.S. Department of Housing and Urban Development, it declared Pruitt-Igoe
unsalvageable and razed the remaining buildings.
.48 If yield stress of steel is 415 Mpa, then strain in tensile reinforcement at the limit state of
collapse shall be at least _____________. For steel, the Young’s Modulus, E = 2 X 105
Mpa.
Answer: 0.0037 to 0.0039
The maximum strain in the tension reinforcement section should not be less than

= 0.002 +

= 0.002 +
= 0.002 + 0 .0018 = 0.0038

Q.49 Match the books in Group I with their authors in Group II


Group I Group II
P. Architecture Now! 1. Ian Mc Harg
Q. Intentions in Architecture 2. Robert Venturi
R. Design with Nature 3. Christopher Alexander
S. Complexity & Contradictions in Architecture 4. Philip Jodidio
5. Christian Norberg Schulz
(A) P-2, Q-3, R-4, S-1 (B) P-4, Q-5, R-1, S-2 (C) P-2, Q-3, R-1, S-5(D) P-3, Q-
1, R-4, S-2
Answer: (B) P-4, Q-5, R-1, S-2

Q.50 Match the common names of the trees in Group I with their botanical names in Group
II
Group I Group II
P. Gulmohar 1. Dalbergia Sissoo
Q. Palash 2. Ficus Benghalensis
R. Indian Mahogany 3. Delonix Regia
S. Banyan 4.Toona Ciliata
5. Butea Monosperma
(A) P-5, Q-3, R-4, S-2 (B) P-4, Q-5, R-2, S-1 (C) P-3, Q-5, R-4, S-2 (D) P-3, Q-1,
R-5, S-2
Answer: (C) P-3, Q-5, R-4, S-2
Note: In Botanical name first name is genera and second is species.

Q.51 A room of internal dimension 4m x 5m x 3.5m (LxBxH) has 20 cm thick walls and two
doors of size 1m × 2m. The required area of Damp Proof Course (sq.m) is
_____________
Answer : 3.3
Soln: Solve using centre line method = (4.2 + 5.2) ×2 =18.8 m
Deduct 2 doors of 1 m each, so 18.8 -2 = 16.8
Total DPC = 16.8 × 0.2 = 3.36 sq.m

Q.52 A load of 30 kN is applied vertically downward at the free end of a cantilever of span 5 m.
If the elastic modulus of the cantilever is 30 Gpa and the section has a width of 0.3 m
and a depth of 0.6 m, then, the elastic deflection (in mm) is ______________
Answer: 7.65
Maximum Elastic deflection in Point load at the end of cantilever beam is given by δmax=

Where P= 30 kN = 30 × N, length (span)= 5m

E= 30 GPa= 30 × 109 Pa (Pascal is N/m2) = = 30 × 103 N/


I= = = 0.0054 m4

δmax= = = = = 7.71
mm
Soln provided by Alam Hasan (B. Arch, NIT Patna)

Q.53 Associate the plans in Group I with the options in Group II


Group I Group II
P. City Development Plan 1. PMGSY
Q. Slum Free City Plan 2. JNNURM
R. Transport Network Plan3. RAY
S. Disaster Management Plan 4. NDMA
5. RSVY
(A) P-2, Q-3, R-1, S-4 (B) P-2, Q-1, R-5, S-4 (C) P-1, Q-3, R-2, S-5 (D) P-3, Q-2,
R-1, S-4
Answer: (A) P-2, Q-3, R-1, S-4
Pradhan Mantri Gram Sadak Yojana or PMGSY is a nationwide plan in India
to provide good all-weather road connectivity to unconnected villages .
JNNURM- Jawahar Lal Nehru National Urban Renewal Mission aims at creating
‘economically productive, efficient, equitable and responsive Cities’ by a strategy of
upgrading the social and economic infrastructure in cities. This project runs under
Ministry of Urban Development, Govt. of India.
Rajiv Awas Yojana (RAY) (2013-22), is a scheme of the Government of India for
the benefit of poor in Urban areas. It is a Centrally Sponsored Scheme, (CSS), to
be implemented in mission mode during 2013-2022. The scheme aims to make
India slum-free by 2022 by providing people with shelter or housing free of cost.
NDMA: National Disaster Management Authority created by Disaster Management
Act of 2005 for disaster preparedness, mitigation and rescue operations.
RSVY: Rashtriya Sam Vikas Yojna was a major 10th Plan initiative of the
Government of India for bringing equity in development. The program had several
objectives – poverty alleviation, employment generation, improving productivities in
agriculture, lessening disparities by ensuring economic participation of hitherto
marginalized social groups and regions etc.

Q.54 The capacity of a hall is 600 persons and its volume is 3000 cu.m. If an optimum
reverberation time of 1.0 second is to be achieved then the required total absorption
(m2sabine) is _____________
Answer: 475 to 485
Soln: A standard reverberation time has been defined as the time for the sound to die
away to a level 60 decibels
below its original level.

Where RT60 = Reverberation time in sec


V = Volume in m³
Se = Total Absorption

1.0 =
Se = 0.16 × 3000 = 480 m2sabine
Q.55 A solid straight steel rod of diameter 100 mm is bent in single curvature into a circular arc
by a moment of 50 kNm applied at its ends. If elastic modulus, E, for steel is 2 x 105
Mpa, the radius of curvature (mm) of the arc assuming π=3.14 is _________
Answer: 19625

Moment Curvature Relationship is = where


ρ= radius of curvature
M= Momentum
E= Young Modulus of Elasticity
I= Inertia
Given rod dia = 100 mm
Moment = 50kNm
E= Young Modulus of Elasticity = 2× MPa
π= 3.14

I of circular cross section of rod =

= = 4906250

ρ= = = 19625

solution provided by : Prashant Anand (PhD scholar, NUS Singapore) B Arch (NIT Patna) M.Tech (IIT Madras )
GATE 2015
Questions 1-5 carry one mark each

.1 Choose the most appropriate word from the options given below to complete the
following sentence.
The principal presented the chief guest with a ……………….as token of appreciation.
(A) momento (B) memento (C) momentum (D) moment
Answer: (B) memento

.2 Choose the appropriate word/phrase, out of the four options given below, to
complete the following sentence:
Frogs……………………
(A) croak (B) roar (C) hiss (D) patter
Answer: (A) croak

Q.3 Choose the word most similar in meaning to the given word:
Educe
(A) Exert (B) Educate (C) Extract (D)Extend
Answer: (C)Extract

Q.4 Operators □ ,◊ and → are defined by : a □ b = ;a◊b= ; a→ b = ab


Find the value of (66 □ 6) → (66 ◊ 6 ).
(A) -2 (B) -1 (C) 1 (D) 2
Answer: (C) 1

Q.5 If logx(5/7) = -1/3, then the value of x is


(A) 343/125 (B) 125/343 (C) -25/49 (D) -49/25
Answer: (A) 343/125

Questions 6-10 carry two marks each


.6 The following question presents a sentence, part of which is underlined. Beneath
the sentence you find four ways of phrasing the underlined part. Following the
requirements of the standard written English, select the answer that produces
the most effective sentence.
Tuberculosis, together with its effects, ranks one of the leading causes of death in India.
(A) ranks as one of the leading causes of death
(B) rank as one of the leading cause of death
(C) has the rank of one of the leading cause of death
(D) are one of the leading causes of death
Answer: (A) ranks as one of the leading causes of death

Q.7 Read the following paragraph and choose the correct statement.
Climate change has reduced human security and threatened human well being. An
ignored reality of human progress is that human security largly depends upon
environmental security. But on the contrary, human progress seems contradictory to
environmental security. To keep up both at the required level is the challenge to be
addressed by one and all. One of the ways to curb the climate change may be
suitable scientific innovations, while other may be Gandhian perspective on small
scale progress with focus on sustainability.
(A) Human progress and security are positively associated with environmental
security.
(B) Human progress is contradictory to environmental security.
(C) Human security is contradictory to environmental security.
(D) Human progress depends upon environmental security.

Answer: (B) Human progress is contradictory to environmental security.

Q.8 Fill the missing value

Answer: 3, Vertical Central grid is sum of Left and Right grid, divided by 2

.9 A cube of side 3 units is formed using a set of smaller cubes of side 1 unit. Find
the proportion of the number of faces of the smaller cubes visible to those which
are not visible.
(A) 1 : 4 (B) 1 : 3 (C) 1: 2 D)2 : 3
Answer: (C) 1: 2

.10 Humpty Dumpty sits on a wall everyday while having lunch. The wall sometimes
breaks. A person sitting on the wall falls if the wall breaks.
Which one of the statements below is logically valid and can be inferred from the above
sentences?
(A) Humpty Dumpty always falls while having lunch.
(B) Humpty Dumpty does not fall sometimes while having lunch
(C) Humpty Dumpty never falls during dinner
(D) When Humpty Dumpty does not sit on the wall the wall does not break.
Answer: (B) Humpty Dumpty does not fall sometimes while having lunch

Q. 11-35 carry 1 mark each and Q 36-65 carry 2 marks each.

.11 A Housing Finance Institution in the private sector is


(A) HUDCO (B) SBI (C) PNB (D) HDFC
Answer: (D) HDFC
A Housing finance institution means the organisation which can give loan for
construction of a house or purchase of a house. Common institutions are Bank,
HUDCO, LIC.
HUDCO : Housing and Urban Development Corporation Limited was found in
1970 and acts as a techno-financial institution and is under the administrative
control of the Ministry of Housing and Urban Affairs.
SBI and PNB : Public Sector banks.
HDFC: Private Sector Bank. Other Private sector banks are ICICI, Yes Bank

Q.12 Which of the following statements regarding PERT is not true ?


(A) Each activity of PERT network has three different time estimates
(B) Expected activity time is estimated based on β- distribution
(C) PERT is a deterministic model
(D) PERT network may have more than one critical path
Answer: (C) PERT is a deterministic model
In CPM, (Critical Path Method) activity times are assumed to be predictable.
(“deterministic”).
In PERT, (Project Evaluation and Review Technique) activity times are
assumed to be random, with assumed probability distribution (“probabilistic”)

Q. 13Damage of foundations due to ‘Soil Liquefaction’ is related to


(A) Cyclones (B) Landslides (C) Floods (D) Earthquakes
Answer: (D) Earthquakes

Soil liquefaction
describes a phenomenon whereby a saturated or partially saturated soil substantially
loses strength and stiffness in response to an applied stress, usually earthquake
shaking or other sudden change in stress condition, causing it to behave like a liquid.
The adjoining picture is of Niigata Earthquake (Japan) 1964, in which buildings tilted
due to liquefaction of soil.
With respect to Earthquake India is divided into FOUR zones, Zone II, Zone III, Zone
IV and Zone V. Earlier there were five zones, now Zone I is merged with Zone II,
thereby reducing it to four. There are two types of waves generated during an
Earthquake ‘P’ and ‘S’ wave. ‘P’ wave (Primary wave) is compressional wave,
longitudinal in nature and travel fastest. Secondary waves (S-waves) are shear
waves that are transverse in nature.

P-wave S- Wave
Travels faster Travel slower than P
-wave
Compression Shear waves
and rarefaction
Longitudinal Transverse
Less Damage More damage
Q. 14Walls with high thermal inertia are suitable in which type of climate?
(A) Hot-dry (B) Hot-humid (C) Temperate (D) Cold
Answer: (A) Hot-dry

Q. 15 The ratio of town area to agricultural land area as


suggested by Sir Ebenezer Howard in ‘Garden City’ concept is
(A) 1 : 20 (B) 1 : 15 (C) 1: 10 (D)1 : 5
Answer: (D) 1 : 5
Garden city would house 32,000 people on a site of 6,000 acres (2,400 ha),
planned on a concentric pattern with open spaces, public parks and six radial
boulevards, 120 ft (37 m) wide, extending from the centre.
Letchworth, UK was the first garden city planned by Raymond Unwin.

Q. 16A ‘Demolition Contract’ for a building is awarded to the


(A) Lowest Bidder (B) Highest Bidder
(C) Second Lowest Bidder (D) Second Highest Bidder
Answer : (B) Highest Bidder
In a demolition contract a contractor demolishes the building and takes all the
scrap by paying a certain amount of money.
In a tender process lowest bidder is awarded the job.

Q. 17Bulking of sand is highest in


(A) Coarse sand (B) Medium sand (C) Fine sand (D) Sand saturated with water
Answer: (C) Fine sand
The options are confusing because of option D. Bulking is the phenomenon which occurs
for saturated sand. But it's highest in fine sand.
Increase in volume of fine aggregate due to the presence of moisture is called
bulking. During making of concrete appropriate measures should be taken when sand
is measured volume wise.

Q. 18The Venice Charter (1964) led to the establishment of


(A) International Centre for the Study of the preservation and Restoration of Cultural
Property (ICCROM)
(B) International Council of Monuments and Sites (ICOMOS)
(C) Indian National trust for Art and Cultural Heritage (INTACH)
(D) Archaeological Survey of India (ASI)
Answer : (B) International Council of Monuments and Sites (ICOMOS)
ICOMOS- International Council on Monuments and Sites. (Headquartered: Paris)
Athens Charter - Restoration of Historic Monuments (1931) - manifesto with 7
points.
Venice Charter- Conservation and restoration of monuments and sites. Total
16 articles.(1964)
Burra Charter: conservation and restoration of monuments and sites in
Australia.
Florence Charter- Historic Gardens.(1982)
Granada Convention: Convention for the Protection of the Architectural
Heritage of Europe.(1985)
History of Architectural Conservation.
Architectural Conservation started during the period of Modernism when sentimental
attachment for old buildings grew due to rapid advancement in building construction
system.
John Ruskin's (1819-1900, British writer) ideas on, preservation of open spaces and
conservation of historic buildings. The Society for the Protection of Ancient Buildings
(SPAB) was founded by William Morris (textile designer), Philip Webb and others, in
1877, to oppose what they saw as destructive 'Restoration' of ancient buildings then
occurring in Victorian England.
Two School of Thoughts
Preservation/Conservation were used interchangeably to refer to the
architectural school of thought that either encouraged measures that would
protect and maintain buildings in their current state, or would prevent
further damage and deterioration to them. Main proponent’s art critic John
Ruskin and artist William Morris.
Restoration was the conservationist school of thought that believed historic
buildings could be improved, and sometimes even completed, using current
day materials, design, and techniques. Main proponent French architect
Eugène Viollet-le-Duc.
Archaeological Survey of India (1861) is an Indian government agency in the
Department of Culture that is responsible for archaeological studies and the
preservation of cultural monuments. ASI's function is to "explore, excavate,
conserve, preserve and protect the monuments and sites of National &
International Importance."The Archaeological Survey of India (ASI) is the
successor of the Asiatic Society of British archaeologist founded in January 15,
1784 in Bengal. ASI was founded in 1861. After independence an Indian act
was passed known as ‘Ancient Monuments and Archaeological Sites and
Remains Act.
INTACH: Indian National Trust for Art and Cultural Heritage established in
1984.
World Heritage Status is awarded by UNESCO

. 19 The ratio between illumination at a working point indoor to total light available
simultaneously outdoor is known as
(A) Daylight factor (B) Sky Component
(C) Internally reflected component (D) Externally Reflected Component
Answer: (A) Daylight factor
Daylight is is the sum of Sky Component, External Reflected Component, Internal
Reflected Component.
Daylight factor is the ratio of internal light level to external light level and is defined
as follows:

DF = 100%
. 20 Which of the following vehicular traffic intersections converts all crossings into
merging and diverging sequences?
(A) Rotary (B) Manual Signaling (C) Grade Separation (D) Automatic Signaling
Answer: (A) Rotary
. 21 The process of spraying Polyster, Polyurethane, Acrylic and Epoxy Plastic,
followed by heat curing onto metals is called
(A) Anodizing (B) Galvanizing (C) Vitreous Enameling (D) Powder Coating
Answer: (D) Powder Coating
Anodizing is an electrochemical process that converts the metal surface into a
decorative, durable, corrosion-resistant, anodic oxide finish. Aluminum is
ideally suited to anodizing, although other nonferrous metals, such as
magnesium and titanium, also can be anodized.
galvanizing: steel is dipped in molten zinc. eg. Galvanized sheet (GI sheet)
Vitreous Enamel is simply a thin layer of glass fused at high temperature on to
the surface of a metal.
Polytetrafluoroethylene (PTFE) is a synthetic fluoropolymer of
tetrafluoroethylene used as a non-stick coating for pans and other cookware.
(commonly used as Teflon)
Difference between painting and powder coating
Painting Powder coating
Solvent based applied electrostatically and
is then cured under heat to
allow it to flow and form a
"skin"
Lighter and thin finish Tougher and thicker finish
Generally same finish throughout Special effect can be
achieved
Longer curing time Fast process
VOC (Volatile organic Solvent) No solvent required.
present in thinner

. 22 The fundamental right pertaining to property ownership in India DOES NOT


embrace:
(A) Sell, Lease, Donate or Bequeath (B) Mortgage (C) Grant
Easement (D) Change in use
Answer: (D) Change in use

. 23 Match the Elements in Group –I with their Applications in Group –II


Group I Group II
P. Bracket 1. Door
Q. Baluster 2. Dome
R. Key stone 3. Cornice
S. Holdfast 4. Arch
5. Staircase
(A) P-2, Q-5, R-3, S-1 (B) P-3, Q-5, R-4, S-1
(C) P-3, Q-1, R-4, S-5 (D) P-2, Q-1, R-3, S-4
Answer: (B) P-3, Q-5, R-4, S-1

. 24 Match the Buildings in Group –I with their Principal Architects in Group –II
Group I Group II
P. Wixner Centre for the Visual Arts, Ohio 1. I. M Pei
Q. Vitra Fire Station, Weilam Rhein, Germany 2. Peter Eisenman
R. AT & T Building, New York 3. Louis Kahn
S. Sher –e-Banglanagar, Dacca 4. Zaha Hadid
5. Philip Johnson
(A) P-2, Q-4, R-5, S-3 (B) P-3, Q-5, R-4, S-1
(C) P-1, Q-2, R-5, S-3 (D) P-2, Q-4, R-1, S-5
Answer: (A) P-2, Q-4, R-5, S-3

Q.25 A combination of colours forming an equilateral triangle in a colour wheel is


called
(A) Analogous Scheme (B) Triad Scheme
(C) Split Complementary Scheme (D) Double Complementary Scheme

Answer: (B) Triad Scheme


In a colour wheel for Triad system three colours are chosen such that they form a
symettric three blade fan.
Munsell developed three dimensional colour concepts, depicting hue, value and
chroma. Munsel five Principle hues are red, yellow, green, blue, and purple.
Munsel Colour representation: eg. 5.3R6.1/14. Here 5.3R is Hue representing a value
of 5.3 on Hue Band (0-10) for Red. 6.1/ represents value i.e. brightness and 14 is the
purity i.e. chroma.
(Refer page 136. Manual of Tropical Housing and Building, Koenigsberger)

Q.26 Desire Line diagram helps in


(A) completion of a project by a desired date
(B) meeting demand and supply in desired category of housing
(C) determining income versus expenditure pattern of individuals
(D) Origin- Destination analysis in transportation planning
Answer: (D) Origin- Destination analysis in transportation planning
Desire line is widely used in transportaion Planning and Land scape
Architecture, both indicating almost same.
Transportation Planning: The path usually represents the shortest or most
easily navigated route between an origin and destination. In the Google
navigation map when you type the destination from your current location the
desired path is called desired line.
Landscape Planning: desire line is a path created as a consequence of foot or
bicycle traffic which is shortcut to original path. These are often visible in a
linear grassless path within the green field.

.27 As per Fire Safety norms of NBC India for buildings having assembly and
institutional occupancies, the maximum travel distance in meters to an exit from
the dead end of a corridor is
(A) 30 (B) 24 (C) 12 (D) 6
Answer: (D) 6 (Refer section 4.4.2.2 , NBC, 2016 Part 1)
Adequate passageway and clearances required for fire fighting vehicles to enter the
premises shall be provided at the main entrance
the width of such entrance shall be not less than 4.5 m.
If an arch or covered gate is constructed, it shall have a clear head-room of not
less than 5m.

Following minimum width shall be provided for staircases as per fire safety norms.:
(a) Residential buildings (dwellings) .........1.0m
(b) Residential hotel buildings ..................1.5m
(c) Assembly buildings like auditorium, theatres and cinemas....2.0m
(d) Educational buildings up to 30m in height.........1.5m
(e) Institutional buildings like hospitals .....2.0m
(f) All other buildings ...............................1.5m

Q.28 Which of the following is a part of a studio apartment


(A) Master Bed Room
(B) Artist’s Room
(C) Multipurpose space
(D) Children’s Room
Answer: (C) Multipurpose space
Studio Apartment: one multipurpose room along with a kitchen and a bathroom

.29 The Saturation level of a colour represents


(A) distribution (B) brilliance
(C) darkness (D) warmth
Answer: (B) brilliance

Q.30 Invert level of a pipe at a given cross section refers to the


(A) highest point of the internal surface (B) lowest point of the internal surface
(C) highest point of the external surface (D) lowest point of the external surface
Answer: (B) lowest point of the internal surface
Highest point of internal surface is obvert and uppermost part of outer surface is
crown.
Q.31 The command DVIEW in AutoCAD permits to view
(A) a selected portion of drawing in detail (B) the entire screen on the monitor
(C) a perspective of the drawing (D) a damaged part of the drawing
Answer: (C) a perspective of the drawing

.32 Match the Land use categories of Group-I with their respective Colour codes in Group-
II as per practice in India.
Group I Group II
P. Residential 1. Red
Q. Commercial 2. Grey
R. Industrial 3. Blue
S. Public/ Semi -public 4. Violet
5. Yellow
(A) P-5, Q-3, R-4, S-1 (B) P-5, Q-4, R-2, S-1 (C) P-1, Q-2, R-4, S-5 (D) P-1, Q-3,
R-2 S-4
Answer: (A) P-5, Q-3, R-4, S-1
These colours are used while preparing Landuse map mostly in Master Plan.
Residential - Yellow
Commercial - Blue
Industrial - Violet
Public/ Semi –public – Red
Transportaion – Grey
Parks, playgrounds, open spaces- Green

.33 A rectangular beam section of size 300 mm (width) X 500 mm (depth) is loaded
with a shear force of 600 kN. The maximum shear stress on the section in N/mm2
is ………………
Answer: 6

The Maximum Shear Stress occurs at the Neutral Axis is where F=force and
b=breadth, d=depth.

So we get Max Shear Stress = = 6 N/mm2


(Here 600 kN is converted to N by multiplying by 1000)

.34 In a 50 meter section of a waste water pipe, if the gradient is 1 in 80, then the fall
in millimeter is ………………..
Answer: 625
For every 80 m there is fall of 1 m

For 1 m fall = (unitry method)

For 50 m = x 50 m = 0.625 m which equals to 625 mm.


.35 A 15 meter long and 3 meter wide driveway needs to be paved with 300 mm X
300mm square tiles. If each packet contains 30 number of tiles, then the number
of packets to be procured to pave the whole area is ……..
Answer: 17
Area to be paved 15 m x 3m = 45 m 2 Area of each tile = 0.3 m x 0.3 m = 0.09 m2

No of tiles needed = = 500 No of packets = ~ 17 packets

.36 Match the Monuments in Group –I with their Features in Group –II
Group I Group II
P. Panch Mahal Fatehpur Sikri 1. Painted Stone Figures
Q. Meenakshi Temple , Madurai 2. Intricated Red Sand Stone Carvings
R. Jor-Bangla Temple, Bishnupur 3. Granite statues
S. Sun temple, Konark 4. Khondalite stone work
5. Terracotta carvings
(A) P-2, Q-1, R-4, S-3 (B) P-2, Q-1, R-5, S-4 (C) P-2, Q-4, R-1, S-3 (D) P-1,
Q-5, R-5, S-4
Answer: (B) P-2, Q-1, R-5, S-4

.37 Match the Monuments in Group –I with their Style of Architecture in Group –II
Group I Group II
P. Pisa Cathedral, Italy 1. Gothic
Q. St. Hagia Sophia, Istanbul 2. Moorish
R. Great Temple Aman, Karnak 3. Egyptians
S. Cathedral of Notre Dame, Paris 4. Byzantine
5. Romanseque
(A) P-5, Q-1, R-3, S-2 (B) P-2, Q-4, R-3, S-5 (C) P-4, Q-2, R-5, S-1 (D) P-5,
Q-4, R-3, S-1
Answer: (D) P-5, Q-4, R-3, S-1

.38 Match the Buildings in Group –I with their Style of Architecture in Group –II
Group -I Group –II
P. Rashtrapati Bhawan, New Delhi 1. Industrial Architecture
Q. German Pavillion for World Exhibition, Barcelona 2. Deconstruction
R. Guggenheim Museum, Bilbao 3. Radical Eclecticism
S. Crystal Palace, London 4. International Style
5. Neo Classical
(A) P-5, Q-3, R-2, S-1 (B) P-5, Q-4, R-2, S-1 (C) P-1, Q-5, R-4, S-3 (D) P-3, Q-
4, R-2, S-5
Answer: (B) P-5, Q-4, R-2, S-1
Guggenheim, Bilbao, Spain – Architect – Frank Gehry. Other Guggenheim
Museum is in New York, by F L Wright. Guggenheim, Bilbao is an example of
Deconstructivism.
Features of Deconstructivism.
(d) Fragmentation, geometrical blocks is fused together.
(e) Non-Rectilinear shapes, eg. Tilted cuboids
(f) Manipulating surface eg. Metallic cladding
The attempt in deconstructivism throughout is to move architecture away from what
its practitioners see as the constricting ‘rules’ of modernism such as “form follows
function,” “purity of form,” and “truth to materials.” Frank Gehry, Daniel Libeskind,
Rem Koolhaas, Peter Eisenman, Zaha Hadid, Coop Himmelbau, and Bernard
Tschumi practice deconstructivism

.39 Match the Terms in Group –I with their Definition in Group –II
Group -I Group –II
P. Kinesthesia 1. Measurement and study of size and proportion of human body
Q. Anthropometry2. Study of man-machine interaction
R. Ergonomics 3. Study of past and present of human race
S. Biomimicry 4. Study of human sensory experience during movement
5. Imitation of model, system and elements of nature
(A) P-5, Q-3, R-4, S-1 (B) P-5, Q-2, R-4, S-3 (C) P-4, Q-1, R-2, S-5 (D) P-4, Q-
1, R-2, S-3
Answer: (C) P-4, Q-1, R-2, S-5

Q.40 Match the following Urban Spaces in Group –I with their names in Group–II
(A) P-4, Q-1, R-2, S-3 (B) P-2, Q-3, R-1, S-5(C) P-4, Q-3, R-1, S-5 (D) P-2, Q-1,
R-4, S-3

Answer: (B) P-2, Q-3, R-1, S-5

.41 Match the Terms in Group –I with the appropriate Items in Group –II
Group -I Group –II
P. Toposheet 1. Path/Row
Q. Satellite Image 2. Contour
R. Wavelength 3. Focal Length
S. Scan Line 4. Spectral Signature
5. Bits/inch
(A) P-5, Q-4, R-2, S-1 (B) P-5, Q-1, R-4, S-3
(C) P-2, Q-1, R-4, S-5 (D) P-2, Q-4, R-1, S-5
Answer: (C) P-2, Q-1, R-4, S-5
Toposheet has contour lines.
Path is satellite image numbered in the direction of satellite movement(West to
East)
Scan lines used in scanners

.42 Match the Concepts in Group –I with the appropriate Explanation in Group –II
Group -I Group –II
nned Unit Development 1. Development occurring on vacant or underused
lots in otherwise built up areas
Q. Infill Development 2. Development providing a fair and equitable way
to integrate peri-urban areas
R. Transit Oriented Development 3. Developing a large area as a single entity
merging
zoning and subdivision control
S. Mixed Use Development 4. Development with compatible land uses integrating
varied activities at different times of the
day
5. Development located within walking
from mass
transit stations along the corridor
(A) P-3, Q-2, R-5, S-4 (B) P-3, Q-1, R-5, S-4 (C) P-2, Q-1, R-4, S-5 (D) P-2, Q-
4, R-1, S-5
Answer: (B) P-3, Q-1, R-5, S-4
A Planned Unit Development (PUD) is a community of homes that could look
like single family residences, townhomes or condos, and can include both
residential and commercial units

Q.43 Particles of soil in descending order of grain size is


(A) Gravel – Sand- Silt- Clay (B) Gravel- Sand- Clay- Silt
(C) Sand-Gravel- Clay- Silt (D) Clay- Gravel- Sand- Silt
Answer: (A) Gravel – Sand- Silt- Clay

Q.44 Match the Units in Group –I with their Definitions in Group –II
Group -I Group –II
P. Hertz 1. Newton-meter
Q. Lux 2. Cycles/ second
R. Joule 3. Lumen/ m2
S. Newton 4. Watt/ ampere
5. kg- meter/ sec2
(A) P-5, Q-4, R-2, S-1 (B) P-3, Q-1, R-5, S-4 (C) P-2, Q-3, R-1, S-4 (D) P-2,
Q-3, R-1, S-5
Answer: (D) P-2, Q-3, R-1, S-5

Q.45 Match the Energy Efficient Building Elements in Group –I with their associated Working
Principles in Group –II
Group -I Group –II
P. Solar Chimney 1. Thermal Storage
Q. Earth Air Tunnel 2. Radiant Cooling
R. Trombe Wall 3. Stack Effect
S. Chilled Slab 4. Cross Ventilation
5. Geothermal Energy
(A) P-3, Q-2, R-4, S-5 (B) P-5, Q-2, R-4, S-3 (C) P-3, Q-5, R-1, S-2 (D) P-4, Q-
5, R-1, S-2
Answer: (C) P-3, Q-5, R-1, S-2
Passive Heating/ Cooling: Building design approach that focuses on heat gain control and
heat dissipation in a building in order to improve the indoor thermal comfort with
low or nil energy consumption
Solar Chimney: solar chimney consists of a black-painted chimney. During the
day solar energy heats the chimney and the air within it, creating an updraft of
air in the chimney. The suction created at the chimney's base can be used to
ventilate and cool the building below. It is most effective in hot windless day.

Trombe Wall: A Trombe wall is a passive solar building technique where a wall is
built on the winter sun side of a building with a glass external layer and a high
heat capacity internal layer separated by a layer of air. Heat in close to UV
spectrum passes through the glass almost unhindered then is absorbed by the
wall that then re-radiates in the far infrared spectrum which does not pass back
through the glass easily, hence heating the inside of the building.
Earth Air Tunnel: this
technique is essentially used for cooling the air in Hot and dry climates. The
cooling process is based on the fact that the temperature a few meters below the
ground is almost constant throughout the year. A wind tower is connected to the
underground tunnel, which runs from the bottom of the wind tower to the
basement of the building. The wind tower catches the wind which is forced down
the tower into the tunnel. The temperature of the tunnel, being lower than that of
the ambient temperature, cools the air before it is circulated into the living space.
In winter, the temperature of the air tunnel is higher than the ambient temperature
and hence warms the air passing through it.
Chilled slab: These deliver cooling through the building structure, usually slabs,
and is also know as thermally activated building systems (TABS).
Night flush cooling – The building structure acts as a sink through the day and
absorbs internal heat gains and solar radiation. Heat can be dissipated from the
structure by convective heat loss by allowing cooler air to pass through the
building at night. The flow of outdoor air can be induced naturally or mechanically.
The next day, the building will perform as a heat sink, maintaining indoor
temperatures below the outdoor temperature.
Brise-soleil - A screen, usually of louvers, placed on the outside of a building to
shield the windows from direct sunlight.
Drum wall- accoustic wall used to absorb unwanted sound.

.46 Match the Vibrator types in Group –I with their related area of application in Group –II
Group -I Group –II
P. Needle Vibrator 1. Concrete Pavement
Q. Shutter Vibrator 2. Pre cast Concrete Unit
R. Surface Vibrator 3. Beam-Column Junction
S. Table vibrator 4. Retaining Wall
5. Slip Forming
(A) P-1, Q-5, R-4, S-3 (B) P-3, Q-4, R-1, S-2 (C) P-1, Q-4, R-2, S-5 (D) P-3,
Q-5, R-1, S-2
Answer: (B) P-3, Q-4, R-1, S-2
Concrete vibrators consolidate freshly poured concrete so that trapped air and
excess water are released
and the concrete settles firmly in place in the formwork.
A vibro-pile is a closed off casing that is vibrated into the ground displacing and
"densifying" all the material in its path. The casing is then filled with concrete
and reinforcement and then extracted

.47 Match the type of Temporary Structures in Group–I with their corresponding
Functions in Group–II
Group -I Group –II
P. Scaffolding 1. To support unsafe structures
Q. Formwork 2. To support platforms for workman and materials at raised height
during construction
R. Shoring 3. Removal of water from pits
S. Underpinning 4. Mould for RCC structure
5. Strengthening the existing foundation
(A) P-2, Q-4, R-1, S-5 (B) P-3, Q-5, R-1, S-2 (C) P-3, Q-4, R-5, S-2 (D) P-2,
Q-3, R-4, S-5
Answer: (A) P-2, Q-4, R-1, S-5
In construction or renovation, underpinning is the process of strengthening the
foundation of an existing building or other structure.
Shoring is the process of supporting a building, structure, or trench with
shores (props) when in danger of collapse or during repairs or alterations.

.48 Match following Scientific Names in Group –I with their common Indian Names in
Group –II
Group -I Group –II
P. Lagerstroemia speciosa 1. Amaltas
Q. Cassia fistula 2. Neem
R. Azadarachta indica 3. Jarul
S. Acacia auriculiformis 4. Babul
5. Peepal
(A) P-2, Q-4, R-3, S-5 B) P-5, Q-3, R-2, S-4 (C) P-3, Q-1, R-4, S-2 (D) P-3, Q-
1, R-2, S-4
Answer: (D) P-3, Q-1, R-2, S-4
Neem (Azadarachta indica), Amaltas (Cassia fistula) and Peepal (Ficus religiosa)
are very common and asked almost every
alternate years in GATE.
Botanical name of common Indian Plants.
No Indian Name Botanical Name
1 Neem Azadirachta indica
2 Mango Mangifera indica
3 Peepal Ficus religiosa
4 Banyan Ficus benghalensis
5 Gulmohar Delonix regia
6 Tamarind (Imli) Tamarindus indica
7 Jack fruit Artocarpus
(kathal) heterophyllus
8 Bottle brush Callistemon viminalis
9 Croton Codiaeum variegatum
10 Jasmine Jasminum
augustifolium
11 Marigold Calendula officinalis
12 Rose Rosa indica
13 Amaltus Castia fistula
14 Asoka Saraca asoca,
Polyalthia longifolia
15 Basil (Tulsi) Ocimum Sanctum
16 Coconut Cocos nucifera
17 Bamboo Dendrocalamus
calostachymus
18 Teak Techtona grandis
19 Sheshame Dalbergia sissoo
20 Indian Toona ciliata
Mahogony
21 Palash Butea monosperma
22 Babul Acacia auriculiformis
23 Jarul Lagerstroemia
speciosa
24 Champa Plumeria alba
(Frangipani)

.49 A man starts from his residence and uses the following mode in sequence to reach the
office- cycle rickshaw to Railway station, then train to destination station, followed by
auto rickshaw to nearby bus stand and finally a bus to his office. Which of the following
describes the sequence of transit usage?
(A) Non Motorosied Transit- Paratransit- Mass Transit- Public Transit
(B) Paratransit- Public Transit – Non Motorosied Transit –Mass Transit
(C) Private Transit- Public Transit – Non Motorosied Transit –Mass Transit
(D) Non Motorosied Transit- Mass Transit- Paratransit- Public Transit
Answer : (D) Non Motorosied Transit- Mass Transit- Paratransit- Public Transit
NMT- Non Motorised Transit- walking, Rickshaw, horse cart
Paratransit is an alternative mode of flexible passenger transportation that
does not follow fixed routes or schedules.
MRTS- Mass Rapid Transit System. Min 5000 trips PHPD (peak hour peak
direction)
BRTS- Bus Rapid Transit System (Pune has BRTS corridor)
MMTS- Multi Modal Transit System
Public transit- fixed route and fixed time.
Mode Route Timing Carrier Example
type
NMT (Non Not Not Pvt/ Walking, cycling,
Motorised fixed fixed public cycle rickshaw,
Transport) tonga
Paratransit Not Not Public Reserve Auto,
fixed fixed ola, uber
Private Not Not Pvt Personal vehicle
transit fixed fixed
Shared Fixed Not Public Shared Auto,
transport fixed City ride bus
IPT( Not Not Pvt Cycle Rickshaw,
Intermediate fixed fixed E- rickshaw,
Public Tonga, Trekker,
Transport) Matador
MRTS Fixed Fixed Public Metro Rail
(Mass Rapid
Transit
System)
BRTS (Bus Fixed Fixed Public Bus
Rapid
Transport
System)
MMTS (Multi Fixed Fixed Public/Pvt Train
Modal
Transit
System)

Q.50 PMGSY and JNNURM are two Indian Government programmes which deal with
(A) rural road development and urban basic service improvement respectively
(B) rural sanitation services and under-developed road maintenance respectively
(C) peri-urban basic services and urban basic service improvement respectively
(D) rural road development and urban transport development respectively
Answer: (A) rural road development and urban basic service improvement
respectively
Pradhan Mantri Gram Sadak Yojana or PMGSY is a nationwide plan in India
to provide good all-weather road connectivity to unconnected villages .
JNNURM- Jawahar Lal Nehru National Urban Renewal Mission aims at creating
‘economically productive, efficient, equitable and responsive Cities’ by a strategy of
upgrading the social and economic infrastructure in cities. This project runs under
Ministry of Urban Development, Govt. of India.
Rajiv Awas Yojana (RAY) (2013-22), is a scheme of the Government of India for the
benefit of poor in Urban areas. It is a Centrally Sponsored Scheme, (CSS), to be
implemented in mission mode during 2013-2022. The scheme aims to make India
slum-free by 2022 by providing people with shelter or housing free of cost.
Recently Introduced program (2015) by Ministry of Urban Development, Govt.
of India. (IMPORTANT)

Smart city Project:


Developing 100 smart cities across india is a flagship program of Ministry of Urban
Development, Govt. of India. Round the clock water and electricity supply, high
internet connectivity, e- governance, smart traffic system, high quality work space and
many more aspects will be the key feature of these cities.
AMRUT: (Atal Mission for Rejuvenation and Urban Transformation). This program will
work replacing JNNURM catering Physical and Social Infrastructure of the 500 urban
areas. (GATE 2016)
PMAY : (Pradhanmantri Aawas Yojna) Providing housing for all by 2022 in the Urban
Areas. More than 2 crore homes would be build across all the urban locations by
2022. Central Govt. will provide a grant of Rs 1 lakh to Rs 2.5 lakh for every home.
HRIDAY: Heritage City Development and Augmentation Yojna. Revitalization of urban
infrastructures in identified 12 heritage cities namely Ajmer, Amravati, Amritsar,
Badami, Dwarka, Gaya, Kanchipuram, Mathura, Puri, Varanasi, Velankani and
Warangal.

.51 Match the Planning Terms in Group –I with their Descriptions in Group –II
Group -I Group –II
P. Gentrification 1. Haphazard and low density outward growth of urban area
Q. Urban core revitalization 2. Primarily dormitory settlement with functional
dependency on parent city
R. Urban sprawl 3. Replacement of low income residents with high income
population
S. Satellite town 4. Physical and socio-economic revival of the inner city
5. Restricted development in an environmentally sensitive zone
(A) P-4, Q-3, R-5, S-2 (B) P-3, Q-4, R-1, S-5
(C) P-1, Q-5, R-2, S-3 (D) P-3, Q-4, R-1, S-2
Answer: (D) P-3, Q-4, R-1, S-2

Gentrification: Gentrification is the


buying and renovating of houses and stores in deteriorated urban
neighborhoods by wealthier individuals, which increases property values but
also can displace low-income families and small businesses.
Difference between Urban Sprawl and Connurbation:
Urban Sprawl: Outward growth of Urban Area. When the urban sprawl is such
large that it engulfs smaller towns it is called Conurbation.
Conurbation: a large densely populated urban sprawl formed by the growth
and coalescence of individual towns or cities. NCR is the Conurbation which
came into existence by Urban Sprawl of Delhi. The engulfed areas are
Gurgaon, Noida, Ghaziabad, Faridabad.
Out Growth- Out Growth should be a viable units such as village or part of a
village contiguous to a statutory town and possess the urban features in terms
of infrastructure and amenities. Examples of OG’s are Railway towns, Port
Towns, University Towns that may come up near a city or statutory town
outside its statutory limit but within the revenue limit of a village or villages
contiguous to the town or city.

.52 Match the Planning Concepts in Group –I with their Corresponding Proponents in
Group –II
Group –I Group –II
P. Broadacre city 1. Le Corbusier
Q. Radiant City 2. F. L. Wright
R. Industrial Town 3. Robert Owen
S. Arcosanti 4. Henry Wright
5. Paolo Soleri
(A) P-1, Q-4, R-3, S-5 (B) P-1, Q-3, R-5, S-2 (C) P-2, Q-1, R-3, S-5 (D) P-2,
Q-1, R-5, S-4
Answer: (C) P-2, Q-1, R-3, S-5
Concept City
Concept Propagated Population Area
by
1 Plato’s city Plato 5040 -
2 Garden City Ebenezer 32,000 6000 acre
Howard
3 Broadacre City F L Wright - 1-5 acre per
household
4 Radiant City Le - -
Corbusier
5 Arcosanti Paolo Soleri 5000 25 acre
6 Neighbourhood Clarence 5000-6000 160 acre
Perry
7 Linear City Soria Y - -
Mata
Garden City concept was given by Ebenezer Howard. Letchworth, UK was
the first garden city built in 1909 planned by Raymond Unwin.
Arcology: Architecture + Ecology first coined by Paolo Soleri. It is
architectural design principles for enormous habitats (hyperstructures) of
extremely high human population density

.53 The housing stock of a town has total number of 9090 dwelling units. Present population
of the town is 45,450. Assuming an average house hold size of 4.5, the housing
shortage in percentage is
Answer: 10 %

No of household = = 10,100 therefore shortage of dwelling units = 10,100- 9090=


1010 units

Shortage in % = x 100 = 10 %
What is the difference between Dwelling unit and a house?
What is the difference between Household and family?
A house is a physical identity where several dwelling unit may exist. Eg. An
apartment(a physical identity) is a house where several dwelling units (flats)
exist.
Household is related with kitchen. In a joint family where several families
stay together but share a common kitchen will be counted as one
household.Families with separate kitchen will be counted as separate
household.
.54 A hall is 15 m long and 12 m wide. If the sum of areas of the floor and ceiling is equal to
the sum of the area of its four walls, then the volume of the hall in cubic meters is
……………
Answer: 1200 cubic m
Area of floor = 15m x 12 m = 180 square m so area of ceiling also 180 sq m.
Sum of area of floor and ceiling = 360 sq m.
Let the height of wall be ‘h’
Then area of wall = (15 + 12) x 2 x h
54 h = 360 so ‘h ‘= 6.66 m
So Volume of hall = 15 x 12 x 6.66 = 1200 m3

.55 The actual roof area of a building is 3,60,000 sqm, which on a site plan measures 25
sqcm. The scale of the site plan is ……………………
Answer: 12000

Area to scale conversion = = = 12000


Input by Vipin V Mukkawar, Maharashtra, B Arch from AIIA.

.56 If the annual net income from a commercial property is Rs 22,000/- and the interest rate
is 8 %, then the capitalized value is rupees of the property in perpetuity
is……………………
Answer: Rs 2,75,000/-
Capitalised value = Net annual Income × Year’s Purchase

= 22,000 × = 2,75,000

.57 A five storied building is constructed on a 100 m X 50 m plot having ground coverage of
60 % (option 1). Alternatively, a four storied building is constructed on the same plot
with a 50 % ground coverage (option 2). The ratio of FARs between option 1 and 2 is
…………….
Answer: 1.5

Option I Total built up area of all floors = 100 x 50 x x 5 = 15,000 sq m

FAR = = 3.0

Option II Total built up area of all floors = 100 x 50 x x 4= 10,000 sq m

FAR = = 2.0

Ratio of FAR = = 1.5

.58 If a roof is treated with a layer of thermal insulation material, the internal heat gain is
reduced by 60 %. The U-value of the roof slab (without thermal insulation) is 3 W /m2
deg C. Assuming a constant temperature difference between indoor and outdoor , the
U-value of the thermal insulation layer in W /m2 deg C is …………………
Answer: 2

R- value is inverse of U-value (R-value= ) which can be directly added. (We


cannot add U-value)
Given the U-value of the roof slab (without thermal insulation) is 3 W /m2 deg C. It is
reduced by 60 % i.e.

3- ( x 3) = 1.2 (This the combined U value after insulation)

So adding R value…. + = (considering U value of insulation material as ‘x’)

= - so = =
Solving x we get U value of insulation material as 2 W /m2 deg C

.59 A simply supported beam having effective span of 5 meter is carrying a centrally
concentrated load of 16 kN. The maximum bending moment in the beam in kN-m is
………………
Answer: 20 kN-m
16 kN

R1R2

5 m span

Maximum BM will occur at centre. = so it will be = 20 kN-m.

.60 A landscaped garden with irregular profile and minor undulations, measuring 35,000
sqm, has a total surface area covered with 20% brick paving, 15 % cement concrete
paving, and rest with grass. The peak intensity of rainfall in that region is 70 mm/hr.
The coefficient of runoff for brick paving, cement concrete paving and grass is 0.8, 0.9
and 0.5 respectively. The estimated quantity of runoff in cubic meter/hr for the entire
garden area is …………………….
Answer: 1519 cubic m/ hr

Total rainfall in 1 hr = 35,000 x = 2450 cubic m

Runoff = 2450 ( x + x + x )
= 2450 ( 0.16 + 0.135 + 0.325)
= 2450 ( 0.620 ) = 1519 cubic m /hr

.61 The number of standard cement bags required to prepare 1400 kg of concrete in the
ratio 1:2:4 (mixed by weight batching) is ………………..
Answer: 4 bags

Cement required in kg = x 1400 = 200 kg


Cement comes in 50 kg bags so 4 bags are required.
(Here ratio is taken in terms of weight. If it was taken in terms of volume than dry vol of
concrete = 1.54 × wet vol of concrete)

.62 A class room measuring 10 m (L) x 8 m (B) x 2.7 m (H) requires an illumination level of
500 lux on the desk level using 40 W fluorescent lamps with rated output of 5000
lumens each. Assuming utilization factor of 0.5 and maintenance factor of 0.8, the
number of lamps required is ………………..
Answer: 20 lamps
Required illumination = 500 lux = 500 lumens/m2
Total lumens needed in room = 10 x 8 x 500 = 40,000 lumens
Let no of lamps be ‘n’
So, 5000 x n x 0.5 x 0.8 = 40,000
n= 20 lamps
.63 Area of tensile steel per meter width of a reinforced concrete slab is 335 sq mm. If 8 mm
rods are used as reinforcement, then centre to centre spacing of the reinforcement in
mm is ………………..
Answer: 145 mm

Area of each rod = π r² = x (4)² = 50.2 mm²

No of rods used = = 6.67 (say 7 rods)

Centre to centre spacing = cm = (say 14.3 cm) approx 145 mm


.64 The population of the town as per Census 2011 was 22,730 and the population as per
census 2001 was 15,770. Corresponding arithmetic projection of growth, the projected
population in 2016 will be ……………..
Answer: 26210
In Arithmatic progression it is equal increase per year.
Gain of population in 10 years : 22,730- 15,770 = 6960

Annual gain = = 696 persons


In 2016 (five years from 2011) toal population will be = (696 x 5) + 22,730 = 26,210

.65 Two concrete mixers of capacity 200 litres each are used in a construction site to
produce 20 cubic meter of concrete. Ingredient charging, mixing and discharge times
are 3 minutes, 7 minutes and 1 minute respectively. Assuming a time loss of 5 minutes
per hour of operation, the total time in hours for the mixture to produce the required
amount of concrete will be ………….
Answer: 10 hr.
Total time required for one discharge= 3+ 7+ 1= 11 min
In one hour it will discharge five times (11x 5= 55 + 5 min of loss in one hour)
Total concrete produced by two mixers in one hour = 2 x 5 x 200 litres = 2000 litres = 2
m3
In order to produce 20 m3 it will take 10 hours.
GATE 2016
Q. 1 – Q. 5 carry one mark each.

Q.1 An apple costs Rs. 10. An onion costs Rs. 8.


Select the most suitable sentence with respect to grammar and usage.
(A) The price of an apple is greater than an onion.
(B) The price of an apple is more than onion.
(C) The price of an apple is greater than that of an onion.
(D) Apples are more costlier than onions.
Answer: (C) The price of an apple is greater than that of an onion.

.2 The Buddha said, “Holding on to anger is like grasping a hot coal with the intent of
throwing it at someone else; you are the one who gets burnt.”
Select the word below which is closest in meaning to the word underlined above.
(A) burning (B) igniting (C) clutching (D) flinging
Answer: (C) clutching

.3 M has a son Q and a daughter R. He has no other children. E is the mother of P and
daughter-inlaw of M. How is P related to M?
(A) P is the son-in-law of M. (B) P is the grandchild of M.
(C) P is the daughter-in law of M. (D) P is the grandfather of M.
Answer: (B) P is the grandchild of M.

Q.4 The number that least fits this set: (324, 441, 97 and 64) is ________.
(A) 324 (B) 441 (C) 97 (D) 64
Answer: (C) 97 and (D) 64
( Both the answers were given marks)

Q.5 It takes 10 s and 15 s, respectively, for two trains travelling at different constant speeds
to
completely pass a telegraph post. The length of the first train is 120 m and that of the
second train is 150 m. The magnitude of the difference in the speeds of the two trains
(in m/s) is ____________.

(A) 2.0 (B) 10.0 (C) 12.0


(D) 22.0
Answer: (A) 2.0

Q. 6 – Q. 10 carry two mark each.


.6 The Velocity V of a vehicle along a straight line is measured in m/s and plotted as shown
with respect to time in seconds. At the end of 7 seconds, how much will the odometer
reading increase by (in m)?

(A) 0 (B) 3 (C) 4 (D) 5


Answer: (D) 5

.7 The overwhelming number of people infected with rabies in India has been flagged by
the World Health Organisation as a source of concern. It is estimated that inoculating
70 % of pets and stray dogs against rabies can lead to a significant reduction in the
number of people infected with rabies.
Which of the following can be logically inferred from the above sentences?
(A) The number of people in India infected with rabies is high.
(B) The number of people in other parts of the world who are infected with rabies is low.
(C) Rabies can be eradicated in India by vaccinating 70 % of stray dogs.
(D) Stray dogs are the main source of rabies worldwide.

Answer: (A) The number of people in India infected with rabies is high.

.8 A flat is shared by four first year undergraduate students. They agreed to allow the
oldest of them to enjoy some extra space in the flat. Manu is two months older than
Sravan, who is three months younger than Trideep. Pavan is one month older than
Sravan. Who should occupy the extra space in the flat?
(A) Manu (B) Sravan (C) Trideep (D) Pavan
Answer: (C) Trideep

.9 Find the area bounded by the lines 3x+2y= 14, 2x-3y=5 in the first quadrant.
(A) 14.95 (B) 15.25 (C) 15.70 (D) 20.35
Answer: (B) 15.25

.10 A straight line is fit to a data set (ln x,y).This line intercepts the abscissa at ln x = 0.1 and
has a slope of – 0.02. What is the value of y at x= 5 from the fit?
(A) -0.030 (B) -0.014 (C) 0.014 (D) 0.030
Answer: (A) -0.030

Q. 1 – Q. 25 carry one mark each.


Q.1 ‘Dharahara’ refers to
(A) Concept of Vastu Shastra
(B) Elevation feature of a Hindu Temple
(C) Heritage tower at Kathmandu
(D) Construction method of Buddhist Architecture
Answer: (C) Heritage tower at Kathmandu
Famous tower of Kathmandu, destroyed in 2015 Earthquake that shook whole Nepal
and entire North India on 25 April 2015. It was also known as Bhimsen tower.
(adjacent picture is of the destroyed tower)
Q.2 ‘Mushroom Column’ is a typical feature of
(A) Pantheon, Rome
(B) Parthenon, Athens
(C) Chrysler Building, New York
(D) Johnson Wax Building, Racine
Answer: (D) Johnson Wax Building, Racine
Johnson wax headquarter, Wisconsin, USA designed by F L Wright.
Pantheon, Rome –famous for oldest and largest unreinforced dome with
opening (oculus) at top.
Parthenon, Athens- famous for proportion and optical correction (Entasis).

Q.3 ‘AMRUT’, the new scheme launched by Government of India, stands for
(A) Atal Mission for Rejuvenation and Urban Transformation
(B) Atal Mission for Renewal of Urban Transportation
(C) Atal Mission for Redevelopment of Urban Transportation
(D) Atal Mission for Renewal and Urban Transformation
Answer: (A) Atal Mission for Rejuvenation and Urban Transformation
500 cities selected with broad objective of
access to tap water and sewerage facilities,
greenery like parks and open spaces are well maintained,
digital and smart facilities like weather prediction, internet and WiFi facilities,
pollution reduction by encouraging the public for using cheaper but secure
public transport
SAAP(State Annual Action Plans) is a consolidated plan of all the city level
SLIPs(Service Level Improvement Plans) of all proposed AMRUT cities in the
respective states.

Q.4 The concept of ‘Dry Garden’ is associated with


(A) Japanese Garden (B) Chinese Garden (C) Mughal Garden (D) Egyptian
Garden
Answer: (A) Japanese Garden and (D) Egyptian Garden
The Japanese rock garden or “dry landscape” garden, often called a zen
garden, creates a miniature stylized landscape through carefully composed
arrangements of rocks, water features, moss, pruned trees and bushes, and
uses gravel or sand that is raked to represent ripples in water.
A zen garden is usually relatively small, surrounded by a wall, and is usually
meant to be seen while seated from a single viewpoint outside the garden.

Q.5 The concept of Transit Oriented Development (TOD) refers to


(A) Transit oriented planning in institutional campuses
(B) High density development along a mass transit corridor
(C) Orienting residential blocks towards the transit in a gated community
(D) Spatial design for persons with disabilities along a mass transit corridor

Answer: (B) High density development along a mass transit corridor


A transit-oriented development (TOD) is a mixed-use residential and
commercial area designed to maximize access to public transport, and often
incorporates features to encourage public transportation and less dependence
on private vehicle.
Transit stops- metro station, bus stops, sub urban train station.
Radius of development from transit station : 400-800 m (this is considered to
be an appropriate scale for pedestrians)

Gated
community: (or walled community) It is a form of residential community or housing
estate containing strictly controlled entrances for pedestrians, bicycles, and
automobiles, and often characterized by a closed perimeter of walls and fences.

Q.6 The components measuring Human Development Index (HDI) of a nation are
(A) Life expectancy, Education and Per Capita Income
(B) Life expectancy, Economy and Housing
(C) Health, Development and Per Capita Income
(D) Housing, Health and Hygiene
Answer: (A) Life expectancy, Education and Per Capita Income
(Earlier it was GDP per capita and now it has been modified as GNI per capita)
Human Development Index is calculated by UNDP (United Nations Development
Programmes) All three parameters are
calculated on scale of 0-1.
Education attainment: Geometric mean of years of schooling for adults aged 25
years and expected years of schooling for children of school entering age.
Life expectancy: The life expectancy at birth component of the HDI is calculated
using a minimum value of 20 years and maximum value of 83.57 years.
The decent standard of living component is measured by GNI per capita
instead of GDP per capita. The HDI uses the logarithm of income, to reflect the
diminishing importance of income with increasing GNI.
The scores for the three HDI dimension indices are then aggregated into a
composite index using geometric mean.

.7 As per the Ancient Monuments and Archaeological Sites and Remains Act, 2010, the
extent of ‘prohibited area’ beyond the ‘demarcated protected area’ in meters is
(A) 50 (B) 100 (C) 200 (D) 500
Answer: (B) 100
prohibited area is 100 m from demarcated protected area.(no mining or
construction allowed)
regulated area is 200 m beyond the prohibited area. (allowed mining and
construction with regulation)
CRZ (Coastal Regulatory Zone) – no development allowed upto 100 m from
high tide line along sea coast.
IPZ (Island Protection Zone) – included for islands of Lakshadweep, Andaman
& Nicobar Islands.
Q.8 In CMYK colour model, ‘K’ represents the colour
(A) White (B) Black (C) Blue (D) Green
Answer: (B) Black
Subtrative and Additive Colour Model:
The CMYK color model (color, four color) is a subtractive color model, used in
color printing, and is also used to describe the printing process itself. CMYK
refers to the four inks used in color printing: cyan, magenta, yellow, and key
(black).
The RGB color model is an additive color model in which red, green, and blue
light are added together in various ways to reproduce a broad array of colors.
(used in screens of computer)
RYB (an abbreviation of red–yellow–blue) is a historical set of colors used in
subtractive color mixing, and is one commonly used set of primary colors. It is
primarily used in art and design education, particularly painting.

Q.9 ‘Segregation at source’ is a concept associated with


(A) Solid waste management (B) Disaster management (C) Traffic management
(D) Water management
Answer: (A) Solid waste management
Waste Can be segregated as :-
A. Bio Degradable Waste:- Bio Degradable waste includes organic waste, e.g.
kitchen waste, vegetables, fruits, flowers, leaves from the garden and paper.
B. Non Bio Degradable Waste:- (Non Biodegradables can be further segregated
into) :
Recyclable Waste- Plastics, Paper, Glass, Metal Etc.
Toxic Waste:- Old Medicine, paints, Chemicals, bulbs, Spray Cans,
fertilizer and pesticide containers, batteries, shoe polish.
Soiled:- Hospital waste such as cloth soiled with blood and other body
fluids. Toxic & soiled waste must be disposed of with utmost care.
Colour code of waste bins
Organic – Green
Recycleable- Blue.

Q.10 In a flat slab, the load is transferred through


(A) Beam action (B) Membrane action (C) Plate action (D) Vector action
Answer: (C) Plate action

Q.11 Map is an
(A) Isometric projection (B) Orthographic projection
(C) Axonometric projection (D) Oblique projection
Answer: (B) Orthographic projection
Orthographic projection is a perspective (or azimuthal) projection, in which the
sphere is projected onto a tangent plane or secant plane. The point of
perspective for the orthographic projection is at infinite distance. It depicts a
hemisphere of the globe as it appears from outer space, where the horizon is a
great circle. The shapes and areas are distorted, particularly near the edges.

Axonometric projection is a
type of parallel projection used for creating a pictorial drawing of an object,
where the object is rotated along one or more of its axes relative to the plane of
projection. There are three main types of axonometric projection: isometric,
dimetric, and trimetric projection.
Oblique projection is a simple type of technical drawing of graphical projection
used for producing two-dimensional images of three-dimensional objects.

Q.12 ‘Indore Slum Networking Programme’ has been planned by


(A) B. V. Doshi (B) Dulal Mukherjee (C) Anil Laul (D) Himanshu Parikh
Answer: (D) Himanshu Parikh
There are two famous housing projects of Indore(both are winner of Aga Khan
award)
Aranya low cost housing project by B V Doshi (1988)
Indore Slum Networking Programme by Himanshu Parikh (1993)
Himanshu Parikh explained that his research into the growth of slums had led to the
realisation that slums always develop along the natural drainage paths. Therefore the
cheapest way to provide sewerage to a city is to build major sewers through the
slums and connect the higher (and usually richer) areas of the city to them. This then
provides sewerage to the whole city for a lower costs than just providing sewerage for
the rich areas.

This approach opens up funding from local


and national government, which can then be matched by donor funding if necessary
to construct sewerage and water supply systems. The costs savings provide leverage
to get local government to provide tenure to slum dwellers. The slum dwellers, once
given tenure, are willing to spend their own money on developing the slums as the
risk of losing their investment is removed. Microcredit schemes mobilise this funding
and allow slum housing to be connected to sewers and water systems and for slums
to be landscaped and lit.
Q.13 ‘Volume Zero’ is a film based on the architectural works of
(A) Peter Eisenman (B) Charles Correa
(C) Daniel Libeskind (D) Roger Angers
Answer: (B) Charles Correa
Director and Producer: Arun Khopkar (2009)
The film deals with the work of Charles Correa, one of world’s prominent architects,
using live footage, extended interview with the architect, diagrams and animation. It
deals with ideas of architecture and cosmic views of the culture of the architect. It
deals with both the physical and spiritual aspects of architecture.

Q.14 The unit of Thermal Conductivity of a material is


(A) W/(m K) (B) W/(m2 K) (C) m K/W (D) m2 K/W
Answer: (A) W/(m K)
K- Value= W/(m K)
U- value= W/(m2 K)
R value= m2 K/W (it is inverse of U-value)

Q.15 A semicircular niche in the wall of a mosque, towards the direction of Mecca is known
as
(A) Sahn (B) Minbar (C) Qibla (D) Mihrab
Answer: (D) Mihrab
Sahn- Courtyard in Islamic Architecture
Minbar- raised platform close to Mihrab in the mosque where the imam stands
to deliver sermons.
Qibla- Qibla, is the direction that should be faced when a Muslim prays.

Kaba : the holiest place in Islam, a large


cube-shaped building inside the al-Masjid al-Haram mosque in Mecca

.16 As per CPWD Guidelines and Space Standards for Barrier Free Built Environment, the
minimum turning radius for a wheelchair in mm is
(A) 900 (B) 1200
(C) 1500 (D) 1800
Answer: Marks to all
Since here CPWD is mentioned, the standards of that organization has to be
followed. Overall circular space required is 1500 mm so turning radius
comes 750 mm.
The turning radius of a vehicle is the radius of the smallest circular turn (i.e. U-turn)
that the vehicle is capable of making.

(This Question was contested by Sowjanya Inala, B.Arch, JNAFAU, Hyderabad and she won it. Earlier the answer
released by GATE was 900mm). She completed her MURP from IIT Roorkee.(2016-18)
Q.17 ‘Summit Curve’ is a term associated with the design of
(A) Roads and flyovers (B) Escalators and elevators
(C) Tensile fabric structures (D) Geodesic domes
Answer: (A) Roads and flyovers
Tensile fabric structures: A tensile structure is a construction of elements
carrying only tension and no compression or bending.
Tensegrity: a structural form with both tension and compression elements.
(truss)
Geodesic Dome: (concept by Buckminster Fuller)

Q.18 As per Census of India 2011, Nagar Panchayat refers to


(A) Rural area surrounded by a growing city (B) Urban area within a panchayat
(C) Rural area adjacent to a nagar (D) Urban area with a statutory
local government
Answer: (D) Urban area with a statutory local government
A Nagar Panchayat or Notified Area Council (NAC) is a settlement which is in
transition from rural to urban. Each Nagar Panchayat has a committee
consisting of a chairman along with ward members. Membership consists of a
minimum of ten elected ward members and three nominated members. NAC
members of the Nagar Panchayat are elected from the several wards of the
Nagar Panchayat on the basis of adult franchise for a term of five years

Q.19 Statutory setback of a building depends on


(A) Building height (B) Property boundary (C) Width of access road (D) Ground
coverage
Answer: (A) Building height, (C) Width of access road
Both the answers were marked correct.

Q.20 Super plasticizer is added in a concrete mix to


(A) Reduce the water-cement ratio for a given workability
(B) Increase the water-cement ratio for a given workability
(C) Reduce the flow
(D) Reduce the content of coarse aggregate
Answer: (A) Reduce the water-cement ratio for a given workability
Superplasticizers, also known as high range water reducers, are chemical
admixtures used where well-dispersed particle suspension is required.
Super absorbant concrete: The concrete works by having a permeable layer on
the surface, made up of relatively large pebbles through which water can drain
almost instantly. This is followed by an ‘attenuation layer’, which feeds the
water into a drainage system that connects with the city’s groundwater
reservoirs.

Q.21 Shodhan House, Ahmedabad was designed by


(A) Anant Raje (B) Le Corbusier
(C) Louis I. Kahn (D) B. V. Doshi
Answer: (B) Le Corbusier
(adjoining photo is of Shodan House, Ahmadabad)
Le Corbusier’s work in India
Town Planning of Chandigarh
High court, Chandigarh
Secretariat, Chandigarh
Governor House, Chandigarh
Govt. College of Art, Chandigarh
Chandigarh College of Architecture, Chandigarh
Museum and gallery of Art, Chandigarh
Shodan House, Ahmedabad (GATE 2016)
Sarabhai House, Ahmedabad
Mill Owners Association Building, Ahmedabad. (also known as ATMA
House)
Sanskar Kendra Museum, Ahmedabad

Other Important works of foreign architect in Independent India, apart from Le


Corbusier.
IIM Ahmedabad- Louis I Kahn
Centre for Performing Arts, Mumbai – Philip Johnson
India International Centre- J. A. Stein
Khalsa Heritage Centre, Anandpur Saheb- Moshe Shafdie
Bihar Museum, Patna- Fumihiko Maki

Q.22 Low-emissivity coating on a glazing unit


(A) Increases the SHGC (B) Increases the VLT
(C) Reduces the SHGC (D) Increases the VLT and SHGC
Answer: (C) Reduces the SHGC
SHGC- Solar Heat Gain Co-efficient. The SHGC is the fraction of incident solar
radiation admitted through a window, both directly transmitted and absorbed
and subsequently released inward. SHGC is expressed as a number between
0 and 1. The lower a window’s solar heat gain coefficient, the less solar heat it
transmits.
VLT: The percentage of visible light that passes through a window or other
glazing unit is called the Visible Light Transmittance (T vis or VLT).
When heat or light energy is absorbed by glass it is either shifted away by
moving air or reradiated by the glass surface. The ability of a material to radiate
energy is known as emissivity. In general, highly reflective materials have a low
emissivity and dull darker colored materials have a high emissivity. All
materials, including windows, radiate heat in the form of long-wave, infrared
energy depending on the emissivity and temperature of their surfaces.
Low- emmisivity coating- These coatings reflect radiant infrared energy, thus
tending to keep radiant heat on the side of the glass where it originated, while
letting visible light pass. This results in more efficient windows because radiant
heat originating from indoors in winter is reflected back inside, while infrared
heat radiation from the sun during summer is reflected away, keeping it cooler
inside.

Q.23 Spatial connectedness in GIS refers to

(A) Tomography (B) Topography (C)


Topiary (D)Topology
Answer: (D)Topology
Spatial is related with place and temporal with time.
Tomography: It refers to imaging by sections through the use of any kind of
penetrating wave. eg CT scan(Computerised Tomography Scan) – medical
term
Sonography- ultrasonic wave is used as in ultrasound.- medical term
Topography is a detailed map of the surface features of land.
Topiary- pruning of trees to give a particular shape or form.
Topology- Topology basically refers the set of rules between things, and
in the GIS, Topology refers to the relationship between spatial features or
objects. It fills in gaps, removes overlapping polygons. When polygons
share a boundary, topology rules enforce data integrity (correctness of
data)
Georeferencing is the process of assigning real-world coordinates to each pixel
of the raster. Many times these coordinates are obtained by doing field surveys
– collecting coordinates with a GPS device for few easily identifiable features in
the image or map.

.24 In a residential neighbourhood, the net area of residential plots is 50 percent of the total
area. If the population is 8000, the ratio of net density to gross density of the
neighbourhood is ________
Answer: 2
Let the total area of neighbouhood be ‘A’ units
50% is designated for Residential use = 0.5A

Net Density= = Gross Density=

= × =2

.25 A hemispherical earth mound of 3 meter diameter is proposed to be constructed in a


children’s park. If the proportion of soil and sand for the construction is 3:2, the
estimated volume of soil in cubic meters is ________
Answer: 4.24 m3

Volume of hemisphere = ( ×π×r3 ) where r = 1.5 m


V= { × ×(1.5)3 }
=7.07 m3

Volume of soil = ×7.07= 4.24 m3

Q. 26 – Q. 55 carry two marks each.

Q.26 Match the ancient cities in Group –I with their characteristic features in Group-II
Group-I Group-II
(P) Mohen-jo-daro 1 Agora
(Q) Babylon 2 Enclosed Court
(R) Kahun 3 Grid Iron Street Pattern
(S) Athens 4 Forum
5 Hanging Garden
(A) P-3, Q-4, R-2, S-5 (B) P-4, Q-5, R-1, S-2 (C) P-3, Q-5, R-2, S-1 (D) P-4, Q-
5, R-2, S-3
Answer: (C) P-3, Q-5, R-2, S-1
Babylon: The city was built upon the Euphrates and divided in equal parts
along its left and right banks, with steep embankments to contain the river’s
seasonal floods. The remains of the city are in present-day, Iraq, about 85
kilometers south of Baghdad, comprising a large tell of broken mud-brick
buildings and debris.
Kahun- ancient Egyptian town,It was erected for the overseers and workmen
employed in constructing the nearby pyramid of Al-Lāhūn, built by Sesostris II
(reigned 1844–37 bce), and it was abandoned when the pyramid was
completed. It revealed a crisscross of streets laid out in a regular pattern, with
houses built of mud brick having beamed, flat mud roofs, open courts and
porticos, and the earliest examples of a supporting wooden column, fluted and
on a raised base.

Q.27 Match the water purification stages in Group-I with the corresponding items in Group-
II
Group-I Group-II
P Coagulation 1 Ammonium hydroxide
Q Filtration 2 Sodium Zeolite
R Disinfection 3 Alum
S Softening 4 Sand
5 Chlorine
(A) P-3, Q-4, R-1, S-5 (B) P-1, Q-4, R-3, S-2 (C) P-2, Q-3, R-5, S-4 (D) P-3,
Q-4, R-5, S-2
Answer: (D) P-3, Q-4, R-5, S-2
A coagulant is added to the water to overcome the repulsive charge and
“destabilize” the suspension. For example, the colloidal particles are negatively
charged and alum is added as a coagulant to create positively charged ions.
Once the repulsive charges have been neutralized ( since opposite charges
attract), the van der Waals force will cause the particles to cling together
(agglomerate) and form micro floc which finally settles down.
Sand filters are used for water purification. There are three main types;
rapid (gravity) sand filters
upflow sand filters
slow sand filters
All three methods are used extensively in the water industry throughout the
world. The first two require the use of flocculant chemicals to work effectively
while slow sand filters can produce very high quality water free from
pathogens, taste and odour without the need for chemical aids.
Shock chlorination is a process used in many swimming pools, water wells,
springs, and other water sources to reduce the bacterial and algal residue in
the water. Shock chlorination is performed by mixing a large amount of
hypochlorite into the water. The hypochlorite can be in the form of a powder or
a liquid such as chlorine bleach (solution of sodium hypochlorite in water).
Water that is being shock chlorinated should not be swum in or drunk until the
sodium hypochlorite count in the water goes down to three parts per million
(PPM) or less.
Sodium Zeolite- One of the most important uses for this mineral is in
detergents. Sodium Zeolite replaces sodium tripolyphosphate in detergents as
a water softener.

Q.28 Match the software tools in Group-I with their field of application in Group-II
Group-I Group-II
P Radiance 1 Outdoor thermal environment
Q Odeon 2 Construction management
R Rayman 3 Air-flow analysis
S Primavera 4 Acoustical design
5 Lighting analysis
(A) P-3, Q-4, R-5, S-1 (B) P-5, Q-3, R-1, S-2 (C) P-5, Q-4, R-1, S-2 (D) P-4,
Q-1, R-2, S-5
Answer: (C) P-5, Q-4, R-1, S-2
Radince- software package for architectural lighting simulation.
Odeon (building), ancient Greek and Roman buildings built for singing
exercises, musical shows and poetry competitions.
RayMan- software for thermal environment
Primavera: Primavera software includes project management, product
management, collaboration and control capabilities.

Q.29 Match the street layouts of ancient Indian


settlements in Group – I with their corresponding types in Group – II
(A) P-2, Q-4, R-3, S-5 (B) P-2, Q-3, R-4, S-5
(C) P-4, Q-3, R-5, S-1 (D) P-4, Q-3, R-2, S-1
Answer: (B) P-2, Q-3, R-4, S-5
Karmuka- It was suitable for port town either at sea or river. Banaras Hindu
University Campus is planned in Karmuka style.
Jaipur- Prastara

Q.30 Associate the terms in Group-I with the building hardware in Group-II
Group-I Group-II
P Parliamentary 1 Bar
Q Aldrop 2 Lock
R Panic 3 Hinge
S Mortise 4 Bolt
(A) P-3, Q-1, R-2, S-4 (B) P-2, Q-4, R-1, S-3 (C) P-3, Q-4, R-1, S-2 (D) P-2,
Q-3, R-4, S-1
Answer: (C) P-3, Q-4, R-1, S-2

A crash bar (also known as a


panic bar, exit device, panic device, or a push bar) is a device for unlocking a
door during emergency conditions. The mechanism consists of a spring-loaded
metal bar fixed horizontally to the inside of an outward-opening door. When the
lever is either pushed or depressed, it activates a mechanism which unlatches
the door allowing occupants to leave quickly from the building.

Q.31 Hoop and meridional forces are associated with


(A) Dome (B) Truss (C) Folded Plate (D) Space Frame

Answer: (A) Dome


Truss- forces acting are Tension and Compression
Folded Plate- Folded plate structures consist of flat components, or plates, that
are interconnected at some dihedral angle.
Space frame or space structure is a truss-like, lightweight rigid structure
constructed from interlocking struts in a geometric pattern. Space frames can
be used to span large areas with few interior supports.

.32 Match the Olympic stadia in Group-I with their Architects in Group-II
Group-I Group-II
P Palazzetto dello Sport, Rome 1 Herzog & de Meuron
Q Olympic Arena, Tokyo 2 Frei Otto
R Bird’s Nest, Beijing 3 Kenzo Tange
S Olympia Stadium, Munich 4 Roger Taillibert
5 P. L. Nervi
(A) P-5, Q-2, R-1, S-4 (B) P-5, Q-3, R-1, S-2 (C) P-2, Q-1, R-4, S-5 (D) P-2,
Q-4, R-1, S-3
Answer: (B) P-5, Q-3, R-1, S-2
Freio Otto was the Pritzker Prize winner for the year 2015. Olympic stadium
was designed by him in 1972.
2016 Olympics in Rio de Janeiro- sports complex designed by BCMF
Architects (Brazil).

Q.33 Match the terms in Group-I with the related terms in Group-II
Group-I Group-II
P Acquisition 1 Ownership
Q Planning permission 2 Construction
R Building plan sanction 3 Land cover
S Mutation 4 Land use
5 Land
(A) P-5, Q-4, R-2, S-1 (B) P-5, Q-3, R-1, S-2 (C) P-3, Q-4, R-5, S-1 (D) P-5,
Q-3, R-2, S-4
Answer: (A) P-5, Q-4, R-2, S-1

Q.34 Associate the structural systems of Group-I with buildings in Group-II


Group-I Group-II
P Diagrid 1 Millennium Dome, London
Q Outrigger truss 2 HSBC, Hong Kong
R Suspended floor 3 Taipei 101, Taipei
S Cable stayed 4 Hearst Tower, New York
5 Sears Tower, Chicago
(A) P-1, Q-2, R-3, S-5 (B) P-5, Q-3, R-4, S-1
(C) P-4, Q-2, R-3, S-1 (D) P-4, Q-3, R-2, S-1
Answer: (D) P-4, Q-3, R-2, S-1
Diagrid : It is a framework of diagonally intersecting metal, concrete or wooden
beams that is used in the construction of buildings and roofs. It requires less
structural steel than a conventional steel frame.

Two important
(iconic) buildings that uses diagrid system is :
Hearst Tower in New York City, designed by Sir Norman Foster
Gherkin in London by Sir Norman Foster.
Outrigger truss: The outrigger and belt truss system is one of the lateral load resisting
system in which the external columns are tied to the central core wall with very stiff
outriggers and belt truss at one or more levels (refer adjacent Figure ).
When the lateral load acts on the building, the bending of the core rotates the stiff
outrigger arms, which is connected to the core and induces tension and compression in the
columns
Suspended floor:
HSBC, Hongkong (designed by Sir Norman Foster) The building is more akin to the
structure of a bridge, with floors suspended from the distinctive chevron-shaped
trusses supported by eight sets of masts that run the height of the building. The
structure is not hidden and is a feature of the façade. Services are accommodated on
the periphery, allowing clear floor spans. Its variation is introduced with double-height
spaces on the floors that meet the bottom of each truss. Express lifts serve only
certain floors; further access to individual levels is via escalators.

Cable stayed : The dome is one of the largest of its type in the world. Externally, it
appears as a large white marquee with twelve 100 m-high yellow support towers, one
for each month of the year, or each hour of the clock face, representing the role
played by Greenwich Mean Time. In plan view it is circular, 365 m (one metre for each
day in a standard year) in diameter.

Q.35 Associate the systems in Group-I with their applications in Group-II


Group-I Group-II
P Nisargruna 1 Renewable energy generation
Q Vortex-DEWAT 2 Ground water recharge
R Swale 3 Solid waste management
S BIPV 4 Desalination
5 Waste water treatment
(A) P-4, Q-1, R-5, S-3 (B) P-1, Q-2, R-5, S-4 (C) P-3, Q-5, R-2, S-1 (D) P-5,
Q-4, R-2, S-1
Answer: (C) P-3, Q-5, R-2, S-1
Nisargruna: developed by BARC (Bhaba Atomic Research Centre)- This plant
can process biodegradable waste such as kitchen waste, paper, grass, gobar
and dry leaves. It offers Zero garbage and Zero effluent and provides high
quality manure and methane gas. This technology of biphasic biomethanation
has high potential of solving the solid waste management problems of the
urban areas and provides organic manure and bio-gas as a fuel.
The DEWATS (decentralized waste water treatment system) can be seen in
use in several places around Auroville. The vortex system is useful in more
compact areas that do not have sufficient space for a planted filter bed.
Flowforms, through which the water travels into the polishing pond, can be
integrated into the landscape in an aesthetically pleasing way.
Swale is low lying shallow water pool from which water slowly percolates to the
ground.
BIPV- Building-integrated photovoltaics (BIPV) are photovoltaic materials that
are used to replace conventional building materials in parts of the building
envelope such as the roof, skylights, or facades.
Q.36 Match the Houses in Group-I with their Architects in Group-II
Group-I Group-II
P Villa Müller, Prague 1 Frank Gehry
Q Farnsworth House, Illinois 2 Frank Lloyd Wright
R Schröder House, Utrecht 3 Adolf Loos
S Dancing House, Prague 4 Mies van der Rohe
5 Gerrit Rietveld
(A) P-5, Q-2, R-4, S-1 (B) P-3, Q-4, R-5, S-1 (C) P-3, Q-2, R-5, S-4 (D) P-5,
Q-4, R-2, S-3
Answer: (B) P-3, Q-4, R-5, S-1

Q.37 Match the Books in Group-I with their Authors in Group-II


Group-I Group-II
P Space, Time and Architecture 1 Bill Hillier
Q The Social Logic of Space 2 Christopher Alexander
R Timeless Way of Building 3 Rob Krier
S Form, Space and Order 4 Sigfried Gideon
5 Francis D. K. Ching
(A) P-3, Q-4, R-1, S-5 (B) P-4, Q-3, R-1, S-2
(C) P-5, Q-4, R-2, S-3 (D) P-4, Q-1, R-2, S-5
Answer: (D) P-4, Q-1, R-2, S-5

Q.38 Associate the green rating system in Group-I with the respective country in Group-II
Group-I Group-II
P CASBEE 1 UAE
Q Green Mark 2 China
R GRIHA 3 Japan
S Estidama 4 Singapore
5 India
(A) P-2, Q-4, R-5, S-1 (B) P-4, Q-1, R-5, S-2 (C) P-3, Q-5, R-1, S-2 (D) P-3,
Q-4, R-5, S-1
Answer: (D) P-3, Q-4, R-5, S-1
Green Rating for various countries. In India LEEDS, GRIHA, IGBC, BEE and EDGE is
used.
S. Country Rating
No. System
1 USA LEED
2 UK BREEAM
3 Canada LEED Canada
4 India GRIHA
5 Japan CASBEE
6 China Label-3
7 Singapore Green Mark
8 UAE Estidma
9 Australia Green Star

Q.39 Match the instruments in Group-I with the corresponding usage in Group-II
Group-I Group-II
P Pyranometer 1 Shadow analysis
Q Heliodon 2 Seismic intensity
R Total Station 3 Wind velocity
S Anemometer 4 Solar radiation
5 Land survey
(A) P-2, Q-1, R-5, S-3 (B) P-4, Q-3, R-1, S-2 (C) P-4, Q-1, R-5, S-3 (D) P-1, Q-4,
R-2, S-3
Answer: (C) P-4, Q-1, R-5, S-3

No Functions Instruments
1. Ambient Thermometer
Temperature
2. Low air speed Kata Thermometer
3. Humidity Dry-Wet Bulb thermometer
(Hygrometer)
4. Wind Velocity Anemometer
5. Solar radiation Solarimeter, actinometer,
pyranometer
6. Sun Path Heliodome
7. Lat/Long, Time GPS
8. Sound Intensity Decibel meter
9. Illumination Lux meter
10. Precipitation Disdrometer
11 Rainfall Raingauge, udometer,
pluviometer
12. Altitude Altimeter
13 Air Purity Eudiometer
14. Intensity of Sunlight Heliograph
15. Dust in air Konimeter
16. Vapour Pressure Manometer
17. Area of plane figure Planimeter
18. Transpiration Potometer
19. Distance measuring Distomat
20 Shadow analysis Heliodon
21 Height of cloud base Celiometer
22 Soil moisture Potentio meter
23 Length of river Opisometer, meilograph,
measured on map map measure

.40 As per URDPFI guidelines, the number of Equivalent S. Vehicle ECS PCU
Car No
Space (ECS) required to accommodate ten cars, 1 Car 1 1
sixteen scooters and forty bicycles will 2 Auto 0.5 0.8
be______________ Rickshaw
Answer: 18
3 Two 0.25 0.5
Total ECS= (10×1) + (16×0.25) + (40×0.1) = 18
Wheeler
Equivalent Car Space (ECS)
URDPFI: Urban and Regional Development Plans 4 Bicycle 0.1 0.2
Formulation & Implementation (URDPFI) 5 Emergency 2.5
Guidelines. Vehicle
(Note: ECS is different from PCU) 6 Rickshaw 0.8 1.5
7 Truck/bus 2.5 3.0

Space standards for Parking


S No Type of parking Area in sqm
per ECS
1 Open 23
2 Ground Floor 28
Covered
3 Basement 32
4 Multi level with 30
Ramps
5 Automated 16
multilevel with lifts
source: URDPFI guidelines 2014, Vol I (page 292)

.41 A steel I-beam section is subjected to a bending moment of 96 kN-m. The moment of
inertia of the beam section is 24,000 cm4. The bending stress at 100 mm above the
neutral axis of the beam in MPa will be ____________
Answer: 40 MPa

Allowable bending stress= = =

= = 40 × 106 N/mm2
= 40 MPa
` 40 × 106 N/mm2 = 40 MPa

.42 A room is mechanically ventilated through four air-conditioning ducts. The opening area
of each duct is 0.35 sqm. The air velocity in the duct is 0.5 m/s. The temperature
difference between the ambient air and supply air is 10 °C. Volumetric specific heat of
air is 1250 J/m3 °C. Assuming one Ton of refrigeration (TR) equals 3.5 kW, the cooling
load of the room in TR will be _____________
Answer: 2.33
Total volume of air sent to room per sec= 4× 0.35 × 0.5 = 0.7 m3 /s
Total heat exchange per second Qv= 1250 J/m3 °C × rate of ventilation×
= 1250 × 0.7 × 10 = 8750 J/s

Watt= , therefore 8750 J/s = 8750 Watt= 8.75 kW

One ton of refrigeration = 3.5 kW, therefore 8.75 kW = = 2.5 TR (ton of


refrigeration)

.43 A CPM network of a construction project is given in the figure below. The activity
durations are mentioned in weeks. The project completion time in weeks will be
_____________

Answer: 18 weeks

.44 For a room with dimensions 4m × 3m × 3m (L×B×H), the details of indoor acoustical
treatment are as follows.
The reverberation time in seconds at 1000 Hz is _______
Answer: 0.45 sec

T60 = where A=
Here A=summation of (area × absorption coefficients )
Dimension of room 4m x 3mx 3m (LxBxH)
Total area of wall = 42 sqm (30 % with coefficient 0.4 and 70 % with coefficient 0.1)
Total area of ceiling= 12 sqm (40 % with coefficient 0.6 and 60 % with coefficient 0.1)
Total area of floor= 12 sqm (100% with coefficient 0.1)

Awall =( × 42 +( × 42 5.04+2.94
= 7.98

Aceiling =( × 12 +( × 12 2.88+ 0.72


= 3.6
Afloor = 12 × 0.1= 1.2

Total A= 7.98 +3.6 + 1.2= 12.78 m2 sabines


Volume of Room = 4

applying formula T60 = =0.45 sec

Suggested Reading

Reverberation time is the time required for a steady-state sound to reach one
millionth or -60dB of its original intensity.
There are several models used in calculating the reverberation time but the first and
most commonly used is that of Wallace Sabine (1868-1919). The Sabine equation

states that the reverberation time (Tr, in seconds) is directly proportional to the
volume of the room (V, [m3]) and inversely proportional to the room's effective surface
area (A, [m2]). The effective surface area is the sum of the product of an area covered
by a particular material and the material's absorption coefficient.

The units of ‘A’ are sabins. For a surface of area ‘A’ and with absorption
coefficient ‘a’, ‘aA’ can be thought of as the equivalent area of a perfect absorber
(open window). The absorption coefficient varies with frequency and so the
reverberation time is a function of frequency.

.45 In 2001, the population and work force participation rate of a town were 30,000 and 30
percent respectively. The work force participation rate in the year 2011 increased to 34
percent. If the decadal population growth rate was 6 percent, the increase in the
number of working people in the town in 2011 was __________

Answer: 1812

Total workforce in year 2001 = × 30,000= 9000 for pop of 30,000


In 2001 Population growth rate (6% decadal) so total population = × 30,000=
31,800

work force participation in the year 2011 (34%) = × 31,800= 10,812


Increase in the number of working people in the town in 2011 was 10,812-9000=
1812

.46 In a 20 storey building with 3m floor to floor height, a passenger lift is hoisted by a steel
rope.Weight of the lift car is 750 kg and ultimate load the steel rope can carry is 39,000
kg. Assuming a factor of safety of 20 for the steel rope and an average passenger
weight of 75 kg, the passenger capacity of the lift is ____________
Answer: 16
After deduction of factor of safety load carriage capacity (including wt of lift car) =

= 1950 kg
Deduction of lift car = 1950- 750= 1200

No of passenger= = 16

.47 One litre of acrylic paint can cover 16 sqm of wall area for the first coat and 24 sqm for
the second coat. The walls of a lecture hall measuring 12m × 8m × 4m (L × B × H)
need to be painted with two coats of this paint. The hall has total glazed fenestration
area of 12 sqm. The number of 4 litre paint containers required will be __________
Answer: 4
(Note: Dimension of hall is given. Total wall area has to be calculated.)
Total vertical area of the hall (including wall + opening) = (12 +8) x 2 x 4 =160 sqm
Area that has to be painted = 160-12= 148 sqm.

Total paint (litre)required for First coat = = 9.25 litres

Total paint (litre)required for Second coat = = 6.16 litres


----------------------------------
Total paint required = 15.41 litres
Since Paint is available in 4 litre container therefore total 4 container is required.

.48 A 250 mm × 250 mm RCC column is reinforced with one percent steel. The permissible
compressive stress of concrete and steel are 8 N/mm2 and 150 N/mm2 respectively.
The axial load carrying capacity of the column in kN is _____________
Answer: 588.75 kN
Area of concrete column = 250 × 250 = 62500 mm2

Cross sectional area of steel = 1% = × 62500= 625 mm2


Sectional area od concrete = 62500- 625= 61875 mm2
Axial load carrying capacity = (625 × 150) + (61875 × 8)
= 93750 + 495000
= 588750 N (588.75 kN)
When permissible stress is not given then Grade of concrete and steel quality
is given
eg. M30 concrete and Fe 415, in that case following formula is applied.
Pu =0.4 fck Ac + 0.67 fy Asc
where,
Pu= factored axial load on the member,
fck= characteristic compressive strength of the concrete,
Ac= area of concrete,
fy= characteristic strength of the compression reinforcement, and
Asc= area of longitudinal reinforcement for columns.

.49 A solar photo-voltaic system is proposed to be installed at the roof top of a hostel. The
cost of installation and the annual maintenance are INR 2,40,000 and INR 6000
respectively. It is expected to generate 600 kWh of electricity per month. Assume unit
price of electricity as INR 5. Ignoring the discount rate, the payback period of the
investment in years is____________
Answer: 8
600kWh is 600 units of power; so total money saved per month = 600 ×5= 3000
Rs/month
yearly saving = 3000× 12= 36,000 Rs
Let the pay back period be ‘p’ years.
Then framing equation we get 2,40,000 + 6000p = 36,000p
Solving for ‘p’ 30,000p= 2,40,000
p= 8 years

.50 A pump is installed in an apartment building to lift water from ground level to the roof top
water tank with the capacity of 10,000 litres. Total head of lift is 18 m and pumping time
is 30 minutes to fill the tank completely. Assuming acceleration due to gravity (g) as 10
m/sec2 and efficiency of the pump as 80 percent, the power requirement of the pump in
kW will be __________
Answer: 1.25 kW
Total work done in lifting water = mgh (Mass ×gravity ×height)
= 10,000 × 10 × 18
= 18,00,000 joules= 1800 kJ
Time taken is 30 min = 30 ×60 =1800 sec

Total power of pump req = = 1 kJ/sec= 1 kW


since the pump operates at 80 % efficiency.

Let the rated watt be ‘Z’ watt , therefore Z× = 1 kW


solving for ‘Z’ we get Z= 1.25 kW
.51 In a housing project, the number of LIG, MIG and HIG units are in the ratio of 1:1:2. The
ratio of areas of the units is 3:5:8. Assume unit cost of construction is same for all the
three types. For a no profit no loss situation, if 10% discount is offered to LIG and MIG
units on sale price, extra charge in percentage payable per HIG unit will be
____________
Answer: 5 %
LIG : MIG : HIG = 1:1:2 and area in the ratio of 3: 5: 8
Ratio of cost of construction of three segments will be 3 : 5 : 16
10 % discount to LIG and MIG so the proportionate cost of LIG and MIG will be 2.7 :
4.5
Adding this both will give 2.7 + 4.5= 7.2 (earlier it was 8)
so the burden of 0.8 will pass to 16 units
16 units –------- 0.8

1 unit-----------

percentage payable per HIG unit × 100 = 5 %

.52 The estimated number of bricks (unit size: 250 mm × 125 mm × 75 mm) for laying one
course of a 250 mm thick brick wall using rat-trap bond for a running length of 3.9
meter will be ___________
Answer: 36 bricks
In rat trap bond bricks are laid on edges.
A module has three bricks (two bricks horizontal to wall and one perpendicular to wall,
all laid on edges.
Length of one module = 250 mm +75 mm = 325 mm i.e. (for a running length of 325
mm we need three bricks)

for a running length of 3.9 m we need = 12 module


Total no. of bricks= 12 × 3= 36 bricks

Q.53 The difference in invert levels between two pits


separated by a distance of 30 meter is one meter. An intermediate pit is required to be
constructed at a distance of 18 meter from the pit at higher level. Maintaining the same
slope, the difference in invert levels of the new pit and the pit at lower level in mm will
be _________

Answer: 400 mm

=
Solving for x
we get x= 0.4m
0.4 m = 400 mm

Q.54 A four-storey building with equal areas in each floor is required to be designed
on a plot with FAR of 2.0. If the FAR is increased to 2.2, the
percentage increase in ground coverage utilizing full FAR in both cases
will be ___________
Answer: 10 %
Let the plot area be ‘A’
Total built up area of all floors= 2A

Since it is a four storey building ground coverage = = 0.5 A


In second case FAR increaded to 2.2 , total built up area of all floors= 2.2A

Ground coverage =
Increase in ground coverage = - = = 0.05 A

% increase in ground coverage = × 100= 10


%

.55 A lamp source of 3200 candela is mounted on a wall at a height of 2 meter from the
work-plane. It subtends an angle of incidence of 60º with the center of the work plane.
The illumination at the centre of the work plane in Lux is __________
Answer: 100 Lux

Sin 30º = and d= 4m

Illumination = ×

= × cos 60= × = 100 Lux


(Note: Angle of incidence is the angle of incident light from normal)
(Note: Normal is perpendicular to work plane.)
GATE 2017
Q.1 The Pritzker Architecture prize for the year 2016 has been awarded to
(A) Alejandro Alevana (B) Frei Otto (C) Stephen Breyer(D) Yung Ho Chang
Answer: (A) Alejandro Alevana

Q.2 As per CPWD Handbook on Barrier Free and Accessibility, 2014 Government of India,
the minimum length of a straight ramp in meter to raise a wheelchair to the plinth level of
600 mm is _________
Answer: 7.2 m
Slope of Ramp: Not more than 1:12 (1:6 slope for ramp for car parking)
Width of Ramp – A ramp shall not be less than 1800 mm in width.

Max. Length of ramp in one go= 9m


Height of hand rail on ramp= 800mm – 900 mm
Min. distance between wall and hand rail= 50 mm
Dia. of hand rail= 40 mm
Access Route: The clear width of an access route shall not be less than 1200 mm
and for two way traffic it should be 1650mm-1800mm wide.
Tactile floor : Three kinds of tactile tiles/blocks are commonly used in constructing a
tactile guide path:
Directional Tile/Block- This has parallel raised bars for guiding the users
along an intended safe path.
Hazard warning Tile/Block- This has raised big dots (35mm in diameter)
arranged in square grid parallel to the sides of the slab for indication of
potential hazards ahead. This type of tile/block could be used alone to form
tactile warning strips at the top and bottom ends of staircase or ramps, and at
dropped kerb.
Positional Tile/Block- This has raised small dots (23mm in diameter) placed
in staggered position for indication of possible change in walking directions.
Tactile warning strip should have a minimum luminous contrast of 70% with the
adjoining surfaces.
Q.3 Tuscan and Composite Orders are associated with
(A) Greek Architecture (B) Islamic Architecture (C) Byzantine Architecture (D)
Roman Architecture
Answer: (D) Roman Architecture
Greek Orders- Doric, Ionic, and Corinthian
Roman Orders- Tuscan and Composite Orders

Q.4 A pointed arch having two centres and radii greater than the span is known as
(A) Lancet arch (B) Gothic arch (C) Roman arch (D) Drop arch

Answer: (A) Lancet


arch
Roman Arch- Semi circular, radius half of span and one centre.
Lancet and Gothic- arch having two centres and radii greater than the span.
Drop arch- radius is smaller than span.
Q.5 The concepts of ‘serial vision’, ‘punctuation’ and ‘closure’ were proposed by

(A) Le Corbusier (B) Louis Kahn


(C) Gorden Cullen (D) Kevin Lynch
Answer: (C) Gorden Cullen
From ‘Townscape’ by Gorden Cullen.

Q.6 In one litre of paint, volume of solid pigment and volume of non-volatile binder are 400 cc
and 600 cc respectively. The Pigment Volume Concentration number of the paint is
___________
Answer: 40

PVC= × 100
= 40
Pigment Volume Concentration (PVC), Volume Solid (VS) and Hiding power are the
properties of paint.
Pigment Volume Concentration or PVC gives an idea of how much pigment
there is in the paint compared to the amount of binder. (in percentage). Volatile
thinner is not taken in account.

PVC= × 100
Volume Solid (VS) is (in percentage)

VS= × 100

Hiding power or ‘opacity’ or


‘coverage’ are words that refer to paint’s ability to completely obliterate the
colour of the surface it is applied over. A paint with excellent hiding power
may completely hide words written in black paint on a white background in one
coat applied at 12 square metres per litre whereas a poor hiding power paint
may need to be applied in 3-4 coats at that spreading rate to achieve the same
effect. Hiding power is obtained by pigment.

Q.7 ‘Cold joint’ refers to the


(A) expansion joint in large span concrete members
(B) interface between an already setting concrete and a fresh batch of concrete
(C) structural crack arrested by embedding metal rods
(D) joining of two similar metals in vacuum
Answer: (B) interface between an already setting concrete and a fresh batch of concrete
Cold welding or contact welding is a solid-state welding process in which joining takes place without
fusion or heating at the interface of the two parts to be welded in vacuum.

.8 Slenderness ratio of a column is represented as:


(A) Effective length / Cross-sectional area (B) Effective length / Radius of gyration
(C) Actual length / Cross-sectional area (D) Actual length / Radius of gyration
Answer: (B) Effective length / Radius of gyration
S. End Effective
No
condition Length
1 Pinned-
pinned L
2 Free end-
fixed 2L
3 Pinned-
fixed 0.7L
4 Fixed- Effective
fixed L/2 Length of Column

Types of load
If the load on a column is applied through the center of gravity (centroid) of its cross section, it is called
an axial load.
A load at any other point in the cross section is known as an eccentric load.
A load from sidewards is called lateral load
A short column under the action of an axial load will fail by direct compression before it buckles, but a long column
loaded in the same manner will fail by springing suddenly outward laterally (buckling) in a bending mode.
[Ref: Q-69 (2007), Q-42(2003), Q18 (1994)]
Q.9 Liquidated damage refers to the
(A) cost borne by the contractor to rectify defects within defect- liability period
(B) compensation paid on breach of contract to the affected party by the other party
(C) money paid by the insurance company to the owner of insured property if it is
damaged
(D) money earned by the owner from selling damaged property through auction
Answer: (B) compensation paid on breach of contract to the affected party by the
other party
Unliquidated damage- When the contractor or owner break the contract, the
amount is received by the party who suffer this.
Q.10 Which of the following process is NOT used for corrosion resistance of cast iron?
(A) Painting (B) Epoxy coating (C) Quenching (D) Galvanizing
Answer: (C) Quenching
Epoxy coating- The raw materials for epoxy resin production are today largely
petroleum derived which are used in coating various metals for corrosion
protection.
Quenching- Quenching is the rapid cooling of a metal to obtain certain material
properties.
Galvanizing- putting iron in molten zinc.
Annealing and Tempering- sudden heating and cooling at room temperature or
gradual cooling.
Pasteurization (milk) – process of heating milk at 72° C for 15 seconds and
then quickly cooling down and storing at 4° C.

Q.11 Data on ‘households with one or more married couples sharing room with a person aged
12 years or more’, is used for computing
(A) house density (B) housing shortage (C) housing price (D) housing
affordability
Answer: (B) housing shortage
Components of Housing Shortage
Obsolescence Factor
Non Serviceable Katcha (temporary) Houses
All bad houses excluding those less than 40 years old
80 years old houses(Old structurally weak)
Congestion Factor (Unacceptable as per social norms)
Households with one or more married couples sharing room with a person
aged 12 years or more
Houselessness
Households living without any house(open, pavements)

Q.12 Excellence in Design for Greater Efficiency (EDGE) programme DOES NOT focus on
(A) lower carbon emission(B) greater resource efficiency (C) cost
effectiveness (D) labour safety
Answer: (D) labour safety
EDGE-Excellence in Design for Greater Efficiencies
An online platform that allows design teams to estimate the efficiency of a building by
using practical solutions and alternative materials.
EDGE defines a green building as one that is projected to reduce the consumption of
energy and water by at least 20 percent, as well as the energy used to make the
construction materials. It is controlled by IFC a member of WBG.
The World Bank Group headquarters in Washington, DC. IFC, (International
Financial Corporation) is a member of the World Bank Group, is the largest global
development institution focused exclusively on the private sector in developing
countries.

.13 Select the right option representing strategic components arranged in ascending order
of specified minimum area under Smart City Mission of Government of India.
(A) Greenfield development- Redevelopment – Retrofitting
(B) Redevelopment – Greenfield development – Retrofitting
(C) Retrofitting – Redevelopment – Greenfield development
(D) Redevelopment – Retrofitting- Greenfield development
Answer: (B) Redevelopment – Greenfield development – Retrofitting
Specified Minimum area for Smart City Mission.
Redevelopment- more than 50 acre
Green Field Development- more than 250 acre
Retrofitting- more than 500 acre.
Pan city development.

Q.14 The grade-separated interchange suitable for 3-legged intersection is:


(A) Trumpet (B) Full Clover leaf (C) Diamond (D) Partial Clover leaf
Answer: (A) Trumpet
Various types of junctions.
1. Underpass
2. Overpass
3. Trumpet Interchange- where one highway terminates at another highway.
4. Diamond Interchange- Diamond interchange is a popular form of four-leg
interchange found in the urban locations where major and minor roads
crosses.
5. Cloverleaf Interchange
6. Partial Cloverleaf Interchange
7. Directional Interchange
8. Bridged Rotary
Trumpet interchanges have been used where one highway terminates at another highway.

Trumpet junction Cloverleaf interchange


Q.15 The design element provided to ensure
safety of a vehicle travelling at a prescribed design speed along the curved segment of
a highway is
(A) shoulder
(B) super-elevation
(C) median
(D) footpath

Answer: (B) super-elevation


Road Median is the portion between the dual carriageway which separates the
traffic flow in opposite direction.
Min. width of footpath as per IRC is 1.8m

Required width of footpath as per adjacent land use


S.No. Description Width
(m)
1 Minimum free walkway width 1.8
and residential/mixed use
areas
2 Commercial/Mixed Use 2.5
Areas
3 Shopping Frontages 3.5-4.5
4 Bus Stops 3
5 High Intensity Commercial 4
Areas
Source: IRC 103‐2012.

.16 Which of the following processes is NOT adopted in solid waste management?
(A) Incineration (B) Pyrolysis (C) Flocculation (D) Sanitary landfill
Answer: (C) Flocculation
Incineration is burning of organic waste.
Pyrolysis is the heating of an organic material, such as biomass, in the
absence of oxygen.
Flocculation, in the field of chemistry, is a process wherein colloids come out of
suspension in the form of floc or flake; either spontaneously or due to the
addition of a clarifying agent.
Sanitary landfills are sites where waste is isolated from the environment until it
is safe. It is considered when it has completely degraded biologically,
chemically and physically.
Q.17 The principle of Eminent Domain is the power to
(A) restrict exercise of rights in land through zoning and environmental laws
(B) control land use No Country Terminology
(C) retain land use 1 USA, Phillipines Eminent
(D) acquire and take possession of property Domain
in order to promote public interest. 2 UK, Ireland, New Compulsory
Zealand purchase
Answer: (D) acquire and take possession
3 HongKong Resumption
of property in order to promote public
interest. 4 France, Italy Expropriation
laissez-faire- abstention by Mexico
governments from interfering in the 5 India Land
workings of the free market. Acquisition
police power – the capacity of the states to regulate behavior and enforce
order within their territory for the betterment of the health, safety, morals, and
general welfare of their inhabitants.

.18 In which of the following models does the private partner own the revenue as well as the
risk associated with the project for a limited period of time ?
(A) Build, Own, Operate (BOO) (B) Build, Own, Operate, Transfer (BOOT)
(C) Design, Build, Finance, Operate (DBFO) (D) Design, BID, Build (DBB)
Answer: (B) Build, Own, Operate, Transfer (BOOT)
Types of PPP:
Service Contract: Under a service contract, the Government (public authority) hires
a private company to carry out one or more specified tasks or services for a
period, typically 1–3 years. The public authority remains the primary provider of the
infrastructure service and contracts out only portions of its operation to the private
partner. The private partner must perform the service at the agreed cost and must
typically meet performance standards set by the public sector. The Government pays
the private partner a predetermined fee for the service, which may be a one time fee,
based on unit cost, or some other basis.
Management Contract : A management contract expands the services to be
contracted out to include some or all of the management and operation of the public
service (i.e., utility, hospital, port authority, etc.). Although ultimate obligation for
service provision remains in the public sector, daily management control and authority
is assigned to the private partner or contractor. In most cases, the private partner
provides working capital but no financing for investment. The private contractor is
paid a predetermined rate for labour and other anticipated operating costs.
Management contract variants include supply and service contract, maintenance
management and operational management.
Lease contract : Under a lease contract, the private partner is responsible for the
service in its entirety and undertakes obligations relating to quality and service
standards. Except for new and replacement investments, which remain the
responsibility of the public authority, the operator provides the service at his expense
and risk. The duration of the leasing contract is typically for 10 years and may be
renewed for up to 20 years. Responsibility for service provision is transferred from the
public sector to the private sector and the financial risk for operation and maintenance
is borne entirely by the private sector operator. In particular, the operator is
responsible for losses and for unpaid consumers’ debts. Leases do not involve any
sale of assets to the private sector.
Concessions : A concession makes the private sector operator (concessionaire)
responsible for the full delivery of services in a specified area, including operation,
maintenance, collection, management, and construction and rehabilitation of the
system.
Build Operate Transfer (BOT) :
BOT and similar arrangements are a kind of specialized concession in which a private
firm or consortium finances and develops a new infrastructure project or a major
component according to performance standards set by the government.
Under BOTs, the private partner provides the capital required to Build the new facility,
Operate & Maintain (O&M) for the contract period and then return the facility to
Government as per agreed terms.
Importantly, the private operator now owns the assets for a period set by contract—
sufficient to allow the developer time to recover investment costs through user
charges.
BOTs generally require complicated financing packages to achieve the large financing
amounts and long repayment periods required. At the end of the contract, the public
sector assumes ownership but can opt to assume operating responsibility, contract
the operation responsibility to the developer, or award a new contract to a new
partner.
Example: Yamuna Expressway (Greater Noida to Agra)
Name of the consortium- JP Infrastructure
Cost of project: 13,000 crore
Period of contract- 36 years
Concessionaire has rights for collecting tolls during concession period of 36 years.
Land development rights of 25 million square meters approximately alongside the Express Way.
Land including 100 meters for the Expressway at five or more locations of which one shall be Greater
Noida/Noida with an area of 5 million sq. meters alongside the Express way for commercial, amusement,
industrial, institutional and residential development would be offered on acquisition cost on lease for the
period of 90 years.
Land for development as well as Expressway transferred to Concessionaire on acquisition cost plus lease
rent of Rs 100/- per hectare per year.

Design Build (DB): The traditional approach for construction projects consists of the
appointment of a designer on one side, and the appointment of a contractor on the
other side. In DB Private sector designs and constructs at a fixed price and transfers
the facility.
Build Transfer Operate (BTO) : Where Private sector designs and builds the facility.
The transfer to the public owner takes place at the conclusion of construction.
Concessionaire is given the right to operate and get the return on investment.
Build-Own-Operate (BOO) : A contractual arrangement whereby a Developer is
authorized to finance, construct, own, operate and maintain an Infrastructure or
Development facility from which the Developer is allowed to recover his total
investment by collecting user levies from facility users. Under this Project, the
Developer owns the assets of the facility and may choose to assign its operation and
maintenance to a facility operator. The Transfer of the facility to the Government,
Government Agency or the Local Authority is not envisaged in this structure;
however, the Government, may terminate its obligations after specified time period.
Design-Build Operate (DBO) : Where the ownership is involved in private hands and
a single contract is let out for design construction and operation of the infrastructure
project.
Design Build Finance Operate (DBFO) : With the design–build–finance–operate
(DBFO) approach, the responsibilities for designing, building, financing, and operating
& maintaining, are bundled together and transferred to private sector partners. DBFO
arrangements vary greatly in terms of the degree of financial responsibility that is
transferred to the private partner.
Build- Operate- Transfer (BOT) : Annuity/Shadow User Charge : In this BOT
Arrangement, private partner does not collect any charges from the users. His return
on total investment is paid to him by public authority through annual payments
(annuity) for which he bids. Other option is that the private developer gets paid based
on the usage of the created facility.
Joint Venture: Joint ventures are alternatives to full privatization in which the
infrastructure is co-owned and operated by the public sector and private operators.
Under a joint venture, the public and private sector partners can either form a new
company (SPV) or assume joint ownership of an existing company through a sale of
shares to one or several private investors. A key requirement of this structure is good
corporate governance, in particular the ability of the company to maintain
independence from the government, because the government is both part owner and
regulator.

.19 In a multi-storied building, the type


of plumbing system suitable for
reusing the sullage for non-
potable use is
(A) single stack system
(B) partially ventilated single stack
system
(C) one pipe system
(D) two pipe system
Answer: (D) two pipe system
Note:
One Pipe system and Two
pipe system are fully
ventilated. In one pipe
system sewage and
sullage is not separated.
Single stack system has
no ventilation.
Partially Ventilated Single stack system has only ventilation to trap of WC.
Kitchen and bathroom trap not ventilated.

.20 The unit for measuring sound absorption in a room is


(A) Sabin(B) Phon (C) Decibel (D) Hertz
Answer: (A) Sabin
Decibel- used to measure the intensity
Phon- phon is a unit of loudness level for pure tones
Hertz- frequency of sound.

.21 In Geographic Information System, DEM represents information on


(A) vegetation cover (B) soil type (C) water table (D) topography
Answer: (D) topography
DEM- Digital Elevation Model (Digital Model Representing 3-D of a Terrain)
Hydrologic modelling – A DEM is used to delineate watersheds, calculate
flow accumulation and find out flow direction.
Terrain stability – Areas prone to avalanches are high slope areas with sparse
vegetation, which is useful when planning a highway or residential subdivision.
Soil mapping – DEMs assist in mapping soils which is a function of elevation
(as well as geology, time and climate).
LIDAR- Light Detection and Ranging
RADAR- Radio Detection (or Direction) and Ranging.
DSM- Digital Surface Model (captures natural built features on Earth surface)
DTM- Digital Terrain Model (In some countries DSM is also known as DTM)
TIN- Triangular Irregular Network.

.22 Minimum point required for GRIHA certification is


(A) 35 (B) 40 (C) 50 (D) 60
Answer: All three answers (A), (B), and (C)
GRIHA: Indian Model of Green Building rating is GRIHA (Green Rating for
Integrated Habitat Assessment). Developed by TERI (The Energy and
Resources Institute) it gives star rating from 1 to 5 Star.
Other two Green Building Rating system in India is IGBC and BEE.
GRIHA keeps on updating its star rating criteria.

Star Griha Griha Version 5, Jan


Rating Version 3 2015
(34 (31 criteria) for built up
criteria) area 2500 sq.m
5- star 91-100 86 or more
4-star 81-90 71-85
3-star 71-80 56-70
2- star 61-70 41-55
1-star 50-60 25-40

.23 ArchiCAD, Auto Desk Revit, Digital Project Designer (CATIA) and Vector Works
Architect are examples of
(A) Stastical Analysis software (B) GIS software
(C) BIM software (D) Image processing software
Answer: (C) BIM software

.24 The CARTOSAT 2C satellite recently launched by ISRO


(A) is a geo-synchronous satellite (B) is a part of IRNSS GPS satellite system
(C) was launched using a GSLV rocket (D) has high spatial resolution
Answer: (D) has high spatial resolution
IRNSS GPS Satellite System- IRNSS is an independent regional navigation satellite
system being developed by India. It is designed to provide accurate position
information service to users in India as well as the region extending up to 1500 km
from its boundary, which is its primary service area.
The space segment consists of the IRNSS constellation of seven satellites,
Navigation with Indian Constellation (NavIC). Three satellites are located in suitable
orbital slots in the geostationary orbit and the remaining four are located in
geosynchronous orbits with the required inclination and equatorial crossings in two
different planes.

Carto Sat 2E launched in Dec, 2017 has a spatial resolution of 60 cm.

.25 Which of the following trees has a columnar form?


(A) Delonix regia
(B) Tamarindus indica
(C) Polyalthia longifolia
(D)Callistemon Lanceolatus
Answer: (C) Polyalthia longifolia
Q.26Match the architectural movements in Group- I with their proponents in Group- II
Group –I Group- II
P. Deconstruction 1. Joseph Paxton
Q. Historicism 2. Kenzo Tange
R. Metabolism 3. Walter Gropius
S. Art Nouveau 4. Victor Horta
5. Frank O. Gehry
(A) P-5, Q-1, R-2, S-4 (B) P-5, Q-4, R-2, S-3
(C) P-5, Q-2, R-3, S-3 (D) P-2, Q-4, R-1, S-5

Answer: (A) P-5, Q-1, R-2, S-4

Q.27 Associate the historic buildings in Group- I with their predominant materials in Group-
II
Group –I Group- II
(P) Lingaraj Temple, Bhubaneshwar, India 1. Red sandstone
(Q) Victoria Memorial, Kolkata, India 2. Timber
(R) Padmanabhapuram Palace, Thuckalay, India 3. Terracotta tiles
(S) Humayun’s Tomb, Delhi, India 4. Sandstone and laterite
5. Marble

(A) P-1, Q-2, R-3, S-5 (B) P-1, Q-4, R-3, S-5
(C) P-2, Q-1, R-3, S-4 (D) P-4, Q-5, R-2, S-1

Answer: (D) P-4, Q-5, R-2, S-1

Padmanabhapuram Palace, Thuckalay,


Kanyakumari, India – A magnificent wooden palace of the 16th century,
Padmanabhapuram Palace lies at the land’s end of mainland India – Kanyakumari.
An enticing to any lover of art and architecture this old palace of the Rajas of the
erstwhile Travancore (1550 to 1750 AD) is a fine specimen of Kerala’s indigenous
style of architecture. The antique interiors are replete with intricate rosewood carvings
and sculptured decor.

Padmanabhaswamy Temple is located in Thiruvananthapuram, Kerala, India. The


temple is built in an intricate fusion of the indigenous Kerala style and the Dravidian
style (kovil) of architecture associated with the temples of Tamil Nadu, featuring high
walls, and a 16th-century Gopuram
Bishnupur group of temples (West Bengal ) have terracotta tiles cladding.

Q.28 Match the terminologies in Group- I with their description in Group- II


Group –I Group- II
(P) Prunning 1. Cutting of trees
(Q) Felling 2. Removing broken branches from trees for better growth
(R) Hoeing 3. Maintaining moisture content in soil by a protective layer
(S) Mulching 4. Indiscrimate cutting of branches to reduce all size of a tree
5. Loosening the ground to remove weeds

(A) P-2, Q-1, R-5, S-3 (B) P-2, Q-1, R-4, S-3
(C) P-2, Q-1, R-3, S-4 (D) P-1, Q-2, R-3, S-1

Answer: (A) P-2, Q-1, R-5, S-3

Q.29 A proposed housing will have HIG, MIG and LIG units on a site measuring 60,750 sq.m.
The buildable area of each category of units with respect to total buildable area will be
30%, 50% and 20% respectively. The maximum allowed FAR is 2.5, ground coverage
45% and height 15m. The maximum available buildable area in sq.m of HIG units,
considering a floor height of 3m for all categories will be______________.
Answer: 41006 sq.m.
Plot Area= 60,750 sq. m
FAR= 2.5
Total Buildable Area = Plot Area × FAR
= 60,750 × 2.5= 1,51,875 sq.m
Ground Coverage = 45 %

Total Ground Coverage = × 60,750 = 27,337.5


Total Buildable Area as per ground coverage and height restriction
27,337.5 × 5= 1,36,687.5
Note: Total Buildable Area as per ground coverage and height restriction will be applicable
rather than that calculated from FAR.

Maximum available buildable area in sq.m of HIG 1,36,687.5 × = 41006.25 sq.m.

Q.30 In 2011, the population of a town was 5,00,000 and the number of housing units were
1,00,000. Calculate the additional number of dwelling units (DU) required by 2031 so
that there is no housing shortage. The assumptions are
i. 5% decadal increase in population
ii New DU to be completed by 2021 is 10,000
iii Number of DU which will become non habitable by 2031 is 5,000
iv Average household size is 4.5

Answer:17,500

Population of the town in 2031= 5,00,000 × (


= 5,51,250

No of dwelling units required by 2031= = = 1,22,500

Already available units in 2011= 1,00,000


Obsolete units by 2031=5,000
Additional unit created by 2021= 10,000
Habitable unit by 2021= 1,00,000- 5000 + 10,000= 1,05,000
Additional dwelling units required by 2031 = 1,22,500- 1,05,000
= 17,500 DU
Q.31 Match the classical urban planning theories in Group- I with their proponents in
Group- II
Group –I Group- II
(P) Concentric Zone Model 1. Berry and Horton
(Q) Sector Model 2. Homer Hoyt
(R) Multiple Nuclei Model 3. Ernest Burgess
(S) Factorial Ecology 4. Shevky and Bell
5. Harris and Ulmann
(A) P-4, Q-1, R-3, S-5 (B) P-3, Q-4, R-3, S-5
(C) P-2, Q-4, R-5, S-1 (D) P-3, Q-2, R-5, S-1

Answer: (D) P-3, Q-2, R-5, S-1

Q.32 Match the distinguished housing projects in Group- I with their architects in Group- II
Group –I Group- II
(P) Nagakin Capsule Tower, Tokyo, Japan 1. Walter Gropius
(Q) Tara Apartments, New Delhi, India 2. Moshe Safdie
(R) Habitat 67, Montreal, Canada 3. Ralph Erskine
(S) Byker Wall, New Castle, England 4. Charles Correa
5. Kisho Kurokawa
(A) P-5, Q-4, R-2, S-3 (B) P-1, Q-3, R-4, S-5
(C) P-5, Q-2, R-1, S-4 (D) P-5, Q-4, R-2, S-1

Answer: (A) P-5, Q-4, R-2, S-3

Q.33 Match the development schemes by Government of India in Group- I with their
objectives in Group- II
Group –I Group- II
(P) PMAY 1. Housing for All
(Q) AMRUT 2. Rural cluster development
(R) NruM 3. Heritage city development
(S) HRIDAY 4. Urban mobility improvement
5. Urban rejuvenation
(A) P-1, Q-5, R-4, S-3 (B) P-1, Q-5, R-2, S-3
(C) P-3, Q-5, R-1, S-2 (D) P-4, Q-2, R-1, S-5

Answer: (B) P-1, Q-5, R-2, S-3


PMAY : (Pradhanmantri Aawas Yojna) Providing housing for all by 2022 in the
Urban Areas. More than 2 crore homes would be build across all the urban
locations by 2022. Central Govt. will provide a grant of Rs 1 lakh to Rs 2.5 lakh
for every home.
AMRUT: (Atal Mission for Rejuvenation and Urban Transformation). This
program will work
replacing JNNURM catering Physical and Social Infrastructure of the 500
urban areas.
(GATE 2016)
NruM: Large parts of rural areas in the country are not stand-alone settlements
but part of a cluster of settlements, which are relatively proximate to each
other. These clusters typically illustrate potential for growth, have economic
drivers and derive locational and competitive advantages. These clusters once
developed can then be classified as ‘Rurban’. Government of India, has
launched the Shyama Prasad Mukherji Rurban Mission (SPMRM), aimed at
developing such rural areas by provisioning of economic, social and physical
infrastructure facilities
HRIDAY: Heritage City Development and Augmentation Yojna. Revitalization of
urban infrastructures in identified 12 heritage cities namely Ajmer, Amravati,
Amritsar, Badami, Dwarka, Gaya, Kanchipuram, Mathura, Puri, Varanasi,
Velankani and Warangal.

Q.34 Match the international events in Group- I with their directives in Group- II
Group –I Group- II
(P) Earth Summit, Rio de Janerio, 1992 1. Kyoto Protocol
(Q) UN Framework Convention on Climate Change, New York, 1992 2. Agenda 21
(R) UN Sustainable Development Summit, New York, 2015 3. Heritage
Conservation
(S) Habitat II, Instanbul, 1996 4. Agenda 2030
5. Housing for All
(A) P-1, Q-5, R-4, S-3 (B) P-1, Q-5, R-2, S-3
(C) P-2, Q-1, R-4, S-5 (D) P-2, Q-1, R-5, S-4

Answer: (C) P-2, Q-1, R-4, S-5


Convention for the Prevention of Pollution of the Sea by Oil, 1962
The Ramsar Convention, 1971- wet lands
Montreal Protocol, 1987- ozone layer
Basel Convention, 1989- Hazardous waste
U.N. Framework Convention on Climate Change, 1992 (Earth Summit)
Kyoto Protocol, 1997
Sustainable Development Goals (SDGs), officially known as Transforming our
world: the 2030 Agenda for Sustainable Development –contains 17 Goals and
169 targets.

Q.35 Match the planning techniques in Group- I with their salient features in Group- II
Group –I Group- II
(P) Land Pooling 1. Assigning specific task on a short time horizon
(Q) Action Plan 2. Assembling privately owned land parcels for development
(R) Land sharing 3. Agreement for reallocation of land between occupiers and owners
(S) Transfer of Development Rights4. Assigning specific task on a long term horizon
5. Incentive based voluntary shifting of FAR of a plot to another plot

(A) P-1, Q-5, R-4, S-3 (B) P-2, Q-1, R-3, S-5 (C) P-2, Q-1, R-3, S-4 (D) P-4, Q-2, R-1,
S-5
Answer: (B) P-2, Q-1, R-3, S-5

.36 For a symmetrical two dimensional truss as shown in the figure, vertical force in kN
acting on the member PQ is ____________

Answer: 0
Q.37 Value of bending moment in kN-m at point C for a beam as shown in the figure is
__________
Answer: 28

Soln: Ʃ Horizontal and Vertical force= 0


RB +RD = 20 + 40
RB +RD = 60 …(i)
Ʃ Moment= 0
Taking Moment about D
(20 × 7.5) + (40×2) = RB × 5

RB = solving for RB we get RB= 46 kN and from eq (i) we


get RD= 14 kN
BMC =14 × 2= 28 kN
(soln. provided by Surbhi Jethani, B.Arch, MITS Gwalior)

Q.38 Fee of contractor for a project has the following provisions


Basic fee = 15 % of actual cost of work incurred
Bonus = 20% of savings from estimated cost of work
Penalty = 20 % of cost overrun
If the estimated cost of the project is Rs 60,000, and the actual cost is Rs 70,000, the
total fee of contractor in Rupees is _____________
Answer: 8500

Basic fee : 15 % of actual cost of work incurred= 70,000 × = 10,500


Bonus : 20% of savings from estimated cost of work =0 (no bonus due to cost
overrun)

Penalty : 20 % of cost overrun= × 10,000= 2,000


Total 10,500- 2,000= 8,500/-

.39 A site has a unidirectional slope of 30° with horizontal along its longer side. The
projected dimensions of the site on the horizontal plane measures 30 m × 40 m. Using
cut and fill method the site has to be levelled parallel to the horizontal plane. The
minimum amount of earth to be excavated in cubic metre is ___________.
Answer: 3464

.40 The optimistic, most likely and pessimistic time for developing a new product are 12
months, 15 months and 17 months respectively. Calculate the expected time in
months.
Answer: 15 months

Expected time =

= = 14.83 months (say 15 months)


.41 A circular plate inclined at an angle θ with horizontal plate generates an ellipse as top
view with major axis and minor axis of 5 cm and 2.5 cm respectively. The value of θ in
degrees is _______________
Answer: 60

.42 Calculate the volume of cement in cubic metre required for making 10 cubic metre of
M20 grade Plain Cement Concrete work, assuming the ratio of dry concrete mix to wet
concrete mix as 1.52.
Answer: 2.76 S No Mix Ratio
In this designation M20 the letter ‘M’ refers to the Cement: sand: chips
mix and 1 M10 1:3:6
the number to the specified 28 day cube strength 2 M15 1:2:4
of mix
3 M20 1:1.5:3
in N/mm2.
4 M25 1:1:2
= 1.52

Dry conc. Mix= 1.52 (wet conc. Mix)


Dry conc. Mix = 1.52 × 10= 15.2
Ratio in M20= 1:1.5:3 (cement: fine agg: coarse agg.)

Amount of cement = × 15.2 = 2.76 cubic metre

.43 One acre of agricultural land has been given on a lease till perpetuity at an annual rent
of Rs 10,000 to be paid at the end of each year. Net Present Value of the land parcel in
Rupees assuming a discount rate of 5% per annum is _____________________
Answer: Rs 2,00,000

NPV= annual rent ×

= 10,000 × = 2,00,000 Rs

.44 In year 2001, a district with 4,000 manufacturing jobs had a 10% share of total
manufacturing jobs within the state. In year 2011, the state recorded 15 % drop in
manufacturing jobs whereas, the share of the district in total manufacturing jobs within
the state increased to 15%. Additional manufacturing jobs created in the district
between year 2001 and 2011 is ___________
Answer: 1100
Total manufacturing job in district in 2001= 4000 (10 % of state share)
Total Manufacturing job of state in 2001= 40,000

In 2011 there is drop of 15% in state job= × 40,000= 6000 job declined
In 2011 total manufacturing job in state = 40,000- 6000 = 34,000

Total Manufacturing job in district in 2011= × 34,000= 5100 (in 2011 district
contribution was 15 %)
Additional manufacturing jobs created in the district between year 2001 and 2011= 5100-
4000= 1100
Q.45 Match the parameters in Group- I with their units in Group- II
Group –I Group- II
(P) Traffic flow 1. Metre
(Q) Traffic Density 2. Cycle/second
(R) Right of Way 3. Seconds
(S) Traffic Signal cycle length 4. Vehicle/km
5. PCU/hr
(A) P-5, Q-4, R-1, S-2 (B) P-5, Q-4, R-1, S-3
(C) P-5, Q-2, R-4, S-3 (D) P-4, Q-5, R-1, S-3

Answer: (B) P-5, Q-4, R-1, S-3

Q.46 Match the planning tasks in Group- I with their tools of analysis in Group- II
Group –I Group- II
(P) Population Projection 1. Input- Output Analysis
(Q) Regional resource allocation 2. Hardy Cross Method
(R) Trip distribution 3. Cohort Analysis
(S) Design of water distribution network 4. Gravity Model
5. Moving Observer method
(A) P-3, Q-1, R-4, S-2 (B) P-3, Q-5, R-4, S-1 (C) P-5, Q-1, R-3, S-4 (D) P-1, Q-3, R-
5, S-2
Answer: (A) P-3, Q-1, R-4, S-2

Q.47 Match the land use classes in Group- I with their use zones in Group- II
Group –I Group- II
(P) Transportation 1. Sports complex
(Q) Commercial 2. Heritage and Conservation areas
(R) Public and Semi –Public 3. Burial ground
(S) Recreational 4. BRT Corridor
5. Service sector
(A) P-4, Q-1, R-3, S-5 (B) P-5, Q-3, R-1, S-2 (C) P-4, Q-5, R-1, S-2 (D) P-4, Q-5, R-3,
S-1
Answer: (D) P-4, Q-5, R-3, S-1

Q.48 Match the structural systems in Group- I with the buildings in Group- II
Group –I Group- II
(P) Folded plates 1. Kurilpa Bridge, Brisbane
(Q) Shell 2. Eden Project, Cornwall
(R) Tensegrity 3. Riverside Museum, Glasgow
(S) Pneumatic 4. MIT Auditorium, Boston
5. 30, St. Mary Axe, London
(A) P-3, Q-4, R-1, S-2 (B) P-5, Q-4, R-3, S-1 (C) P-3, Q-2, R-1, S-5 (D) P-1, Q-3,
R-4, S-2
Answer: (A) P-3, Q-4, R-1, S-2
Kurilpa Bridge, Brisbane Riverside Museum, Glasgow

MIT Auditorium, Boston Eden Project, Cornwall

.49 As per National Building Code of India, 2005, the maximum number of occupants per
unit exit width of a doorway is 60, where unit width is 500 mm. The maximum
permissible occupants in a theatre having four number of 2.2 m wide doors will be
______________
Answer:960
2.2 m width will be considered as 4 unit only, so there will be 4 × 4= 16 unit only.
Therefore total occupant = 16 × 60= 960

Q.50 Match the instruments in Group- I with the corresponding tests in Group- II
Group –I Group- II
(P) Pycnometer 1. Initial and final setting time
(Q) Brinell’s Apparatus 2. Abrasion test
(R) Los Angeles Apparatus 3. Surface hardness test
(S) Vicat’s Apparatus 4. Slump test
5. Apparent Specific gravity
(A) P-5, Q-3, R-2, S-1 (B) P-5, Q-4, R-2, S-1 (C) P-3, Q-2, R-1, S-5 (D) P-2, Q-
3, R-4, S-1
Answer: (A) P-5, Q-3, R-2, S-1

.51 Water flows through a constricted pipe whose diameter at the constricted end is half of
the non-constricted end. Velocity of water at the non-constricted end is 2 m/s. Velocity
of water in m/s at the constricted end using the principle of continuity of flow is
_______________
Answer: 8 m/s
V1A1= V2A2

Let the diameter of non constricted end be ‘d’ then radius =

Diameter of constricted end = , so radius =

Applying formula 2 × π( = V2 π(

2 π ) = V2 π )
V2= 8 m/s

. 52 A drainage basin of 180 hectares comprise 40 % wooded area, 45 % grassed area and
15 % paved area. Run off coefficients for wooded, grassland and paved areas are
0.01, 0.2 and 0.95 respectively. The composite runoff coefficient for the drainage basin
is_______________
Answer: 0.2365
Let us assume that there is 100 litre of rainfall uniformly distributed
40 litre will fall in wooded area, 45 litre in grassed land and 15 litre in paved area.
Now calculating runoff
Wooded area = 40 × 0.01= 0.4 Litre
Grass land= 45 × 0.2= 9.0 litre
Paved area= 15 × 0.95= 14.25 litre
Total= 0.4 + 9.0+ 14.25 = 23.65 (this is the total runoff for 100 litre of rainfall)

Composite runoff = = 0.2365


Note: Area of drainage basin was not needed.

.53 A fluorescent light source consumes 40 W electric power and has a luminous efficacy of
40 lm/W. Illumination in lux at a distance of 3 m from this light source is ____________
Answer:14.147 Lux
Total lumens= 40 ×40 = 1600 lumens

Lux= = = 14.147 Lux (since the light is distributed in all the directions)
(the answer provided by GATE is 175-180 Lux, which is wrong.)

.54 A room measures 3 m (width) × 4 m (length) × 3 m (height). The outdoor temperature is


36 °C. The volumetric specific heat of air is 1300 J/cu.m.degC. The ventilation heat
flow rate in Watts required to attain an internal room temperature of 26°C with 3 air
changes per hour is________________
Answer: 390 W
Volume of room = 3 × 4 × 3= 36 cubic m.

Total air change = 36 ×3 = 108 cubic m in one hr (3600 sec)= = 0.03


Diff in temperature= 10° C
Qv = 1300 J/cu.m.degC × rate of ventilation ( in ×ΔT
Ventilation heat flow rate= 1300 J/cu.m.degC = 390 W

Q.55 Match the equipment in Group- I with their applications in Group- II


Group –I Group- II
(P) PIR 1. Air conditioning
(Q) FCU 2. Lighting
(R) OLED 3. Power generation
(S) BIPV 4. Motion detection
5. Daylight sensing
(A) P-4, Q-5, R-2, S-1 (B) P-1, Q-4, R-5, S-3 (C) P-4, Q-1, R-2, S-3 (D) P-4, Q-
2, R-5, S-1

Answer: (C) P-4, Q-1, R-2, S-3


PIR- A passive infrared sensor (PIR sensor) is an electronic sensor that
measures infrared (IR) light radiating from objects in its field of view. They are
most often used in PIR-based motion detectors. The term passive in refers to
the fact that the devices do not generate or radiate any energy for detection
purposes. PIR sensors don’t detect or measure “heat”; instead they detect the
infrared radiation emitted or reflected from an object .
FCU- Fan coil unit.
OLED- An organic light-emitting diode is a light-emitting diode (LED) in which
the emissive layer is a film of organic compound that emits light in response to
an electric current.
Building-integrated photovoltaics (BIPV) are photovoltaic materials that are
used to replace conventional building materials in parts of the building
envelope such as the roof, skylights, or facades.

GENERAL APTITUDE

Q.56 He was one of my best __________ and I felt his loss ________________
(A) friend, keenly (B) friends, keen (C) friend, keener (D) friends, keenly
Answer: (D) friends, keenly

Q.57 As the two speakers became increasingly agitated, the debate became
________________
(A) lukewarm (B) poetic (C) forgiving (D) heated
Answer: (D) heated

Q.58 A right angled cone (with base radius 5 cm and height 12 cm) as shown in the figure
below, is rolled on the ground keeping the point P fixed until the point Q (at the base of
the cone, as shown) touches the ground again

By what angles (in radians) about P does


the cone travel

(A) (B)

(C) (D)

Answer: (D)
While rotating Q the whole cone wil also rotate in a circle of radius which will be equal
to its and slant height
So rotating Q it will cover 2 distance in horizontal circle
So angle made will be
.59 In a company with 100 employees, 45 earn Rs 20,000 per month, 25 earn Rs 30,000,
20 earn Rs 40000, 8 earn Rs 60,000 and 2 earn Rs 1,50,000. The median of the
salaries is
(A) Rs 20,000 (B) Rs 30,000 (C) Rs 32,300 I Rs 40,000
Answer: (B) Rs 30,000
Medium is the middle term of the data arranged in increasing under if no of terms are
odd, if is even then median will be the average of two middle terms.
So for above question, arranged data

60 P, Q and R talk about S’s car collection. . P states thst S has at least 3 cars. Q believes
that S has less than 3 cars. R indicates that to his knowledge, S has at least one car.
Only one of P, Q and R is right. The number of cars owned by S is
(A) 0 (B) 1 (C) 3 (D) Cannot be determined
Answer: (A) 0

.61. “Here throughout the early 1820s, Stuart continued to fight his losing battle to allow his
sepoys to wear their caste- marks and their own choice of facial hair on parade, being
again reprimanded by the commander –in – chief. His retort that ‘ A stronger instance
than this of European prejudice with relation to this country has never come under my
observations’ had no effect on his superiors.”

According to this paragraph, which of the statements below is most accurate ?


(A) Stuarts commander-in- chief was moved by this demonstration of his prejudice.
(B) The Europeans were accommodating of the sepoys’ desire to wear their caste-
marks.
(C) Stuart’s ‘losing battle’ refers to his inability to succeed in enabling sepoys to wear
caste- marks.
(D) The commander-in- chief was exempt from the European prejudice that dictated
how the sepoys were to dress.
Answer: (C) Stuart’s ‘losing battle’ refers to his inability to succeed in enabling sepoys
to wear caste- marks.

Q.62 What is the sum of the missing digits in the substration problem below ?

(A) 8 (B) 10 (C) 11 (D) Cannot be determined

Answer: (D) Cannot be determined.

.63 Let S1 be the plane figure consisting of the points (x,y) given by the inequalities ǀx-1ǀ ≤ 2
and ǀy+2ǀ ≤ 3. Let S2 be the plane figure given by inequalities x-y ≥ -2, y ≥ 1 and x ≤ 3.
Let S be the union of S1 and S2. He area of S is
(A) 26 (B) 28 (C) 32 (D) 34
Answer: (C) 32
.64 Two very famous sportsmen Mark and Steve happened to be brothers, and played for
country K. Mark teased James, an opponebt from country E, “here is no way you are
good enough to play for our country. “James replied, “ Maybe not, but at least I am the
best player in my own family.”
Which one of the following can be inferred from this conversation ?
(A) Mark was known to pla beter than James
(B) Steve was known to play better than Mark
(C) James and Steve were good friends
(D) James played better than Steve

Answer: (B) Steve was known to play better than Mark

Q.65 The growth of bacteria (lactobacillus)


in milk leads to curd formation. A minimum bacterial population density of 0.8 (in
suitable units) is needed to form curd. In the graph below, the population density of
lactobacillus in 1 litre of milk is plotted as a function of time, at two different
temperatures, 25 °C and 37 °C .

Consider the following statements based on the data shown above:

i The growth in bacterial population stops earlier at 37 °C as compared to 25 °C


iiThe time taken for curd formation at 25 °C is twice the time taken at 37 °C

Which one of the following options is correct ?


(A) Only i (B) Only ii (C) Both i and ii (D) Neither i nor ii

Answer: (A) Only i


(i) the growth in bacterial population stops almost 140s in 370C as compared to 180s
in 250C
(ii) time taken for curd formation at 250C is approximately 90s while it is 130s in 370C
which is not double
GATE 2018
Q. 1 – Q.5 carry one mark each.

Q.1 “When she fell down the _______ she received many _______ but little help.”
The words that best fill the blank in the above sentence are
(A) stairs, stares (B) stairs, stairs (C) stares, stairs (D) stares, stares
Ans: (A) stairs, stares

Q.2 “In spite of being warned repeatedly he failed to correct his _________ behaviour.”
The word that best fills the blank in the above sentence is
(A) rational (B) reasonable (C) errant (D) good
Ans: (C) errant

Q.3 For 0 ≤ ?? ≤ 2 ?? , sin ?? and cos ?? are both decreasing functions in the interval
________.
(A) (0, ?? 2 ) (B) ( ?? 2 , ?? ) (C) ( ?? , 3 ?? 2 ) (D) ( 3 ?? 2 , 2 ?? )
Ans: (B) ( ?? 2 , ?? )

Q.4 The area of an equilateral triangle is √3. What is the perimeter of the triangle?
(A) 2 (B) 4 (C) 6 (D) 8
Ans: (C) 6

Q.5 Arrange the following three-dimensional objects in the descending order of their volumes:
(i) A cuboid with dimensions 10 cm, 8 cm and 6 cm
(ii) A cube of side 8 cm
(iii) A cylinder with base radius 7 cm and height 7 cm
(iv) A sphere of radius 7 cm
(A) (i), (ii), (iii), (iv) (B) (ii), (i), (iv), (iii) (C) (iii), (ii), (i), (iv) (D) (iv), (iii), (ii), (i)
Ans: (D) (iv), (iii), (ii), (i)

Q. 6 – Q. 10 carry two marks each.

Q.6 An automobile travels from city A to city B and returns to city A by the same route. The
speed of the vehicle during the onward and return journeys were constant at 60
km/h and 90 km/h, respectively. What is the average speed in km/h for the
entire journey?
(A) 72 (B) 73 (C) 74 (D) 75
Ans: (A) 72

Q.7 A set of 4 parallel lines intersect with another set of 5 parallel lines. How many
parallelograms are formed?
(A) 20 (B) 48 (C) 60 (D) 72
Ans: (C) 60

Q.8 To pass a test, a candidate needs to answer at least 2 out of 3 questions correctly. A total
of 6,30,000 candidates appeared for the test. Question A was correctly
answered by 3,30,000 candidates. Question B was answered correctly by 2,50,000
candidates. Question C was answered correctly by 2,60,000 candidates. Both questions
A and B were answered correctly by 1,00,000 candidates. Both questions B
and C were answered correctly by 90,000 candidates. Both questions A and C were
answered correctly by 80,000 candidates. If the number of students
answering all questions correctly is the same as the number answering none,
how many candidates failed to clear the test?
(A) 30,000 (B) 2,70,000 (C) 3,90,000 (D) 4,20,000
Ans: (D) 4,20,000

Q.9 If + ?? – 1 = 0 what is the value of + ?


(A) 1 (B) 5 (C) 7 (D) 9
Ans: (C) 7

Q.10 In a detailed study of annual crow births in India, it was found that there was relatively no
growth during the period 2002 to 2004 and a sudden spike from 2004 to 2005.
In another unrelated study, it was found that the revenue from cracker sales in India
which remained fairly flat from 2002 to 2004, saw a sudden spike in 2005 before
declining again in 2006. The solid line in the graph below refers to annual sale
of crackers and the dashed line refers to the annual crow births in India. Choose the
most appropriate inference from the above data.

(A) There is a strong correlation between crow birth and cracker sales.
(B) Cracker usage increases crow birth rate.
(C) If cracker sale declines, crow birth will decline.
(D) Increased birth rate of crows will cause an increase in the sale of crackers.
Ans: (A) There is a strong correlation between crow birth and cracker sales.

Q. 1 – Q. 25 carry one mark each.

Q.1 In a Colour Wheel, Red and Blue colours are


(A) Tertiary (B) Complementary (C) Secondary (D) Primary

Ans: (D) Primary


Primary Colours: Red, Blue, Yellow
Secondary Colour: Orange, Green and Violet
Tertiary Colour: Red-Orange, Yellow-Orange, Yellow Green, Blue- Green, Blue
Violet, Red Violet
The first circular colour diagram was designed by Newton in 1666. It is two
dimensional colour scheme depicting variables of hue. Colour wheel is helpful in
suggesting colour scheme. Six prominent colour schemes are

Complementary- exactly opposite


Analogous – neighbour
Split complimentary- neighbour of complementary colour.
Triadic- three colours evenly spaced on colour wheel
Tetradic- four colors arranged into two complementary pairs.
Square – all four colours spaced evenly.

Q.2 In a bird’s eye perspective view of a cuboid, the maximum number of vanishing points
is
(A) 1 (B) 2 (C) 3 (D) 6
Ans: (C) 3

There are some points to consider when working in three


point perspective.
All vertical construction lines lead to the third vanishing point
The closer you place your vanishing points to the center of your canvas, the
greater the distortion
All horizontal construction lines lead to a single vanishing point, based on the
plane on which they are located on
The further away you place your vanishing points, the closer your image will
appear as captured through a tele-photo lens (very little distortion)
If you place your third vanishing point above the horizon line, you create an image
from the “Ant’s Perspective” (looking up)
If you place your third vanishing point below the horizon line, you create an image
from the “Bird’s Eye View” (looking down)
Q.3 The compressive strength of M-25 concrete is
(A) 25 kg/sqm (B) 25 N/sqmm (C) 250 N/sqmm (D) 2.5 N/sqmm
Ans: (B) 25 N/sqmm
Mpa= Mega Pascal (Pascal is N/ , so 1 Mpa= )
Interesting to note : 6 Mpa= 6 N/ (see the conversion below)
6 Mpa= 6 × N/ (Mega is )

= (Converting to )
= 6 N/
S. Item Min
No Compressive
Strength
(N/mm²)
1 Mortar for brick 1 N/mm²
Masonry
2 Clay Brick 3.5 N/mm²
3 Plain Cement 15 N/mm²
Concrete (PCC)
4 RCC 20 N/mm²
5 Post Tensioned 30 N/mm²
Concrete
6 Pre Tensioned 40 N/mm²
Concrete

Q.4 In Critical Path Method (CPM) for time scheduling, ‘forward pass calculation’ is carried
out for determining
(A) Late start and early finish time (B) Early start and early finish time
(C) Late start and late finish time (D) Early start and late finish time
Ans: (B) Early start and early finish time
Forward Pass and BackwardPass: These terms are related to ways of determining the
early or late start [forward pass] or early or late finish [backward pass] for an
activity. Forward pass is a technique to move forward through a diagram to
calculate activity duration. Backward pass is its opposite

Q.5 Collapse of the World Trade Center (WTC), New York, in 2001, was due to
(A) Wind load failure
(B) Foundation failure
(C) Thermal performance failure of reinforcement steel in RCC
(D) Thermal performance failure of structural steel
Ans: (D) Thermal performance failure of structural steel
Q.6 During the construction of tall buildings, the equipment used for hoisting building
materials to the upper floors is a
(A) Goods lift (B) Capsule lift (C) Gantry crane (D) Tower crane

Ans: (D) Tower crane


(adjacent fig is tower crane and Gantry crane)

Q.7 A Rock-cut style of architecture is represented by


(A) Shyama Rama Temple, Bishnupur (B) Kailasa Temple, Ellora
(C) Kandariya Mahadeva Temple, Khajuraho (D) Sanchi Stupa, Sanchi
Ans: (B) Kailasa Temple, Ellora

Q.8 ‘Area based development’ and ‘Pan city development’ are part of
(A) Smart City Mission (B) Digital India Mission (C) Swachh Bharat Mission (D) Atal
Innovation Mission
Ans: (A) Smart City Mission
Specified Minimum area for Smart City Mission.
Redevelopment- more than 50 acre
Green Field Development- more than 250 acre
Retrofitting- more than 500 acre.
Pan city development-Pan-city development envisages application of selected
Smart Solutions to the existing city-wide infrastructure.

Q.9 In mass transportation, LRTS stands for


(A) Light Rail Transit System (B) Linear Rail Transit System
(C) Light Rail Transportation System (D) Linear Rail Transportation System
Ans: (A) Light Rail Transit System
MRTS- Mass Rapid Transit System
BRTS- Bus Rapid Transit System

Q.10 The structural grid type shown in the figure below is a


(A) Tartan Grid
(B) Square Grid
(C) Rectangular Grid
(D) Irregular Grid
Ans: (A) Tartan Grid

Q.11 Assuming other variables remaining constant, the Tropical Summer Index
(A) Increases with increase in air velocity (B) Decreases with increase in wet-bulb
temperature
(C) Decreases with increase in globe temperature (D) Increases with increase in
vapour pressure
Ans: (D) Increases with increase in vapour pressure
"Tropical Summer Index" (TSI) i“ defined as the air/g”obe temperature of the still air at
50% RH which produces the same overall thermal sensation as the
environment under investigation. This index takes into account all four
environmental variables.
i. air temperature,
ii. globe temperature,
iii. humidity,
iv. air velocity
Use of the TSI is justified in the prevailing hot-dry and warm-humid conditions in India
when radiant flux is not excessively high and the subjects have sufficient air motion for
any visible perspiration to evaporate off. Determining the thermal comfort conditions in
this index is according to observations of four effective environmental variables and
Bradford’s five scales thermal sensation. A simple and approximate equation for the
rapid determination of TSI values for any combination of environmental
variables is as follows:

TSI = tw + tg – 2
Where: TSI = Tropical Summer Index, tw= Wet-bulb Temperature (°C), tg = Globe
temperature (°C), = Square root of air velocity (m/s).

Q.12 Government of India’s urban development program ‘HRIDAY’ stands for


(A) Heritage Rejuvenation Implementation Development Aayog Yojana
(B) Heritage Review Implementation Development Augmentation Yojana
(C) Heritage City Development and Augmentation Yojana
(D) Heritage City Improvement and Development Aawas Yojana
Ans: (C) Heritage City Development and Augmentation Yojana

Q.13 As per the Urban and Regional Development Plan Formulation and Implementation
(URDPFI) guidelines, the plan period considered in a ‘Perspective
plan’ is
(A) 1-10 years (B) 11-15 years (C) 20-30 years (D) 35-45 years
Ans: (C) 20-30 years

Q.14 The Hall of Nations, New Delhi, was designed by


(A) Charles Correa (B) Raj Rewal
(C) Joseph Allen Stein (D) A. P. Kanvinde
Ans: (B) Raj Rewal
Hall of Nations and Halls of Industries, Pragati Maidan, New Delhi 1971 – 72.
Designed as space frame in reinforced concrete; the first of its
kind in India.
Demolished in 2017.

Q.15 As per the National Building Code of India 2016, the minimum turning radius (in
metres) required for fire tender movement is
(A) 8.0 (B) 8.5 (C) 9.0 (D) 9.5
Ans: (C) 9.0
Minimum width of road for fire tender movement= 6 m
The road surface must be mettled and free from obstruction.

Q.16 Sidi Bashir Mosque with ‘Shaking Minarets’ is located in


(A) Ajmer (B) Allahabad (C) Ahmedabad (D) Amritsar

Ans: (C) Ahmedabad


Q.17 ‘Sight Distance’ is considered in the design of
(A) Road intersection (B) Fenestration
(C) Open kitchen (D) Auditorium
Ans: (A) Road intersection

Q.18 In India, the term ‘Town Planning Scheme’ refers to


(A) Land renewal (B) Land rejuvenation
(C) Land reclamation (D) Land readjustment
Ans: (D) Land readjustment
[Refer GATE 2005, Q70]
Land Renewal: no such term (renewal is used for driving license, passport
after expiry of term)
Land rejuvenation: give newer life to land
Land reclamation: get newer land from sea by filling the low lying area
Land readjustment: A legal procedure that allows pooling of lands by owners,
preparation of layout and redistribution of final plots also known as ‘Town
Planning Scheme’.

Q.19 Bamboo is a type of


(A) Shrub (B) Timber (C) Evergreen tree (D) Perennial grass
Ans: (D) Perennial grass
Scientific name: Bambusoideae

Q.20 According to the UN, one of the components for measuring ‘inclusive growth’ is
(A) Economic well-being (B) Physical infrastructure
(C) Education (D) Life expectancy
Ans: (A) Economic well-being

Q.21 The unit of measurement of Damp Proof Course (DPC) in building construction is
(A) kg (B) cum (C) sqm (D) rm
Ans: (C) sqm

Q.22 Which of the following is NOT a Building Information Modeling software tool
(A) Adobe Illustrator (B) Bentley Microstation (C) Autodesk Revit (D) Graphisoft
ARCHICAD
Ans: (A) Adobe Illustrator
BIM (Building Information Modeling) is an intelligent
3D model- based process that gives architecture, engineering, and construction
(AEC) professionals the insight and tools to more efficiently plan,
design, construct, and manage buildings and infrastructure.

Q.23 The concentric circles in a solar chart represent


(A) Azimuth angle
(B) Altitude angle
(C) Horizontal shadow angle
(D) Vertical shadow angle
Ans: (B) Altitude angle

Q.24 A room of 3m × 3m × 3m has a reverberation time of 0.8 sec. Using Sabine’s method,
the total absorption in the room is _____________ sabin (up to one decimal place).
Ans:54 sabine

RT=

Volume of room = 3m ×3m×3m =27

0.8 =

Sabine = = 54 sabine

Q.25 A 25 storeyed building has 5 lifts. The resulting waiting time is 35 sec and ‘Return Travel
Time’ is 175 sec. The number of lifts required for reducing waiting time
to 25 sec, without increasing the lift speed, is _______________
Ans: 7 lifts

Soln: Waiting time (T) =


Waiting time has to be reduced to 25 sec, while return time remains the same i.e. 175 sec

So, 25sec = which gives N= 7 lifts.


Q. 26 – Q. 55 carry two marks each.

Q.26 Match the planning documents in Group-I with their respective government schemes
in Group-II
Group-I Group-II
P. Integrated Cluster Action Plan 1. NULM
Q. Service Level Improvement Plan 2. Make in India
R. Housing for All Plan of Action 3. RuRBAN mission
S. City Livelihood Centre Development Plan 4. PMAY
5. AMRUT
(A) P-4, Q-1, R-5, S-2 (B) P-3, Q-5, R-4, S-2 (C)P-5, Q-1, R-4, S-3 (D) P-3, Q-5, R-
4, S-1
Ans: (D) P-3, Q-5, R-4, S-1

Q.27 Associate the fire safety requirements for high rise buildings in Group-I with
corresponding standards of the National Building Code of India 2016 in Group-
II
Group-I Group-II
P. Minimum Refuge area 1. 12.5 sqm/person
Q. Maximum Travel distance 2. 2.0 m
R. Maximum Occupant load 3. 0.3 sqm/person
S. Minimum Stair case width 4. 12.0 ton
5. 30.0 m
(A) P-4, Q-1, R-5, S-2 (B) P-3, Q-5, R-4, S-1 (C) P-3, Q-5, R-1, S-2 (D) P-4, Q-5, R-
1, S-3
Ans: (C) P-3, Q-5, R-1, S-2

Q.28 Match the photometric quantities in Group-I with their respective units in Group-II
Group-I Group-II
P Illuminance 1 Candela
Q Luminous Intensity 2 Candela/sqm
R Luminance 3 Lumens/sqm
S Luminous Efficacy 4 Lumens/watt
5 Lumens
(A) P-3, Q-2, R-5, S-4 (B) P-5, Q-4, R-2, S-1 (C) P-5, Q-1, R-2, S-3 (D) P-3, Q-1, R-
2, S-4
Ans: (D) P-3, Q-1, R-2, S-4

Q.29 Associate the symbols in Group I with their meanings in Group II

Group II
1. Hearing Impaired
2. Emergency Lamp
3. Electical and Electronic waste disposal
4. Biohazard
5. Speech impaired

(A) P-5, Q-3, R-1, S-2 (B) P-1, Q-5, R-3, S-4 (C) P-1, Q-3, R-4, S-5 (D) P-5, Q-3,
R-4, S-2
Ans: (D) P-5, Q-3, R-4, S-2

Q.30 Match the elements in Group-I with the building components in Group-II
Group-I Group-II
P King post 1 Curtain glazing
Q Grade beam 2 Door
R Metal decking 3 Plinth
S Jamb 4 Intermediate floor
5 Truss
(A) P-5, Q-3, R-4, S-1 (B) P-2, Q-4, R-3, S-1 (C) P-2, Q-4, R-5, S-3 (D) P-5, Q-3, R-4, S-
2
Ans: (D) P-5, Q-3, R-4, S-2

Q.31 Match the iconic architectural examples in Group-I with their predominant structural
systems in Group-II Group-I
Group-II
P. S. Maria del Fiore Cathedral, Florence 1. Shell
Q. Notre Dame Cathedral, Paris 2. Suspended roof
R. Olympic Arena, Tokyo 3. Space frame
S. Bahá’i Temple, Delhi 4. Double-walled dome
5. Flying buttress
(A) P-1, Q-3, R-5, S-4 (B) P-4, Q-1, R-2, S-3 (C) P-4, Q-5, R-2, S-1 (D) P-5, Q-4, R-
3, S-2
Ans: (C) P-4, Q-5, R-2, S-1

Q.32 Match the building materials in Group-I with their distinctive properties in Group-II
Group-I Group-II
P. Cement 1. Charring
Q. Steel 2. Brittle
R. Wood 3. Evaporation
S. Glass 4. Tensile strength
5. Setting Time
(A) P-5, Q-3, R-2, S-1 (B) P-5, Q-4, R-1, S-2 (C) P-1, Q-4, R-5, S-2 (D) P-4, Q-3, R-1,
S-5

Ans: (B) P-5, Q-4, R-1, S-2


Charred Wood is the process of lightly applying open flame to a wood plank to
char the surface of the board. The charred exterior helps to weatherproof the
siding and act as a deterrent to insects.
Brittle- hard but liable to break easily.
Tensile Strength- (Ultimate tensile strength, often shortened to tensile strength) is
the capacity of a material or structure to withstand loads tending to elongate.
Q.33 Match the built forms in Group-I with their descriptions in Group-II
Group-I Group-II
P. Agora 1. Custodial precincts
Q. Ziggurat 2. Place of Jewish worship
R. Mastaba 3. Built in diminishing stages of masonry with buttressed wall
S. Synagogue 4. Market place or public square
5. Tomb made of mud bricks
(A) P-1, Q-4, R-3, S-2 (B) P-4, Q-3, R-1, S-5 (C) P-4, Q-3, R-5, S-2 (D) P-3, Q-1, R-
5, S-2
Ans: (C) P-4, Q-3, R-5, S-2

Q.34 Match the building configuration characteristics in Group-I with their seismic
consequences in Group-II Group-I
Group-II
P. Re-entrant corner 1. Soft storey
Q. Floating column 2. Stress concentration at corner
R. Irregular storey stiffness 3. Load path discontinuity
S. Gap between adjacent buildings 4. Vertical asymmetry
5. Pounding
(A) P-3, Q-1, R-2, S-4 (B) P-2, Q-3, R-1, S-5 (C) P-4, Q-3, R-1, S-5 (D) P-3, Q-5, R-
2, S-1

Ans: (B) P-2, Q-3, R-1, S-5


Floating column is a vertical member which rest on a beam but doesn’t transfer the
load directly to the foundation. The floating column acts as a point load on the beam
and this beam transfers the load to the columns below it.
Types of irregularity in Buildings
Q.35 Match the landscaping terms in Group-I with their descriptions in Group-II

Group-I Group-II
P. Xeriscaping 1. Wide vegetated drain
Q. Drip line 2. Tree rings
R. Swale 3. Outermost circumference of a tree canopy
S. Turf block paver 4. Solution to topsoil erosion and water permeability
5. A little or no irrigation
(A) P-5, Q-3, R-1, S-4 (B) P-3, Q-5, R-1, S-4
(C) P-2, Q-3, R-1, S-5 (D) P-5, Q-2, R-4, S-1

Ans: (A) P-5, Q-3, R-1, S-4


Xeriscaping is landscaping designed specifically for areas that are susceptible to
drought, or for properties where water conservation is practiced. Derived from the
Greek xeros meaning “dry,” the term means literally “dry landscape.”
Drip line is used in drip irrigation where plant is watered at roots. It is also Outermost
circumference of a tree canopy.
A swale is a low tract of land, especially one that is moist or marshy. The term can
refer to a natural landscape feature or a human-created one. Artificial swales are often
infiltration basins designed to manage water runoff, filter pollutants, and increase
rainwater infiltration.
Turf Block grass paving unit turns driveway, parking area, and emergency access
lane into a beautiful greenbelt that is easily mowed and cared.

Q.36 Match the planning principles in Group-I with their descriptions in Group-II
Group-I Group-II
P. Transit oriented development 1. Four stage model of regional development
Q. Core periphery theory 2. Compact and walkable mixed use development
R. Bid rent theory 3. Geographic concentration of inter-connected institutions
S. Cluster theory 4. Change of land price with relative distance from the CBD
5. Interactive and participatory planning process
(A) P-2, Q-1, R-4, S-3 (B) P-2, Q-1, R-5, S-3 (C) P-4, Q-2, R-5, S-3 (D) P-2, Q-3, R-5, S-
4

Ans: (A) P-2, Q-1, R-4, S-3


TOD Model: a transit-oriented development (TOD) is a type of urban development that
maximizes the amount of residential, business and leisure space within walking
distance (10 min) of mass rapid public transport.

Core Periphery theory


Friedman’s four stage of model of regional development.
STAGE 1: When towns/regions are developing independently with very minimal contact
between them – possibly some basic trading of goods.
STAGE 2: One town or region becomes dominant, probably because it has more
physical or human resources. The core starts attracting people and investment from
other regions.
STAGE 3: New smaller cores (semi-periphery) start to develop and there is an increase in
flows between the core and the semi- periphery (two ways flows e.g.
FDI)
STAGE 4: All areas are now developed and fully dependent upon each other with flows of
capital and people in both directions.

Bid rent theory is a geographical economic


theory that refers to how the price and demand for real estate change as the distance
from the central business district (CBD) increases.
The sharpest decline in Rent per square foot is observed in Retail as one
moves from Centre to Periphery.

Cluster theory: Cluster theory is a theory of strategy. Alfred Marshall, in his book
Principles of Economics, published in 1890, first characterised clusters as a
"concentration of specialised industries in particular localities" that he termed industrial
districts.
Industry clusters are groups of similar and related industry in a defined
geographic area that share common markets, technologies, worker skill needs,
and which are often linked by buyer-seller relationships. The product of one
industry is directly used by other industry.
Healthcare Cluster is independent organization of hospitals, clinics, medical
professionals and the government in a specific city, state, or region.
Q.37 Match the cities in Group-I with their planners in Group-II
Group-I Group-II
P. Islamabad 1. Patrick Geddess
Q. Tel Aviv 2. C.A. Doxiadis
R. Bhubaneswar 3. Lucio Costa
S. Brasilia 4. B. V. Doshi
5. O. Koenigsberger
(A) P-2, Q-4, R-1, S-3 (B) P-4, Q-1, R-5, S-2 (C) P-2, Q-1, R-5, S-3 (D) P-2, Q-3, R-
4, S-5
Ans: (C) P-2, Q-1, R-5, S-3
Other planned cities of India.
i. Auroville- Ar Roger Anger along with Mirra Alfassa, the Spiritual collaborator of Sri
Aurobindo.
ii. Bhubaneshwar- Otto Konigsberger (German Architect and planner)
iii. Chandigarh- Le Corbusier
iv. Durgapur- Joseph Allen Stein and Benjamin Polk
v. Gandhi Nagar- H K Mewada and Prakash M Apte
vi. Jaipur- Maharaja Sawai Jai Singh
vii. Jamshedpur- FC Temple, Otto Konigsberger, Maple (Master plan prepared by
Konigsberger)
viii. Kolkata- Job Charnok
ix. New Delhi- Leutyens
x. Vidyadhar Nagar (city close to Jaipur) – B V Doshi.
xi. Navi Mumbai- Charles Correa

Major planned cities of World


i. Brasilia- Lucio Costa (urban planner), Oscar Niemeyer (architect) Brasila is the
capital of Brazil planned in 1956. This city is a World Heritage Site listed by
UNESCO.
ii. Canberra- Walter Burley Griffin and Marion Mahony Griffin
iii. Horizon City, Texas- Lucio Costa (GATE 2004)
iv. Islamabad- Constantine Doxiadius
v. London- Wren, Abercrombie
vi. Paris- Voisin
vii. Philadelphia- Louis Kahn (City of Philadelphia is based on ‘Stop and Go Roads’
and emphasis on parking towers)
viii. Radburn, New Jersey- Clarence Stein and Henry Wright (known as ‘Town for the
Motor Age) (GATE 2011)
ix. Tokyo- Kenzo Tange’s (metabolism)

Conceptual Cities
i. Broadacre cities- F L Wright (GATE 1995)
ii. Garden City- Ebenezar Howard (GATE 2012)
iii. Eco Skyscrapers- Ken Yeang (GATE 2012)
iv. Radiant City- Le Corbusier (GATE 2012)
v. Space City- Yona Friedman

List of urban planners chronological by initial year of plan.


c. 332 BC Dinocrates – Alexandria, Egypt (City of Alexander the Great)
c. 408 BC Hippodamus of Miletus – Piraeus, Thurii, Rhodes (Hippodamus is
known as Father of City Planning)
c. 1450 AD Nezahualcoyotl – Texcoco (altepetl), Aztec Mexico
c. 1590 Tokugawa Ieyasu, Tokugawa Hidetada, Tokugawa Iemitsu, Takatora Todo-
Edo, later Tokyo.

1666 Christopher Wren – London


1682 William Penn and Thomas Holme – Philadelphia (later on Louis Kahn)
1727 Maharaja Jai Singh II, astronomer, city planner – Jaipur, Rajasthan, India
1791 Peter Charles L’Enfant and Andrew Ellicott – Washington, D.C.
1805 Augustus B. Woodward – Detroit
1811 Gouverneur Morris, John Rutherfurd, and Simeon De Witt – Commissioners’ Plan of
New York City
c. 1838 Joseph Smith and later Brigham Young – Nauvoo, Illinois and Salt Lake City.
1853 Georges-Eugène Haussmann – responsible for the broad avenues of Paris
1859 Ildefons Cerdà – planner of the Eixample district of Barcelona
c. 1880 Solon Spencer Beman and George Pullman – Pullman, Chicago
1882 Arturo Soria y Mata – the Ciudad Lineal, Madrid
1898 Ebenezer Howard – Garden city movement
1901 Charles Follen McKim – Washington, D.C. revised plan
1909 Daniel Burnham – Chicago
1912 Walter Burley Griffin – Canberra (His tomb is in Lucknow graveyard)
1924 Clarence Stein – Sunnyside Gardens, Queens, New York; Chatham Village,
Pittsburgh; Baldwin Hills Village, Los Angeles
1925 Ernst May – city plan and housing units in Frankfurt, Germany, including
Siedlung
Römerstadt
1927 Bruno Taut – Hufeisensiedlung (Horseshoe Projects), Berlin
1928 Henry Wright – Radburn, New Jersey
c. 1930 Robert Moses, responsible for the urban renewal of New York City
1935 Frank Lloyd Wright – Broadacre City (concept)
1950 Le Corbusier – Chandigarh, India
1957 Lucio Costa – Brasília, Brazil
1958 Ludwig Mies van der Rohe, Ludwig Hilberseimer, Alfred Caldwell – Lafayette Park,
Detroit
1960 Edmund Bacon – engaged in the redevelopment of parts of Philadelphia
1960 Konstantinos Doxiadis – Islamabad, Pakistan
1970 Paolo Soleri – Arcosanti, Arizona, as well as his concept of arcologies
1970 William Pereira, Ian McHarg – The Woodlands, Texas
1971 H. K. Mewada, Prakash M. Apte – Gandhinagar, Gujarat, India
1973 Moshe Safdie – Coldspring New Town, Baltimore
2003 Christopher Charles Benninger, Thimphu, Bhutan

Q.38 Match the Temples in Group-I with their Dynastic period in Group-II
Group-I Group-II
P. Brihadeshvara Temple 1. Gupta
Q. Kailasanatha Temple 2. Chalukya
R. Bhitragaon Temple 3. Lodhi
S. Lad Khan Temple 4. Chola
5. Pallava
(A) P-4, Q-5, R-1, S-2 (B) P-5, Q-1, R-2, S-3 (C) P-2, Q-5, R-1, S-3 (D) P-4, Q-1, R-2, S-
5
Ans: (A) P-4, Q-5, R-1, S-2

Q.39 Match the Buildings in Group-I with their Architects in Group-II


Group-I Group-II
P. Guggenheim Museum, Bilbao 1. Richard Rogers
Q. The Shard, London 2. Norman Foster
R. Commerz Bank, Frankfurt 3. Frank Gehry
S. Heydar Aliyev Centre, Baku 4. Renzo Piano
5 Zaha Hadid
(A) P-3, Q-4, R-2, S-5 (B) P-3, Q-4, R-1, S-2 (C) P-2, Q-4, R-1, S-5 (D) P-2, Q-5, R-4, S-
3
Ans: (A) P-3, Q-4, R-2, S-5
Q.40 Match the following urban conservation themes in Group-I with their respective
descriptions in Group-II
Group-I Group-II
P. Restoration 1. Piece by piece re-assembly
Q. Reconstitution 2. Returning to previous stage
R. Reconstruction 3. Physical addition
S. Replication 4. Re-creation of vanished elements
5. Reproduction of an exact copy
(A) P-2, Q-5, R-4, S-3 (B) P-2, Q-1, R-4, S-5 (C) P-3, Q-2, R-1, S-4 (D) P-3, Q-1, R-3,
S-5
Ans: (B) P-2, Q-1, R-4, S-5

Q.41 A Single Phase Neutral (SPN) electrical circuit has a power consumption of 330W.
Considering a voltage of 110V and power factor of 0.8, the electrical current
drawn is ___________ Amp (up to one decimal place).
Ans: 3.75 Amp
Soln: Power factor of an AC electrical power system is defined as the ratio of the ‘real
power flowing to the load’ to the apparent power in the circuit.
Wattage = Volt × current × power factor
330 watt = 110 volt × current ×0.8

Current= = 3.75 Amp

Q.42 A building with 100 sqm roof area is connected to a 72 cum rainwater collection tank. If
the rainfall is 60 mm per hour and the loss during water storage is
20%, then the time taken to fill the tank completely
is_______ hours.
Ans:15 hr

Soln: Intensity of rainfall = 60 mm/hr= (converting to SI unit)


Let the tank fill in ‘t’ hr
Total rainfall collected = surface area of roof× Intensity of rainfall × effective storage
× ‘t’
Note- Here effective storage is 100-20=80 %

So 72 cum= 100 sqm × × ‘t’ hr


Solving for ‘t’we get t= 15 hr
Q.43 The planning norms for provision of schools in a given town is shown in the table
below

Schools Population Land requirement


norm per school
Elementary One per 2500 0.4 hectare
School persons
Primary School One per 5000 1.0 hectare
persons
Secondary One per 2.0 hectare
School 12500
persons
Total land area required for providing all types of schools for a population of 200,000 is
____________hectares
Ans: 104 ha
Soln:
Total no. of Elementary School = = 80 so land required = 80 × 0.4= 32
ha

Total no. of Primary School = = 40 so land required = 40 × 1.0 = 40


ha

Total no. of Secondary School = = 16 so land required = 16 × 2.0 = 32


ha
Total Land area required = 32+ 40 + 32= 104 ha

School Education System in India


Education Level Classification
Pre- Primary Level Nursery and Kinder
Garten
Primary Education Class I to Class V
Upper Primary Class VI to Class VIII
Education
Secondary Class IX to Class X
Education
Senior Secondary Class XI to Class XII
Source: Indian Standard Classification of Education MHRD, GoI, 2014

Q.44 In a mixed use development on a 2.0 hectare site with 2.0 FAR, the ratio of residential
to commercial floor area is 3:2. The minimum parking (in ECS)
needed per 100 sqm of residential and commercial floor area is
1.0 and 1.25 respectively. Considering full FAR utilization, the total parking
requirement is ____________ ECS.
Ans:440
Soln: 2 ha= 2×10,000 sqm= 20,000 sqm
FAR=2 so total floor area = 20,000 ×2= 40,000 sqm

Total Residential area = ×40,000= 24,000 sqm

Total Commercialal area = ×40,000= 16,000 sqm

Total ECS for Residential and Commercial respectively is × 1)+( ×1.25) =


240 + 200= 440

Space standards for Parking


S No Type of parking Area in sqm
per ECS
1 Open 23
2 Ground Floor 28
Covered
3 Basement 32
4 Multi level with 30
Ramps
5 Automated 16
multilevel with lifts
source: URDPFI guidelines 2014, Vol I (page 292)
Q.45 A plotted housing scheme on a site of 12 hectare has 60% saleable area. The average
unit cost of land development is INR 300 million per hectare. If the profit margin
is 20%, then the selling price of land per hectare is __________ million INR.
Ans: 600 million INR

Soln: saleable area= ×12 ha = 7.2 ha


Total land development cost = 300 million ×12 ha= 3600 million INR

A profit of 20 % is expected so total selling price = 3600 × = 4320 million INR

This is obtained by selling 7.2 ha of land, so selling price of land per ha =


= 600 million INR

Q.46 An isolated enclosure shown in the Figure has inlet P and


outlet Q of 2 sqm each, on the opposite walls. The outdoor wind speed is 5
m/sec. If the coefficient of effectiveness is 0.6, the rate of natural ventilation in
the enclosure due to wind action is _________ cum/hr.
Ans: 21600 cum/hr
Wind speed is 5m/s and window opening through which wind will pass is 2 sqm.

Amount of wind entering per second = 2 sqm × 5 × 0.6= 6 cum/sec

Ventillation in cum/hr = 6 ×3600 = 21600 cum/hr

Q.47 A 5m × 5m × 3m room has four 230 mm thick external brick walls. Total wall fenestration
is 10 sqm. The temperature difference between indoor and outdoor is 2 degC.
The air to air transmittance values for 230 mm thick brick wall and 200 mm
thick aerated concrete block wall are 2.4 and 1.7 W/sqm degC respectively. If
the brick walls are replaced with the aerated concrete block walls, then the change in
conductive heat flow through the walls is _________W.
Ans: 70 Watt
Total wall area = 60 sqm- 10 sqm= 50 sqm
U-value of brick wall = 2.4 W/sqm degC and that of aerated concrete block= 1.7 W/sqm
degC
ΔT= 2° C
Conductive heat flow = surface area × U-value × ΔT
For brick wall = 50 m² × 2.4 W/sqm degC × 2 deg C= 240 watt
For aerated concrete block wall = 50 m²×1.7 W/sqm degC × 2 deg C= 170 watt
Change in heat flow rate = 240-170= 70 watt

Q.48 For an activity, ‘optimistic time duration’ is 4 days, ‘pessimistic time duration’ is 11 days
and ‘most-likely time duration’ is 8 days. The PERT value of time duration
is______ days (up to one decimal place).
Ans: 7.8 days
PERT value of time duration =

= = = 7.83 days

Q.49 In the Figure, the negative bending moment at point A of the cantilever is ________
kNm.

Ans:225 kNm
Moment due to udl = concentrated load at centre × 2.5 m
= (10kN/m ×5m) × 2.5m = 125 kNm
Moment due to point load at end= 20 kN × 5 m = 100 kNm
Total BM at A= 225kNm

Q.50 The water consumption of a high rise apartment building with 60 dwelling units having an
average household size of 5 persons is 135 lpcd. Assuming 80% of the total
use is met with recycled water supply, the daily domestic demand for the
building is _________ litres.
Ans: 8,100 litres
Soln: Total no. of person = 60 ×5= 300
Total water needed = 135 lpcd ×300= 40,500 litres
80 % is met through recycled so, 20 % will be domestic demand

× 40,500= 8,100 litres

Q.51 In India, for 1.0 cum of M-20 grade concrete, the number of cement bags required is
________ (up to two decimal places).
Ans: 5-9 bags (any one who answerd 5,6,7,8 or 9 bags got marks)
NOMINAL MIX CONCRETE DESIGN FOR ONE CUBIC METER OF M20 CONCRETE
BY VOLUME
It takes 1.55 cubic meters of total dry material to prepare one cubic of concrete with
20mm aggregate because cement and sand will fill the gaps between the
aggregate(crushed stone) in the beginning before it increases the volume of
concrete. But, if we use 40mm aggregate then total dry materials needed will be 1.52
cubic meter.
Let’s calculate material required for M20 grade to prepare one cubic meter concrete.
Cement sand and aggregate ratio for M20 grade concrete is 1:1.5:3
Cement = 1 Part
Sand = 1.5 Part
Aggregate = 3 Part
Total Parts = 1 + 1.5 + 3 = 5.5
Total Material Required per cubic meter of concrete= 1.55

Volume of 1 part= x 1.55


= 0.282 cu.m and density of cement is 1440 kg/cu.m.
Weight of = volume of cement x
cement density
= 0.282 cu.m x 1440
kg/cu.m
= 406
(approximately 400 kg)
No of cement bags required to prepare 1 cubic meter = 400 / 50 = 8 bags

Q.52 The sound power level of an outdoor non-directional point source is 90 dB. Considering
an atmospheric impedance of 400 rayls, the sound pressure level at 10 m
distance from the source is _______ dB.
Ans: 59 dB
For full sphere propagation the sound power level (measured in watt) is equal to sound
pressure level (measured in Mpa) or intensity level (measured in ) at
the distance of r = 0.2821 m from the source.
When it is given ‘Sound power Level’ is 90dB, it means 90dB is at a distance of 0.2821 m.
(this 0.2821 m is the radius of a sphere whose surface area is 1 sqm)
Now we calculate the power (in watt) of the source.

dB= 10 log 10(


here dB= 90 dB

90 = 10 log 10(

log 10( =

log 10( = 9 (removing log) we get =

Id= ×

Id= (this is intensity at distance of 0.2821 m from source)


Note : This is distributed in the surface area of a sphere whose radius is
0.2821 m and surface area of 1 sqm.
So the power of the source is .
Now this watt is distributed in a larger sphere whose radius is 10 m (since dB at 10 m is
asked)

So intensity of sound (in watt/ ) is

=
Now again putting it in equation of sound (dB)

dB= 10 log 10(

= 10 log 10

= 10 log 10 [ ]
= 10 [log 10 – log 10 (4 π)]
= 10 (7-1.09)
= 10 (5.90)
= 59 dB
(Note: No role of atmospheric impedence here)
Q.53 The live load and dead load in a three storeyed residential building, transferred through a
single column, is 12 tons and 18 tons respectively. If the soil bearing capacity is
10 ton/sqm and the factor of safety is 1.5, the area of column footing is
_____________ sqm (up to one decimal place).
Ans: 4.5 sqm
Soln: Total load on column= live load + dead load = 12 ton + 18 ton= 30 ton
Soil bearing capacity is 10 ton/sqm, so for 30 ton→ 3 sqm would have been used.
But factor of safety is 1.5 so 3sqm×1.5= 4.5 sqm of column footing required.

Q.54 The indoor illumination requirement for a building is 350 Lux. If the daylight factor is 2.7
and the design sky illuminance is 9000 Lux, then the required supplementary
artificial lighting is ____________ Lux.
Ans:107 Lux

DF = × 100%
where, Ei = illuminance due to daylight at a point on the indoors working plane,
Eo = simultaneous outdoor illuminance on a horizontal plane from an
unobstructed hemisphere of overcast sky.

Here, 2.7= = × 100 solving for Ei we get Ei= = 243 Lux


We need 350 Lux so the required supplementary artificial lighting is 350- 243= 107 Lux.

Q.55 Two design options of a business building on a 10.0 hectare site are being compared
for built up area. Floor to floor height of Option A is 3.6 m and that of Option B
is 4.5 m. If the maximum allowable building height is 45 m with same ground
coverage for both options, the additional built up area achievable in Option A over
Option B is______percent.
Ans: 20 percent
Soln: It says about same ground coverage but FAR is not mentioned.
Let us assume ground coverage as 50 %
Option A
Total Ground coverage = 5 ha = 50,000 sqm

No of floors= (since height is max. 45 m and fllor to floor height is 3.6 m in Option A)
= 12.5 ( since 12.5 storey is not possible, so max 12 storey can be built)
Total Built up area = 50,000 × 12 = 6,00,000 sq m
Option B
Total Ground coverage = 5 ha = 50,000 sqm

No of floors= (since height is max. 45 m and fllor to floor height is 4.5 m in Option B)
= 10 storey
Total Built up area = 50,000 × 10 = 5,00,000 sq m
Additional built up area in Option A over Option B = 1,00,000 sqm

Percentage = × 100 = 20 %
GATE 2019
Section: General Aptitude
Q.1 The fisherman, _______ the flood victims owed their lives were rewarded by the
government.
(A) whom (B) to which (C) to whom (D) that
Answer: (C) to whom

Q.2 Some students were not involved in the strike.


If the above statement is true, which of the following conclusion is/are logically
necessary?
1. Some who were involved in the strike were students.
2. No students were involved in the strike.
3. At least one student was involved in the strike.
4. Some who were not involved in the strike were students.
(A) 1 and 2 (B) 3 (C) 4 (D) 2 and 3
Answer: (C) 4

Q.3 The radius as well as the height of a circular cone increases by 10%. The
percentage increase in its volume is _______.
(A) 17.1 (B) 21.0 (C) 33.1 (D) 72.8
Answer: (C) 33.1
Q.4 Five members 10, 7, 5, 4 and 2 are to be arranged in a sequence from left to right
following the directions given below:
1. No two odd or even numbers are next to each other.
2. The second number from the left is exactly half of the left-most number.
3. The middle number is exactly twice the right most number.
Which is second number from the right?
(A) 2 (B) 4 (C) 7 (D) 10
Answer: (C) 7
According to given data the only possible arrangement is 10, 5, 4, 7, 2
So, second from right will be 7.

Q.5 Until Iran came along, India had never been ____________________in kabaddi
(A) defeated (B) defeating (C) defeat (D) defeatist
Answer: (A) defeated

Q.6 Since the last one year, after a 125 basis point reduction in repo rate by the reserve
Bank of India, banking institutions have been making a demand
to reduce interest rates on small saving schemes. Finally, the
government announced yesterday a reduction in interest rates on small
saving schemes to bring them on par with fixed deposit interest
rates.
Which one of the following statements can be inferred from the given passage?
(A) Whenever the Reserve Bank of India reduces the repo rate, the interest rates on
small saving schemes are also reduced.
(B) Interest rates on small saving schemes are always maintained on par with fixed
deposit interest rate.
(C) The government sometimes takes into consideration the demand of banking
institutions before reducing the interest rates on small
saving schemes.
(D) A reduction in interest rates on small saving schemes follows only after a
reduction in repo rate by the Reserve Bank of India.
Answer: (C)
The argument says that banking institutiions had been demanding for a reduction in
interest rates for the last one year. Finally the government
decided to reduce the interest rates of small saving schemes thus implying
that the govt. does consider the demands of banking institutions
before making any such policy decision

Q.7 In a country of 1400 million population, 70% own mobile phone. Among the mobile
phone owners, only 294 million access the Internet. Among
these Internet users, only half buy goods from e-commerce portals. What
is the percentage of these buyers in the country?
(A) 10.50 (B) 14.70 (C) 15.00 (D) 50.00
Answer: (A) 10.50

Q.8 The nomenclature of Hindustani music has changed over the centuries. Since the
medieval period dhrupad styles were identified as baanis.
Terms like gayaki and baaj were used to refer to vocal and instrumental styles,
respectively. With the institutionalization of music education the term gharana
became acceptable. Gharana originally referred to hereditary
musicians from a particular linage, including disciples and grand disciples.
Which one of the following pairings is NOT correct?
(A) Dhrupad, baani (B) Gayaki, vocal (C) Baaj, institution (D) Gharana,
linage
Answer: (C) Baaj, institution

Q.9 Two trains started at 7AM from the same point. The first train travelled north at a
speed of 80km/h and the second train travelled south at a
speed of 100km/h. The time at which they were 540km apart is _____AM.
(A) 9 (B) 10 (C) 11 (D) 11.30
Answer: (B) 10

Q.10 “I read somewhere that in ancient times the prestige of a kingdom depended upon
the number of taxes that it was able to levy on its people. It was
very much like the prestige of a head- hunter in his own community.”
Based on the paragraph above, the prestige of a head-hunter depended upon
_______
(A) The prestige of the kingdom (B) The prestige of the heads
(C) The number of taxes he could levy (D) The number of heads he could gather
Answer: (D) The number of heads he could gather

Section: Architecture and Planning


(one mark each)

Q.1 Which of the following commands in AUTOCAD is used to create 3D solid


between various cross sections?
(A) LOFT (B) MESH
(C) XEDGES (D) PFACE
Answer: (A) LOFT
Q.2 Name the architect who criticized ornament in useful objects in his
essay ‘Ornament and Crime’
(A) John Ruskin (B) H.P. Berlage (C) Adolf Loos (D) Walter Gropius
Answer: (C) Adolf Loos
In the essay, Loos explains his philosophy, describing how ornamentation can
have the effect of causing objects to go out of style and thus become
obsolete. It struck him that it was a crime to waste the effort needed to add
ornamentation, when the ornamentation would cause the object to soon go
out of style.

Q.3 A sanitary landfill is provided with High Density Poly Ethylene (HDPE) lining along
the ground surface. This is provided primarily to
prevent

(A) Bleaching (B) Leaching


(C) Rodents (D) Planth growth
Answer: (B) Leaching
A leachate is any liquid that, in the course of passing through matter, extracts
soluble or suspended solids, or any other component of the material through
which it has passed.
When landfill waste degrades and rain water rinses the resulting products
out, leachate is formed. The black liquid contains organic and inorganic
chemicals, heavy metals as well as pathogens; it can pollute ground water.

Q.4 Super-elevation of a road with pre-determined radius of curvature is primarily dependent


on
(A) Altitude (B) Soil bearing capacity (C) Traffic volume (D) Design traffic speed
Answer: (D) Design traffic speed
Advantages of providing Super
elevation:-
Super elevation is provided to achieve the higher speed of vehicles.
It increases the stability of fast-moving vehicles when they pass through a horizontal
curve, and it also decreases the stresses on the foundation.
In the absence of super elevation on the road along curves, potholes are likely to occur
at the outer edge of the road.
IRC specifies a maximum super-elevation of 7 percent for plain and rolling terrain,
while that of hilly terrain is 10 percent (snow clad) and urban road is 4 percent. The
minimum super elevation is 2-4 percent for drainage purpose, especially for large
radius of the horizontal curve.

S. No Type of Road Super Elevation


In Ratio In Percentage
1 In Hilly and Snow 1 in 10 10
fall area
2 In Plain and 1 in 15 7
Rolling Terrain
3 Urban Roads 1 in 50 to 1 2-4
in 25

Q.5 In mono-centric urban model, land rent is expected to

(A) diminish as one moves towards the


center
(B) diminish as one moves away from the center
(C) remain constant across the whole urban area
(D) be unrelated with distance from centre
Answer: (B) diminish as one moves away from the center
Land users, whether they be retail, office, or residential, all compete for the most
accessible land within the CBD. The amount they are willing to pay is called bid rent. This
can generally be shown in a "bid rent curve", based on the reasoning that the most
accessible land, generally in the centre, is the most expensive land.
Q.6 Fineness modulus of sand measures its
(A) Compressive strength (B) Grading according to particle size
(C) Bulking of sand (D) Ratio of coarse and fine sand
Answer: (B) Grading according to particle size
Fineness modulus of sand (fine aggregate) S. No Type of sand Finess modulus
is an index number which represents the range
mean size of the particles in sand 1 Fine sand 2.2 - 2.6
Fine aggregate means the aggregate 2 Medium sand 2.6 - 2.9
which passes through 4.75mm sieve. To
3 Coarse sand 2.9 - 3.2
find the fineness modulus of fine aggregate
we need sieve sizes of 4.75mm, 2.36mm, 1.18mm, 0.6mm, 0.3mm and 0.15mm.
(example)

Sieve Opening Wt. of % retained Cumulative % finer


Size (mm) material % retained
retained (g)
No 4 4.76 0 0 0 100
No 8 2.36 1.3 0.26 0.26 99.74
No 16 1.18 2.4 0.48 0.74 99.26
No 30 0.6 4.3 0.86 1.6 98.4
No 50 0.3 164.6 32.88 34.68 65.52
No 100 0.15 259.6 51.86 86.34 13.66
Pan 68.4 13.66 100 0

Wt. of sample = 500gm and fineness modulus is 1.23

Q.7 The spherical surface of the geodesic dome comprises of


(A) Equilateral triangles of various sizes
(B) Isosceles triangles of various sizes
(C) Equilateral triangles of uniform size
(D) Isosceles triangles of uniform size
Answer: (C) Equilateral triangles of uniform size
Geodesic domes don't have one canonical form, but the most popular is
based on an icosahedron whose triangular faces are then subdivided into
smaller triangles. Each triangles are equilateral triangles.

Q.8 The abrupt change or junction between two


ecological zones is termed as
(A) Ecological niche (B) Ecosystem
(C) Ecotype (D) Ecotone
Answer: (D) Ecotone
An ecological niche
describes how a species interacts with, and lives in, its habitat. Ecological
niches have specific characteristics, such as availability of nutrients,
temperature, terrain, sunlight and predators, which dictate how, and how well, a
species survives and reproduces. A species carves out a niche for itself in a
habitat by being able to adapt and diverge from other species.
Ecosystem: a biological community of interacting organisms and their physical
environment.
Ecotype: a distinct form or race of a plant or animal species occupying a
particular habitat.
Ecotone: abrupt change or junction between two ecological zones.

Q.9 Complementary colours in a Munsell pigment colour wheel refers to


(A) Colours in alternate positions (B) Colours opposite to one another
(C) Colours adjacent to each another (D) A pair of secondary colours

Answer: (B) Colours opposite


to one another
S. Colour Position on colour Examples
No Composition wheel
1 Complementary Opposite to one
another
2 Analogous Neighbours Red Orange- Orange-
Yellow Orange
3 Split Neighbours of
Complementary complementary
4 Triad Three Colours at an Yellow- Red -Blue
angle of 120º from
Centre of colour
wheel
5 Tetrad Four Colours at an Yellow- Red orange-
angle of 90º from Purple- Blue green
Centre of colour
wheel
[Refer: GATE 2018(Q1), 2015(Q25), 2009(Q3), 2006(Q58), 2005(Q47), 2002(Q2.3), 1995(Q1,iii), 1994(Q1,iv)]
[Refer:UPSC 2016(Q13, 2015(Q29)] [Refer ISRO 2015(Q8)] [Refer KCET 2013(Q58)]

Q.10 The closing syntax, for an executable command line in C or C++ program, is
(A) : (B) , (C) ; (D) ,
Answer: (C) ;

Q.11 The term ‘Necropolis’ refers to


(A) Organically growing settlement (B) Origin of a settlement
(C) A dead settlement (D) Merging of two settlements
Answer: (C) A dead settlement
Necropolis: The city in the worst stage and unfit for dwelling.
Patrick Geddes Classification of towns
S. Types of Characteristics
No settlement
1 Primary Agricultural Village
2 Secondary Marketing town
3 Tertiary Residential, Educational
and Recreational

Lewis Mumford Classification of Cities


S. Types of Characteristics
No settlement
1 Eopolis Town grows as entire unit. Its economy
is based on agriculture
2 Polis Town grows into a small urban unit of
self contained community. It has
commerce and industry.
3 Metropolis City grows with full strature. High
population density and large
potentialities.Water supply, drainage,
Electricity, transport, commerce and
industries.
4 Megapolis Overgrown cities with mess with
expansion of industries, multi track
roads, mass housing, mass
transportation, city starts decaying
5 Tyrannopolis City shows further decay
6 Necropolis Worst stage and unfit for dwelling, Dead
city

Doxiadis Ekistic Unit


S. Types of Population Settlement Description
No settlement Scale Classification
(Four Types)
1 Anthropos 1 Elementary
2 Room 2 Minor Shells units, man,
3 Dwelling 5 room, dwelling
4 Dwelling group 40 Units smaller
than, or as
5 Small 250 small as, the
Neighbourhood traditional town
Micro where people
6 Neighbourhood 1500 Settlements
used to and
7 Small polis 10,000 still do achieve
(town) interconnection
by walking
8 Polis 75,000 Between the
9 Small 5,00,000 traditional town
Metropolis and
Meso conurbation
10 Metropolis 4 million Settlement
11 Small 25 million within which
Megalopolis/ one can
Conurbation commute daily
12 Megalopolis 150 million From the
13 Small 750 million Megalopolis to
Eperopolis the largest
14 Eperopolis 7500 possible
Macro expression of
million
settlement Ecumenopolis

15 Ecumenopolis 50,000
million

Classification of large cities in India


Classification of Remarks
large cities
Cities (Class I UA/ Population more than 1,00,000
town)
Million plus UA/ Towns with Population more than 1
UA/Town million (10,00,000)
Mega Cities UA/ Cities having more than 10 million
population (1 crore)
Classification of Area in India:
Types of Classification Definition Remarks
Settlement
Statutory All places with a
Town Municipality,
corporation,
cantonment board or
notified town area
committee declared
by state law.
Census Town Places that satisfy Classification Population
the following criteria
(a) A minimum Class I 1,00,000 and
Population of 5000 above
(b) Atleast 75% of
male working Class II 50,000-99,999
population Class III 20,000-49,999
engaged in non
agricultural pursuits Class IV 10,000-19,999
(c) A density of
population of atleast Class V 5000-9999
Urban
400 person/ square
km.
Outgrowth A viable unit such as Outgrowths are Railway
a village or a part of colonies, University Campus,
village that is Port areas that may come up
contiguous to a near a city or statutory town
statutory town and outside the statutory unit but
possess the urban within the revenue limit of a
feature in terms of village or villages contiguous
infrastructure and to the town or city
amenities such as
pucca roads,
electricity, taps,
drainage system,
education
institutions, post
offices, medical
facilities, banks.
Rural All area other than Basic Unit is Revenue village
urban

DDA (Settlement hierarchy)

Nomenclature Population Major Facilities


1 Housing 5000 totlot
2 Neighbourhood 10,000 Primary School, Senior Secondary
School, 11 KVA substation
3 Community 1,00,000 Hospital, Auditorium, 33 KVA
substation, Community Centre
4 District 5,00,000 Hospital, Vetinary Hospital, School for
special need,220 KVA substation
5 Zone/Sub City 10,00,000 Medical College, Whole sale market,
Head Post Office
6 City More than Police Line, District Jail, ISBT
25,00,000

(UA):An urban agglomeration is a continuous urban spread constituting a


town and its adjoining outgrowths (OGs), or two or more physically
contiguous towns together with or without outgrowths of such towns. An
Urban Agglomeration must consist of at least a statutory town and its total
population (i.e. all the constituents put together) should not be less than
20,000 as per the 2001 Census. In varying local conditions, there were
similar other combinations which have been treated as urban
agglomerations satisfying the basic condition of contiguity. Examples:
Greater Mumbai UA, Delhi UA, etc.

Q.12 Which of the following projection types is adopted in Universal Transverse Mercator
(UTM)?
(A) Spherical (B) Conical (C) Planar (D) Cylindrical
Answer: (D) Cylindrical
A map projection is a systematic transformation of the latitudes and
longitudes of locations from the surface of a sphere or an ellipsoid into
locations on a plane.
Maps cannot be created without map projections.
All map projections necessarily distort the surface

Common types of map projections are cylindrical, conic, and azimuthal.

Cylindrical map projections are


rectangles, but are called cylindrical because they can be rolled up
and their edges mapped in a tube, or cylinder.
Distortion- The disadvantage of cylindrical map projections are that they are
severely distorted at the poles. While the areas near the Equator are the
most likely to be accurate compared to the actual Earth.
Advanatages: Cylindrical map projections are good for comparing latitudes to
each other and are useful for teaching and visualizing the world as a
whole.
Types of cylindrical map projections include the popular Mercator projection,
Cassini, Gauss- Kruger, Miller, Behrmann,
Hobo- Dyer, and Gall- Peters.

Conic map projections are


designed to be able to be wrapped around a cone on top of a sphere (globe), but
aren’t supposed to be geometrically accurate.
Conic map projections are best suited for use as regional or hemispheric maps, but
rarely for a complete world map. The distortion in a conic map makes it
inappropriate for use as a visual of the entire Earth but does make it great for use visualizing
temperate regions, weather maps, climate projections, and more.

Azimuthal map projection is angular- given three points on a map (A, B, and
C) the azimuth from Point B to Point C dictates the angle someone
would have to look or travel in order to get to A. These angular
relationships are more commonly known as great
circle arcs or geodesic arcs. The main features of
azimuthal map projections are straight meridian lines,
radiating out from a central point, parallels that are circular
around the central point, and equidistant parallel
spacing. Light paths in three different categories
(orthographic, stereographic, and gnomonic)
can also be used. Azimuthal maps are beneficial for finding
direction from any point on the Earth using the central point as a
reference.

Q.13 The ingredient to be added to produce


Aerated Cement Concrete, is
(A) Aluminium (B) Calcium chloride
(C) Gypsum (D) Sulphur
Answer: (A) Aluminium
Autoclaved aerated concrete (AAC) is made with fine aggregates, cement, and an
expansion agent that causes the fresh mixture to rise like bread dough. In fact,
this type of concrete contains 80 percent air.
Manufacturing Process: several ingredients are blended into a slurry: cement,
lime, water, finely ground sand, and often, fly ash. An expansion agent like
aluminum powder is added and the fluid mixture is cast into a large billet. As the
slurry reacts with the expansion agent to generate air bubbles, the mixture
expands. After its initial set, the resulting “cake” is wire cut into precisely sized
blocks or panels and then baked (autoclaved). The heat helps the material to cure
faster so that blocks and panels maintain their dimensions.

Q.14 The cause of short column effect, during seismic occurrence, is due to

(A) Centralized rupture of the


column (B) Tearing of reinforcement bars
(C) Buckling of columns (D) Stress concentration
Answer: (D) Stress concentration
The distribution of seismic forces is critical when all columns are not of all the
same height. In this case shorter columns,
although with the same cross section will receive increased
seismic load due to higher overall stiffness. The
stiffness of a column is inversely proportional to
the column length cubed (L³). Thus , the column which
is half of other column’s length is in fact eight times stiffer.

Q.15 The solar protection system consisting of fixed slates or grids, outside a building
façade in front of openings, is known as
(A) Brise-soleil (B) Solarium (C) Malqaf (D) Trombe wall
Answer: (A) Brise-soleil
Brise soleil systems use a series of horizontal or vertical blades to control
the amount of light and solar heat that enters a building.
Solarium: a room in which you can tan (= make brown) your skin using either
light from the sun or special equipment.

Fig: Brise Soleil Fig: Solarium

Malqaf: A windtower (wind catcher) is a traditional Iranian architectural


element to create natural ventilation in buildings.

Trombe Wall-A Trombe wall is a passive solar building design where a wall is
built on the winter sun side of a building with a glass external layer and a high
heat capacity internal layer separated by a layer of air.

Fig: Trombe Wall Fig: Malqaf

Q.16 The Indian property inscribed by UNESCO on the World Heritage List in the year 2018
is
(A) Mattanchery Palace, Emakulam (B) The Victorian Gothic and Art Deco
Ensembles of Mumbai
(C) Ancient Buddhist Site, Sarnath (D) Mughal Gardens in Kashmir
Answer: (B) The Victorian Gothic and Art Deco Ensembles of Mumbai
Name of Cultural World Year of
S No. State
Heritage Site Notification
1 Agra Fort Uttar Pradesh 1983
2 Ajanta Caves Maharashtra 1983
Buddhist Monuments at Madhya
3 1989
Sanchi Pradesh
Name of Cultural World Year of
S No. State
Heritage Site Notification
Champaner-Pavagadh
4 Gujarat 2004
Archaeological Park
Chhatrapati Shivaji
5 Terminus (Formerly Maharashtra 2004
Victoria Terminus)
Churches and Convents
6 Goa 1986
of Goa
7 Elephanta Caves Maharashtra 1987
8 Ellora Caves Maharashtra 1983
9 Fatehpur Sikri Uttar Pradesh 1986
Great Living Chola
10 Tamil Nadu 1987
Temples
Group of Monuments at
11 Karnataka 1986
Hampi
Group of Monuments at
12 Tamil Nadu 1984
Mahabalipuram
Group of Monuments at
13 Karnataka 1987
Pattadakal
Rajasthan
(Chittorgarh,
Kumbhalgarh,
Ranthambhore,
14 Hill Forts of Rajasthan 2013
Amber Sub-
Cluster,
Jaisalmer,
Gagron)
15 Humayun’s Tomb, Delhi Delhi 1993
Khajuraho Group of Madhya
16 1986
Monuments Pradesh
Mahabodhi Temple
17 Bihar 2002
Complex at Bodh Gaya
Mountain Railways of
18 Tamil Nadu 1999
India
Qutb Minar and its
19 Delhi 1993
Monuments, Delhi
Rani-Ki-Van (the Queen’s
20 Stepwell) at Patan, Gujarat 2014
Gujarat
21 Red Fort Complex Delhi 2007
Rock Shelters of Madhya
22 2003
Bhimbetka Pradesh
23 Sun Temple, Konarak Orissa 1984
24 Taj Mahal Uttar Pradesh 1983
25 The Jantar Mantar, Jaipur Rajasthan 2010
Archaeological Site of
Nalanda Mahavihara
26. Bihar 2016
(Nalanda University) at
Nalanda
Name of Cultural World Year of
S No. State
Heritage Site Notification
The Architectural Work of
Le Corbusier, an
27. Chandigarh 2016
Outstanding Contribution
to the Modern Movement
Historic City of Ahmedabad,
28. 2017
Ahmedabad Gujarat, India
The Victorian Gothic and
29. Art Deco Ensembles of Mumbai 2018
Mumbai
30. Walled City of Jaipur Rajasthan 2019

Q.17 Typical features of Buddhist architecture are


(A) Mandapa, Chattri, Amalaka, Torana (B) Stambha, Torana, Vimana, Harmika
(C) Vedika, Chattri, Torana, Harmika (D) Vedika, Stupa, Chaitya, Vimana
Answer: (C) Vedika, Chattri, Torana, Harmika

Q.18 Identify the Queen closer

(A) A (B) B (C) C (D) D


Answer: (A)

Q.19 Identify the role of Vermiculate in vertical landscapes


(A) Fertilizer (B) Holding material
(C) Binder material (D) Water retention element
Answer: Marks to all
(it is vermiculite !)
Perlite and vermiculite are two different substances used for a common purpose, that
is for moisture retention and aeration in soil. That means to hold water within soil
for plants and also to improve air circulation or aeration of potting mix.
Actual soil is not used for Potting Plants. It contains a mix of peat moss, vermiculite,
perlite, sand, and shredded bark or compost.
Peat Moss- Decayed, dried sphagnum moss has the name of peat or peat
moss. This is used as a soil conditioner which increases the soil's capacity to
hold water and nutrients by increasing capillary forces and cation exchange
capacity. Moss is ( काई )
Vermiculite- Vermiculite is the name of a group of hydrated laminar minerals
(aluminum-iron magnesium silicates) which look like mica. Horticultural
vermiculite is processed with massive heat that expands it into accordion
shaped pellets composed of multiple layers of thin plates.
Perlite- Perlite is an amorphous volcanic glass that has a relatively high water
content

Q.20 Which of the following parameters is essential to estimate the Envelope Performance
Factor (EPF) of a building as per the Energy Conservation Building Code
(ECBC), 2011?
(A) Building Type (B) Maximum humidity
(C) Maximum and minimum monthly temperature (D) Building occupancy duration
Answer: (D) Building occupancy duration
Building Envelope :
The building envelope refers to the exterior façade, and is comprised of opaque
components and fenestration systems. Opaque
components include walls, roofs, slabs on grade (in touch with ground), basement
walls, and opaque doors. Fenestration systems include windows,
skylights, ventilators, and doors that are more than one-half glazed.
The envelope protects the building’s interior and occupants from the weather
conditions and shields them from other external factors e.g.
noise, air pollution, etc.

Day Time 24- Hour


Occupancy Occupancy
U- factor SHGC U- factor SHGC
Mass Walls 6.01 - 13.85 -
Curtain Walls, 15.72 - 20.48 -
Others
Roofs 11.93 - 24.67 -
North -1.75 40.65 -4.56 58.15
Windows
Non- North -1.25 54.51 0.68 86.57
Windows
Skylights -96.35 311.71 -294.66 918.77
Source: Envelope Performance Factor Coefficients-Composite Climate (under review) ECBC User Guide 2011.
(Appendix D)
EPF calculation through Trade off Factor
=

Where = A coefficient for the “Roof” class of construction


= The U-factor for the envelope component referenced by the subscript “s”
= The Area of a specific envelope component referenced by the
subscript “s”

Similarly EPF (Envelope Performance Factor) of wall and fenestration can


be calculated. All will sum up to give EPF of building
(For more information refer ECBC User Guide)

Q.21 The illumination level of a room is 300 lux and the efficacy of the lamps is 60. The Light
Power Density (LPD) of the room in Watt/m² is _________.
Answer: 5 Watt/m²
Efficacy of a lamp source is lumens per watt. Given here is 60 that means 60
lumens/watt.
The illumination level of Room is 300 lux= 300 lumens/m² (Technically Lux is
lemens/m²)
After consuming 1 watt, the lamp can give only 60 lumens.
Consider 1 square m of area in a room. To get 300 lumens for this area total wattage
needed is

= 5 watt
Light Power Density (LPD) = 5 Watt/m²
[Light Power Density (LPD) was asked for the first time in GATE AR]

Q.22 The load on a RCC column is 150 kN. The soil bearing capacity is 80 kN/m². Assuming
a factor of safety of 1.2, the side of the square column footing is
______ meter (rounded off to one decimal place).
Answer: 1.5 m
Soln: Soil bearing capacity is 80kN/m².

Factor of safety to be taken as 1.2 , it means soil bearing capacity mut not exceed
= 66.67 kN/m².

1 sqm of soil can take 66.67 kN, so in order to take 150 kN= = 2.25 sqm
Since it is a square footing the side of square will be sq m = 1.5 m
Q.23 A room is separated by a partition wall. The average intensities of sound in the source
and receiving sides across the partition are W/m² and
W/m² respectively. The transmission loss (TL) of the partition wall is
_______dB.
Answer: 30 dB
Sound Pressure Level (dB) and Intensity formula

(dB) = 10 ( where

Sound Pressure Level at the source where intensity is W/m²

(dB) = 10 (

= 10
= 10 × 8 ×
= 80 dB
Similarly Sound Pressure Level at the receiving side where intensity is W/m²

(dB) = 10 (

= 10
= 10 × 5 ×
= 50 dB
Transmission Loss = 80-50 = 30 dB

Q.24 The purchase price of 2BHK flat rises 10 percent, the demand for such flats is
observed to decrease by 8 percent. The given
elasticity of the housing demand for 2BHK flats is _____ (rounded off to one decimal
place).
Answer: 0.8
Price elasticity of demand is measured by using the formula:

Price Elasticity of demand (PED) = =


= = =0.8
There are four types of elasticity, each one measuring the relationship between two
significant economic variables. They are:
Price elasticity of demand (PED), which measures the responsiveness of the
quantity demanded to a change in price. PED can be mmeasured over a price
range, called arc elasticity, or at one point, called point elasticity.
Price elasticity of supply (PES), which measures the responsiveness of the
quantity supplied to a change in price.
Cross elasticity of demand (XED), which measures the responsiveness of the
quantity demanded of one good, good X, to a change in the price of another
good, good Y.
Income elasticity of demand (YED), which measures the responsiveness of the
quantity demanded to a change in consumer incomes.

Q.25 ‘Threshold enclosure’ created by vertical surface or series


of vertical elements in an urban plaza, represented by the ratio of height and distance,
is given by an angle of _____ degrees (rounded off to one decimal
place)
Answer: to 30º

Soln: Tan θ = (since =

S. Type of Ratio Angle


No enclosure (Ht/distance) (degrees)
1 Full 1:1 45º
Enclosure
2 Threshold 1:2 30º
of
Enclosure
3 Minimum 1:3 18º
Enclosure
4 Loss of 1:4 14º
Enclosure
source: Architecture of Towns and Cities by Paul D. Sprieregen (page 75)

Section: Architecture and Planning (2 marks)


Q.26 Match the instruments in Column I with the various types of surveying in Column II and
select the appropriate option.
Column- Column-II
I
P Cross staff 1 Indoor wall to
wall
measurement
Q Alidade 2 Traversing
R Sextant 3 Chain survey
S Distomat 4 Plane table
survey
5 Contour
survey
(A) P-3, Q-4, R-2, S-5 (B) P-2, Q-4, R-1, S-5 (C) P-5, Q-3, R-2, S-1 (D) P-3,
Q-4, R-2, S-1
Answer: (D) P-3, Q-4, R-2, S-1
Cross Staff: The cross staff is used for finding out foot of the perpendicular from a
given point to a line and setting right angle at a given point on
a line.
An alidade or a turning board is a device that allows one to sight a distant object
and use the line of sight to perform a task. This task can be, for example, to draw a
line on a plane table in the direction of the object or to measure the angle to the
object from some reference point

Sextant: A sextant is a doubly reflecting navigation instrument that measures the angular
distance between two visible objects.
Distomat:
Distance measuring
instrument.
Q.27 Match the characteristics of settlement systems in Column- I with their corresponding
theory/rules in Column- II and select appropriate option.
Column-I Column-II
P Primacy of settlements 1 Central Place
Theory
Q Settlement size and 2 Gravity Model
location
R Random componenets 3 Rank size rule
in location of settlements
S Interaction between 4 Entropy of
settlements settlements
5 Core Peripheral
Model
(A) P-4, Q-1, R-2, S-5 (B) P-2, Q-5, R-3, S-1 (C) P-3, Q-5, R-4, S-2 (D) P-3, Q-
1, R-4, S-2

Answer: (D) P-3, Q-1, R-4, S-2


Central Place Theory- Central place theory is a geographical theory that seeks to
explain the number, size and location of human settlements in a
residential system. The theory was created by the German geographer
Walter Christaller.

Gravity Model- The Gravity Model is a model used to


estimate the amount of interaction between two cities. It is based on Newton's
universal law of gravitation, which measured the attraction of two objects based
on their mass and distance.
Rank Size Rule- The rank-size rule (or rank-size distribution) of city populations, is a
commonly observed statistical relationship between the population sizes and
population ranks of a nation's cities. According to the rank-size rule, if the largest city
in a country has a population of 10,00,000, then the fourth largest city in that country
would have a population of approximately 2,50,000.
Core peripheral model:
According to the centre–periphery model, underdevelopment is not the result of
tradition, but is produced as part of the process necessary for the development of
capitalism in the central capitalist countries—and its continued reproduction on a world
scale. The theory assumes a central core of capitalist countries, in which the economy
is determined by market forces, there is a high organic composition of capital, and
wage-levels are relatively high. In the peripheral countries, on the other hand, there is
a low organic composition of capital and wage-levels do not meet the cost of
reproduction of labour.
Primacy of Settlement: The law of the primate city was first proposed by the
geographer Mark Jefferson in 1939. He defines a primate city as being "at least twice
as large as the next largest city and more than twice as significant." Aside from size
and economic influence, a primate city will usually have precedence in all other
aspects of its country's society, such as being a center of politics, media, culture and
education and receive most internal migration.
S. Theory Propounded by
No
1 Central Place Theory Walter Christaller
(1933)
2 Primate City Mark Jefferson
(1939)
3 Rank Size Rule G.K. Zipf (1949)
4 Strategy of Albert O.
Unbalanced Growth Hirshman(1950)
5 Input Out Analysis Wassily W.
Leontief (1951)
6 Growth Pole Theory Francois Perroux
(1955)
7 Export Base Theory Douglos C. North
(1955)
8 Economic Base Hans Blumenfeld
Theory (1955)
9 Circular Cumulative Gunnar Myrdal
Causation (1956)
10 Stages of Economic Walt Whitman
Growth Rostow (1960)
11 Core Periphery John Friedmann
Model (1963)
Refer GATE 2008 (Q.27)

Q.28 Match the architectural projects in Column I with the architect in Column II, and select
the appropriate option.
Column-I Column-II
P India Habitat 1 Christopher Charles
Centre, New Delhi Benninger
Q United World 2 Charles Correa
College (UWC),
Mahindra College,
Pune
R Brain and 3 Joseph Allen Stein
Cognitive Science
Centre- MIT,
Cambridge
S Habitat -67, 4 Norman Foster
Montreal
5 Moshe Safdie
(A) P-3, Q-1, R-2, S-5 (B) P-1, Q-2, R-5, S-3 (C) P-2, Q-1, R-5, S-4 (D) P-3, Q-
4, R-1, S-5
Answer: (A) P-3, Q-1, R-2, S-5
Q.29 Match the Name of book provided in Column I with the corresponding author in
Column II and select the appropriate option.

Column-I Column-II
P Earthscape 1 Ian McHarg
Q Synthesis of Form 2 John O
Simonds
R Design with Nature 3 Cristopher
Alexander
S The City of Tomorrow 4 Lewis Mumford
and its Planning
5 Le Corbusier
(A) P-2, Q-3, R-1, S-5 (B) P-5, Q-2, R-3, S-4 (C) P-5, Q-3, R-1, S-4 (D) P-2, Q-1,
R-4, S-5
Answer: (A) P-2, Q-3, R-1, S-5

Q.30 Match the thermal properties in the Column- I and their respective units in Column- II
and select the appropriate option.
Column- I Column- II
P Thermal 1
Resistance J
Q Thermal 2
Transmittance
R Specific Heat 3
S Thermal 4
Conductivity
5
J
(A) P-4, Q-1, R-5, S-2 (B) P-4, Q-3, R-1, S-2 (C) P-5, Q-3, R-1, S-4 (D) P-3,
Q-4, R-2, S-1
Answer: (B) P-4, Q-3, R-1, S-2

Q.31 Match the application in the field of construction in the Column I and the respective
itemsin Column II and select appropriate option

Column-I Column-II
P Polytetrafluoroethylene 1 Tendon
(PTFE) membrane
Q Isolated compression 2 TMT
component inside a
network of continuous
tensile member
R Cable used for pre- 3 Tensigrity
stressed concreting
S Reinforcement bar used 4 TMD
in RCC construction
5 Teflon
(A) P-5, Q-1, R-4, S-3 (B) P-4, Q-3, R-1, S-5 (C) P-5, Q-3, R-1, S-2 (D) P-3,
Q-4, R-2, S-1
Answer: (C) P-5, Q-3, R-1, S-2
PTFE Membrane is a material made by coating
the superfine glass fiber fabric with PTFE resin.
Such membrane has good welding
performance, excellent ultraviolet resistance,
anti- ageing performance and flame
retardation. In addition, it has the best anti-
dirt and self-cleaning performance among all
building membranes, but it is characterized
by poor ductility, which results in the difficult
construction and surprisingly high cost. The
processing method is the fiber fabrics are
placed quickly into Teflon melt for times, so that both
sides of the fabrics are uniformly
applied with Teflon coating.

A tendon is basically a steel cable or wire used in


Prestressed Concrete structural elements like
beam, column etc. It is a medium through which
tensile stresses are induced into the concrete. It maybe one single steel wire or a
group of wires twisted together to pass through required amount of tensile
stresses.
Tendons are made of individual units called strands.
Ex: 19T15 means 15.2 mm strands ,total 19nos of strands which will make a
single tendon.

Tensigrity: Term coined by Sir Buckminster


Fuller. Tensegrity, tensional integrity or floating compression is a structural principle based
on the use of isolated components in compression inside a net of continuous tension, in
such a way that the compressed members (usually bars or struts) do not touch each other
and the prestressed tensioned members (usually cables or tendons) delineate the system
spatially

Teflon:
polytetrafluoroethene (PTFE).Teflon is also used as the trade name for a polymer with
similar properties, perfluoroalkoxy polymer resin (PFA). :Teflon is the
plastic with the lowest coefficient of friction. It is also used as a non-stick
coating for pans and other cookware.

TMT bars or Thermo-Mechanically


Treated bars are high-strength reinforcement bars having a tough outer core and a soft
inner core. The very first step of the manufacturing process involves passing the steel
wires through a rolling mill stand. Thereafter, these rolled steel wires are again passed
through the Tempcore water cooling system. While passing the wires through the water
cooling system, the water pressure is optimised. The sudden quenching and drastic
change in temperature toughen the outer layer of the steel bar, thus making it super
tough and durable. Once this process is over, the TMT bars are subject to atmospheric
cooling. This is done in order to equalise the temperature difference between the soft
inner core and the tough exterior. Once the TMT bar cools down, it slowly turns into a
ferrite-pearlite mass. The inner core remains soft giving the TMT bar great tensile
strength and elongation point. This design is unique to the TMT bars and gives
superior ductility to the bars. Also, this unique manufacturing technique and the
absence of Cold stress make this bar corrosion-resistant and boost its weldability.

Q.32 Match the following types of masonry joints in Column I with their respective
corresponding description in Column II, and select the appropriate option.
(A) P-1, Q-3, R-2, S-4
(B) P-4, Q-3, R-2, S-5
(C) P-3, Q-4, R-5, S-2
(D) P-4, Q-3, R-1, S-5
Answer: (B) or (D) both correct
Pointing is the term given to the ‘finish’ that is between the bricks or stone used to build
your house. Depending on the age of the building, the mortar used to lay the stone or
brick will either be made from lime, or more recently, cement. Incorrect pointing
causes irreparable damage to older buildings. It is essential to understand what the
mortar joints actually do for the fabric of the house.
The function of the mortar in the wall is to act as a bedding between stones and varies
from fine joints in ashlar stonework to larger joints in rubble masonry walls. Joints are
effectively reduced in size by inserting small stones and ‘snecked’ pieces of stone.
Whilst acting as a bedding the mortar must also perform other functions:
Prevent water penetration through the joints by its physical presence almost like a
masonry‘ sponge’, yet it must allow the wall to breathe and drain, porosity being a key
factor in the choice of a repointing mortar.

Q.33 Match the following in Column I with their suitable description in Column II, and
select the appropriate option.
Column-I Column-II
P Tolerance 1 100 mm
Q Precast concrete 2 Non modular
rings for well dimension
R 3 Acceptable
M variation
S Weather joints 4 3-D prefabricate
5 Resilient sealants
(A) P-2, Q-4, R-1, S-3 (B) P-2, Q-4, R-3, S-5 (C) P-1, Q-2, R-3, S-4 (D) P-3,
Q-4, R-1, S-5
Answer: (D) P-3, Q-4, R-1, S-5

Q.34 Match the units in Column I with their corresponding items in Column II and select the
appropriate option.
Column-I Column-II
P dB 1 Sound Intensity
Q 2 Absorption of
Phon sound
R 3 Frequency of
W/m² sound
S Sabine 4 Loudness
5 Sound Pressure
level
(A) P-5, Q-1, R-4, S-3 (B) P-2, Q-3, R-4, S-5 (C) P-1, Q-2, R-3, S-4 (D) P-5,
Q-4, R-1, S-2
Answer: (D) P-5, Q-4, R-1, S-2
Q.35 Match the scientific names of the trees provided in Column- I with the corresponding
colour of their bloom in Column II, and select the appropriate option.
Column-I Column-II
P Cassia fistula 1 White
Q Lagerstroemia flos- 2 Red
reginae
R Cordia sebastena 3 Blue
S Piumeria alba 4 Yellow
5 Mauve
(A) P-4, Q-5, R-4, S-1 (B) P-1, Q-5, R-2, S-3 (C) P-5, Q-4, R-1, S-3 (D) P-4, Q-
5, R-2, S-1

Answer: (D) P-4, Q-5, R-2, S-1


Q.36 Match the items in Column- I and their respective location in building/site in Column-
II, and select the appropriate option.

Column-I Column-II
P Nahani Trap 1 Between waste
water pipe and main
house drain
Q 2 Between Septic tank
Gully Trap and soak pit
R 3 Junction of house
Bottle Trap drain and sewer
S Intercepting 4 Bathroom and
Trap kitchen floor
5 Below the wash
basin

(A) P-4, Q-5, R-2, S-3 (B) P-5, Q-1, R-3, S-2 (C) P-4, Q-1, R-5, S-3 (D) P-3,
Q-4, R-5, S-2
Answer: (C) P-4, Q-1, R-5, S-3
Q.37 As per the Handbook on Barrier Free and Accessibility, CPWD – 2014, match the
design guidelines in Column- I with their appropriate standards in Column- II and
select the appropriate option.

Column-I Column-II
P Minimum clear width 1 600 mm
of ramp
Q Maximum height of 2 1500 mm
wash basin (rim)
above the finished
floor level
R Minimum length of 3 750 mm
grab rail
S Minimum clear width 4 900 mm
for maneuvering
space (wheel chair)
5 1800 mm

(A) P-3, Q-4, R-1, S-5


(B) P-5, Q-3, R-2, S-4
(C) P-5, Q-3, R-1, S-2
(D) P-1, Q-4, R-3, S-1
Answer: (C) P-5, Q-3, R-1, S-2

Q.38 Match the contemporary Urban Design Movements listed in Column- I with the
corresponding principles listed in Column- II and select the
appropriate option.
Column-I Column-II
P Park Movement 1 Self-contained, self-sufficient
community surrounded by
green belts
Q New Urbanism 2 Revival of the relationship
between man and nature
R City Beautiful 3 Relationship between work
Movement and living, environment and
sustainability
S Garden City 4 Unity, cohesion and balanced
and New Town relationship between urban
Movement components and elements
5 Technical and socio economic
process resulting in growth,
energy production and waste
elimination.
(A) P-2, Q-3, R-4, S-1 (B) P-1, Q-5, R-3, S-2 (C) P-5, Q-3, R-1, S-2 (D) P-2,
Q-5, R-4, S-1
Answer: (A) or (D) both correct
Park Movement: It started in USA.
The public park movement, which started in the 1830s, sprang mainly out of a desire
to improve health in the over-crowded conditions of the rapidly growing industrial
towns. By the end of the Victorian era the need for public open space had become
widely appreciated. Increasingly, parks additionally became symbols of civic pride,
providing inhabitants and visitors alike with attractive surroundings in which to enjoy
their leisure time. In the minds of their promoters they also assumed a social role as
places of betterment for the lower levels of society. In the United States, the best
known park advocate was Frederick Law Olmsted, who, with his partner Calvert Vaux,
conceived of and promoted the construction of Central Park in New York City (1858)
and the Emerald Necklace in Boston (1878–80), as well as some of the most notable
parks in other large cities in the United States.

New Urbanism is an urban design movement which promotes environmentally friendly


habits by creating walkable neighborhoods containing a wide range of housing and job
types. It arose in the United States in the early 1980s, and has gradually influenced
many aspects of real estate development, urban planning, and municipal land-use
strategies.
Principles of Urbanism
1. Walkability: Most things must be within a 10-minute walk of home and work.
Pedestrian friendly street design. Pedestrian streets free of cars in
special cases.

2. Connectivity:
Interconnected street grid network disperses traffic and eases
walking. A hierarchy of narrow
streets, boulevards, and alleys. High-
quality pedestrian network and public realm
makes walking pleasurable.
3. Mix-Use and Diversity: Mix of shops, offices, apartments, and homes on site.
Mixed-use within neighborhoods, within blocks, and within buildings. Diversity
of people (of ages, income levels, cultures, and races.)
4. Mixed Housing:A range of types, sizes and prices in closer proximity.
5. Quality Architecture and Urban Design: Emphasis on beauty, aesthetics,
human comfort, and creating a sense of place. Human scale architecture and
beautiful surroundings nourish the human spirit.
6. Traditional Neighborhood Structure: Discernible center and edge. Public
space at center. Importance of quality public realm; public
open space designed as civic art. Contains a range of uses and densities
within 10-minute walk.
7. Increased Density:ore buildings, residences, shops, and services closer
together for ease of walking, to enable a more efficient use of services
and resources, and to create a more convenient, enjoyable place to live.
8. Green Transportation: A network of high-quality trains connecting cities, towns,
and neighborhoods together. Pedestrian-friendly design that encourages a
greater use of bicycles, rollerblades, scooters, and walking as daily
transportation.
9. Sustainability: Minimal environmental impact of development and its
operations. Eco-friendly technologies, respect for ecology and value of natural
systems. Energy efficiency. Less use of finite fuels. More local production. More
walking, less driving.
10. Quality of life: Taken together these add up to a high quality of life well worth
living, and create places that enrich, uplift, and inspire the human spirit.

The City Beautiful Movement was a reform philosophy of North American architecture
and urban planning that flourished during the 1890s and 1900s with the intent of
introducing beautification and monumental grandeur in cities.

The New Town Movement was derived from the Garden City Movement, founded by
Ebenezer Howard in the late 1800s, as an alternative to the overcrowded, polluted,
chaotic and miserable industrial cities that had appeared in Britain. Towards the end of
World War I (1914-18) a group developed – the ‘New Townsmen’ – whose members
were Howard, F.J. Osborn, C.B. Purdom and W.G. Taylor. They began advocating the
development of 100 new cities to be built by the government.
New Town Movement started after World War II (1939- 45)

S. Urban Planning Propounded by


No Concept
1 Satellite Town Concept Raymond Unwin
2 Linear City Concept Soriya Y. Mata
(1882)
3 City Beautiful Movement Daniel Burnham
(1893)
4 Garden City Concept Ebenezar Howard
(1898)
5 Industrial City Tony Garnier (1918)
6 Concentric Zone Model Ernest Burgess
(1925)
7 Neighbourhood Concept Clarence Perry
(1929)
8 A town for the motor Clarence Stein/Henry
age- Radburn Plan Ford (1929)
9 Broadacre City F. L. Wright (1932)
10 Sector Model Homer Hoytt (1939)
11 Multiple Nuclei Model Harris and Edward
Ullman (1945)
Q.39 Match the figures of vault in Column- I with their corresponding types in Column- II
and select the appropriate option.

(A) P-3, Q-4, R-1, S-2


(B) P-3, Q-1, R-4, S-5
(C) P-2, Q-1, R-5, S-3
(D) P-2, Q-3, R-1, S-5
Answer: (A) P-3, Q-4, R-1, S-2

Q.40 A colony of 50 people is served by a septic tank. The rate of water supply is 90 lpcd in
the colony and 40% of it is going to the septic tank. The retention
period of the tank is 24 hours. The length of the septic tank is _____
meter (rounded off to two decimal places).
Assume, storage capacity/person = 0.085m³( 3 years)
Space for digestion = 0.0425m³/person
Depth of tank = 1.4m
Length: Width = 2:1
Answer: 3.4 m
Rate of water supply to septic tank (40% of 90 lpcd) = 36 lpcd
Amount of water to be retained for 24 hr in septic tank 36 × 50 = 1800 litres/ 24 hr= 1.8

Space for storage for 3 years 0.085 m³ × 50 = 4.25 m³
Space for digestion 0.0425 × 50= 2.125 m³
Total volume required for septic tank (1.8 + 4.25+ 2.125) = 8.175 m³

Depth of tank = 1.4m (given) so surface area of tank = = 5.84 m²


Let one side of tank be ‘x’ m so other side = ‘2x’ m
x×2x = 5.84 m²
2x² = 5.84 m²
x= m²
= 1.7 m and 3.4 m

Q.41 A cone, with a base of 10cm diameter and axis of 12cm, is lying on Horizontal Plane
(HP) along its generator. The internal angle which the base of the cone
makes with HP is ______ degrees.

Answer: 67.0 – 68.0

Tan θ=
Tan θ=2.4
θ=
= 67.3º

Q.42 A public utility building of 5000 m² was constructed 5 years before, on a site of 1
hectare. The present value of open land in that location is Rs.
100/m² and present construction cost of such building is Rs. 2500/m². If the
value of the building is assumed to be depreciating at a constant
rate of 6 percent per annum, then the present value of
the property using ‘Valuation by Cost Method’ is ______ (in Rs. Lakh) (rounded off to one
decimal place).
Answer: 101.2 lakh
Cost of land @ Rs 100 /m²= 10,000 × 100= 10,00,000 Rs
Cost of construction @ Rs 2500 /sqm= 5000 × 2500 = 1,25,00,000 (here we assume
that the cost of const. 5 years ago was this much)
Depreciated value of the building = 1,25,00,000
=1,25,00,000 (
= 1,25,00,000 (0.73)
= 91,25,000 (value of building)
Value of Property = Value of land + value of building
10,00,000 + 91,25,000 = 1,01,25,000

Q.43 A residential area of 20 hectares is planned for three different types of plots of 500m²,
300m² and 200m² with numbers of plot in each category are 100,
120, 150 respectively. The rest of the area is allowed for roads and
facilities such as school, shops and parks. Each plot has one dwelling unit and the
average household size is 5 persons. The net residential density
of the area in person per hectare is ______.
Answer: 159.4 person/ ha
There are two terms Net Residential density and Gross Residential density
20 hectare= 2,00,000 sqm (since 1 ha = 100m ×100 m)
Size of No. of Total area Total area in hectare under Residential use
plot (m²) plot (sqm) 116000 sqm = 11.6 ha
1 500 100 50000 Population = Total Dwelling unit (DU) ×
2 300 120 36000 avg. household size
370 × 5= 1850
3 200 150 30000
Total area 1,16,000 Net Residential density=
= 159.4 person/ha

Q.44 In a single lane road, traffic volume of 1000 vehicles/hr moving at 20Km/h, comes to
halt due to an accident. If jam density is 150 vehicle/km, the
velocity of the shock wave generated (in absolute value) is _____ km/h.
Answer: 10 km/h (since absolute value is asked, otherwise -10 km/h)
The equation that is used to estimate the propagation velocity of shock waves is given
below.

=
Where

= propagation velocity of shock wave (km/hour)


= flow prior to change in conditions (vehicles/hour)
= flow after change in conditions (vehicles/hour)
= traffic density prior to change in conditions (vehicles/km)
= traffic density after change in conditions (vehicles/km)

Here = 1000 vehicles /hr and = 0 vehicles/hr


= 50 vehicles/km and = 150 vehicles/km
To find , in one hr under nomal condition there are 1000 vehicles in a stretch of
20 km, so vehicles per km can be found out

=
= - 10 km/hr
Note the magnitude and direction of the shock wave.
(+) Shock wave is travelling in same direction as traffic stream.
(-) Shock wave is traveling upstream or against the traffic stream.

Q.45 In a site map, a rectangular residential plot measures 150 mm x 40 mm, and the width
of the front road in the map measures 16 mm. Actual
width of the road is 4m. If the permissible F.A.R is 1.2, the maximum built up
area for the residential building will be _____ m².
Answer: 450 sqm

Scale of the map = = = = = 1: 250

Actual size of plot = 150 mm × 250= 37,500 mm= 37.5 m


= 40 mm × 250 = 10,000 mm= 10 m
Size of plot 37.5 m × 10 m and area = 375 sqm
FAR = 1.2 so maximum built up area = 375 × 1.2= 450 sqm
Q.46 The internal dimension of a room is 10m x 10m x 4m (height). The total area of the
doors and windows are 16m². Keeping the doors and windows
closed, the reverberation time of the room becomes 1.2 second. Assume
all the interior surfaces including doors and windows have some absorption
coefficient. If all doors and windows of the room are
kept fully open, the reverberation time will be _____ second (rounded of to one
decimal place).
Answer: 0.95 sec
Volume of Room 10× 10× 4= 400 m³
Surface area of room (total wall area + floor and ceiling area) = 4(10 × 4) + 2(10
×10)
= 360 m²

= 0.148
Case 2 ( when door and window of 16 m² is open)

= = 0.95 sec
(Note: Open fenestration has absorption coefficient of 1)

Q.47 A depressed portion of a land is identified by three closed contours, as shown in the
figure below. The area bound by three contour lines is 6m²,
24m² and 96m² respectively. The contour interval is 1m. Using prismoidal
method, the volume of the earth needed to fill land depression is ________m³.

Answer: 66 m³
Soln:Prismoidal Volume formula

= [ × Length

= (96 + 4(24) + 6) × 2

= × (198) × 2
= 66 m³
Q.48 Solar panels are proposed to be installed on a building roof top to generate electricity.
The size of each solar panel is 2m². The efficiency of each panel is 75%. The
orientation of the solar panel and related solar data are given in the table
below.
Orientation No of Average daily Average solar
panel solar radiation hours per day
in W/m²
South 10 400 4
West 5 300 2
As per above proposal ______ kWh solar power will be generated daily (rounded off to
one decimal place).
Answer:
South Direction: Total size of solar panel 10 = 20 sqm
Total solar power received 400 W/m × 20m² = 8000 W
total solar energy received in 4 hour 8000 × 4= 32,000 Watt.hr

Efficiency 75% therefore 32000× = 24,000 Watt. hr


= 24 KWh
North Direction: Total size of solar panel 5 = 10 sqm
Total solar power received in 300 W/m² × 10 m²= 3000 W
total solar energy received in 2 hour 3000 × 2 = 6000 Watt.hr

Efficiency 75% therefore 6000 × = 4500 Watt.hr


= 4.5 KWh
Total solar power generated 24 + 4.5= 28.5 KWh

Q.49 A power shovel is having 1.8m³


excavation output per batch of operation. The average cycle time of the batch
operation is 45 seconds. The lost time per hour of the excavation activity is
10 minutes. Assume six working hours of operation per
day. The amount of soil excavated by the power shovel per day is ____________ m³
(rounded of to one decimal place).
Answer: 720 m³
Lost time per hour = 10 min
In 6 hour it will loose 6 × 10 = 60 min (1 hour)
So effectively it will work for 5 hour = 5 hr × 60 = 300 min= 300 ×60 = 18000 sec

Avg. cycle time 45 seconds so no of cycles = 400 cycles


Amount of excavation = 1.8 m³ × 400= 720 m³

Q.50 A room having dimension 12m x10m x 3.5m is required to be mechanically ventilated
by air-conditioner. The temperature difference outdoor ambient air
and the supply air is 12 ℃ . Consider three air exchanges per hour.
The volumetric specific heat of the air is 1250J/m³ ℃ . Assume one ton of
refrigeration (TR) is equal to 3.5kW. The
capacity of the air-conditioner for the room in TR will be _______.
Answer: 1.5 Ton
Soln: Volume of Room = 12 m × 10 m × 3.5m = 420 m³
Total amount of air replaced in 1 hour 420 × 3= 1260 m³ (Since three air changes per
hour)
Amount of heat transfer per hour = Vol × Specific heat × ∆T (where ∆T is temp
difference)

= 1260 m³ × 1250 × 12 ℃

Amount of heat transfer per second = 5250 watt (Joules/sec=


watt)

Tonnage of Refrigeration (TR) = 1.5 Ton

Q.51 A simply supported beam AB has a clear span of 7


meter. The bending moment diagram (BMD) of the beam due to a single concentrated
load is shown in the figure below.
The magnitude of the concentrated load in kN is _________.
Answer: 21 kN

Moment =

36 kN-m=

36 kN-m=

P= = 21 kN

Q.52 For a symmetrical trapezoidal open drain in a landscape with grass and loose rock,
surface, the velocity of flow of water is ___m/sec, (round off to two
decimal places), given the following data.

Answer: 0.425 m/s


Water edge width at the top = 750 mm
Water edge width at the bottom = 450 mm
Water depth = 600 mm
Manning’s coefficient of roughness = 0.05
Slope along the drain = 1 in 250
Answer:
Manning Formula for open drain

Where V= velocity or flow of liquid in m/s


n = Manning’s coefficient of roughness
R= Hydraulic Radius (area of cross section of fluid/ wetted perimeter)
S= channel slope

Calculation of hydraulic radius =

Area of cross section of fluid = (0.45 m ×0.6 m) + ×2


= 0.27 + 0.09 m²
= 0.36 m²

Wetted Perimeter= 0.45m + )m


= 0.45m + 1.24 m
= 1.69 m

Hydraulic Radius = 0.213 m

V= ×( ×(
= 20 × 0.355 × 0.06
= 0.425 m/s

Q.53 The stack pressure is created by 10m height of stack and 15 ℃ temperature
difference. The motive force due to the stack pressure over a cross section
area of 2.5m² is _____N.
Answer: 15.75 N
The Stack Pressure ( ) can be calculated from the equation = 0.042 x h x ∆T

Where
h = height of stack in m
∆T= difference in Temp (deg C)
(the constant is N/
Stack pressure = 0.042 x 10 x 15 = 6.3 N/ sq.m
Motive Force= Stack pressure × cross-sectional area = 6.3 x 2.5 = 15.75 N

Q.54 An industrial building contains 3000 kg of combustible materials, in dry state,


distributed over three rooms of area 100m², 500m² and 300m²
each, in a proportion of 30%, 50% and 20% of the contents, respectively.
Calorific value of the material is 4400kCal/kg. The total fire load of
the rooms is equal to __________ kCal/m²
Answer: 14667 kCal/m² (Two ranges were given 61500 – 61700 and 14600- 14700)
Total Combustible material = 3000 kg @ 4400 kCal/kg
Total Energy in material 3000 kg × 4400 kCal/kg = 1,32,00,000 kCal
It is distributed over (100 + 500+ 300) = 900 m²

Total fire load = = 14667 kCal/m²

Q.55 A simple truss is shown in the figure below. The truss is loaded with horizontal and
vertical force 15 kN and 25 kN respectively. The force in
member AB wil be ____ kN.
Answer: 20 kN

Taking Moment about fixed end support 15(L) + 25(L) = (2L)


40 (L) = 2 (L)
= 20 kN
GATE 2020
Section: General Aptitude
(One mark each)
Q.1 Rajiv Gandhi Khel Ratna Award was conferred _____________Mary Kom, a six time
world champion in boxing, recently in a ceremony______________the Rashtrapati
Bhawan (the President’s official residence) in New Delhi
(A) on, at (B) to, at (C) on, in (D) with,at
Answer: (A) on, at

Q.2 Despite a string of poor performances, the chances of K. L. Rahul’s selection in the
team are
(A) slim (B) bright (C) obvious (D) uncertain
Answer: (B) bright

Q.3 Select the word that fits the analogy:


Cover : Uncover : : Associate: _______
(A) Inassociate (B) Disssociate (C) Unassociate (D) Misassociate
Answer: (B) Disssociate

Q.4 Hit by floods, the kharif (summer shown) crops in various parts of the country have been
affected. Officials believe that the loss in production of the kharif crops can be
recovered in the output of the rabi (winter sown) crops so that the country can
achieve its food grain production target of 291 million tonnes in the crop year 2019-20 (July-
June). They are hopeful that good rains in July- August will help the soil retain
moisture for a longer period helping winter sown crops such as wheat and pulse
during the November- February period.
Which of the following statements can be inferred from the following passage
(A) Officials feel that the food- grain production target cannot be met due to floods
(B) Officials want the food-grain production target to be met by the November- February
period.
(C) Officials hope that the food gain production target will be met due to a good rabi
produce.
(D) Officials declared that the food grain production target will be met due to good rains.
Answer: (C)

Q.5 The difference between the sum of the first 2n natural numbers and the sum of the first
n odd natural number is __________
(A) 2 (B) -1 (C) (D) +n
Answer: (D) +n
(Two marks each)
Q.6 Repo rate is the rate at which Reserve Bank of India (RBI) lends commercial banks, and
reverse repo rate is the rate at which RBI borrows money from commercial banks.
Which of the following statements can be inferred from the above passage?
(A) Decrease in repo rate will decrease cost of borrowing and increase lending by
commercial banks
(B) Increase in repo rate will decrease cost of borrowing and decrease lending by
commercial banks
(C) Increase in repo rate will decrease cost of borrowing and increase lending by
commercial banks
(D) Decreae in repo rate will increase cost of borrowing and decrease lending by
commercial banks
Answer: (A)
Q.7 P, Q, R, S, T, U, V, and W are seated around a circular table:
I. S is seated opposite to W
II. U is seated at the second place to the right of R.
III. T is seated at the third place to the left of R.
IV. V is the neighbor of S
Which of the following must be true
(A) P is a neighbor of R (B) R is the left neighbor of S (C) Q is the neighbor of R (D) P is
not seated opposite to Q
Answer: (D) P is not seated opposite to Q

Q.8 The distance between Delhi and Agra is 233 km. A car P started travelling from Delhi to
Agra and another car Q started from Agra to Delhi along the same road 1 hour after
the car P started. Two cars crossed each other 75 minutes after the car Q started.
Both cars wre travelling at constant speed. The speed of car P was 10 km/hr more than the
speed of car Q. How many kilometers the car Q had travelled when the cars
crossed each other?
(A) 66.6 (B) 88.2 (C) 75.2 (D) 116.5
Answer: (C) 75.2

Q.9 For a matrix M = [ ; I,j= 1,2,3,4 the diagonal elements are all zero and .
The minimum number of elements required to fully specify the matrix is ________

(A) 16 (B) 0 (C)


12 (D) 6
Answer: (D) 6
Q.10 The profit shares of two companies P and Q are shown in the figure. If the two
companies have invested a fixed and equal amount every year, then the
ratio of the total revenue of company P to the total revenue of company Q, during
2013-18 is
(A) 15: 17
(B) 17: 16
(C) 17: 15
(D) 16: 17
Answer: (D) 16: 17
Section: Architecture Planning
(one Mark each)
Q.1 In the architectural style of ancient North Indian Temples, the term ‘Adhisthana’ refers
to
(A) Base Platform (B) Transept (C) Pinnacle (D) Vestibule
Answer: (A) Base Platform
Greek Temple Foundation and North Indian Temple Foundation
Base and Base

Q.2 Who among the following architects has NOT won the Pritzker Architecture Prize till
2019
(A) Moshe Safdie (B) Arata Isozaki (C) I. M. Pei (D) B. V. Doshi
Answer: (A) Moshe Safdie or (B) Arata Isozaki
2020- Yvonne Farrell, Shelley McNamara (Grafton Architects)
2019- Arata Isozaki
2018- B V Doshi
2017- Rafael Aranda, Carme Pigem and Ramon Vilalta of Spanish
studio (RCR Arquitectes)
2016- Alejandro Aravena (GATE 2017)
2015- Frei Otto
2014- Shigeru Ban (Japanese Architect)
2013- Toyo Ito (Japanese Architect)
2012- Wang Shu (Chinese Architect)
2011- Edwardo Souto de Moura
2010- Kazuyo Sejima & Ryue Nishizawa
2009- Peter Zumthor (GATE 2010)
2008- Jean Nouvel
2007- Richard Rogers
2006- Paulo Mender de Rocha (GATE 2005)
2005- Thom Mayne
2004- Zaha Hadid
2003- Jorn Utzon (Sydney Opera House)
2002- Glenn Murcutt
2001- J. Herzog & Pierce de Meuron
2000- Rem Koolhas
1999- Norman Foster
1998- Renzo Piano
1997- Sverre Fehn
1996- Rafael Moneo (GATE 2008)
1995- Tadeo Ando
1994- Christian de Portzampare
1993- Fumihiko Maki (Japanese Architect)
1992- Alvaro Siza
1991- Robert Venturri
1990- Aldo Rossi
1989- Frank Gehry
1988- Oscar Niemayor
1988- Gorden Bunshaft
1987- Kenzo Tango
1986- Gotfried Bohn
1985- Hans Hollein
1984- Richard Meier
1983- I M Pei
1982- Kevin Roche
1981- James Sterling
1980- Luis Barragn
1979- Philip Johnson

Q.3 The stone used in the construction of Kailash temple at Ellora is


(A) Limestone(B) Basalt (C) Sandstone (D) Marble
Answer: (B) Basalt
Temple Kingdom Period Stones Used/carved
Lomas Rishi Ashoka 322–185 Granite
Cave, BCE Q.4 Four
Jehanabad,
Bihar
Ajanta/Ellora Gupta 2nd- 5th Basalt
Caves Period Century
CE
Shore Temple, Pallava 700-728 Granite
Mahabalipuram Kingdom CE
Meenakshi Pandya Earlier Granite, Marble, Slate, vertical lines having
Temple Dynasty temple- 6th Limestone same thickness
Extension century appear to be of the
work by BC same height in
Nayak perspective as
Lingraj Temple, Soma 1000 AD Laterite shown in
Bhubaneshwar dynasty the figure.
Brihadeshwara Chola 1010 AD Granite which line actually
Temple, has the maximum
Tanjore height?
Kandariya Chandela 1017-1029 Sandstone with (A) 1 (B) 2 (C)
Mahadeva, Rajputs CE Granite foundation 2 (D) 4
Khajuraho Answer: (B)
Sun Temple, Chalukya 1026-27 Sandstone
Modhera, dynasty CE Q.5 As per
Gujarat URDPFI Guidelines
Dilwara Chalukya 11th -16th Marble 2015, Government
Temple, Mount dynasty Century of India, choose the
Abu correct hierarchy of
Sun Temple, Eastern 13th Khondolite- plans from higher
Konark Ganga Century predominant material to lower order.
dynasty Chlorite- door jamb (A) Perspective
and some sculptures plan, Regional
Laterite- interior core Plan, Development
plan, Zonal plan
Fatehpur Sikri Akbar 1571 Red Sandstone
(B) Zonal plan,
Monuments (Mughal)
Development plan,
Taj Mahal Shahjahan 1632 Brick and cladded with Regional Plan,
Marble Perspective plan
(C) Perspective plan, Development plan, Regional Plan, Zonal plan
(D) Regional Plan, Perspective plan, Development plan, Zonal plan
Answer: (A) Perspective plan, Regional Plan, Development plan, Zonal plan
Q.6 Which of the following shapes can be used as in interlocking paver block without adding
any other shape?
(A) 3(B) 4 (C) 2 (D) 1
Answer: (A)

Q.7 In India, the Constitution (Seventy Fourth Amendment) Act, 1992, delegates powers to
institutions forming the third tier of government, which are
(A) Development Authority, Municipal Corporation and Municipality
(B) Municipal Corporation, Municipality and Nagar Panchayat
(C) Improvement Trust, Nagar Panchayat and Panchayat
(D) Development Authority, Improvement Trust and Panchayat
Answer: (B) Municipal Corporation, Municipality and Nagar Panchayat

Q.8 As on 2018, ‘Right to Property’ in India is a


(A) Secondary Right (B) Constitutional Rights (C) Fundamental Rights (D) Tertiary
Rights
Answer: (B) Constitutional Rights

The fundamental rights are the basic and


inalienable rights granted to each citizen of India through Part III of the
constitution, and in some cases to non-citizens too.
a) Right to Equality– Article 14,15,16 and 18.
b) Right to Freedom– Article 19,20, 21, 21A and 22.
c) Right against Exploitation – Article 23 and 24.
d) Right to Freedom of Religion – Article 25,26,27 and 28.
e) Cultural and Educational Rights – Article 29 and 30.
f) Right to Constitutional Remedies – Article 32.
Q.9 Tendon is primarily used
(A) to pre-stress concrete (B) as a compression member (C) to prepare a tender
document (D) as roof sheathing
Answer: (A) to pre-stress concrete
Tendon — A steel element, such as a wire, cable, bar, rod or strand, or a bundle of
such elements used to impart prestress to concrete when the element is tensioned.

Q.10 Emergency preparedness for risk reduction does NOT include


(A) rehabilitation (B) revision of code (C) relief distribution (D) rescue
Answer: (B) revision of code
Crisis
Is an event or series of events representing a critical threat to the health, safety,
security or wellbeing of a community, usually over a wide area. Armed conflicts,
epidemics, famine, natural disasters, environmental emergencies and other major
harmful events may involve or lead to a humanitarian crisis.
Disaster
A serious disruption of the functioning of a community or a society causing
widespread human, material, economic or environmental losses that exceed the
ability of the affected community or society to cope using its own resources. A
disaster is a function of the risk process. It results from the combination of hazards,
conditions of vulnerability and insufficient capacity or measures to reduce the
potential negative consequences of risk.
Any occurrence that causes damage, ecological disruption, loss of human life or
deterioration of health and health services on a scale sufficient to warrant an
extraordinary response from outside the affected community or area.
Emergency
A sudden occurrence demanding immediate action that may be due to epidemics, to
natural, to technological catastrophes, to strife or to other man-made causes .
Hazard- Any phenomenon that has the potential to cause disruption or damage to
people and their environment.
Risk
The probability of harmful consequences, or expected losses (deaths, injuries,
property, livelihood, economic activity disrupted or environment damaged) resulting
from interactions between natural or human-induced hazards and vulnerabilities .
Vulnerability
The conditions determined by physical, social, economic and environmental factors
or processes, which increase the susceptibility of a community to the impact of
hazards (1).
The degree to which a population or an individual is unable to anticipate, cope with,
resist and recover from the impact of a disaster.

Q.11 If Beam : Column :: Transom : X


Which of the following options can replace ‘X’ ?
(A) Balustrade (B) Ceiling (C) Sill (D) Mullion
Answer: (D) Mullion

Q.12 The correct chronological order of the given architectural movements is


(A) Romanesque; Roman; Renaissance; Gothic; Baroque
(B) Romanesque; Roman; Baroque; Gothic; Renaissance;
(C) Roman; Romanesque; Gothic; Renaissance; Baroque
(D) Roman; Romanesque; ; Gothic; Baroque; Renaissance
Answer: (C)
Timeline Architecture Important Structures
Periods
11,600- Prehistoric Stonehenge, Menhir, Pylons
3500 BC Times
3050-900 Ancient Pyramids, Great Bath at
BC Civilization Mohenjodaro, Ziggurat
850BC- Classical Pantheon, Rome; Parthenon-
476AD Athens; Colloseum- Rome
527- 565 Byzantine Hagia Sophia, Istanbul
AD
800-1200 Romanesque Basilica of St. Sernin in Toulouse,
AD France
1100- Gothic Notre Dame Cathedral, Paris
1450 AD
1400- Renaissance Villa Rotonda, Venice, Italy
1600
1600- Baroque Palace of Versailles, France
1830
1650- Rococo Pilgrimage Church of Wies
1790
1730- Neo White House, Washington DC
1925 Classicism
1890- Art Nouveau Secession Building – Vienna
1914
1925- Art Deco Chrysler Building in New York City
1937
1900- Modernism Works of Rem Koolhaas, I.M. Pei,
Le Corbusier, Philip Johnson, and
Mies van der Rohe.
1972- Post Works of Robert Venturi, Philip
Modernism Johnson
1997- Neo Works of Frank Gehry, Zaha
Modernism Hadid, Moshe Shafdie
and
Parametrcism
Q.13 The decay of sound in large room is indicated in the following figure. The spike within
the dashed zone denotes
(A) Flutter echo (B) Early reflection
(C) Perfect sound diffusion (D) Echo
Answer: (D) Echo

Q.14 For the same thickness of material layers, relative


position of insulation in the wall sections 1 and 2 shown below will have an
impact on
(A) Thermal Time Constant (B) Thermal Conductivity
(C) Thermal Resistivity (D) Thermal Trnsmittance
Answer: (A) Thermal Time Constant
Thermal inertia of a building can be quantified by thermal time constant which
describes how long it takes, where the heating/cooling of a building is
discontinuous for building temperature to change. Thermal time constant is defined
as the ratio of a building's thermal mass and overall heat loss.

Q.15 The solar altitude angle on April 16 at 7:00 AM in


Kochi is 16º. The same solar altitude angle will occur at the same time in the same year at
the same location on
(A) October 21 (B) September 23 (C) August 27(D) July 21
Answer: (C) August 27
Calculate the number of days to 22 June from 16 April = 14 April days + 31 May days
+ 22 June days = 67th day

Add 67th day to 22 June = August 27.


Q.16 In perspective drawing, the picture plane is in
between the Object and the Observer. If the Observer comes closer straight towards the
picture Plane, without changing the distance between Object and Picture Plane the
perspective image will be
(A) Bigger than the previous image
(B) Smaller than the previous image
(C) Will become the mirror image of the previous
(D) Will remain the same as the previous image
Answer: (B) Smaller than the previous image

Q.17 Shyama- Rai temple of Bishnupur in West Bengal, is an example of


(A) Stone Carved Nagara Style Temple (B) Panch-Ratna type terracotta temple
(C) Nava-Ratna type terracotta temple (D) Stone Carved Dravidian Style Temple
Answer: (B) Panch-ratna type terracotta temple
Ratna Style of Temple (Bishnupur, West Bengal)
First and most prominent is the Ratna style, which has a flat roof with a canopy or
shikhara on top. Here the temples do not have a flat roof, but a slightly
curved roof, which is a depiction of the style of roofs in the region.
Ek (One) Ratna Panch (Five) Ratna Nav (Nine) Ratna
If there is only one If there are 5 shikhara’s Total nine shikhara’s;
shikhara in the – one on center and 4 Main at Centre, set of 4
center of the roof it is on the four corners of shikhara’s on 4 sides and
called Ek-Ratna the roof, it is called 4 on the four corners of
Panchratna the roof, it is called a
Navratna temple.
Bishnupur, West Shyama- Rai Temple. Dakshineshwaar Kali
Bengal Prominent The central shikhara is Temple, Kolkata
Ek-Ratna temple is octagonal while the
Madan Mohan corner shikhara’s are
temple. Built-in 1694 rectangular in shape
CE by King Durjan with interesting curvi-
Singh and has conical roofs.
noteworthy
terracotta work.
Other examples are
Radha Madhab and
Radha Gobind
temple, which has a
unique mini chariot
in the form of a
temple.
Q.18 Which one of the following is NOT a land use S. Use Category (URDPFI
zone? No 2016)
(A) Heritage Zone (B) Industrial Zone 1 Residential
(C) Commercial Zone (D) Agriculture Zone 2 Commercial
Answer: (A) Heritage Zone
3 Industrial
URDPFI Simplified Urban Landuse Classification.
4 Public and Semi Public
Q.19 ‘Formulation of GIS based Master Plan’ is a sub 5 Mixed
scheme of 6 Recreational
(A) Jawahar Lal Nehru National Urban Renewal 7 Transport and
Mission (B) Shyama Prasad Mukherjee Rurban Communication
Mission 8 Primary Activity
(C) Atal Mission for Rejuvination and Urban 9 Protective and
Transformation (D) Smart City Mission Undevelopable Use Zone
Answer: (C) Atal Mission for Rejuvination and Urban 10 Special Area
Transformation

Q.20 One Hectare is equal to


(A) 10000 (B) 4840 (C) 4048 (D) 4000
Answer: (A) 10000

Q.21 One of the cities added to the list of UNESCO World Heritage Sites in 2019 is
(A) Chandigarh (B) Walled City, Jaipur (C) Walled City, Ahmadabad (D) Fatehpur Sikri
Answer: (B) Walled City, Jaipur
Q.22 In the given contour map, the angle at ‘A’
(in degrees, rounded off to two decimal places)
is ______
Answer: 18.2 to 18.5

tan θ =
tan θ = 0.33
θ= (0.33)

θ = 18.26º

Q.23 A 1.2 m high window is located on a south facing wall. The solar azimuth angle is
equal to the wall azimuth angle and the solar altitude angle is 60º. The minimum depth (in
meters, rounded off to two decimal places) overhang required to
completely shade the window is _______________
Answer: 0.68-0.70

tan 60º =

x= = 0.69 m

Solar The solar


Azimuth azimuth angle is
the azimuth
angle of the
sun. It is most
often defined as
the angle from
due north in a
clockwise
direction
Wall The direction
Azimuth that you are
looking at when
looking out
through the
window in the
wall from inside
the building.
Roof The roof
Azimuth azimuth angle is
the azimuth
angle of the
equivalent wall.
For example
increase the tilt
angle until it is
vertical and it is
the orientation
looking out from
inside. A
horizontal roof
has an azimuth
angle of 0o and
a tilt angle of 0o

Q.24 In the given figure, the area of the shaded portion is


Answer: 8

Q.25 Average density of a highway is 25 vehicles per km. Average volume of the vehicles
on the highway is 520 vehicles per hour. The mean speed (in km/hr,
rounded off in one decimal place is) _________________
Answer: 20.8 km/hr
Mean Speed =
= 20.8 km/hr

(Two marks each)


Q.26 Match the terminologies of Munsell Colour Wheel in Group I with their corresponding
descriptions in Group II
Group I Group II
(P) Hue (1) Addition of black colour to the base colour
(Q) Chroma (2) Radial colour variation

(R) Value (3) Addition of White to the base colour


(S) Tint (4) Colour variation through angular differences
(5) Vertical colour variation
(A) P-4, Q-2, R-3, S-1 (B) P-2, Q-4, R-1, S-3
(C) P-4, Q-2, R-5, S-3 (D) P-2, Q-4, R-5, S-1
Answer: (C) P-4, Q-2, R-5, S-3
Hue- (Angular) 1st Dimension. By definition it is pure colour, containing no white,
black or grey. Hue is displayed on Horizontal ring in Munsell Colour solid. There are
ten hues. Five principle hues are Red, Yellow, Green, Blue, Purple and five others are
Y-R, G-Y, B-G, P-B, R-P.
Value:(Vertical) 2nd Dimension. Degree of colous luminosity i.e. brightness.
Displayed on vertical bar, ranging from 0 (black) at bottom to 10 (white)
at top. 5 is grey.
Chroma: (Radial) 3rd Dimension. Measure of quality of relative colourfulness or
greyness. Also known as intensity, purity or saturation. It starts from 0 and there is no
intrinsic upper limit to chroma.
Eg : 10 PB 7/12
Here PB represents Purple- Blue
Number 10 before PB represents purity….. it could be 2.5PB, 5PB (purest) ,7.5PB or
10PB.
7 represents Value
12 is Chroma

Q.27 Match the plants form in Group I with their botanical names in Group II, as per ‘A
Handbook of Landscape’, CPWD 2013, Government of India
Group I Group II
(P) Columnar (1) Pinus roxburghii
(Q) Globular (2) Ipomoea grandiflora
(R) Weeping (3) Juniperus chinensis
(S) Pyramidal (4) Salix babylonica
(5) Mimusops elengi
(A) P-3, Q-5, R-4, S-1 (B) P-1, Q-5, R-2, S-3 (C) P-3, Q-4, R-2, S-1 (D) P-1, Q-3, R-4, S-
5
Answer: (A) P-3, Q-5, R-4, S-1
Q.28 Match the images of Garden in Group 1 with their names in Group II

(A) P-5, Q-3, R-1, S-4


(B) P-1, Q-4, R-3, S-2
(C) P-3, Q-2, R-4, S-5
(D) P-3, Q-4, R-1, S-2
Answer: (D) P-3, Q-4, R-1, S-2

Q.29 Match the architects in Group I with their projects in Group II


Group I Group II
(P) Victor Horta (1) Farnsworth House
(Q) Gierrit Rietvelt (2) Robie House
(R) Mies van der Rohe (3) Tassel House
(S) Frank Lloyd Wright (4) Schroder House
(5) Vanna Venturi House
(A) P-3, Q-4, R-1, S-2 (B) P-2, Q-5, R-4, S-1 (C) P-4, Q-3, R-1, S-2 (D) P-3, Q-4, R-5, S-
2
Answer: (A) P-3, Q-4, R-1, S-2

Q.30 Match the graphical representation in Group I with corresponding elements in Group II

(A) P-2, Q-1, R-4, S-5 (B) P-1, Q-2, R-3, S-5 (C) P-4, Q-1, R-3, S-2 (D) P-2, Q-3, R-4, S-
5
Answer: (A) P-2, Q-1, R-4, S-5
Q.31 Based on the psychrometric chart given below, match the vector in Group I with the
respective process in Group II
Group I Group II
(P) V1 (1) Heating and humidification
(Q) V2 (2) Cooling and humidification
(R) V3 (3) Heating and dehumidification
(S) V4 (4) Sensible heating
(5) Humidification
(A) P-4, Q-1, R-5, S-2 (B) P-3, Q-1, R-5, S-2
(C) P-5, Q-3, R-4, S-1 (D) P-4, Q-3, R-1, S-5
Answer: (A) P-4, Q-1, R-5, S-2
In sensible heating process the temperature of air is increased without changing
its moisture content.

Q.32 Match the software tools in Group I with their primary applications in Group II
Group I Group II
(P) ETabs (1) Acoustic analysis
(Q) Carto (2) Structural analysis
(R) eQuest (3) Statistical analysis
(S) SPSS (4) Energy simulation
(5) Geo-spatial analysis
(A) P-4, Q-1, R-2, S-3 (B) P-2, Q-5, R-4, S-3 (C) P-4, Q-5, R-1, S-3 (D) P-2, Q-4, R-5,
S-1
Answer: (B) P-2, Q-5, R-4, S-3

Q.33 Match the structural form in Group I with their corresponding illustration in Group II
(A) P-5, Q-4, R-2, S-1 (B) P-4, Q-1, R-5, S-2
(C) P-2, Q-4, R-5, S-1 (D) P-2, Q-4, R-1, S-5
Answer: (C) P-2, Q-4, R-5, S-1
Shell roof is three-dimensional
structures consisting of thin membrane slabs, curved in one or more directions which
transfer the loads on points of support, i.e. columns, beams, walls etc.

Q.34 Match the books in Group I with the corresponding authors in Group II
Group I Group II
(P) The Autobiography of an Idea (1) Christopher Charles Beninger
(Q) Letters to a Young Architect (2) Sunil Khilnani
(R) A Pattern Language (3) Francis D. K. Ching
(S) Architecture: Forms, Space and Order (4) Louis H. Sullivan
(5) Christopher Alexander
(A) P-4, Q-1, R-5, S-3 (B) P-3, Q-1, R-5, S-4 (C) P-4, Q-2, R-1, S-3 (D) P-3, Q-2, R-1,
S-4
Answer: (A) P-4, Q-1, R-5, S-3

Q.35 Match the names of tactile paving in Group I with their patterns in Group II
(A) P-3, Q-5, R-4, S-1 (B) P-2, Q-5, R-1, S-4
(C) P-4, Q-2, R-3, S-1 (D) P-3, Q-2, R-4, S-1
Answer: (D) P-3, Q-2, R-4, S-1
Tactile paving was first developed in Japan by Seiichi Miyake in 1965. In Japanese
these tactile tiles are called Tenji blocks. These blocks were first used in Okayama
City in 1967. Such blocks are also called truncated domes.
Most persons with visual impairment have some degree of sight. This is the reason
tactile tiles are made in bright colors like yellow and red. These colors are easier to
be seen by partially-sighted people.
Red titles are often used for indicating a controlled crossing. For example, you will see
red colored tactile tiles on crossings where there is a traffic and/or pedestrian light
available. This indicates to the partially-sighted person that she can use pedestrian
light to cross the road. Yellow colored tactile tiles are used for uncontrolled
crossings.
Persons with visual impairment use a cane to feel the path in front of them. When cane
touches these patterned tactile tiles, the bearer gets various types of information on
how to navigate the way ahead.

Type of Meaning Images


Tactile
Tiles
Tiles with Tiles with embossed flat-
Parallel topped blisters in a square
Blister pattern are used to indicate
Lines that there is a road
crossing.
Tiles with These tiles indicate that
Offset there is a train platform
Blisters ahead. If one is not careful,
she might fall into the ditch.
Such tiles can be used to
indicate a sudden level
change.
Tiles with Placement of these tiles
Across show that there are stairs or
Stripes some other hurdle ahead.
(Corduroy) Be careful. These tiles are
also called corduroy hazard
warning tiles.Places where
these tiles are often used
include:
The top and bottom of
stairs
At the foot of a ramp
At level crossing
Where people may
unintentionally walk
directly on to the platform
at a railway station
Where a footway joins a
shared route

Tiles with These tiles indicate that it is


Along safe to keep going ahead.
Stripes The path has no hurdles
and it’s safe.
Lozenge- These tiles warn the blind
shaped people that they are
Tactile approaching the edge of an
Tiles on-street transit line (e.g. a
tram line).

Q.36 Match the name of architects in Group I with the buildings designed by them in Group
II
Group I Group II
(P) Brinda Somaya (1) Museum of Tribal Heritage, Bhopal
(Q) Sheila Sai Prakash (2) St. Thomas Cathedral, Mumbai
(R) Revathy Kamath (3) Bait-ir- Rauf Mosque, Dhaka
(S) Marina Tabassem (4) Indian Navel Academy, Kerala
(5) Cholamandal Artist’s Village, Chennai
(A) P-2, Q-1, R-4, S-5 (B) P-2, Q-5, R-1, S-3 (C) P-4, Q-5, R-1, S-3 (D) P-3, Q-1, R-4,
S-2
Answer: (B) P-2, Q-5, R-1, S-3
St. Thomas Cathedral, Mumbai was not designed by Brinda Somaya, but it was
renovated by her. It was opened in 1718.

Q.37 Match the terms in Group I with the parameters in Group II


Group I Group II
(P) Frontal Area Density (1) Active Green Area
(Q) Sky View Factor (2) Urban Density in Third Dimension
(R) Drift Index (3) Built Density in Two Dimension
(S) Biotope Factor (4) Lateral Stiffness
(5) cross Sectional Property of Urban Canyon
(A) P-3, Q-2, R-1, S-4 (B) P-2, Q-5, R-4, S-1 (C) P-3, Q-4, R-5, S-1 (D) P-2, Q-3, R-4,
S-5
Answer: (B) P-2, Q-5, R-4, S-1
Plan Area Density and Frontal Area Density:

Plan Area Density = Frontal Area Density =


Sky View Factor: A Sky View Factor (SVFs) represents the ratio at a point in space
between the visible sky and a hemisphere centered over
the analyzed location.

Drift Index: The drift index is a simple estimate of the lateral stiffness of the building
and is used almost exclusively to limit damage to
nonstructural components.

Biotope Area Factor: The BAF expresses the ratio of the ecologically effective surface
area to the total land area.

Biotope Area Factor =

Types of surfaces and weighting factors per m2


(Surface types not mentioned can be calculated as long as they have a positive effect on
the ecosystem)
Sealed surfaces- (Weighting factor: 0.0 per m2): Surface is impermeable to air
and water and has no plant growth (e.g., concrete, asphalt, slabs with a solid
subbase)
Partially sealed surfaces- (Weighting factor: 0.3 per m2) :Surface is permeable
to water and air; as a rule, no plant growth (e.g., clinker brick, mosaic paving,
slabs with a sand or gravel subbase)
Semi-open surfaces - (Weighting factor: 0.5 per m2); Surface is permeable to
water and air, water infiltration and plant growth (e.g., gravel with grass, wooden
cobbles, grass paving blocks)
Surfaces with vegetation,unconnected to the soil below- (Weighting factor:
0.5 per m2) :Surfaces with vegetation that have no connection to the ground and
less than 80 cm of soil covering
Surfaces with vegetation, unconnected to the soil below- (Weighting factor:
0.7 per m2) :Surfaces with vegetation that have no connection to the ground but
more than 80 cm of soil covering
Surfaces with vegetation, connected to the soil below- (Weighting factor: 1.0
per m2) :Vegetation connected to soil below, available for development of flora
and fauna
Rainwater infiltration per m2 of roof area- (Weighting factor: 0.2 pro m2)
:Rainwater infiltration for replenishment of groundwater; infiltration over surfaces
with existing vegetation
Vertical greenery with connection to the ground- (Weighting factor: 0.5 per
m2): Direct connection of the vertical greenery with the soil, supply with nutrients
and water directly over the roots in the soil
Vertical greenery without connection to the ground- (Weighting factor: 0.7
per m2) Vertical or horizontal vegetation on a wall without direct connection to
soil on the ground, permanent planters supplying the vegetation areas, artificial
irrigation
Extensive roof greening- (Weighting factor: 0.5 per m2) : Nature-like design of
the roof surfaces with a substrate thickness ˂ 20 cm without artificial irrigation
Semi-intensive roof greening -(Weighting factor: 0.7 per m2); Mixture of
extensive and intensive roof greening with a substrate thickness › 12 cm
(depending on the chosen plantings), usually in combination with artificial
irrigation
Intensive roof greening (Weighting factor: 0.8 per m2): Design of the roof
similar to ground-based green areas with a substrate thickness › 15 cm, usually
in combination with artificial irrigation

Q.38 Match the structural system in Group I with their potential causes of failure in Group II
Group I Group II
(P) Flat Slab (1) Thrust
(Q) Long Column (2) Flutter
(R) Arch (3) Punching Shear
(S) Tensile Fabric (4) Buckling
(5) Moment
(A) P-1, Q-3, R-5, S-2 (B) P-3, Q-5, R-4, S-1 (C) P-2, Q-4, R-1, S-3 (D) P-3, Q-4, R-
1, S-2
Answer: (D) P-3, Q-4, R-1, S-2
Punching shear is a type of failure of reinforced concrete slabs subjected to high
localized forces. In flat slab structures this occurs at
column support points. The failure is due to shear. This type of failure is catastrophic
because no visible signs are shown prior to failure.
Buckling of Columns is a form of deformation as a result of axial- compression forces.
This leads to bending of the column, due to the instability of the
column.
Thrust in Arch: The thrust is the resultant of two forces: the weight of the arch and the
horizontal thrust. Thus, the thrust always pushes downwards with
an angle which depends on the arch profile and weight. The intensity of the
horizontal thrust is generated by the weight of the voussoirs, which rest on
each other, and the flatness of the arch. The flatter the arch is, the
more intense the horizontal thrust is.

The horizontal thrust


(HT) is applied on both springers, but it is also found on top of the arch, as it represents the
balance of the second half of the arch. The horizontal thrust can
be minimized by the optimization of the arch profile. Nevertheless,
there will always be a thrust which can be neutralized by means of buttresses, truss rods or
ring beams.
Flutter in Tensile Fabric: If any section of a fabric loses tension, it "bags" and
"flutters"

Q.39 Match the masonry bond type in Group I with the corresponding illustration in
Group II
(A) P-2, Q-1, R-4, S-5 (B) P-4, Q-1, R-2, S-3
(C) P-2, Q-5, R-1, S-3 (D) P-4, Q-1, R-2, S-5
Answer: (D) P-4, Q-1, R-2, S-5

Q.40 Match the characteristics type in Group I with the type of settlements in Group II as
given in URDPFI Guidelines 2015, Government of India
Group I Group II
(P) Zones of transition from (1) Counter Magnets
natural to urban land uses
located between the outer
limits of urban and regional
centres and rural environmnet
(Q) Towns having potential for (2) Satellite Towns
investment and development
identified on the basis of their
inter-aerial relationship with
the regional nodal centre
(R) Settlements that are growing (3) Peri- Urban Areas
sub- nodal centres but
located out of the direct
functionally linked areas of
the growth node / nodal
centre in the region
(S) Located near or within (4) Priority Towns
reasonable distance, well
connected by transportation
route of the growth node or
metropolitan city and
dependent on growth node
largely for employment
(5) Statutory Towns

(A) P-4, Q-3, R-1, S-5 (B) P-3, Q-5, R-2, S-4 (C) P-3, Q-5, R-1, S-4 (D) P-3, Q-4, R-1,
S-2
Answer: (D) P-3, Q-4, R-1, S-2

Q.41 A population of 2500 persons require a minimum area of 3000 for primary
schools. For the population in four different sectors given in the
table below, the sector having maximum shortage of school area per person.
Sector Population Number Existing
of area of
existing each
schools school (

1 20000 5 2000
2 15000 4 4500
3 12500 2 2500
4 10000 4 1500
Answer: Sector 3
For 2500 person school area required is 3000
Sector Population Number Existing Aggregate Aggregate Deficit shortage of
of area of school School school school area
existing each area area area per person.
schools school ( required available
= No. of (
schools ×
area of
each
school
1 20000 5 2000 24,000 10,000 14,000 0.7
2 15000 4 4500 18,000 18,000 nil nil
3 12500 2 2500 15,000 5000 10,000 0.8
4 10000 4 1500 10,000 6000 4000 0.6

Q.42 Number of married couples in a household along with number of rooms (for a
household) are given in the table. Assuming each married couple needs one
separate room, the total number of additional rooms required for them is
_____________
Number Number of households with
of Married
couples in a 1 Room 2 Room 3 Room
household
0 2500 450 100
1 4700 3000 2000
2 3600 5500 1100
3 432 750 400
Answer: 5214
Every married couples need a separate room.
Number Number of households Extra room
of Married with needed
couples in
a 1 2 3
Room Room Room
household
0 2500 450 100 nil
1 4700 3000 2000 nil
2 3600 5500 1100 3600 ×1=
3600
3 432 750 400 (432 ×2) +
(750 × 1) =
1614
Additional Rooms Required 3600 +1614= 5214

Q.43 In a residential complex, the central play area is to be converted as a detention pond
for storm water management. For a 24 hour rainfall event of 100 mm, a
100 % storm water of central play area and 70 % storm water run-off from rest of
the complex is to be held at a detention pond. Area distribution in the residential
complex is given in the table.
Type Area Run-off
coefficient
(
Apartment 1250 0.80
Blocks
Central Play 150 0.60
Area
Other 200 0.70
Permeable
Areas
Other 400 0.90
Imperrmeable
Areas
The required depth of the detention pond (in mm) is _________
Answer: 760 mm
Type Area Run-off Total Percentage Total vol
coefficient Rainfall of ruoff of
( (m) collected rainfall
collected
(
Apartment 1250 0.80 0.1 0.7 70
Blocks
Central Play 150 0.60 0.1 1 9
Area
Other 200 0.70 0.1 0.7 9.8
Permeable
Areas
Other 400 0.90 0.1 0.7 25.2
Imperrmeable
Areas
Total vol. of water to be collected
114

Required depth = = 0.76 m = 760 mm

Q.44 In the plot shown below , ‘S1’ and ‘S2’ are two non- directional point, having a
sound intensity level of 95 dB and 60 db, respectively, at a distance of 1 m from
each point source. Considering free field conditions, the effective sound
intensity level at the receiver location ‘R’ (in dB, rounded off to two decimal
places) is
Answer: 75- 79 dB
95 dB has more than 1000 times more intensity than 60 dB. Since only two places of
decimal is required we ignore the intensity of 60 dB.
Calculating power at source for 95 dB sound.
Intensity corresponding to 95 dB

95 dB = 10 ( )

( ) = 9.5

=
I= ×
I= ×
Power at source = Intensity × 4π (here distance is 1 m so r= 1)

= × 4π Watt

Calculating intensity at a distance of 9 m =

Now calculating dB = 10 ( )
= 10 ( )

= 10 [ - 81]
= 10 (9.5- 1.90)
= 10 (7.6) = 76 dB

Q.45 A room measures 5m × 10m × 3m (L×B×H). Consider the following conditions


Total solar radiation incident on the roof surface = 800 W/
Outdoor air temperature = 40 ℃

Outside film coefficient of the roof surface = 18 W/ deg C


The outdoor mean radiant temperature is equal to outdoor air temperature

The minimum reduction


required in solar absorption of the roof (rounded off to two decimal places) to achieve a
20º reduction in sol air temperature is
________________
Answer: 0.43- 0.47
What is sol air temperature?
The outside air temperature which, in the absence of solar radiation, would give the
same temperature distribution and rate of heat transfer through a wall (or roof) as
exists due to the combined effects of the actual outdoor temperature distribution plus
the incident solar radiation.

Sol Air Temperature = +


where = sol air temperature in ℃
l = radiation intensity, W/
a= absorbance of the surface
= surface conductance (outside), W/ deg C
(refer Manual of Tropical Housing and Building by Koenigsberger section 3.1.18 page 74,
2016 reprint)

Calculating sol air temperature considering absorbance of roof surface as 1 (total


absorbance and no reflectance)

Sol Air Temperature = 40 ℃ +


= 40 ℃ + 44.44 ℃
= 84.44 ℃
Now as per Question Sol air temperature has to be reduced by 20 deg C( 84.44 ℃ - 20
℃ = 64.44 ℃ )

64.44 ℃ = 40 ℃ + (solving for ‘a’)

= 64.44 - 40

a= 24.44 ×
a= 0.5499
Now is the Twist in Question …. Question is asking ‘minimum reduction required in
solar absorption of the roof’
So the reduction required is 1- 0.5499 = 0.45

Q.46 The activity duration, early start, early finish, late start and late finish of the three
activities ‘P’, ‘Q’ and ‘R’ are shown in the following figure. The independent float of
activity ‘Q’ is _________

Answer: 1 day
Independent Float of activity Q=
Early Successor (R) - Late Predessor (P) – Activity duration of Q
22 - 15 – 6 = 1 day

Q.47 A square based regular pyramid has all sides equal to 10 units. Its height (in the
same units, rounded off to two decimal places) is
_______________

Answer: 7.07
BCDE is the base of the pyramid while height is AF

BD is diagonal of square base and half of it will be =5


Consider the triangle AFB
+ = (Pythagoras Theorem)
+ =
= 100 – 50
AF = = 7.07 units

Q.48 A parking area measuring 52 m × 4.67 m is approached through a driveway as shown


in the given illustration. The parking is designed at an angle of 30º with the parking
bay of 2.5m × 5m
The number of cars that can be parked in the designated parking area considering no car
overshoots the length of the parking area is _____________
Answer: 10

For 30º parking no. of cars =

= = 10. 15 (say 10 cars)

Q.49 Plan and section of an isolated foundation is given below. The volume of concrete up
to Ground Level (GL)
(in , rounded off to two decimal places) is___________.
Portion of Volume Volume Answer: 3.1 to 3.4
foundation
(
Rectangular 2.5× 2 × 0.4 2.00
base
(L× B × H)
Truncated 1.03
pyramid × 0.5 × [ 5 +
1 + 0.2]
×h [ +ab
+
Column 0.2 × 1.1 0.22
3.23
Volume (
Q.50 Top floor of a 25 story building is using a flush valve system with a minimum fixture
pressure of 1.0 kg/ . If static pressure increases by 0.3 kg/ per metre length and
friction loss is zero, then height of the bottom of the water tank
from the top fixture (in meters, rounded off to two decimal places) is
___________.
Answer: 3.30 to 3.40 m
For a fixture to operate minimum pressure required is 1.0 kg/
Pressure develops @ 0.3 kg/ for every metre of vertical length

= = 3.33 m

Q.51 In a single phase alternate current circuit, an electric lamp is rated 100 watts. If 220
volts is impressed on it and the power factor is 0.85, the energy (in watt hour,
rounded off to one decimal place) delivered in an hour is _____________ Answer:
116 to 117 watt hour
In electrical engineering, the power factor of an AC electrical power system is defined as
the ratio of the real power absorbed by the load to the apparent power flowing in
the circuit,

Power Factor =

0.85 =

Apparent power = = 117.64 watt


Energy delivered in one hour = 117.64 watt × 1 hour = 117.64 watt.hr

Q.52 A simply supported RCC beam of cross section 0.4 × 0.6 m covers a span of 8 m. It is
subjected to a uniformly distributed load of 30 kN/m. If the unit weight of concrete is
24 kN/ , the tensile stress (in N/ rounded off to two decimal places at the
bottom of the beam at mid-span is ________________
Answer:
Weight of 1 m of beam

Self Weight: = 0.4m × 0.6m × 24


= 5.76 kN/m
Weight Imposed: = 30 kN/m
= 35.76 kN/m
Moment due to UDL in simply supported beam at midspan is given by:

M=

M = 35.76 ×
= 286.08 kNm (286.08 × Nmm)

Now =

=
= 11.32 N/

Q.53 A basement wall resists lateral pressure


exerted by soil and water. The soil pressure amounts to 4.5 kN/ for every meter
of depth below Ground Level (GL). The sub-soil water level is 1.0 m below GL and
hydrostatic pressure of water is 9.8 9.8 kN/ for every meter of depth
below GL. The total lateral pressure (in kN/ , rounded off to one decimal
place) exerted on the wall 2 m below GL is
Answer: 18.8 kN/
Soil Pressure at 2 m depth = 4.5 kN/ ×2
= 9.0 kN/
Hydrostatic Pressure = 9.8 kN/
Total Pressure (Soil Pressure + hydrostatic Pressure) = 18.8 kN/
Q.54 Assuming that the population growth trend given in the table will continue, the
population (in persons) for the year 2031 will be
S. No. Year Population
(in person)
1 1981 1,30,440
2 1991 1,69,572
3 2001 2,20,444
4 2011 2,86,577
Answer: 4,84,315
S. No. Year Population Increase Increase
(in person) in decadal in %
population decadal
population
1 1981 1,30,440 -
2 1991 1,69,572 39,132 29.99
3 2001 2,20,444 50,868 29.99
4 2011 2,86,577 66,133 29.99
We observe a Geometric growth with a decadal growth rate of 29.99 %
= × 1.30 = 2,86,577 × 1.30 = 3,72,550
= × 1.30 = 3,72,550 = 4,84,315
Q.55 A developer would like to select a residential plot of 3000 for group housing in a
city. Different options with varying development controls are given. In every group
housing plot, 15% of the Floor Area Ratio (FAR) over and above the
maximum permissible FAR has to be utilized for Economically Weaker Section (EWS) units.
The maximum built up area (in available from the options given below
is _________

Area Ground FAR


Coverage (%)
1 30 1.5
2 20 2.0
3 40 2.0
4 15 3.0
Answer: 10,350
Area of the Residential plot is 3000
Area Ground FAR Total Built up
Coverage area as per
(%)
FAR (
1 30 1.5 4500
2 20 2.0 6000
3 40 2.0 6000
4 15 3.0 9000
In option 4 adding 15% area above maximum permissible FAR for EWS units = 9000 +

(9000 × ) = 10,350
Number of candidates appeared and cut off marks for GATE AR

ARCHITECTURE/ PLANNING

GATE, for long, has been known to test the Arch/Planning basics in a smart way.
Complaints of “lengthy” problems have been rare. But the task of mastering an
entire course of syllabus (around 30 undergraduate subjects) for a three-hour test,
itself gives the test a certain level of toughness. Each year, only around 18 % of
all appearing candidates qualify.

Cut off Marks


Year Appeared Open OBC SC/ST/PH Highest Topper
(NCL)
2020 9649 34.8 31.30 23.20 72.00 Kintan
Sah
2019 9170 41.00 36.90 27.30 83.33 Akshay
Rathi
2018 7494 43.90 39.50 29.20 79.33 Anup
Bharti
2017 5607 35.70 32.10 27.80 - Abhishek
Bisht
2016 5812 38.90 35.00 25.90 75.67 B K Das
2015 4147 45.17 40.65 30.11 80.67
2014 3087 32.50 29.25 21.67 59.33
2013 2718 40.51 36.46 27.01 70.67
2012 2500 44.72 40.25 29.81 80.67
2011 2516 42.63 38.37 28.42 78.67
2010 1996 - - - -

You might also like